Das Universum dehnt sich ja bekanntlich aus – und wie wir diese Woche gelernt haben, gab es dabei mit ziemlicher Sicherheit eine Zeit, in der diese Ausdehnung extrem schnell war. Aber was passiert eigentlich, wenn sich der Raum ausdehnt? Warum dehnt sich z.B. unser Sonnensystem nicht mit aus?

Die Ausdehnung des Universums wird ja gern mit folgenden Bildchen veranschaulicht:

Universe expansion-de.png
Von Leipnizkeks, CC BY-SA 3.0, Link

Hier sieht man ein “flaches” Universum, in dem sich die Objekte immer weiter voneinander entfernen. Was an diesem Bild nicht so schön ist, ist, dass es so aussieht, als wäre der Punkt in der Mitte irgendwie speziell – aber das ist er nicht. Alternativ wird deswegen oft das Ballonbild verwendet:

expansionFG26_005

Quelle: E. Siegel, Starts with a Bang

Man stellt sich die Objekte wie kleine Punkte vor, die auf der Oberfläche eines Ballons angeheftet sind und die sich dann alle voneinander entfernen.

Und das wirft natürlich folgende Frage auf: Wenn zwei Punkte am Anfang einen bestimmten Abstand zueinander haben und sich das Universum dann ausdehnt, dann vergrößert sich ja der Abstand zwischen ihnen. Wenn an jedem dieser Punkte ein Objekt sitzt, entfernen sich die beiden voneinander. Warum wird dann zum Beispiel unser Sonnensystem oder unsere Galaxis nicht mit expandiert?

Bei der Suche nach einer Antwort auf diese Frage bin ich nebenbei auf folgendes Filmzitat gestoßen:

Mrs Felix: Why don’t you do your homework?
Allen Felix: The Universe is expanding.  Everything will fall apart, and we’ll all die.  What’s the point?
Mrs Felix: We live in Brooklyn.  Brooklyn is not expanding!  Go do your homework.

(from Annie Hall by Woody Allen)

Diese Frage wird im Internet an vielen Stellen diskutiert – allerdings sind die Erklärungen manchmal sehr problematisch. Meistens heißt es, dass die Kräfte, die zum Beispiel das Sonnensystem zusammenhalten, eben einfach größer sind als die Kraft, die die Objekte des Sonnensystems oder die Galaxien auseinandertreibt. In dieser Diskussion zum Beispiel wird das Ballonbild verwendet und die Galaxien mit Münzen. Die Kraft, die die Münzen zusammenhält, ist “größer als die Reibungskraft”. Die Galaxien werden nicht auseinandergetrieben, weil die zusammenhaltende Kraft der Galaxien (die Schwerkraft) größer ist als die Kraft, mit der sie durch die Expansion des Universums auseinangetrieben werden.

Ich gebe zu, dass ich das bisher auch immer so verstanden hatte. Dummerweise ist dieses Bild aber nicht wirklich richtig – es liefert eine falsche Vorstellung davon, welche Kräfte da wirken können. Ich habe das gestern gemerkt, als ich spaßeshalber ausrechnen wollte, wie groß die Kraft ist, mit der zwei Punkte im Universum durch die Expansion auseinandergetreiben werden – dummerweise kam immer Null heraus. Also wie denn nun?

Betrachten wir den einfachsten Fall: wir nehmen ein Universum, dass sich vollkommen gleichmäßig und mit zeitlich konstanter Rate ausdehnt. So eine Ausdehnung wird durch den sogenannten Hubble-Parameter beschrieben. Laut Wikipedia hat der den Wert

H_0 \approx (74{,}3 \pm 2{,}1) \ \frac{\mathrm{km}}{\mathrm{s \cdot Mpc}}

Was bedeutet das? Zwei Objekte, die sich in einer Entfernung von einem Megaparsec (1 Million Parsec, also etwa 3 Millionen Lichtjahre) befinden, entfernen sich mit einer Geschwindigkeit von etwa 74km/s voneinander. So jedenfalls die gängige Interpretation – die ist auch nicht falsch, aber ein bisschen irreführend. Tatsächlich ist es ja der Raum, der sich ausdehnt – man kann auch sagen: Zwischen den beiden Objekten entsteht neuer Raum, und zwar (in dieser Richtung) 74 Kilometer Raum pro Sekunde.

Klingt viel – aber ein Megaparsec ist auch ziemlich lang. Man kann den Hubble-Parameter auch umrechnen, wenn man sich erinnert, dass ein Megaparsec ja auch eine Längeneinheit ist. rechnet man das Megaparsec in Meter um, dann bekommt man (ebenfalls laut Wiki) den Wert 2,3·10−18 s−1.
Das mag auf den ersten Blick seltsam aussehen, weil die Dehnung die Einheit pro Sekunde hat – müsste da nicht eine Länge drin stecken? Es ist aber alles in Ordnung – eine doppelt so lange Strecke dehnt sich ja auch doppelt so viel, so dass man die Länge herauskürzen kann. (Alle IngenieurInnen kennen das – die Dehnung ist eine Größe ohne Einheit, weil es eine Längenänderung bezogen auf eine Ausgangslänge ist.) Eine beliebiges “Stück Raum” wächst also in einer Sekunde um das 2,3·10−18-fache.

1 / 2 / 3 / Auf einer Seite lesen

Kommentare (394)

  1. #1 AP
    21. März 2014

    (1) “… weil Teilchen ja nicht an Raumpunkte ‘angeheftet’ sind.”

    Hmm, ich bin mir nicht sicher, ob meine Frage thematisch wirklich dazu passt, aber mir kommt da die Stringtheorie in den Sinn: Soweit ich es verstanden habe, sind ja Strings an höherdimensionalen Branen (Raum) gebunden, wären also tatsächlich “angeheftet”.
    Wie wird dort dieses Thema der Expansion mit diesem Hintergrund erklärt?

    (2) “Zwischen den Teilchen entsteht neuer Raum und so entfernen sie sich voneinander, und zwar um so mehr, je weiter sie entfernt sind.”

    Ist deine Konklusion also die, dass Solarsysteme und Galaxien sehr wohl mitgedehnt werden, weil ja keine exakt definierte gegengerichtete Geschwindigkeit existiert, diese aber nur gebremst ist?

    Faszinierendes Thema, übrigens. Mir gefallen deine Beiträge über die scheinbar einfachen Dinge immer sehr gut (zB deine Teilchen A nach B Serie).

  2. #2 MartinB
    21. März 2014

    @AP
    Die Enden der Strings sind an die Branen geheftet, so wie ich es verstehe, aber auf diesen Branen beweglich. ich bin aber kein String-Experte.

    “weil ja keine exakt definierte gegengerichtete Geschwindigkeit existiert,”
    Nein. Die exakt entgegengesetzte Geschwindigkeit existiert – jedenfalls in dem einfachen fall mit den zwei Teilchen und dem Band dazwischen. Aber ich muss diese GEschwindigkeit eben nur “einmal” aufbringen, wenn sich die Expansion ändert, danach bleibt sie gleich.

    Ich gebe aber zu, dass mir die Übertragung auf ein rotierendes System nicht 100% klar ist – die rechnungen dazu zeigen aber das gleiche: Eine Kraft existiert nur, wenn sich die Expansionsrate ändert.

  3. #3 Niels
    21. März 2014

    @MartinB
    Damit hab ich mich vor ein paar Jahren auch schon mal beschäftigt und sogar die selben Paper gelesen.
    “Cosmological expansion and local physics” gibt es ebenfalls frei einsehbar bei arXiv, man muss sich nicht unbedingt mit Physical Review rumärgern.
    Mir hat auch noch “Influence of global cosmological expansion on local dynamics and kinematics” geholfen, da werden viele andere Arbeiten und Ansätze besprochen.
    https://arxiv.org/abs/0810.2712

    Jetzt zur Beckmesserei:

    Ob ein einzelnes Objekt “relativ zum Raum” ruht, könnt ihr ja nicht feststellen – es gibt kein absolutes Bezugssystem

    Na ja, in diesem Fall kann man das aber durchaus feststellen. Die mit der Expansion mitbewegten Beobachter beschreiben doch genau die Objekte, die “relativ zum Raum” ruhen. Das sind wiederum deswegen auch genau die Beobachter, für die die kosmische Hintergrundstrahlung (perfekt) isotrop ist.
    Die Hintergrundstrahlung kann man doch problemlos messen. Wahrscheinlich wolltest du diese aber absichtlich außen vor lassen.

    Aber könnten wir die Teilchen nicht – genau wie vorher beim Zwei-Teilchen-Beispiel – mit passenden Geschwindigkeiten versehen, so dass sich ihre Abstände nie ändern?
    Ja, theoretisch könnten wir das – es würde aber bedeuten, dass wir jedem Teilchen ganz am Anfang eine ganz exakt definierte Geschwindigkeit zuweisen müssen, die genau zur (konstanten) Expansionsrate passt. Soweit ich es sehe, könnten wir in diesem Fall an der Teilchenbewegung tatsächlich keine Expansion feststellen, egal auf welchem Teilchen wir sitzen.

    Objekte entfernen sich aber doch von allen Beobachtern nach dem Hubble-Gesetz, egal wo sich diese Beobachter befinden. D.h. für unterschiedliche, räumlich getrennte Beobachter entfernt sich das gleiche Objekt mit jeweils ganz unterschiedlichen Geschwindigkeiten. Das bedeutet ja die Einheit km/(s*Mpc) des Hubble-Parameter.
    Eine solche Präparierung würde also nur für einen einzigen ausgezeichneten Beobachter im ganzen Universum funktionieren, oder?
    Eigentlich nicht einmal für diesen, weil sich durch die Expansion sehr weit entfernte Objekte zwangsläufig mit “Überlichtgeschwindigkeit” entfernen. Teilchen im Raum kann man nicht auf solche Geschwindigkeiten bringen.

    Eine Kraft entsteht durch die Expansion des Alls also nur dann, wenn sich die Expansionsrate ändert – und tatsächlich findet man in allen papers zum Thema (zum Beispiel hier oder hier) immer nur Kräfte durch die Änderung der Expansionsrate

    Die Kraft ist proportional zu a‘‘ (t)/a(t), also zur zweiten Ableitung des Skalenfaktors geteilt durch den Skalenfaktor.

    Das kann man durch Herumstümpern mit Newton erstaunlich schnell erkennen.
    Man verwendet die lineare Hubble-Beziehung v = H * D.
    (v ist die “Rezessionsgeschwindigkeit”, H der Hubble-Parameter und D die Entfernung.)
    Das benennt man um zu
    R‘ (t) = H(t) * R(t)
    (R(t) ist der Abstand, R ‘(t) seine erste Ableitung.)
    Ableiten von R‘(t) ergibt die Beschleunigung
    R‘‘ (t) = H‘ (t) * R(t) + H(t) * R‘ (t)
    Wenn man jetzt die Definition des Hubble-Parameters H = a‘ (t)/a(t) einsetzt, bekommt man als Kraft, die durch die Expansion auf Körper einwirkt,
    F = m * R‘‘ (t) = m*R(t) *[a‘‘(t)/a(t)].
    (a(t) ist der Skalenfaktor)
    Wenn man das dann als äußere Störung ins newtonsche Zwei-Körper-Problem einsetzt und weiter vereinfacht, kommt man laut “Influence of global cosmological expansion on local dynamics and kinematics” auf folgendes Beispielergebnis:
    As an example, the deviation in the radius for an hypothetical spacecraft orbiting around the Sun at 100 AU would be just of the order of 1 mm.

    Es gibt dann mehrere Möglichkeiten:
    1)
    In unserem frühen Universum und in Universen ohne dunkle Energie, aber mit Materie, ist die Expansion gebremst.
    Damit ist a‘‘ (t) negativ, die Kraft bzw. die “kosmologische Beschleunigung” R‘‘ (t) wirkt also in dieselbe Richtung wie die Gravitation. Der Radius, in dem ein Planet um die Sonne kreist, ist kleiner als er ohne Expansion wäre.
    2)
    Beim Universumsalter von etwa 7 Milliarden Jahren hatten dunkle Energie und Materie ungefähr den gleichen (gegenläufigen) Einfluss auf a‘‘ (t), damit war a‘‘ (t) = 0 und damit auch die “kosmologische Beschleunigung” Null.
    (Das gilt natürlich auch für Fall eines Universums mit konstanter Expansion.)
    Ein Planet ist genau nach dem Zwei-Körper-Modell um die Sonne gekreist, die Expansion hatte keinen Einfluss.
    3)
    In unserem momentanen Universum ist die Expansion durch die dunkle Energie beschleunigt, a‘‘ (t) ist positiv, die “kosmologische Beschleunigung” wirkt der Gravitation entgegen, der Radius eines um die Sonne kreisenden Planeten ist größer als ohne Expansion.
    Für die Zukunft läuft die “kosmische Beschleunigung” in unserem Universum gegen einen Grenzwert.

    Erstaunlicherweise erhält man auch dann eine Proportionalität zu [a‘‘(t)/a(t)], wenn man versucht, das Ganze allgemein relativistisch zu rechnen. Man hat aber noch keine Ahnung, wie man das Problem allgemein lösen könnte, deswegen versucht man es mit Abschätzungen durch verschiedene Ansätzen und Vereinfachungen. Man ist sich aber wohl einig, dass auf jeden Fall kleinere Änderungen als beim newtonschen Ansatz herauskommen müssten.
    Das Fazit von “Influence of global cosmological expansion on local dynamics and kinematics” bezüglich der ART-Abschätzungen ist auch nicht gerade überschwänglich. However, the general problem of gaining a qualitative and quantitative understanding of how the cosmological dynamics influences local systems remains challenging, with only partial clues being so far provided by exact solutions to the field equations of General Relativity.

  4. #4 hubert taber
    wien
    21. März 2014

    https://science.orf.at/stories/1734585
    https://cdvolko.blogspot.co.at/2013/10/einstein-widerlegt.html
    https://science.orf.at/stories/1687300

    soviel zur wirren “urknall”-theorie und der angeblichen “ausdehnung” des universums !

    an diesen fakten ändern auch neunmalkluge, aber in wahrheit wirre “erklärungen”, nichts daran.

    mfg. hubert taber

  5. #5 Bjoern Feuerbacher
    21. März 2014

    Ich habe nichts zu meckern (danke an Niels für die Ergänzungen! 🙂 ), nur eine kleine Anmerkung dazu:

    Eine beliebiges “Stück Raum” wächst also in einer Sekunde um das 2,3·10−18-fache.

    Diese Rechnung habe ich vor langen Jahren auch schon mal gemacht – und dabei festgestellt, dass man ein schönes Ergebnis erhält, wenn man diese Ausdehnungsrate pro Milliarden Jahre statt pro Sekunde berechnet. Dann erhält man nämlich, dass sich das Universum ziemlich genau um H/10 Prozent pro Milliarde Jahre ausdehnt (wenn H wie üblich in km/(s*Mpc) gemessen wird).

    Für H = 70 km/(s*Mpc) also: 7% pro Milliarde Jahre.

    Diese Interpretation des Hubble-Parameters finde ich persönlich deutlich anschaulicher als das übliche “das heißt, dass sich ein Objekt, das so und so weit entfernt ist, mit der und der Geschwindigkeit von uns entfernt”.

  6. #6 DasKleineTeilchen
    21. März 2014

    ah, ist das neuer rekord? gleich mit dem vierten “kommentar” trollt die übliche fraktion mit ihrer “wahrheit” über den urknall. ham diese leute sonst nix zu tun?

  7. #7 Niels
    21. März 2014

    @Bjoern
    Du berechnest aber die prozentuale Änderung des Skalenfaktors, nicht die des Volumens, oder?
    Hm, beim Nachrechnen sehe ich, dass MartinB das ebenfalls so gemacht hat.
    Na ja, eine Strecke würde ich eher nicht “Stück Raum” nennen, ist aber wahrscheinlich Geschmackssache.

    Mit einer Angabe der Rezessionsgeschwindigkeit kann wahrscheinlich niemand besonders viel anfangen, das hat sich halt einfach historisch so ergeben.
    Wird in wissenschaftlichen Arbeiten aber auch kaum benutzt, soweit ich das überblicke.

  8. #8 hubert taber
    21. März 2014

    @ # 6 :
    hier wohnen vermutlich keine drachen, sondern nichtaufnahmefähige, durch anterograde amnesie behinderte.

    u.a. wird erklärt, das die rotverschiebung nicht durch “ausdehnung” zustande kommt, sonder das licht über grosse entfernungen langsamer wird!

    und diese ausage ist durch laufzeitmessungen in langen glasfasern beweisbar.
    vom langsamen rot bis zum schnellen blau.
    auch die spektralfarben sind bewiesenermassen verschieden schnell.

    und deine methode mich in allen foren als belämmert darzustellen greift bei mündigen lesern nicht.

    ich habe angeblich I.Q. 145 und ein fotografisches gedächtnis.

    h.t.

  9. #9 Thomas
    21. März 2014

    Hm. Ich hab das mal spaßeshalber nachgerechnet, also für eine Robertson-Walker-Metrik mit flachen Raumanteil die Geodätengleichung gelöst. Und wenn ich da den Abstand zweier Teilchen im “kosmologischen System” definiere (also bei festem t), komme ich auch bei konstanter Expansionsrate a’ auf eine Beschleunigung. Die ist zwar extrem klein (proportional zu H^4), aber halt nicht null.
    Hab ich mich verrechnet, oder muss man eine bessere Definition für den Abstand verwenden?

  10. #10 Thomas
    21. März 2014

    @hubert taber(#8):
    Glasfasern sind halt nicht das Vakuum. Vakuumdispersion, also dass sich auch im Vakuum die Ausbreitungsgeschwindigkeit von Licht verschiedener Wellenlängen unterscheidet, ist bislang nicht beobachtet worden. Und das liegt nicht daran, dass man nicht danach sucht: Viele Ansätze zur Quantengravitation sagen eine leichte Vakuumdispersion bei hohen Energien voraus, deshalb würden viele Physiker das nur zu gerne finden…

  11. #11 Bjoern Feuerbacher
    21. März 2014

    @Niels:

    Du berechnest aber die prozentuale Änderung des Skalenfaktors, nicht die des Volumens, oder?

    Ja. ‘tschuldige, da habe ich mich etwas unpräzise ausgedrückt.

  12. #12 ADHSapiens
    21. März 2014

    Eine Frage die wir heute kurz diskutiert hatten und die einen bezug zur Expansion des Raumes hat:
    Die Hintergrundstrahlung ist ja durch die Expansion enorm rotverschoben, die Energie von Lichtquanten berechnet man mit E= h mal Frequenz.
    Wo steckt die Energiedifferenz?

  13. #13 mju
    22. März 2014

    Erstmal danke für den tollen Artikel. Normalerweise lese ich hier ja nur mit, jetzt habe ich aber paar Fragen.

    Es klingt so, als ob nicht feststellen könnte, wenn sich das Universum mit konstanter Rate ausdehnt. Aber könnte man nicht folgendes Experiment machen? (Ich lasse mal Relativität und Gravitation aussen von.)

    Ich schicke eine Sonde mit vorgegebener Geschwindigkeit los und messe ständig ihre Geschwindigkeit relativ zum Ausgangspunkt, z.B. indem die Sonde jede Sekunde einen Lichtimpuls zurückschickt und ich am Ausgangspunkt die Zeitdifferenz messe. Wenn das Universum sich ausdeht, oder irgendwie der Raum sich verändert, sollte ich doch messen, dass sich die Geschwindigkeit der Sonde relativ zum Ausgangspunkt beschleunigt. Nämlich wird der Zeitunterschied der empfangenen Impulse größer, je weiter die Sonde weg ist, weil die Signale nicht nur den von der Sonde zurückgelegten Raum zusätzlich überbrücken müssen, sondern auch den Raum, der durch das Raumwachstum dazu gekommen ist.

    Ist die Vorstellung richtig?

    @niels (#3):

    Die mit der Expansion mitbewegten Beobachter beschreiben doch genau die Objekte, die “relativ zum Raum” ruhen. Das sind wiederum deswegen auch genau die Beobachter, für die die kosmische Hintergrundstrahlung (perfekt) isotrop ist.

    Bedeutet das, die üblichen Bilder von der Hintergrundstrahlung (z.B. https://upload.wikimedia.org/wikipedia/commons/3/3c/Ilc_9yr_moll4096.png ) beschreiben nicht wirklich die Strahlung, sondern in Wirklichkeit wie sich der Raum seit dem Urknall ausgedeht hat? Weil, zu Anfang war die Hintergrundstrahlung isotrop, also gleichmäßig? Also beschreiben die Bilder insbesondere, dass sich der Raum nicht gleichmäßig ausgedehnt hat? Das ist absolut faszinierend.

    Ich finde die Bilder gerade um eine Größenordnung interessanter als vorher. Danke für die Bemerkung. (Und irgendwie erklären die üblichen Artikel in den Medien das nie.)

  14. #14 Niels
    22. März 2014

    @Thomas
    Na ja, wie hast du denn den Abstand zweier Teilchen im “kosmologischen System” definiert?
    Wie die comoving distance oder die proper distance?
    Noch mal anders?

    Üblicherweise berechnet man bei so einer Frage doch direkt die geodätische Abweichung mit der zugehörigen Formel, ohne den von dir gewählten Umweg.
    Das wurde im Paper The influence of the cosmological expansion on local systems”” dann auch genau so gemacht, das wird in “2 Equations of motion in the LIF” und im Anhang vorgerechnet.
    Die dadurch erhaltene Bewegungsgleichung (2.8) enthält keine Abhängigkeit von a’.

    @mju
    Sorry, ich hätte (fast perfekte) Isotropie schreiben sollen. Das war missverständlich.
    Diese Bilder zeigen schon genau die Hintergrundstrahlung, die ein mitbewegter Beobachter messen würde. Die Färbung steht ja für winzige Temperaturunterschiede. Für Leute, die nicht gerade diese winzigen Unterschiede untersuchen, ist das Ganze in guter Näherung perfekt isotrop.
    Nach den Friedmann-Modellen dehnt sich das Universum überall gleichmäßig aus, mit örtlich ungleichmäßiger Expansion haben die Temperaturdifferenzen nichts zu tun.

  15. #15 hubert taber
    22. März 2014

    @ #10 Thomas :
    in der glasfaser sind bei verschiedenfärbigen monochromatischen lichtquellen laufzeitunterschiede messbar.
    und das ist auf das vakuum umlegbar.

    das licht über grosse entfernungen LANGSAMER wird, wird sehrwohl beobachtet, nämlich durch die ROTVERSCHIEBUNG.
    für die keinesfalls eine “ausdehnung” und der dopplereffekt verantwortlich ist !

    alle “erklärungen” zur ausdehnung und mathematische “beweise” sind unisono nonsens.
    nur wird den herrschaften leider deren irrung nicht bewusst !

    mfg. hubert taber

  16. #16 Thomas
    22. März 2014

    @Niels:
    Dann liegt wohl an der Abstandsdefinition. Ich hatte einfach das genommen was ohne Expansion des Raumes als Abstand rauskäme, und das mit dem Skalenfaktor multipliziert. So einfach ists wohl doch nicht.
    Und klar nimmt man normalerweise gleich die geodätische Abweichung, aber das hier ist ja nur zum Spaß; ich finds in und wieder nett, solche einfachen Rechnungen komplett von Hand durchzuziehen.

    @all (wg hubert taber) sorry for having fed the troll

  17. #17 MartinB
    22. März 2014

    @Niels
    “Die mit der Expansion mitbewegten Beobachter ”
    Die gibt es aber doch nur, weil wir eine 3K-Hintergrundstrahlung haben, oder nicht? In einem Universum mit nur 2 teilchen und ohne slche Strahlung und ohne alles kann ich doch nie wissen, ob ich mit der Expansion mitbewegt bin, oder?
    Das war irgendwie genau der Punkt, der mich immer verwirrt hat, weil immer so getan wird, als müsse isch alles mit dem Raum mitbewegen. Oder hab ich’s doch falsch verstanden.

    “Eine solche Präparierung würde also nur für einen einzigen ausgezeichneten Beobachter im ganzen Universum funktionieren, oder?”
    Aber auch hier gilt das wieder nur, wenn wir ein Bezugssystem wie die 3K-Hintergrundstrahlung haben, oder? Ansonsten ist nur wichtig (ich argumentiere mal 1 einer Dimension), dass die Geschwindigkeit aller Objekte jeweils einen Wert proportional zur -x-Koordinate hat; da ich eine beliebige konstante Geschwindigkeit draufaddieren kann , gilt das immer noch für alle Beobachter.

    Deine “Herumstümper-Rechnung” ist klasse.

    @hubert Taber
    “durch anterograde amnesie behinderte.”
    Solche able-istischen Beleidigungen sind hier gar nicht gern gesehen. “Behinderte” sind auch Menschen.
    “ich habe angeblich I.Q. 145 und ein fotografisches gedächtnis.”
    Es kommt eben nicht nur drauf an, welche Ausrüstung man hat, sondern auch, wie man sie einsetzt.
    Komisch, dass es immer nur cranks sind, die mit ihrem IQ angeben…

    @Thomas
    Hast du bei deiner Rechnung angenommen, dass die teilchen anfangs zueinander in Ruhe sind? Dann gibt es einen winzigen Effekt dadurch, dass ja der Abstand wächst und deswegen auch imemr mehr Raum pro Zeit zwischen den Teilchen dazukommt. (Das meinte ich oben mit Effekt 2. Ordnung.) Wenn du aber ein gebundenes System hast, bleiben die Teilchen ja auf konstantem Abstand.

    @ADHSapiens
    “Wo steckt die Energiedifferenz?”
    Verdammt gute Frage. Antwort: weiß keiner. (Niels hattei rgendwann mal eine gute referenz aus Spektrum der Wissenschaft, ich weiß aber den link nicht).
    Es ist aber ohnehin problematisch, die Energieerhaltung für das Universum als ganzes zu betrachten.

    @mju
    Die Idee mit der Sonde ist gut – ich denke, theoretisch müsste das so klappen.

    @DIe Üblichen Verdächtigen
    Hey, wenn Ihr alle solche Rechnung spaßeshalber per Hand macht, an denen ich mich stundenlang verwirre, warum schreibt IHR dann nicht solche Sachen auf – gern als Gastbeitrag hier – statt diese ganzen klugen Sachen in der hintersten Ecke meiner Kommentarspalten zu verstecken? Ist doch irgendwie schade…

  18. #18 MartinB
    22. März 2014

    Mal ne sehr simple Frage (weiß einer hier bestimmt): Ist der Hubble-Parameter in einem Universum, das sich mit konstanter Geschwindigkeit ausdehnt, eigentlich Lorentz-invariant? Meine Intuition (und das simple Argument, dass sich Längenkontraktion und Zeitdilatation komepnsieren) sagt mir, dass das so sein müsste, aber gelesen habe ich das nirgends.

  19. #19 KOzi
    22. März 2014

    @Hubert Taber

    Ich stimme Ihnen vollends zu, jedoch machen Sie es sich unnötig kompliziert. Der Beweis ist auch eleganter mittels Todoroffscher Beweisformel abgetan 🙂 Gruß nach Wien!

  20. #20 Chemiker
    22. März 2014

    Ich muß zugeben, daß mich der Artikel verwirrt zurückläßt (was im Drachen­blog sonst eher nicht vorkommt). Dehnt sich Brooklyn jetzt aus oder nicht?

    Wenn ich diese Seite richtig ver­standen habe, dann nicht. Aber die bemühen die Gravitation dafür.

    Wenn ich den Beitrag hier richtig ver­standen habe, dann dehnt sich der Raum zwar aus, aber „flutscht“ unter unseren Füßen durch, was wir natür­lich nicht bemerken, weil er nicht reibt und weil Raum­punkte keine Mar­kierun­gen tragen. Die Gravitations­kräfte ersetzen dabei die im Gedanken­experiment vor­gebrach­ten genau ab­gestimmten Anfangsgeschwindigkeiten.

    Lieg ich damit richtig, oder bewege ich mich auf Cellulose und Lignin?

  21. #21 Niels
    22. März 2014

    @MartinB

    Die mit der Expansion mitbewegten Beobachter ”
    Die gibt es aber doch nur, weil wir eine 3K-Hintergrundstrahlung haben, oder nicht?

    Nein, das hat doch überhaupt nichts miteinander zu tun?
    Mitbewegte Beobachter sind die Beobachter, die bestimmten Geodäten folgen, nämlich denen, für die man das Linienelement ds^2 = dt^2 in die Geodätengleichung einsetzt (schließlich ruhen sie ja per Definition in den mitbewegten Koordinaten). Weil sie sich auf Geodäten bewegen, wirken auf sie natürlich auch keine Kräfte, sie sind frei fallend.
    Wenn man die Skala groß genug wählt, kann man letztlich alle gravitativ gebundenen Systeme einfach als einzelne Punkt-Teilchen verstehen. Jedes dieser Punkt-Teilchen verhält sich wie ein mit der Expansion mitbewegter Beobachter.
    Das ist aber klar, oder? Das ist die Sache, die man mit dem Ballon-Bild oder dem „Napfkuchen mit Rosinen“-Bild veranschaulichen will. Was hat das jetzt mit der Hintergrundstrahlung zu tun?

    In einem Universum mit nur 2 teilchen und ohne slche Strahlung und ohne alles kann ich doch nie wissen, ob ich mit der Expansion mitbewegt bin, oder?

    Doch? Ich beobachte das zweite Teilchen. Wenn es sich von mir mit der Geschwindigkeit 74,3 km/s entfernt, wenn es sich in einem Megapersac Entfernung befindet, aber mit 2*74,3 km/s bei einer Entfernung von zwei Megapersac, 3*74,3 km/s bei drei Megaparsec, … und wenn auch alle Zwischenschritte dem linearen Hubble-Beziehung folgen, weiß ich, dass wir beide uns in guter Näherung mit der Expansion mitbewegen müssen. (Oder aber jemand gibt sich verdammt viel Mühe und wirkt ununterbrochen auf das zweite Teilchen ein, um mir das vorzugaukeln.)
    Nochmal mathematischer: Die Formel für die kosmologische Rotverschiebung sieht doch völlig anders aus als die für die relativistische Dopplerschiebung. Ich muss also nur messen, welcher Kurve das vom zweiten Teilchen abgestrahlte Licht folgt.

    “Eine solche Präparierung würde also nur für einen einzigen ausgezeichneten Beobachter im ganzen Universum funktionieren, oder?”
    Aber auch hier gilt das wieder nur, wenn wir ein Bezugssystem wie die 3K-Hintergrundstrahlung haben, oder? Ansonsten ist nur wichtig (ich argumentiere mal 1 einer Dimension), dass die Geschwindigkeit aller Objekte jeweils einen Wert proportional zur -x-Koordinate hat; da ich eine beliebige konstante Geschwindigkeit draufaddieren kann , gilt das immer noch für alle Beobachter.

    Siehe oben, das dürfte eigentlich nicht mal für einen einzigen Beobachter funktionieren, wenn man lange genug Messen darf.
    Für zwei Beobachter funktioniert das aber nicht einmal für einen einzigen Zeitpunkt. Gehen wir doch mal zu zwei Dimensionen über.
    Ich habe 4 Teilchen, auf denen vier Beobachter sitzen. Diese Teilchen sitzen auf den Eckpunkten eines Quadrates.
    https://commons.wikimedia.org/wiki/File:SquareDefinition.svg
    Die x-Achse sei parallel zur Verbindunglinie durch A und B, die y-Achse parallel zur Verbindungslinie von A und D. Die Seitenlänge sei genau ein Megaparsec.
    Das Universum expandiert, es gilt die lineare Hubble-Formel.
    Für den Beobachter in A entfernt sich D mit 74,3 km/s in y-Richtung, C entfernt sich mit Sqrt[2]* 74,3 km/s entlang der Geraden y(x)=x. (Ich lege einfach mal den Nullpunkt des Koordinatensystems in den Punkt A)
    Für den Beobachter in B entfernt sich C dagegen mit mit 74,3 km/s in y-Richtung, D mit Sqrt[2]* 74,3 km/s entlang der Geraden y(x)=-x+1.
    Jetzt erklär mir mal, wie du diese Verhalten nachstellen willst, wenn du die Teilchen in C und D nur in einem statischen Raum bewegen darfst.

    @ADHSapiens @MartinB

    “Wo steckt die Energiedifferenz?”
    Verdammt gute Frage. Antwort: weiß keiner. (Niels hattei rgendwann mal eine gute referenz aus Spektrum der Wissenschaft, ich weiß aber den link nicht.)

    Diesen?
    https://www.spektrum.de/alias/astronomie-physik/verliert-das-universum-energie/1044837
    In der allgemeinen Relativitätstheorie gilt im Allgemeinen keine Energieerhaltung.
    Die Existenz der Energieerhaltung folgert man üblicherweise aus dem Noether-Theorem. Das klappt in der ART aber nicht so richtig. Das liegt daran, dass die aus Symmetrien über das Noether-Theorem abgeleitete Energiedichte ausschließlich die an das “Gravitationsfeld” koppelnde Materie und Felder beinhaltet und eben gerade nicht das “Gravitationsfeld” selbst.
    Die Ursache bzw. das Hauptargument für die Existenz einer erhaltenen Energie fällt also weg.
    Es wird aber trotzdem immer noch versucht, eine erhaltene Gesamtenergie des Universums zu definieren, auch wenn es in den letzten Jahrzenten noch nicht gelungen ist. Das ist ganz aktuelle Forschung. Tatsächlich weiß aber niemand, ob das überhaupt möglich.
    Selbst wenn doch noch ein Weg gefunden wird: Es ist mittlerweile sicher, dass das dann nur noch wenig mit dem zu tun haben wird, was wir momentan unter dem Begriff Energie verstehen.

  22. #22 MartinB
    22. März 2014

    @Niels
    “Wenn es sich von mir mit der Geschwindigkeit 74,3 km/s entfernt, wenn es sich in einem Megapersac Entfernung befindet, aber mit 2*74,3 km/s bei einer Entfernung von zwei Megapersac, 3*74,3 km/s bei drei Megaparsec”
    Ja, wenn du ein teilchen über einen hinreichend langen zeitraum beobachten kannst, dann ist das so, das war ja auch die Idee mit der Sonde.
    Aber das können wir ja nicht, dazu sind die Effekte zu klein. Wen ich also nur eine Momentaufnahme habe, dann kann ich doch bei einem teilchen, das sich von mir mit einer bestimmten Geschwindigkeit entfernt, erst mal nicht sagen, ob es das auf Grund einer bestimmten Anfangsgeschwindigkeit tut oder weil es sich mit dem Raum “mitbewegt” oder sehe ich das falsch.

    Das Geodäten-Argument verstehe ich ehrlich gesagt auch nicht (anscheinend habe ich es doch nicht richtig verstanden): Wenn ich bei einem Teilchen, das sich auf einer Geodäte bewegt, eine konstante Geschwindigkeit hinzuaddiere und der Raum völlig homogen ist, ist dann die neue Weltlinie nicht immer noch eine Geodäte (außer durch Effekte zweiter Ordnung wie eben den zunehmenden Abstand etc.)?

    Bei dem 2D-Problem hast du die Definition umgedreht: Es geht doch gerade darum, eine Möglichkeit zu finden, die Teilchen statisch am Ort zu lassen, obwohl sich das Universum ausdehnt. Und wenn ich ein Gummituch ausdehne, dann können doch die teilchen darauf reibungsfrei alle an ihrem Ort bleiben, oder nicht?

  23. #23 MisterX
    22. März 2014

    @ Niel:

    Hier ein Artikel von John Baez dazu:

    https://math.ucr.edu/home/baez/physics/Relativity/GR/energy_gr.html

  24. #24 MartinB
    22. März 2014

    @Niels
    Nochmal zum Quadrat, ich glaube, ich bin gerade doof: Wenn ich Punkte auf einem regelmäßigen Gitter anordne, so dass jeder Punkt mit Koordinaten (x,y) eine Geschwindigkeit v(x,y) hat, dann entfernt sich ein punkt am Ort (x,y) von einem am Ort (x2,y2) doch genau mit einer Geschwindigkeit, die proportional zum Abstand ist, oder nicht?
    Wenn in deinem Beispiel C mit einer Geschwindigkeit von (1,1) fliegt (um die Zahlen einfacher zu machen) und B mit (1,0), dann ist dch die Differenz der geschwindigkeiten genau wie gewollt (0,1), oder nicht? Wo ist mein Denkfehler?

    (Das funktioniert natürlich wieder nur als Momentaufnahme, weil die Geschwindigkeiten real ja zunehmen, aber das ist ja ein Effekt 2. Ordnung.)

    Nochmal anders gefragt:
    1. Kann man ohne jede Materie und 3K-Hintergrundstrahlung, wenn man ganz allein im Universum ist, jemals feststellen, ob der Raum expandiert oder ob man sich im passend mitbewegten Bezugssystem befindet?
    2. Wenn wir ein gas aus Teilchen nehmen, die alle brav im mitbewegten Bezugssystem sitzen und sich entsprechend nach Hubbnle voneinander entfernen, und ich addiere zur Geschwindigkeit jedes teilchens einen konstanten Geschwindigkeitsvektor, würde man das jemals messen können? (das ist wieder die Frage von oben: Ist die Hubble-Konstante lorentz-invariant?)

    Beim suchen habe ich noch das hier gefunden:
    https://einstein.stanford.edu/content/relativity/a10854.html
    https://au.answers.yahoo.com/question/index?qid=20100415182125AA5qBrv
    https://www.astro.ubc.ca/people/scott/faq_basic.html

    Auch da steht, wenn ich es richtig verstehe, dass jedes Bezugssystem, das sich konstant relativ zum CMB bewegt, gleichberechtigt ist.

    Ich bin verwirrt…

  25. #25 Niels
    22. März 2014

    @MartinB

    Wen ich also nur eine Momentaufnahme habe, dann kann ich doch bei einem teilchen, das sich von mir mit einer bestimmten Geschwindigkeit entfernt, erst mal nicht sagen, ob es das auf Grund einer bestimmten Anfangsgeschwindigkeit tut oder weil es sich mit dem Raum “mitbewegt” oder sehe ich das falsch.

    Das siehst du meiner Meinung nach richtig.

    Kann man ohne jede Materie und 3K-Hintergrundstrahlung, wenn man ganz allein im Universum ist, jemals feststellen, ob der Raum expandiert oder ob man sich im passend mitbewegten Bezugssystem befindet?

    Nein, meines Wissens nicht.

    Das Geodäten-Argument verstehe ich ehrlich gesagt auch nicht (anscheinend habe ich es doch nicht richtig verstanden)

    Ich glaube, hier habe ich dich missverstanden.
    Ich dachte, mit

    Die mit der Expansion mitbewegten Beobachter ”
    Die gibt es aber doch nur, weil wir eine 3K-Hintergrundstrahlung haben, oder nicht?

    wolltest du darauf hinaus, dass es mit der Expansion mitbewegte Beobachter nur dann gibt, wenn es auch eine Hintergrundstrahlung gibt. Mit der Geodäten-Sache wollte ich darauf hinweisen, dass mitbewegte Beobachter einfach Beobachter sind, die ganz bestimmten Geodäten folgen. Die Existenz solcher Geodäten bzw. die Existenz von Teilchen, die solchen Geodäten folgen, muss doch völlig unabhängig von der Existenz einer Hintergrundstrahlung sein.
    Es ging dir aber gar nicht um die Existenz. Sondern darum, wann man herausfinden kann, ob man sich in einem solchen Bezugsystem befindet.
    Richtig?

    Bei dem 2D-Problem hast du die Definition umgedreht

    Stimmt, das war Quatsch. Deswegen hast du beim Quadrat recht. Sorry, da hab ich mich selbst verwirrt.

    Ist der Hubble-Parameter in einem Universum, das sich mit konstanter Geschwindigkeit ausdehnt, eigentlich Lorentz-invariant?

    Die Frage verstehe ich nicht ganz.
    Der Hubble-Parameter ist definiert als H = a‘ (t)/a(t), das ist natürlich lorentzinvariant.

    Andererseits messen alle Beobachter, die keine mitbewegten Beobachter sind, selbstverständlich unterschiedliche Rotverschiebungen für gleich weit entfernte, mit dem Universum mitexpandierende Objekte, wenn sie sich auf unterschiedlichen “Seiten” ihres jeweiligen beobachtbaren Universums befinden.
    Das sieht man sehr einfach daran, dass eben genau für die mitbewegten Beobachter die Hintergrundstrahlung isotrop aussieht. Die Objekte, die die Hintergrundstrahlung ausgesendet haben, sind ja alle gleich weit von uns weg. Sie haben für einen mitbewegten Beobachter deswegen auch alle dieselbe Rotverschiebung, nämlich ungefähr z = 1100.
    Bewegt sich ein Beobachter in Bezug auf das lokales System des mitbewegten Beobachters, ist die eine Hälfte der Hintergrundstrahlung rotverschoben und die andere blauverschoben, also auf der einen Seite zum Beispiel maximal zu z = 1000 und auf der anderen maximal zu z = 1200.
    Daraus ergeben sich dann natürlich auch unterschiedliche Rezessionsgeschwindigkeiten für gegenüberliegende Regionen des Himmels.

    Die Eigenbewegung der Erde (des Sonnensystems) wird immer herausgerechnet, wenn man einen Wert für H0 ermittelt. Dafür gibt’s dann die relativistische Dopplerformel.

    Wenn wir ein gas aus Teilchen nehmen, die alle brav im mitbewegten Bezugssystem sitzen und sich entsprechend nach Hubbnle voneinander entfernen, und ich addiere zur Geschwindigkeit jedes teilchens einen konstanten Geschwindigkeitsvektor, würde man das jemals messen können?

    Wie ist das gemeint? Alle Teilchen bekommen zum Beispiel + 5000 m/s in x-Richtung?
    Die Teilchen sind nicht gravitativ aneinander gebunden, wenn sie sich entsprechend Hubble voneinander entfernen, richtig?

    Meiner Meinung nach müsste man das dann merken. Wenn wir als Beobachter im Nullpunkt sitzen, ist bei den Teilchen in negativer x-Richtung ist unter Berücksichtigung der Expansion die resultierende Gesamt-Geschwindigkeit geringer, bei denen in positiver x-Richtung ist sie größer. Entsprechend ist auch die gemessene Rotverschiebung unterschiedlich.

    Auch da steht, wenn ich es richtig verstehe, dass jedes Bezugssystem, das sich konstant relativ zum CMB bewegt, gleichberechtigt ist.

    Na ja, in der ART sind alle Bezugssysteme gleichberechtigt. Allerdings natürlich nur in Bezug auf die Physik (die Physik ist in allen Bezugsystemen gleich), nicht in Bezug auf die jeweilige Eigenwahrnehmung von Ereignissen.

    Die drei Links versuchen meiner Meinung nach Folgendes zu vermitteln:
    Eine Geschwindigkeitsdefinition über die Hintergrundstrahlung bringt vor nicht viel, weil das System der Hintergrundstrahlung kein Inertialsystem ist, sondern mit dem Universum mitexpandiert. Koordinatenpunkte dieses System entfernen sich also in “physikalischen Koordinaten“ (proper distance) voneinander.

    Zwei Beobachter, die beide jeweils bezüglich der von ihnen selbst gemessenen Hintergrundstrahlung ruhen, entfernen sich dennoch voneinander. Solche “ruhenden” Beobachter haben also eine Geschwindigkeit gegenüber anderen “ruhenden” Beobachtern.
    Man kann also trotzdem keine “absoluten” Geschwindigkeiten definieren oder messen. Man misst nur, ob oder wie man sich bezüglich seines lokalen(!) “Hintergrund-Isotropie-Systems“ bewegt.

    Weil das Ganze deswegen kein Inertialsystem ist, gibt es auch keine Probleme mit der SRT. Weil es aber überhaupt mitbewegte Beobachter gibt, können wir unmöglich in einer Minkowski-Raumzeit leben.

    Ein besserer Link dazu:
    https://www.xn--relativittsprinzip-ttb.info/faq/cmb-ausgezeichnetes-inertialsystem.html

  26. #26 MartinB
    22. März 2014

    @Niels
    “Es ging dir aber gar nicht um die Existenz. Sondern darum, wann man herausfinden kann, ob man sich in einem solchen Bezugsystem befindet.
    Richtig?”

    ” das ist natürlich lorentzinvariant.”
    Naja, da da ne Skala und ne zeit drinsteckt, ist mir das nicht auf Anhieb klar, dass das lorentz-invariant ist, ich hab’s zwar aus dem Bauch heraus gedacht, hätte aber schwierigkeiten, das sauber zu zeigen. (Kontrahiert der Skalenfaktor per Längenkontraktion? Ich vermute ja und das kompensiert dann die Dilatation, aber ist das so offensichtlich?)

    ” Alle Teilchen bekommen zum Beispiel + 5000 m/s in x-Richtung?”
    Genau, so war das gemeint.

    “Wenn wir als Beobachter im Nullpunkt sitzen, ist bei den Teilchen in negativer x-Richtung ist unter Berücksichtigung der Expansion die resultierende Gesamt-Geschwindigkeit geringer”
    Warum? Wir selbst bekommen ja auch die 5000m/s dazu (oder hatte ich das missverständlich ausgedrückt?).

    Worauf ich hinaus will, ist folgendes: Wenn ich für alle Materie im Universum einen Geschwindigkeitsvektor hinzuaddiere, könnte man das merken (ohne CMB, an dem würde man es ja sehen)? Meiner Ansicht nach nicht, weil eben die Hubble-Konstante lorentz-invariant ist.

    Der Link, den du angibst, scheint das ja genau zu bestätigen, denn da steht:
    “Man kann also durch ein Experiment innerhalb eines abgeschlossenen Systems nicht feststellen, ob sich dieses System relativ zum Mikrowellenhintergrund bewegt oder nicht.”

    Ich kann meine Frage auch auf *noch* eine andere Weise stellen: Wir machen zwei Kopien unseres Universums direkt nach dem Urknall. In der einen addieren wir zu aller Materie zusätzlich dieselbe Geschwindigkeit (und zwar bevor das Universum durchsichtig wird und meinetwegen auch bevor die Gravitationswellen wichtig werden). Könnte man das jemals feststellen? Ergibt das überhaupt sinn? Ich meine, nein, weil man eben keinen absoluten Raum definieren kann, gegenüber dem man sich bewegt.

  27. #27 MartinB
    22. März 2014

    @Chemiker
    Ich hatte dich nicht übersehen, ich wollte aber erst mal ein paar Punkte klären, damit ich keinen Blödsinn vertelle.
    Ich glaube, du liegst fast richtig – nur die Formulierung, dass die Gravitationskräfte die Angfangsgeschw. ersetzen, ist ein bisschen problematisch – nimm an, wir starten die Expansion jetzt, dann müssen wir nur einmal Kräfte aufbringen, um das Teilchen am wegfliegen zu hindern, danach hat es ja die passende GEschwindigkeit und alles ist in Ordnung.
    (Wobei das bei einem rotierenden System wohl tatsächlich etwas komplizierter ist, aber wenn ich es richtig sehe, wird in einem der paper vorgerechnet, dass sich ein solches System eben nicht ausdehnt, wenn man die Gravtitation und die Metrik des ausdehnenden Raumes überlagert.)
    Ich hoffe, das war jetzt nicht noch verwirrender…

  28. #28 Niels
    22. März 2014

    @MartinB

    Ich kann meine Frage auch auf *noch* eine andere Weise stellen: Wir machen zwei Kopien unseres Universums direkt nach dem Urknall. In der einen addieren wir zu aller Materie zusätzlich dieselbe Geschwindigkeit (und zwar bevor das Universum durchsichtig wird und meinetwegen auch bevor die Gravitationswellen wichtig werden). Könnte man das jemals feststellen? Ergibt das überhaupt sinn? Ich meine, nein, weil man eben keinen absoluten Raum definieren kann, gegenüber dem man sich bewegt.

    Ach so.
    Werden beide Universen denn durch exakt dasselbe Friedmann-Weltmodell beschrieben?
    (Gilt z.B. für das Universum “mit addierter Geschwindigkeit” überhaupt noch das kosmologische Prinzip, also Homogenität und Isotropie? Bei der Isotropie bin ich mir spontan nicht ganz sicher.)
    Wenn die Antwort auf diese Fragen ja ist, stimme ich dir zu.

  29. #29 MisterX
    22. März 2014

    Ist Niel eine Art Crackpot?? Bin mir nicht sicher, hab aber auch nicht seine ganzen posts gelesen.

  30. #30 MartinB
    22. März 2014

    @Niels
    “Gilt z.B. für das Universum “mit addierter Geschwindigkeit” überhaupt noch das kosmologische Prinzip, also Homogenität und Isotropie?”
    Ich denke, das ist doch genau das, was das Zitat aus deinem Link oben aussagt, oder nicht?

  31. #31 MartinB
    22. März 2014

    @MisterX
    War das ein Scherz???
    Niels ist derjenige, der von Relativitätstheorie, Interpretationen der Qm und diesem ganzen Zeug etwa 1000 mal mehr Ahnung hat als ich und ohne den hier im Blog jede Menge Blödsinn stehen würde.
    Nicht immer sind Leute, die lange Kommentare schreiben, Cranks, manchmal haben die auch einfach viel zu sagen.

  32. #32 Niels
    22. März 2014

    @MartinB
    Nein, das sagt das Zitat sicher nicht. Da geht es natürlich nur um unser tatsächliches Universum, nicht um irgendein anderes, hypothetisches, in dem so etwas wie ein globales “Geschwindigkeitsfeld” existiert.

    Ich versuch es noch mal anders.
    Die mitbewegten Beobachter sind genau die Beobachter, für die das Universum homogen und isotrop aussieht. Das ganze Universum, nicht nur die Hintergrundstrahlung.

    A comoving observer is the only observer that will perceive the universe, including the cosmic microwave background radiation, to be isotropic. Non-comoving observers will see regions of the sky systematically blue-shifted or red-shifted. Thus isotropy, particularly isotropy of the cosmic microwave background radiation, defines a special local frame of reference called the comoving frame.

    https://en.wikipedia.org/wiki/Comoving_distance#Comoving_coordinates

    If by “isotropic observer” you mean “an observer that sees the spacetime as isotropic”, then yes, this is true.
    […]
    the more standard way of stating it requires you to first set up the standard FRW coordinate chart, and then observe that what you are calling isotropic observers (the more usual term is “comoving” observers) remain at the same spatial coordinates in this chart for all time, hence are “at rest” in this chart
    […]

    From this it seems “isotropic ” observer means measuring isotropic CMBR readings , is this right?

    In a universe with a CMBR, such as ours, yes. More generally, whatever matter/energy/radiation is present will appear isotropic to an “isotropic observer”. (Obviously this only holds approximately in our actual universe.)

    https://www.physicsforums.com/showthread.php?t=620137

    The simplest universe consistent with SR is one which appears isotropic (the same in all directions) to a set of privileged observers, called co-moving observers
    […]
    Next one can show that in such a universe there could be a co-moving observer at every point, all of whom see the universe as isotropic, i.e. 3 co-moving observers imply an infinite number of co-moving observers.(1)
    […]
    (1) : This does not mean the universe seems isotropic to every possible observer. Although SR emphasises that all observers should be treated equally, in cosmology the co-moving observers are more equal than others: anyone who presumes to move relative to them will get a distorted view of the universe in which the observer’s direction of motion relative to the co-movers will pick out a special direction in space.

    https://www.jb.man.ac.uk/~jpl/cosmo/RW.html

    Wir sind also überhaupt nicht auf die Hintergrundstrahlung angewiesen. Es reicht, wenn wir das “Restuniversum” untersuchen.
    Die Hintergrundstrahlung ist für unsere Zwecke auch eigentlich nichts Besonderes. Das sind auch einfach nur Atome, die Licht abgestrahlt haben.
    Der Witz ist nur, dass das so furchtbar viel Licht zu einem ganz bestimmten Zeitraum ist. Deswegen ist es so nützlich.

    Man kann aber stattdessen auch Cepheiden, Supernovae vom Typ Ia oder andere astronomische Standardkerzen verwenden, um festzustellen, ob man das Universum als isotrop wahrnimmt.
    (Ich habe schon in #25 versucht, das zu erklären. Hoffentlich ist es jetzt ein bisschen klarer geworden.)

    Das ist natürlich erst mal nur eine prinzipielle Möglichkeit. Die Erde bewegt sich nur mit etwa 500 km/s in Bezug auf den lokalen mitbewegten Beobachter. Das kann man daher ohne Rückgriff auf die Hintergrundstrahlung kaum feststellen.
    Das Universum ist nur auf relativ großen Skalen homogen und isotrop. (Sonst würden wir ja auch gar nicht existieren.)

    Uns setzt also eigentlich nur die Messtechnik grenzen. Wenn ich mein (geheimes) Prinzipometer anwerfe, kann ich daher problemlos feststellen, ob mein System ein mitbewegtes System ist.

    Das Zitat aus dem Link stimmt aber trotzdem. Wenn ich Photonen in mein abgeschlossenes Laborsystem hineinlasse, ist es ja nicht mehr abgeschlossen. Selbst wenn ich die Hintergrundstrahlung ausfiltere.
    Dieser Satz war also meiner Meinung nach so gemeint, dass man durch lokale Experimente keinen Aufschluss gewinnen kann.

    .

    @Bjoern Feuerbacher
    @Thomas
    Habt ihr dazu zufällig auch eine Meinung dazu? Würde mich freuen, die zu hören! 🙂

  33. #33 Niels
    22. März 2014

    Argh, Formatierung versaut. Ich versuch es noch einmal:

    @MartinB
    Nein, das sagt das Zitat sicher nicht. Da geht es natürlich nur um unser tatsächliches Universum, nicht um irgendein anderes, hypothetisches, in dem so etwas wie ein globales “Geschwindigkeitsfeld” existiert.

    Ich versuch es noch mal anders.
    Die mitbewegten Beobachter sind genau die Beobachter, für die das Universum homogen und isotrop aussieht. Das ganze Universum, nicht nur die Hintergrundstrahlung.

    A comoving observer is the only observer that will perceive the universe, including the cosmic microwave background radiation, to be isotropic. Non-comoving observers will see regions of the sky systematically blue-shifted or red-shifted. Thus isotropy, particularly isotropy of the cosmic microwave background radiation, defines a special local frame of reference called the comoving frame.

    https://en.wikipedia.org/wiki/Comoving_distance#Comoving_coordinates

    If by “isotropic observer” you mean “an observer that sees the spacetime as isotropic”, then yes, this is true.
    […]
    the more standard way of stating it requires you to first set up the standard FRW coordinate chart, and then observe that what you are calling isotropic observers (the more usual term is “comoving” observers) remain at the same spatial coordinates in this chart for all time, hence are “at rest” in this chart
    […]

    From this it seems “isotropic ” observer means measuring isotropic CMBR readings , is this right?

    In a universe with a CMBR, such as ours, yes. More generally, whatever matter/energy/radiation is present will appear isotropic to an “isotropic observer”. (Obviously this only holds approximately in our actual universe.)

    https://www.physicsforums.com/showthread.php?t=620137

    The simplest universe consistent with SR is one which appears isotropic (the same in all directions) to a set of privileged observers, called co-moving observers
    […]
    Next one can show that in such a universe there could be a co-moving observer at every point, all of whom see the universe as isotropic, i.e. 3 co-moving observers imply an infinite number of co-moving observers.(1)
    […]
    (1) : This does not mean the universe seems isotropic to every possible observer. Although SR emphasises that all observers should be treated equally, in cosmology the co-moving observers are more equal than others: anyone who presumes to move relative to them will get a distorted view of the universe in which the observer’s direction of motion relative to the co-movers will pick out a special direction in space.

    https://www.jb.man.ac.uk/~jpl/cosmo/RW.html

    Wir sind also überhaupt nicht auf die Hintergrundstrahlung angewiesen. Es reicht, wenn wir das “Restuniversum” untersuchen.
    Die Hintergrundstrahlung ist für unsere Zwecke auch eigentlich nichts Besonderes. Das sind auch einfach nur Atome, die Licht abgestrahlt haben.
    Der Witz ist nur, dass das so furchtbar viel Licht zu einem ganz bestimmten Zeitraum ist. Deswegen ist es so nützlich.

    Man kann aber stattdessen auch Cepheiden, Supernovae vom Typ Ia oder andere astronomische Standardkerzen verwenden, um festzustellen, ob man das Universum als isotrop wahrnimmt.
    (Ich habe schon in #25 versucht, das zu erklären. Hoffentlich ist es jetzt ein bisschen klarer geworden.)

    Das ist natürlich erst mal nur eine prinzipielle Möglichkeit. Die Erde bewegt sich nur mit etwa 500 km/s in Bezug auf den lokalen mitbewegten Beobachter. Das kann man daher ohne Rückgriff auf die Hintergrundstrahlung kaum feststellen.
    Das Universum ist nur auf relativ großen Skalen homogen und isotrop. (Sonst würden wir ja auch gar nicht existieren.)

    Uns setzt also eigentlich nur die Messtechnik grenzen. Wenn ich mein (geheimes) Prinzipometer anwerfe, kann ich daher problemlos feststellen, ob mein System ein mitbewegtes System ist.

    Das Zitat aus dem Link stimmt aber trotzdem. Wenn ich Photonen in mein abgeschlossenes Laborsystem hineinlasse, ist es ja nicht mehr abgeschlossen. Selbst wenn ich die Hintergrundstrahlung ausfiltere.
    Dieser Satz war also meiner Meinung nach so gemeint, dass man durch lokale Experimente keinen Aufschluss gewinnen kann.

    .

    @Bjoern Feuerbacher
    @Thomas
    Habt ihr dazu zufällig auch eine Meinung dazu? Würde mich freuen, die zu hören! 🙂

  34. #34 MisterX
    22. März 2014

    Was ich viel interessanter finde. Jetzt da man Gravitationswellen aus der Inflationären phase gemessen hat man ja direkt einen Quantengravitationsphänomen gemessen oder? Das heisst das Gravitonen auch direkt gemessen wurden oder? Auf jedenfall sind sie viel wahrscheinlicher geworden.

  35. #35 stone1
    23. März 2014

    Für H = 70 km/(s*Mpc) also: 7% pro Milliarde Jahre.

    Das ist im Angesicht eines eventuellen Big-Rip Exitus unseres Universums eine immer wieder beruhigende Umformung des Hubbleparameters. 😉

  36. #36 hubert taber
    23. März 2014

    da die von mir erwähnten links offenbar nicht gelesen wurden erkläre ich nochmals den ursprung der “idee” urknall:

    die urknall-theorie ist nonsens, da dieses “ereignis” logisch erklärbar niemals stattfand.
    diesen unsinn “erdachte” ursprünglich ein belgischer katholischer landpfarrer und hobby-astronom namens george lemaitre.
    und die päpstliche akademie der wissenschaft sorgte mit vehemenz dafür, dass diese trottelei tatsächlich zur “lehrmeinung” in der physik wurde !

    und zwar deshalb mit vehemenz, das diese angebliche und unerklärbare “ereignis” wieder einen “schöpfungsakt” voraussetzen würde !

    h.t.

  37. #37 hubert taber
    23. März 2014

    und hier die erklärung diverser scheinbegriffe wie “raumzeit”, “multidimensionales nicht lineares diskontinuum” etc. :

    Dienstag, 1. Oktober 2013Einstein widerlegt?
    Da Hubert T. regelmäßig diesen Blog mitliest (wie man auch an seinen zahlreichen Kommentaren erkennen kann), hier etwas speziell für ihn:

    https://www.youtube.com/watch?v=pu15MdmSQ2c&feature=youtu.be

    In diesem Video behauptet Peter Donald Rogers (ein Australier, der angeblich einen IQ von 175 hat), dass Einsteins Relativitätstheorie falsch sei und es möglich sei, verschiedene Phänomene, welche die Relativitätstheorie zu beschreiben versucht, mit einer Theorie zu erklären, die nur mit drei Dimensionen auskommt (statt mit vier, wie Einsteins Theorie).

    Das dürfte dir, Hubert, doch gefallen, denn du bist ja auch der Meinung, dass es nur drei Dimensionen gibt.

    Ich kann mir im Moment dazu noch keine Meinung bilden, dafür habe ich zu wenig Ahnung von Theoretischer Physik. Das gebe ich gerne offen zu. Eingestellt von Claus-Dieter Volko um 01:14
    Diesen Post per E-Mail versendenBlogThis!In Twitter freigebenIn Facebook freigebenAuf Pinterest teilen1 Kommentar:
    hubert taber19. März 2014 07:17
    ES EXISTIEREN NUR 3 DIMENSIONEN !
    nämlich das achsenkreuz x,y,z
    die lage dieses kreuzes ist arbiträr.
    und kardanisch kugelig drehbar in jeder lage möglich.

    in wievielen verschiedenen lagen möglich ?
    bis das volumen einer 3D-kugel vollgeschrieben ist, darüber hinaus wären es wiederholungen.

    es gibt also nur 3 dimensionen.
    das hat der dimensionenbegriff so an sich.

    für den scheinbeweis der wirren “string-theorie” wurden z.b.10 dimensionen “benötigt” !

    scheinbegriffe wie “raumzeit”, “raum-zeit-kontinuum” sind daher unsinnig.
    da die beiden rechnungen RAUM oder ZEIT voneinander unabhängig getrennt möglich sind !

    h.t.

  38. #38 Drogenberatung
    23. März 2014

    Hallo Hubert,
    egal was du nimmst – hör auf damit, es tut dir nicht gut.

    Mit kardanisch kugeligen Grüßen
    “Der-immer-Recht-hat-weil-sein-IQ-angeblich-über-10000-liegt”

  39. #39 hubert taber
    23. März 2014

    @ Drogenberatung :
    schön, dass auch leser mit 1,5 hirnwindungen posten.
    und sich anmassen einen anderen zu qualifizieren.

    und übrigens schreibt der von MatinB gelobte Niels ausschliesslich wirres, widerlegbares zeug.

    h.t.

  40. #40 DasKleineTeilchen
    23. März 2014

    “…und sich anmassen einen anderen zu qualifizieren”

    was DIR natürlich nicht IM TRAUM einfallen würde, gell hubi?

  41. #41 MartinB
    23. März 2014

    @Hubert taber
    Das hier ist die letzte Warnung: Solche Beleidigungen möchte ich hier nicht haben, und wirre youtube-.Widerlegungen der RT interessieren hier wirklich niemanden..

    @Drogenberatung
    Drogen sind ein ernstes Problem – sie als Grundlage für einen Scherz zu nehmen, finde ich nur bedingt witzig.

    @Niels
    Ich glaube, wir reden *immer noch* aneinander vorbei. (Vermutlich drücke ich mich sehr wirr aus…)

    In *unserem* Universum ist völlig klar, dass ein Beobachter die Expansion an Hand der Galaxien, des CMB und überhaupt von allem, was in unserem Universum herumfliegt, beobachten kann.

    Das ist aber nach wie vor nicht meine Frage. Meine Frage ist, ob es einen Unterschied macht, wenn ich *alles* im Universum mit einer zusätzlichen konstanten Geschwindigkeit versehe. Wenn ich sämtliche Galaxien, Teilchen etc. mit einem zusätzlichen Geschwindigkeitsvektor versehen würde, wäre daraus irgendein Effekt beobachtbar?
    Stellenw ir uns vor, ich würde alle Objekte im Universum kopieren, jeweils lokal zur Geschwindigkeit jeder Kopie einen konstanten Wert addieren und dann die Originale verschwinden lassen. Wäre das so entstandene Universum von unserem unterscheidbar?

    Oder, noch mal ganz anders gefragt:
    Genügt es, damit ich die Expansion beobachten kann, dass ich kurz nach dem Urknall hinreichend viele Objekte habe, die alle relativ zueinander entsprechend kleine Geschwindigkeiten hatten und so ein passendes Bezugssystem bilden, an dem ich nachher die Geschwindigkeiten durch die Ausdehnung messen kann? (Und meiner Ansicht nach ist das so, weil die Hubble-Konstante lorentz-invariant ist.)
    Oder müssen diese Objekte *zusätzlich* noch in irgendeiner Form *relativ zum Raum* ruhen (und das ist, wo mein Verständnis hakt – kann man relativ zum Raum ruhen)?

  42. #42 CM
    23. März 2014

    Niels & Martin: Eure Diskussion ist super interessant! Lasst euch bitte nicht von cranks die Stimmung verderben. (Ich versuche euch zu folgen – mein Problem ist mangelnde Zeit und was anfänglich klar war, ist jetzt etwas verwirrt und ich bin mir nicht sicher, ob das einfach an meinem mangelnden Textverständnis liegt. Gelegentlich ist der Blog etwas mathematischer. Vielleicht wäre eine etwas mathematischere – und damit komprimierte – Zusammenfassung eine Überlegung wert?)

  43. #43 MisterX
    23. März 2014

    Wie es existiert nur ein Achsenkreuz?? Und was wenn ich eine vierte Achse dazu male? Implodiert dann das Universum ? 😀

  44. #44 Herr Senf
    23. März 2014

    #41 “Oder müssen diese Objekte *zusätzlich* noch in irgendeiner Form *relativ zum Raum* ruhen (und das ist, wo mein Verständnis hakt – kann man relativ zum Raum ruhen)?”
    Ich bin hier schon Jahre Mitleser, heute glaube meine erste “Einmischung”.
    Für mich scheint das “Vorbeireden” an der bildhaften Deutung zu liegen.
    Die Lorentz-Invarianz ist Bestandteil der SRT, die ist in der ART, wo Hubble und Lambda zusätzlich auftauchen, lokaler Grenzfall. Wir haben bzw. definieren eine kosmische Zeit t=13,8 Mrd J und einen Skalenfaktor a(t). Diese Zeit wird mit einer lokalen Atomuhr gemessen, die Frage der Zeitdilatation stellt sich nicht. Die Expansionsrate bestimmen wir nach einem lokalen Längenmaß, wobei wir annehmen, daß dieses innerhalb der Galaxie bzw. des Haufens selbst nicht expandiert, auch hier kann man nicht nach der Längenkontraktion fragen.
    Die “Fluchtgeschwindigkeit” ist eine Rate, aber keine Relativgeschwindigkeit von inertialen Systemen. Der Hubble-Parameter ist eine empirische Konstante, der nach dem CMB z<10 geringfügig von der kosmischen Zeit abhängig ist, er wird also nur lokal bestimmt, wieso die Frage nach der Lorentz-Invarianz?
    Mit der Annahme der Homogenität und der Isotropie des Universums gehen wir nur davon aus, daß alle anderen lokalen Beobachter mit ihren lokalen Maßstäben denselben Wert des Hubble-Parameters unabhängig von ihrer Pekuliargeschwindigkeit bestimmen würden.
    Die Eigengeschwindigkeit ist aber gering, so daß jeder Beobachter eine relative Ruhe zu seinem Raum hinkriegt.
    Hoffentlich nicht am Thema vorbeigeschlaumeiert.

  45. #45 Niels
    23. März 2014

    @MartinB

    Oder müssen diese Objekte *zusätzlich* noch in irgendeiner Form *relativ zum Raum* ruhen (und das ist, wo mein Verständnis hakt – kann man relativ zum Raum ruhen)?

    Vielleicht liegt hier ein Teil des Problems? Relativ zum Raum ruhen hat in einem statischen Universum keinen sinnvolle Bedeutung. Für mich bedeutet relativ zum Raum ruhen in einem dynamischen Universum, dass man ein mitbewegter Beobachter ist. Damit ruht man dann aber natürlich eigentlich nur relativ zur Expansion.
    Keine Ahnung, wie hier die offizielle Nomenklatur aussieht.

    .
    Na ja, das hat jetzt aber alles nicht mehr besonders viel mit deinem Artikel zu tun. Ich beschreibe mal, wie ich den Ablauf unserer Diskussion bisher verstanden habe. Hoffentlich hilft uns das weiter, um die letzten Missverständnisse aufzuklären.

    In meinem ersten Beitrag ging es um Folgendes:

    Erster Punkt:
    Ich habe auf die freie Verfügbarkeit des einen Papers bei arXiv hinweisen und eine weitere Arbeit erwähnt, die versucht, einen Überblick über das Thema zu geben. Außerdem wollte ich ergänzen, dass man mit einer “newtonschen Stümperrechnung” sehr schnell einsehen kann, warum ausschließlich bei einer Änderung der Expansionsrate Kräfte wirken. Dann habe ich beschrieben, in welche “Richtung” diese Kraft bei unterschiedlichen Vorzeichen wirkt, was das für unser eigenes Universum bedeutet und wie sich das Ganze quantitativ auf ein um die Sonne kreisendes Raumschiff auswirkt. Schließlich ging es noch darum, dass das anscheinend auch unter Berücksichtigung der ART gilt. Wobei es dabei aber bisher nur Abschätzungen gibt, keine exakte Lösung.
    Darum ging es mir in der Hauptsache!

    Zweiter Punkt:
    Da sollten eigentlich nur zwei kleinere Nichtpicker-Einwürfe werden:

    A)

    Wenn also unsere Teilchen am Anfang relativ zueinander ruhen, während sich der Raum ausdehnt, dann bleiben sie auch relativ zueinander in Ruhe – sie fliegen mit genau der Geschwindigkeit aufeinander zu, mit der zwischen ihnen Raum entsteht. Ob ein einzelnes Objekt “relativ zum Raum” ruht, könnt ihr ja nicht feststellen

    “Relativ zum Raum ruhen” hatte ich ganz automatisch als “mitbewegter Bobachter sein” übersetzt und “Objekt” mit “Materie-Teilchen”. Dann habe ich darauf hingewiesen, dass man das eben doch feststellen kann, solange die Hintergrundstrahlung trotzdem existiert.
    Dazu muss man keine anderen Materie-Teilchen beobachten, sondern nur den Grad der Isotropie der Hintergrundstrahlung überprüfen.

    Dazu fragtest du, wie das aussieht, wenn man jetzt mal ausschließlich zwei Teilchen betrachtet.
    Meine Antwort war, dass man selbstverständlich herausfinden kann, ob sich ein Objekt nur aufgrund einer Eigengeschwindigkeit in einem statischen Universum von einem entfernt, oder ob das durch die Expansion des Universums verursacht wird. Dazu müsse man nur längere Zeiträume betrachten.

    Darum ging es dir aber gar nicht.
    Dir ging es darum, ob man bei nur bei der alleinigen Betrachtung zweier zueinander ruhender Objekte feststellen kann, ob man sich in einem statischen oder einem expandieren Universum befindet, richtig?
    Nein, das kann man tatsächlich nicht wenn es um ein expandierendes Universum mit konstanter Expansion geht.

    B)

    Aber könnten wir die Teilchen nicht – genau wie vorher beim Zwei-Teilchen-Beispiel – mit passenden Geschwindigkeiten versehen, so dass sich ihre Abstände nie ändern?
    Ja, theoretisch könnten wir das – es würde aber bedeuten, dass wir jedem Teilchen ganz am Anfang eine ganz exakt definierte Geschwindigkeit zuweisen müssen, die genau zur (konstanten) Expansionsrate passt. Soweit ich es sehe, könnten wir in diesem Fall an der Teilchenbewegung tatsächlich keine Expansion feststellen, egal auf welchem Teilchen wir sitzen.

    Das kam mir irgendwie sehr komisch vor.

    Da habe ich mir wieder vorgestellt, man selbst wäre ein mitbewegter Beobachter. Dann funktioniert das Ganze meiner Meinung nach nicht.

    Das wollte ich mit dem Quadrat-Beispiel verdeutlichen. Dabei habe ich das Problem aber umgedreht, damit hattest du völlig recht.
    Man kann die 4 Ecken sehr wohl passend bewegen, damit das Ganze in einem konstant expandieren Universum konstant bleibt.

    Mein nächster Einwand war:
    Okay, wenn es keine Hintergrundstrahlung gibt, können die Beobachter an den 4 Ecken durch die Beobachtung der jeweils anderen drei Ecken nicht feststellen, ob konstante Expansion stattfindet.
    Aber sie befinden sich doch per Anfangspräparation irgendwo in einem expandieren Universum. Sie können also den Rest des Universums betrachten und überprüfen, ob das für sie isotrop aussieht.

    Das habe ich dann anschließend noch einmal sehr ausführlich dargelegt, als es um die Bedeutung des Zitates im “Definiert der kosmische Mikrowellenhintergrund ein ausgezeichnetes Inertialsystem?”-Link ging.
    (Das war Kommentar #33.)
    War aber wahrscheinlich unnötig, weil wir hier ein wenig aneinander vorbei geredet haben.

    Dir ging es nämlich eigentlich darum, dass alle Teilchen im Universum passend bewegt werden bzw. dass außer den 4 “Eck-Teilchen” keine weitere Materie oder Strahlung im Universum existiert.
    Richtig?

    Das ist mir erst in Kommentar #28 klargeworden.
    Du setzt also ein konstant expandierendes Universum als “Hintergrund-Raumzeit” voraus, in der sich dann Teilchen bewegen.
    Zum Beispiel passend, damit man diese Hintergrund-Expansion nicht mehr wahrnehmen kann. Die Teilchen sind also eigentlich nur Testteilchen, bei deiner Betrachtung ignorierst du nämlich ihren Einfluss auf die Raumzeit.
    Oder nicht?

    Dazu ist dann mein aktueller Einwand aus #28:
    Das kannst du aber doch vermutlich nicht so machen, wenn es um das ganze Universum geht.
    Wo soll denn diese “Hintergrund-Raumzeit” herkommen?
    Schließlich geht die Materie und die Bewegung der Materie doch in den Energie-Impuls-Tensor ein und “erzeugt“ sozusagen erst die Form der Raumzeit.
    Deswegen bin ich zuerst nicht auf die Idee gekommen, dass du das so meinen könntest.

    Noch mal anders:
    Die Zustandsgleichung der Materie und die Struktur der Raumzeit, einschließlich ihrer Entwicklung, kann man im Rahmen der ART doch nicht getrennt betrachten.
    Bildet ein ganzes Universum ausschließlich gefüllt mit solchen “passend bewegten” Teilchen überhaupt ein Friedmann-Universum, in dem Isotropie und Homogenität zumindest in guter Näherung gelten müssen? Gibt es in diesem Universum überhaupt eine räumlich konstante Expansion? Kann diese dann zusätzlich auch zeitlich konstant sein?
    Kann man also überhaupt einen all diesen Ansprüchen genügenden Energie-Impuls-Tensor basteln?
    Vielleicht denke ich hier ja viel zu kompliziert, aber das ist für mich absolut keine triviale Frage.
    Ist jetzt klar, worum es mir geht?

    .

    Jetzt noch mal zu den Fragen, die sich meiner Meinung nach dann nur mehr oder weniger um diesen letzten Aspekt drehen.
    (Alles andere ist jetzt geklärt, hoffe ich?)

    Meine Frage ist, ob es einen Unterschied macht, wenn ich *alles* im Universum mit einer zusätzlichen konstanten Geschwindigkeit versehe.

    Ich bin mir unter anderem wegen der Isotropie-Voraussetzung nicht sicher, ob das dann überhaupt noch ein Friedmann-Universum ist.
    Wenn es das nicht ist, kann man die Expansion auch nicht mehr so einfach mit einer Hubble-Konstanten beschreiben, die Beziehung v=H*d gilt dann nicht mehr. Was dann stattdessen gelten würde, ist mir aber auch nicht klar.
    Da geht es meiner Ansicht nach also um viel mehr als einfach nur um die Frage, ob die Hubble-Konstante Lorentz-invariant ist.

    Stellenw ir uns vor, ich würde alle Objekte im Universum kopieren, jeweils lokal zur Geschwindigkeit jeder Kopie einen konstanten Wert addieren und dann die Originale verschwinden lassen. Wäre das so entstandene Universum von unserem unterscheidbar?

    Siehe oben. Wird das Universum vor und nach diesem äußeren Eingriff durch exakt dieselbe Lösung der Einsteinschen Feldgleichungen beschrieben?
    Oder gilt vor dem Eingriff das kosmologische Modell A, sofort danach aber das sich davon unterscheidende kosmologische Modell B?
    Wenn unterschiedliche kosmologische Modelle gelten, kann man das höchstwahrscheinlich auch feststellen.
    Und selbst wenn es bei der Addition ein konstanten Geschwindigkeit für alle Teilchen passt, wie sieht es dann es, wenn wir die Expansion “ausgleichen” wollen?

    .

    Huch, so viel wollte ich gar nicht schreiben.
    In der Nachschau ist die Ursache vieler Missverständnisse bei mir zu suchen. Tut mir leid. Ich muss mir angewöhnen, länger nachzudenken, bevor ich kommentiere.

  46. #46 MartinB
    23. März 2014

    @Herr Senf
    Was meinst du mit dem letzten Satz:
    “Die Eigengeschwindigkeit ist aber gering, so daß jeder Beobachter eine relative Ruhe zu seinem Raum hinkriegt.”
    Was ist “mein raum” und wie unterscheidet er sich vom Raum von jemandem, der mit 100m/s an mir vorbeifliegt? (Und genau darauf lief auch meine Frage nach der lorentz-invarianz der Hubble-Konstante hinaus.)

  47. #47 Alderamin
    23. März 2014

    @MartinB

    Ich hätte Dir nach Lektüre des Artikels fast zugestimmt, bin aber mittlerweile anderer Ansicht. Wenn man zwei Teilchen betrachtet, die sich genau mit der richtigen Geschwindigkeit bewegen, so dass sie die Raumexpansion kompensieren, sollten sie ewig so stehen bleiben können (abgesehen von der Schwerkraft, die sie dann in Bewegung aufeinander zu versetzen würde).

    Was ist aber, wenn sie diese Geschwindigkeit nicht genau haben? Dann änderte sich ihr Abstand und somit auch der Raumzuwachs auf der Strecke zwischen ihnen, denn ein Streckenelement wächst ja mit einer konstanten Rate, es kommt also proportional zur Länge absolut mehr hinzu (um Björns 7% pro Milliarde Jahre aufzugreifen, 7% von 1m sind 7 cm, aber 7% von 1 km sind 70 m pro Milliarde Jahre). Mithin wirkt scheinbar eine Beschleunigung, wenn sich die Teilchen immer weiter voneinander entfernen.

    Wenn man also nicht genau die Hubble-Expansionsrate trifft, dann wird sich eine positive (wenn die Teilchen sich langsamer als die Hubble-Expansionsrate aufeinander zubewegen) oder negative (wenn sie es schneller tun) Beschleunigung ergeben. Und das gilt auch bei konstanter Expansionsrate. Man muss nur lange genug zuschauen, was zwei Teilchen tun (und die Schwerkraft herausrechnen).

    Und wenn man mehr als zwei Teilchen hat, dürfte einem auch nicht gelingen, einen Punkt zu finden, relativ zu dem sie sich alle mit der richtigen Geschwindgikeit bewegen könnten, um die Hubble-Expansion zu kompensieren, denn sonst gäbe es einen ausgezeichneten Massenmittelpunkt des Universums, von dem alles wegströmt. Tatsächlich sieht die Expansion aber von jedem Punkt gleich aus. Man kann sich nicht bzgl. jedes Punktes mit der richtigen Geschwindigkeit bewegen.

    Deswegen müsste man in einem Universum nur aus Teilchen und ohne Hintergrundstrahlung mit der Zeit schon erkennen können, ob es expandiert. Zumal die Teilchen am Horizont der Reihe nach ausgeknipst würden, weil ihr Licht irgendwann mit der Expansion nicht mehr mithalten könnte und sie hinter den Horizont verschwinden würden.

    Da es also eine Beschleunigung gibt, gibt’s auch eine Kraft, und die Frage bleibt, warum diese sich nicht etwa auf die Umlaufbahnen der Planeten auswirkt (aber Niels sagte ja oben, dass sie es um einen sehr kleinen Betrag tut).

    @Niels
    Würdest Du Dich diesem Argument anschließen?

  48. #48 Alderamin
    23. März 2014

    @Niels

    Würdest Du Dich diesem Argument anschließen?

    Nein, würdest Du nicht, Du hast ja oben hergeleitet, das nur bei einer Änderung der Expansionsrate eine Beschleunigung vorläge.

    Das verstehe ich jedoch nicht: wenn sich zwei Galaxien im Abstand von 1 Mpc mit 74 km/s von einander entfernen und irgendwann mal 2 Mpc entfernt sind, dann entfernen sie sich doch dann mit 148 km/s voneinander, d.h. in der Zwischenzeit haben sie ihre (scheinbare) Entfernungsgeschwindigkeit trotz konstanter Expansionsrate verdoppelt. Hmm.

  49. #49 MartinB
    23. März 2014

    @Alderamin
    “Was ist aber, wenn sie diese Geschwindigkeit nicht genau haben? ”
    Dan geht’s nicht. Aber mir ging es hier ja genau darum, dass bei einem gebundenen System die wirkende Kraft – egal wie klein – für stabile Verhältnisse sorgt, solange die Ausdehnung konstant ist.

    “Und wenn man mehr als zwei Teilchen hat, dürfte einem auch nicht gelingen, einen Punkt zu finden, relativ zu dem sie sich alle mit der richtigen Geschwindgikeit bewegen könnten, um die Hubble-Expansion zu kompensieren”

    Doch das geht (das war ja auch Niels Quadrat-Idee). Mankann es zum Beispiel durch ein Stück Metall veranschaulichen: Wenn ich das aufheize, entfernen sich alle Punkte mit einer rate, die genau proportional zum Abstand ist (thermischer Ausddehungskoeffizient). Dabei ist es unerheblich, welches Atom ich als koordinatenursprung wähle.

    Natürlich ist (und da hast du mit deinem puntk 1 völlig recht) dazu eine exakte Feinabstimmung notwendig – und wenn sich die Expansionrate ändert, geht es gar nicht mehr (außer man passt die Geschwindigkeiten dazu an).

  50. #50 Alderamin
    23. März 2014

    @MartinB

    Ok, die Idee mit dem Quadrat habe ich verstanden (sorry, die Posts oben sind alle so lang, ich war etwas ungeduldig darin, meine Anmerkung loszuwerden, und habe sie nur quergelesen, ohne jeden Gedanken bis zum Ende mit zu verfolgen, was bei Niels’ Posts eine gewisse Konzentation erfordert).

  51. #51 roel
    *****
    23. März 2014

    @MartinB Ebenfalls erstmal (wenn auch nach 50 Kommentaren) vielen Dank für den tollen Artikel. Das hilft mir deutlich weiter. Wenn ich diesen Artikel und die anschließende Diskussion lese, ist dieses für mich vormals schwierige Thema so einfach zu verstehen, einfach super.

  52. #52 Niels
    23. März 2014

    @MartinB @Alderamin
    Habt ihr meinen letzten Beitrag #45 gesehen?

    Da geht es ausführlich darum, ob meiner Meinung nach

    “Und wenn man mehr als zwei Teilchen hat, dürfte einem auch nicht gelingen, einen Punkt zu finden, relativ zu dem sie sich alle mit der richtigen Geschwindgikeit bewegen könnten, um die Hubble-Expansion zu kompensieren”

    Doch das geht

    tatsächlich gilt.
    Zusammengefasst: Mir ist nicht wirklich klar, ob das geht.

    @Herr Senf

    Wir haben bzw. definieren eine kosmische Zeit t=13,8 Mrd J und einen Skalenfaktor a(t). Diese Zeit wird mit einer lokalen Atomuhr gemessen, die Frage der Zeitdilatation stellt sich nicht.

    Die kosmische Zeit ist aber per Definition die Zeit, die ein (schon seit dem Urknall) mitbewegter Beobachter auf seiner (beim Urknall gestarteten) Uhr abließt.
    Alle Beobachter, die anderen Weltlinien gefolgt sind, messen ein jüngeres Universumsalter als t=13,8 Mrd Jahre.
    Es gibt also durchaus noch Zeitdilatation.

    Die Eigengeschwindigkeit ist aber gering, so daß jeder Beobachter eine relative Ruhe zu seinem Raum hinkriegt.

    Damit ist gemeint, dass aufgrund der geringen Eigengeschwindigkeiten praktisch jeder auf einem Stück Materie sitzende Beobachter in guter Näherung bezüglich des jeweiligen lokalen mitbewegten Bezugssystemes ruht?

  53. #53 Herr Senf
    23. März 2014

    @MartinB #46
    “Was ist “mein raum” und wie unterscheidet er sich vom Raum von jemandem, der mit 100m/s an mir vorbeifliegt?”
    Die Diskussion ist spannend, aber ich “begreif” das konstruierte Problem nicht.
    Das kosmologische Prinzip sagt als Postulat, daß alle dasselbe Geschehen “sehen”, Beobachtungstatsache ist die Expansion. Das Standardmodell macht es ganz einfach, es nimmt strömende Testteilchen, wobei jedes im Gravitationsfeld der anderen frei fällt. Jedes Teilchen ist also in seinem Anschauungsraum in Ruhe. Mache ich eine Momentaufnahme des gesamten Systems (z.B. ein paar Jahre lang als gute Näherung alle Supernovae beobachten) habe ich einen momentanen Ruheraum für alle oder einen Gleichzeitigkeitsraum für den Beobachter zugelassen. Das muß für alle so “mitschwimmenden” Teilchen also gleich sein, ist damit sogar absolut.
    Alle Beobachter sehen in diesem Falle nur Isotropie, tanzt ein Testteilchen mit z.B. 100 m/s aus der Reihe, wird dieses einen Dipol sehen. Sein Anschauungsraum ist also im momentan definierten Ruhesystem nicht in Ruhe. Bei der Messung am CMB z=1100 reicht die Meßgenauigkeit und man kann diese “Eigengeschwindigkeit” herausrechnen, mit den Supernovae bis z<10 fällt es nicht weiter auf. Mit dem CMB kann ich mangels Aufhängepunkten kein Koordinatensystem festmachen, nur gucken ob die Null steht, an den Supernovae kann ich dreidimensional es dann ausrichten.
    Das Modell benutz deshalb eine kosmische Zeit und darin Gleichzeitigkeitsräume, weil wir über kosmologische Entfernungen keine Uhren synchronisieren können, das geht nur lokal in der SRT.

  54. #54 Niels
    23. März 2014

    @Herr Senf
    Bei #53 kann ich allem zustimmen.
    Nur

    Mache ich eine Momentaufnahme des gesamten Systems (z.B. ein paar Jahre lang als gute Näherung alle Supernovae beobachten) habe ich einen momentanen Ruheraum für alle

    verstehe ich nicht ganz.
    Meinst du damit, dass sich ein Koordinatensystem finden lässt, in dem alle Beobachter ruhen?

    Die Diskussion ist spannend, aber ich “begreif” das konstruierte Problem nicht.

    Ich versuche, es anders zu erklären.
    Wir warten in unserem Universum einfach mal 1000 Milliarden Jahre ab. Die Hintergrundstrahlung ist so weit abgekühlt, dass man sie nicht mehr messen kann. Alle Galaxien, die nicht gravitativ gebunden sind, sind schon lange hinter den kosmologischen Ereignishorizont verschwunden, die können wir also auch nicht mehr wahrnehmen.
    Wir können nur noch die Lokale Gruppe beobachten.
    (Sonden sind uns ebenfalls ausgegangen.)

    Haben wir trotzdem eine (prinzipielle) Chance, die Expansion des Universums zu entdecken, auch wenn wir nur diese gravitativ gebundenen Teilchen beobachten?
    Ja, aber nur weil das Universum beschleunigt expandiert.
    Durch die beschleunigte Expansion wirken nämlich Kräfte auf die gebundenen Systeme, die wir unter Umständen messen können.
    Wäre die Expansion konstant, hätten wir keine Möglichkeit.

    (Diese Veranschaulichung ist nur ziemlich schlecht, weil diese Kraft ebenfalls nur einen furchtbar winzigen Effekt bewirkt. Wenn wir das messen könnten, könnten wir wahrscheinlich auch die rotverschobene Strahlung messen.)

    MartinB hat in seinem Artikel eine Veranschaulichung erfunden, bei der er die Bindung der Teilchen durch die Gravitation dadruch ersetzt, dass ungebundene Teilchen genau passend gewählte Geschwindigkeiten mitbekommen.

    Bei der Diskussion über diese “passend bewegte Teilchen” fahren wir auf einem Nebengleis, bei dem es nur darum, ob die spezielle Veranschaulichung Martinbs unter verschiedenen Umständen wasserdicht ist.

    Am Grundsachverhalt, dass nur durch bei einer Änderung der Expansionsrate Kräfte entstehen, ändert sich dadurch nichts. Egal wie die Diskussion ausgeht.

  55. #55 Alderamin
    23. März 2014

    @Niels

    Wo ist der Denkfehler in #48? Warum existiert bei konstanter Expansionrate keine Beschleunigung und damit keine Kraft?
    Danke im voraus.

  56. #56 Herr Senf
    23. März 2014

    @Niels #54, jein
    “Meinst du damit, dass sich ein Koordinatensystem finden lässt, in dem alle Beobachter ruhen?”
    Soll nur heißen, das der “momentane Radius” festgeschrieben wird, nicht aber die Expansionsrate.
    In der ART muß man zwischen Koordinatensystem und Bezugssystem (Anschauungsraum) unterscheiden, die Koordinaten sind nur noch Markierungen der Weltpunkte, weil kein starres Bezugssystem existiert (Kovarianz).
    Damit ruht jeder “mitschwimmende” Beobachter, der Rest ist seine Beschreibung.

  57. #57 Herr Senf
    23. März 2014

    Noch’ne Frage, was hat eine Beschleunigung der Expansion mit einer Newton’schen Kraft zu tun?

  58. #58 MartinB
    24. März 2014

    @Niels
    “Zusammengefasst: Mir ist nicht wirklich klar, ob das geht.”
    Bei konstanter Expansionsrate muss das gehen – wie gesagt, in jedem thermisch ausdehnenden Körper ist die Relativgeschwindigkeit zweier Punkte proportional zum Abstand, unabhängig vom Ursprung.

    @Herr Senf
    Was genau ist ein “mitschwimmender Beobachter”? Womit schwimmt der mit? Das ist letztlich der Kern meiner Frage.

    @Alderamin
    Da ist meiner Ansicht nach kein Denkfehler, aber das ist zum einen ein Effekt zweiter Ordnung, zum anderen tritt der Effekt ja gerade dann nicht auf, wen die teilchen aneinander gebunden sind, weil sich dann der Abstand eben nicht ändert.

  59. #59 Alex
    24. März 2014

    Wenn sich der Abstand zweier Objekte durch die Ausdehnung des Universums erhöht, müsste sich doch auch ihre potienzelle Energie (gravitativ oder elektrostatisch) zunehmen. Wo kommt diese Energie her? Stammt sie aus der Dunklen Energie?

  60. #60 MartinB
    24. März 2014

    @Alex
    Energieerhaltung im ganzen Universum ist eine problematische Angelegenheit, dazu hat Niels oben bei #21 was geschrieben und einen Spektrum-Artikel verlinkt.

  61. #61 Alderamin
    24. März 2014

    @Alex

    Lawrence Krauss argumentiert genau so in Bezug auf die Inflation (die ja die große Schwester der Dunklen Energie ist). Potenzielle Energie rechnet er als negative Energie, die zusätzlich Vakuumenergie, die mit dem neu entstandenen Raum einher geht, wieder ausgleicht. Das ist die Schule der Nullenergie-Universums-Anhänger.

    @MartinB

    Das ist wirklich eine verzwickte Sache. Ab einer gewissen Entfernung greift dann plötzlich doch die Expansion (jenseits der Galaxienhaufen, so bei 10-20 Mpc), obwohl ja immer noch eine gewisse Schwerkraft vorhanden ist. Vielleicht liegt es einfach daran, dass das Weltall auf dieser Skala schon so homogen ist, dass sich die Kräfte zwischen den Galaxien auf gegenüberliegenden Seiten eines Testkörpers so weit ausgleichen, so dass der Testkörper kräftefrei unterwegs ist. So könnte ich mir einen Reim darauf machen.

  62. #62 MartinB
    24. März 2014

    @Alderamin
    Liegt das nicht dann auch daran, dass die Expansion eben nicht zeitlich konstant ist?

  63. #63 Alderamin
    24. März 2014

    @MartinB

    Die zeitliche Variation des Hubble-Parameters macht sich auf ein paar Megaparsec noch nicht bemerkbar. Siehe etwa Fig. 2 hier, bis ca. 200 Mpc.

  64. #64 Alderamin
    24. März 2014

    @MartinB

    Den Link auf das Paper habe ich übrigens von hier wo auch die Messmethode kurz erklärt wird. Cool.

  65. #65 Herr Senf
    24. März 2014

    @MartinB #58 “mitschwimmender Beobachter”
    Das ergab sich so aus der Diskussion, der macht die Expansion “ungestört” mit. Wir sind ein gestörter Beobachter, das liegt an einer lokalen Inhomogenität der Gravitation, deshalb “sehen” wir einen Dipol im CMB.
    Die näherungsweise lokalen Inertialsysteme bewegen sich in einer auf kleinen Skalen inhomogenen Materieverteilung, die auf die Geometrie des Raumes rückwirkt. Wenn wir zwei kosmologisch entfernte IS nehmen, gibt es keine Lorentz-Trafo, die die Systeme “zusammenführen” kann. Die Relation zwischen solchen IS ist nichtlinear, das macht dann so die ART, die LT sind linear, also nicht geeignet.
    Das hat mich ja gestört bei der Frage nach der Lorentz-Invarianz.
    Genau deshalb ging es ja nicht, das Newtonsche Gravitationsgesetz innerhalb der SRT einfach zu verallgemeinern, die Gravitation wirkt auf den Raum zurück und ist unverträglich mit einem globalen IS.
    Die ART ist wegen des Korrespondenzprinzipes eine “Art nichtlinearer Kompromiß”, lokal SRT mit LT, bei geringer Gravitation Newton, aber im Großen krumm mit “anderen” Trafos, die für sinnvolle Lösungen gesucht werden müssen.

  66. #66 MartinB
    24. März 2014

    @Herr Senf
    Danke, war aber nicht genau meine Frage – die zeilt nach wie vor darauf ab ob es dazu, ein solcher Beobachter zu sein, ausreciht sich so zu bewegen wie es die Materie im Universum (die am Anfang mehr oder weniger in ruhe war) jetzt tut oder ob noch ein weiteres Kriterium erfüllt sein muss.

  67. #67 Niels
    24. März 2014

    @Alderamin
    Wenn man ein perfekt isotropes und homogenes Universum betrachtet, sind zwangsläufig alle Teilchen mitbewegte Beobachter.
    Die Materie verhält sich wie ein ideales Gas, die Teilchen wechselwirken also nur über die Gravitation.
    Alle Teilchen bewegen sich entlang bestimmter Geodäten, fallen also frei. (Das ist die Definition von Geodäten). Damit wirken auf sie auch keine Kräfte. Das ist nämlich die Definition von frei fallend.

    Dabei spielt es überhaupt keine Rolle, ob die Expansion beschleunigt ist oder nicht.
    (Klingt komisch, aber genau das kommt heraus, wenn man für ein perfekt homogenes und isotropes Universum die einscheinschen Feldgleichungen löst.)

    In einem echten Friedmann-Universum gibt es also keine Verklumpung, es gibt keine Sterne, keine Galaxien, kein gar nichts. Nur einzelne Teilchen, die sich durch die Expansion kräftefrei voneinander entfernen, obwohl sie gravitativ wechselwirken.

    Deswegen leben wir offensichtlich nicht in einem echten Friedmann-Universum, sonst würden wir nämlich gar nicht existieren.
    (Das liegt im letztlich daran, dass es Quanteneffekte, insbesondere Vakuum-Fluktuationen, gibt. Die berücksichtigt die ART nicht.)

    Das Universum ist also nur näherungsweise ein Friedmann-Universum, die Robertson–Walker-Metrik gilt nur auf großen Skalen.

    In unserem lokalen, durch die Gravitation gebundenen Klumpen liegt also keine Robertson-Walker-Raumzeit vor, sondern etwas unglaublich Kompliziertes.
    Dieser Klumpen inhomogener Materie beeinflusst schließlich die Robertson-Walker-“Hintergrundraumzeit”, und zwar nichtlinear, weil die Diff-Gleichungen der ART nichtlinear sind.

    Wählt man die Skala so groß, dass unser Klumpen gut durch ein Punktteilchen beschrieben wird, können wir wieder von einer reinen Friedmann-Raumzeit ausgehen. Deswegen können wir in der Regel eben doch die Friedmann-Gleichungen verwenden, weil bei der Kosmologie eigentlich immer mit gewaltigen Entfernungen arbeiten.

    Dieses Punktteilchen, das unseren ganzen Klumpen beschreibt, ist dann auf dieser Skala wieder einfach ein mitbewegter Beobachter, folgt also einer Geodäten und ist damit kräftefrei.

    Worauf ich mit dem ganzen Geschwafel hinaus will:

    Ab einer gewissen Entfernung greift dann plötzlich doch die Expansion (jenseits der Galaxienhaufen, so bei 10-20 Mpc), obwohl ja immer noch eine gewisse Schwerkraft vorhanden ist. Vielleicht liegt es einfach daran, dass das Weltall auf dieser Skala schon so homogen ist, dass sich die Kräfte zwischen den Galaxien auf gegenüberliegenden Seiten eines Testkörpers so weit ausgleichen, so dass der Testkörper kräftefrei unterwegs ist. So könnte ich mir einen Reim darauf machen.

    Das kann man meiner Meinung nach so sagen.
    (Das Problem ist nur, dass das “Newton-Spache” ist. In der ART geht es um Raumzeit-Krümmung, da gibt es keine Gravitations-“Kräfte”.)

    @Herr Senf
    Dann stimmt soweit ich es verstehe alles, was du gesagt hast.

    was hat eine Beschleunigung der Expansion mit einer Newton’schen Kraft zu tun

    Eigentlich natürlich gar nichts, man muss selbstverständlich allgemein relativistisch rechnen.

    Wenn man trotzdem eine newtonsche Abschätzung macht, kommt aber dieselbe Proportionalität heraus, als wenn man allgemein relativistisch die Gleichung der geodätische Abweichung löst.

    Die Spaghettisierung eines Objektes beim Fall in ein schwarzes Loch ist doch ebenfalls Resultat einer geodätische Abweichung.
    Da würde man doch auch sagen, dass Kräfte auf dieses gebundene Objekt wirken, oder? Selbst wenn das nicht unbedingt die “relativistisch saubere” Sprechweise ist.

    Die näherungsweise lokalen Inertialsysteme bewegen sich in einer auf kleinen Skalen inhomogenen Materieverteilung, die auf die Geometrie des Raumes rückwirkt.

    Und genau so eine kleine inhomogene Materieverteilung im expandieren Universum betrachtet man, zum Beispiel in dem annimmt, dass sich die dortige Raumzeit durch die sogenannte “McVittie spacetime” beschreiben lässt.
    Dann berechnet man in dieser Raumzeit zum Beispiel die “Exact condition for non-expanding circular orbits” und stellt überrascht fest (jedenfalls finde ich das überraschend), dass das etwas mit dem Ergebnis der newtonschen Abschätzung zu tun hat.

    @MartinB

    Was genau ist ein “mitschwimmender Beobachter”? Womit schwimmt der mit?

    Das ist der mitbewegte Beobachter, um den es die ganze Zeit geht. Der “schwimmt“ mit der Expansion mit.

    Danke, war aber nicht genau meine Frage – die zeilt nach wie vor darauf ab ob es dazu, ein solcher Beobachter zu sein, ausreciht sich so zu bewegen wie es die Materie im Universum (die am Anfang mehr oder weniger in ruhe war

    In Ruhe in Bezug worauf?
    Die Materie in unserem Universum ist auf großen Skalen doch schon seit dem Urknall mit der Expansion mitbewegt. Weil die Expansion im frühen Universum viel schneller ablief, “bewegte“ sich die Materie auch viel schneller auseinander. (D.h. die erste Ableitung des Skalenfaktors war viel größer.)

    Bei konstanter Expansionsrate muss das gehen – wie gesagt, in jedem thermisch ausdehnenden Körper ist die Relativgeschwindigkeit zweier Punkte proportional zum Abstand, unabhängig vom Ursprung.

    Hast du #45 gesehen?
    Da stimme ich dir ja zu. Wenn man davon ausgeht, dass es sich um (masselose) Testteilchen handelt, die man in einer Friedmann-Raumzeit bewegt, geht das.

    Wenn man aber berücksichtigt, dass diese Teilchen selbst Masse haben, also auf die Geometrie des Raumes rückwirken, bekommt man Probleme.

    Passend bewegte Testteilchen in einer Friedmann-Raumzeit können durch die Beobachtung der anderen Testteilchen nicht feststellen, ob konstante Expansion vorliegt.
    Irgendwie muss die Friedmann-Raumzeit, in der sie sich bewegen, aber erzeugt werden. Und wenn man die Teilchen beobachtet, die das tun, bemerkt man die konstante Expansion.

    Wenn man dagegen alle Materie des Universums passend bewegt, ist das Universum wahrscheinlich nicht homogen und isotrop. Dann gibt es aber auch keine Friedmann-Raumzeit mehr, also auch räumlich oder zeitlich konstante Expansion und dieses Modell wird sinnlos.

    .

    Zur Lorentzinvarianz der Hubble-Konstanten:
    Das ist keine Größe, die man lokal in einem abgeschlossenen Laborsystem messen kann. Die Einheit ist schließlich km/(s* Mpc).
    Die SRT gilt in der ART schließlich nur noch lokal, also im Prinzip jeweils nur für einen einzigen Punkt.
    Herr Senf hat also recht, die Frage nach Lorentzinvarianz ist hier eigentlich sinnlos.
    (@Herr Senf: Habe ich dich richtig verstanden?)

    Andererseits kann jeder Beobachter die gleiche Hubble-Konstante messen. Er muss nur den Dipol aus seinen Messdaten der Hintergrundstrahlung herausrechnen. Wenn verschiedene Beobachter so in ihren verschiedenen Messdaten jeweils Isotropie hergestellt haben, lesen sie daraus dieselbe Hubble-Konstante ab.
    Deswegen schrieb ich, die Hubble-Konstante wäre lorentz-invariant.
    Wie gesagt, Herr Senf hat aber recht. Ein einzelner Beobachter kann die Hubble-Konstante nicht lokal in seinem abgeschlossenen Laborsystem messen, deswegen ist die Frage nach der Lorentzinvaranz dieser Größe genau genommen unsinnig.

  68. #68 Alderamin
    24. März 2014

    @Niels

    Danke.

    Kann man also verkürzt sagen, im Sonnensystem expandiert der Raum deswegen nicht, weil die Bestandteile des Sonnensystem im wechselseitigen Potentialtrichter gefangen sind, und weil sie sich nicht voneinander weg bewegen können, expandiert folglich auch der Raum zwischen ihnen nicht?

    Wenn man es von der Seite betrachtet, dass der Raum nicht die Materie auseinandertreibt (oder dazwischen wächste), sondern die Materie den Raum wie ein Gummituch auseinander zieht (wo sie das eben tut), dann wird manches klarer.

  69. #69 Alderamin
    24. März 2014

    @myself

    (oder dazwischen wächste)

    Mit einem schönen Gruß an Giovanni Trapatoni 😉

  70. #70 Herr Senf
    24. März 2014

    Hallo @Niels #67, jetzt passt es.
    Noch zu #66 ” bewegen wie es die Materie im Universum (die am Anfang mehr oder weniger in ruhe war) jetzt tut oder ob noch ein weiteres Kriterium erfüllt sein muss”
    Die Materieverteilung ist auch jetzt noch fast in Ruhe “am jeweiligen Ort” im Raum, am Anfang gab es aufgrund der Quantenfluktuationen mit der Unschärferelation Dichteschwankungen und Geschwindigkeitsschubser. Aus den Dichteschwankungen wurden Galaxienhaufen, die Geschwindigkeitsschubser sind heute die unterschiedlichen Pertikulargeschwindigkeiten. Die sind einige 100 bis einige 1.000 km/s, also v<<c. Die Rezessionsgeschwindigkeit der Expansion ist keine relativistische Geschwindigkeit, deswegen würde ich hier aus Gründen der Unmißverständlichkeit eben nicht von Bewegung sprechen, auch wenn's auf den ersten Blick so aussieht. Man muß den ungestörten Hubble-Fluß nehmen, d.h. das einzige Kriterium ist "keine Pertikulargeschwindigkeit".

  71. #71 MartinB
    24. März 2014

    @Niels
    Danke – ich muss nochmal drüber nachdenken, irgendwas hakt bei mir am Verständnis.

  72. #72 Niels
    24. März 2014

    @Herr Senf
    Bezieht sich das “passt” nur auf den direkt an dich gerichteten Abschnitt von #67?
    Was sagst du zum Rest und vor allem zum letzten Teil von #67, der mit “Zur Lorentzinvarianz der Hubble-Konstanten:”
    beginnt?

    @Alderamin

    Kann man also verkürzt sagen, im Sonnensystem expandiert der Raum deswegen nicht, weil die Bestandteile des Sonnensystem im wechselseitigen Potentialtrichter gefangen sind, und weil sie sich nicht voneinander weg bewegen können, expandiert folglich auch der Raum zwischen ihnen nicht?

    Schon, dass ist aber wieder “newtonsche Spache.”
    Wenn du von Potentialtrichtern sprichst, gehst du von newtonschen Gravitationspotentialen bzw. newtonschen Gravitationsfeldern aus und landest dann wieder bei Gravitationskräften.
    Da kannst du letztlich auch einfach sagen:
    Das Sonnensystem expandiert nicht, weil es gravitativ gebunden ist.

    Für die ART-Sprechweise siehe zum Beispiel auch die Beschreibung in diesem Eintrag bei den Physics FAQ:
    If the universe is expanding, does that mean atoms are getting bigger? Is the Solar System expanding?

    Bei beiden Beschreibungen wird aber genau er Effekt vernachlässigt, um den es MartinB in seinem Artikel geht. 😉

  73. #73 Herr Senf
    25. März 2014

    @Niels, du fragst nach dem Rest von #67, der ist auch ok.
    Ich wollte ja auf die Problematik des Ruhebegriffs bei der Expansion hinaus, da muß man die Expansion rauslassen, also eine Momentaufnahme betrachten. Selbst dann kann ich aber entfernte Inertialsysteme nicht linear ineinander überführen.
    An anderer Stelle
    https://www.scilogs.de/relativ-einfach/kosmische-expansion-der-skalenfaktor/
    wurde das Thema auch besprochen, ich habe da mal 3 Fragen eingebracht:
    1. Die Galaxienhaufen entfernen sich voneinander, als ob dazwischen Raum entsteht.
    2. Die Galaxien in den Haufen nehmen (noch) nicht an der Raumdehnung teil
    3. Die Galaxien sind gravitativ gebunden, ist der Raum hier auch gebunden, oder bleiben die Galaxien an der “Stelle” und der “entstehende Raum darunter” dehnt sich auch weg?

    Oder wegen #68 noch anders gefragt, entsteht “zusätzlicher” Raum dazwischen, wo wenig Materie ist. Dort wo Materie geklumpt ist, wird die Raumbildung gebremst. In der ART ist es so, daß Materie den Raum krümmt und sie auch noch dieser Krümmung selbstbezüglich folgt. Dann wird praktisch die Gravitation (als Kraft) ins Spiel gebracht, die der Expansion innerhalb von Galaxien und Haufen entgegenwirkt. Das ist für mich in der Geometrodynamik nicht konsequent. Wenn der Raum also gleichmäßig entstehen würde, wären die Schwerpunkte festgenagelt, der Raum würde nach außen gedrängt. Ein spekulatives Szenario wäre, daß dort “viel weniger” Raum entsteht. Wie will man das konsistent mathematisch auf die Reihe kriegen? MP wollte am Thema dran bleiben, bin gespannt auf weitere “Deutungen”.

  74. #74 Chemiker
    26. März 2014

    Ich finde diese Diskussion hier sehr schön. Mit meinen beschränkten (=nicht vorhandenen) Kennt­nissen der ART kann ich natürlich nur die „Folklore der Folklore“ verstehen, würde aber zu­min­dest gerne dieses Level schaffen.

    Ich hatte immer den Eindruck, die Metrik der ART ist so etwas wie der Hamilton-Operator in der Schrödinger­gleichung: Da kann ich hinein­schreiben was ich will, dann fülle ich die Teilchen ein, und die müssen nach der Musik des Operators tanzen. Das macht man in der Chemie dauernd so, nach dem Motto „Alles was zu kompliziert zum Be­rech­nen ist wird durch irgend­einen Einteilchen­operator ersetzt, den wir einfach dazunehmen“.

    Das scheint in der ART aber nicht so zu sein, weil (nur?) die Masse die Metrik generiert. Und deshalb sind ver­schie­dene Gedanken­experimente (ein aus­dehnen­der Raum mit vier einander zu­winken­den Teilchen und sonst nichts) offen­bar problematisch.

    Auch wenn das keine Lösung ist, ist es doch einmal gut, das Problem ver­standen zu haben.

    BTW, vor ein paar Tagen habe ich herausgefunden, daß man die „100 Autoren gegen Einstein“ auch aus dem Netz runter­laden kann. Ich habe es gelesen und bin begeistert. Proto­typische Trolle in Hardcopy! Mit allem was dazugehört, wie multiplen Ruf­zeichen und einem in Ver­salien ge­druck­ten DIE RTH IST DAS KRANKE PROD­UKT EINER KRAN­KEN ZEIT!, dazu Rechen- und Inter­pretations­fehler, gelegentlich auch einmal ein ehrliches „Es wider­spricht Kant und kann daher nicht richtig sein“. Einige Beiträge sind nur einen Absatz lang und bestehen nur aus einem Ver­weis auf das Buch des Autors, in dem er Einstein „wissen­schaftlich“ wider­legt haben will; das erinnert sehr an die moder­nen Trolle, die überall eine Duft­marke in Form eines Links zu ihrem Forum hinterlassen.

    Wirklich ganz großes Kino! MartinB, willst Du nicht eine Rezension schreiben?

  75. #75 MartinB
    26. März 2014

    @Chemiker
    “weil (nur?) die Masse die Metrik generiert. ”
    Nein, nicht nur. Genauso wie elektromagnetische Felder sich auch selbst erzeugen können, tun das auch Gravitationsfelder – sonst gäbe es keine Wellen im Vakuum. Und darüber hinaus gibt es eine Selbstwechselwirkung des Gravtitationsfeldes.

    Die 100 Autoren gegen Einstein hatten wir hier schon:
    https://scienceblogs.de/hier-wohnen-drachen/2010/12/27/spezielle-relativitatstheorie/#comment-2795
    Siehe auch #213 und #219, da zitieren wir ein paar Highlights
    Mir das alles anzutun, um ne Rezension zu schreiben – grusel… Kann ich lieber ins Tal der 1000 Spinnen in Mexiko?

  76. #76 Chemiker
    26. März 2014

    @ MartinB
    Danke für die Klarstellung beim „nur“.

    Und danke für den Link zur alten Diskussion (die Kommentar­schlacht hatte ich mir beim Lesen sträflicher­weise nicht angetan). Die Zitate sind so ziemlich die­selben Stellen, die ich auch ausgewählt hätte. Hinweisen muß ich aber noch auf die prachtvolle Argu­mentation zum Merkur-Perihel (S. 20/21), wo Herr Le Roux den Erfolg der „RTH“ lückenlos nachweist und das selbst nicht versteht.

    Daß Oscar Kraus, einer der Helden meiner Schulzeit („Meyeriade“, würdest Du wohl auch mögen) da mitgemacht hat, finde ich traurig.

    Disclaimer: Ich sehe mir keine Horror­filme an, wahr­schein­lich ist solche Lek­türe daher die für mich geeignete Kom­pensa­tion. Nach­machen auf eigene Gefahr.

  77. #77 MartinB
    26. März 2014

    ” Ich sehe mir keine Horror­filme an”
    Ich auch nicht, das Tal der 1000 Spinnen in Mexiko ist ein Zitat aus “Bernd&Friends” (“…dann ist dein name also Quetzlotzquotquot … vom Stamm der furchtsamen Scheepumas aus dem tal der 1000 Spinnen in Mexiko?”)
    Ich glaube, das hier ist der rictige:

  78. #78 volker
    waakirchen
    9. April 2014

    Mal eine Frage zum Verständnis, oder Anschaulichkeit der Raumkümmung bei der ART:

    Wenn man Darstellungen der Raumkrümmung durch eine Schwere Masse, z.B. durch ein massereiches SL sieht, dann ist meistens ein zweidimensionaler Trog mit der Punktmasse am Boden dargestellt.. Der Raum ist aber 3+1 dimensional. Ich habe da Verständnisschwierigkeiten.
    Was ist da z.B. mit einer weiteren Masse, die sich in Richtung der”Achse” dieses Troges nähert, wie ist ihre Raum-Bahnkurve?
    Gibt es irgendwelche anschaulicheren Darstellungen oder zumindest Erklärungen, und wenn ja, wo? Danke an jeden, der sich bemüht.

  79. #79 MartinB
    9. April 2014

    @volker
    Solche Darstellungen sind immer problematisch.
    Ein bisschen was dazu findest du in der Raumkrümmungsserie (rechts bei den Artikelserien), wo ich das alternativ mit Landkarten erkläre.

  80. […] bewegt, auch er beobachtet die Expansion des Universums in seinem Bezugssystem. Da ich die Expansion des Universums vor einiger Zeit ausführlicher diskutiert habe, brauche ich hier auf die Details nicht einzugehen. […]

  81. #81 Loner
    Terra
    6. März 2015

    Hallo MartinB,

    toller Beitrag mit vielen spannenden Überlegungen.
    Trotz dem, dass Raum nur ein Konzept und ein Synomym von “nichts” ist. Zeit ist übrigens ein abstraktes Konzept. Nun überlege was der Begriff Raumzeit aussagt.

    Selbst dann, wenn Raum physikalische Eigenschaften wie ein Objekt hätte, sehe ich am Ende Deines Beitrags ein Problem.
    Du willst dabei einer Gruppe von Teilchen eine Eigengeschwindigkeit geben, die der Expansionsrate der postulierten Raumexpansion entspricht.
    Nun, dieses Expansionspostulat sagt aus, dass die Raumexpansion von jedwedem Punkt in jedwede gleichberechtigt stattfinden soll. Wie willst Du bei einer Gruppe von Teilchen die Abstandsbildung bei einer Expansion in alle Richtungen verhindern, ohne dass Du die Teilchen, wie im oberen Textteil, aneinanderbindest?
    Bei einer gleichzeitigen Expansion in jedwede Richtung gibt es keine bevorzugte Richtung, aus der Du eine Kraft wirken lassen könntest, um eine Veränderung der Teilchen zueinander zu verhindern. Sebst dann, wenn Du aus allen Richtungen gleichzeitig eine Kraft wirken lassen würdest, würdest Du damit nur einen gleichmäßigen Oberflächendruck auf die Teilchen Auswirken, doch das Raumexpansionspostulat würde dennoch wirken.

    Nette Grüße

    Loner

  82. #82 MartinB
    7. März 2015

    @Loner
    Ich glaube, ich verstehe nicht, worauf du hinaus willst.
    Wenn ich einen Kristall nehme und ihn erwärme, entfernen sich auch alle Atome darin in genau dieser Weise. Ich sehe also nicht genau, wo da das Problem liegt.

  83. #83 Karl-Heinz
    16. September 2017

    @MartinB
    Alderamin hat gemeint, ich solle doch die Diskussion auf deiner Seite weiterführen.
    Hätte ich doch seinen Rat beherzigt.
    https://scienceblogs.de/astrodicticum-simplex/2017/09/08/sternengeschichten-folge-250-die-hubble-konstante/#comment-1401760

    Ich Streite mich gerade mit DerZimmermann.

    Expansion, wo die Dunkle Energie dominiert:

    R(t)=R0 * e^(H*t)

    R'(t)=R0*H*e^(H*t)
    R”(t)=R0*H^2*e^(H*t)

    a(t) = R(t)/R0 = e^(H*t)
    a'(t) = H*e^(H*t)
    a”(t) = H^2*e^(H*t)
    a'(t)/a(t)=H(t)=H
    a”(t)/a(t) =H^2
    Es sollte gelten a”(t)/a(t) = H’+H^2
    H'(t)=0
    H’+H^2= H^2 … Ist also erfüllt.
    Die Expansionsrate ist konstant
    H(t)=H = konstant
    Beschleunigung ist ∝ H^2

    Beispiele für Skalenfaktor
    Strahlung: a(t) ∝ t^(1/2)
    Materie: a(t) ∝ t^(2/3)
    Vakuum: a(t) ∝ e^(H*t)
    ————————————

    Ich nehme mal den Skalenfaktor für Materie und versuche die Expansion zu rechnen
    Die Dichte der Materie nimmt mit ρM ∝ a^(−3)
    ab. Die dunkle Energie und die Strahlung werden nicht berücksichtig.

    a(t) ∝ t^(2/3)
    a(t) = k* t^(2/3)
    a'(t) = 2 *k * t^(-1/3) / 3
    a”(t) = – 2 * k * t^(-4/3) / 9

    a'(t)/a(t)=H(t)=(2 *k * t^(-1/3) / 3) / (k* t^(2/3))
    a'(t)/a(t) = H(t) = 2 * t ^(-1) / 3 = 2 / (3 *t)

    a”(t)/a(t) = (- 2 * k * t^(-4/3) / 9) / (k* t^(2/3))
    a”(t)/a(t) = – 2 * t^(-2) / 9 = -2 / (9 * t^2 )

    Es sollt gelten a”(t)/a(t) = H’+H^2
    H'(t)=(2 * t ^(-1) / 3)’ = – 2 * t^(-2) / 3
    H^2= 4 * t^(-2) / 9
    H’+H^2= -6 * t^(-2) / 9 + 4 * t^(-2) / 9
    H’+H^2 = -2 * t^(-2) / 9
    a”(t)/a(t) = – 2 * t^(-2) / 9
    a”(t)/a(t) = H’+H^2 ist also erfüllt

    Fazit:
    Expansionsrate: a'(t)/a(t) = H(t) = 2 / (3 *t)
    a”(t)/a(t) = -2 / (9 * t^2 )
    Beschleunigung ist ∝ -2 / (9 * t^2 )
    Also das heißt, dass die Expansionsrate mit der Zeit abnimmt und gegen 0 strömt.
    Die Beschleunigung ist negativ und geht mit der Zeit gegen 0.

  84. #84 Karl-Heinz
    16. September 2017
  85. #85 MartinB
    16. September 2017

    @Karl-Heinz
    Ich habe im Moment leider keine Zei,t um das nachzurechnen oder zu recherchieren, worum es dir gerade geht, sorry.
    Wenn es (was ich nach kurzem Überfliegen so sehe) darum geht, ob bei konstanter Ausdehnungsgeschwindigkeit eine permanente Kraft auf alles wirkt: Nein, das ist nicht der Fall, steht im Artikel hier ja auch drin.
    Aber wie gesagt, bin gerade ziemlich beschäftigt.

  86. #86 Karl-Heinz
    16. September 2017

    @MartinB
    Danke für die Info.
    Macht nix.
    Vielleicht gibt’s ja Kommentatoren, die sich mit dem Zeug auskennen.
    Deswegen ist ja so ein Blog, was wunderbares. 😉

    Nochmal Danke für die Info

  87. #87 MartinB
    16. September 2017

    @Karl-Heinz
    Ansonsten hilft auch das physicsforum weiter (auf Englisch), da muss man allerdings die Fragen relativ sauber formulieren, sonst gibt’s mecker…

  88. #88 Karl-Heinz
    16. September 2017

    @MartinB

    Wenn es (was ich nach kurzem Überfliegen so sehe) darum geht, ob bei konstanter Ausdehnungsgeschwindigkeit eine permanente Kraft auf alles wirkt.

    Ja genau.
    Mit konstanter Ausdehnungsgeschwindigkeit meinst du eh, dass die Hubble-Konstante zeitlich konstant ist, oder?

  89. #89 MartinB
    16. September 2017

    Ja.

  90. #90 Karl-Heinz
    16. September 2017

    @MartinB
    Irgendwie werde ich das Gefühl nicht los, dass deine Beweisführung in etwa so abläuft.
    Gegeben sei eine Bahnkurve eines Geschosses.
    Bei gleichen Anfangsbedingung wird die Bahnkurve immer die selbe sein, das heisst sie ändert sich zeitlich nicht. Zu jedem Punkt der Bahnkurve kann man natürlich auch eine Geschwindigkeit zuordnen. Die Geschwindigkeit im Punkt P (P ist Element der Bahnkurve) ist zeitlich konstant. Jetzt folgerst du, das die Beschleunigung dort 0 ist. Sorry zuerst kompensiert du natürlich die Geschwindigkeit auf 0 und dann folgerst du, dass die Geschwindigkeit im Punkt P zeitlich konstant ist und es daher keine Beschleunigung geben kann.
    Ich hoffe ich habe jetzt keinen Blödsinn erzählt.

  91. #91 MartinB
    16. September 2017

    @Karl-Heinz
    Ich konnte dem Beispiel nicht wirklich folgen, sorry.
    Meine “Beweisführung” ist letztlich nur der Versuch das, was man ausrechnen kann (Kraft gibt es nur bei sich ändernder Ausdehnung) anschaulich zu machen.
    Dass bei konstanter Ausdehnungsrate keine Kräfte Objekte auseinanderreißen, steht auch in den verlinkten papers (und lässt sich wirklich nicht so schwer ausrechnen…)

  92. #92 DerZimmermann
    16. September 2017

    Da muss ich mich jetzt auch nochmal einmischen 🙂

    Eine Kraft entsteht durch die Expansion des Alls also nur dann, wenn sich die Expansionsrate ändert – und tatsächlich findet man in allen papers zum Thema (zum Beispiel hier oder hier) immer nur Kräfte durch die Änderung der Expansionsrate (in den Gleichungen taucht immer die zweite Ableitung des Skalenfaktors des Universums, also die Änderung der Expansionsgeschwindigkeit, auf).

    Die Kraft hat also was mit der zweiten Ableitung des Skalenfaktors zu tun. Mein Standpunkt war, wenn diese zweite Ableitung gleich null ist (d.h. die erste Ableitung a’ ist konstant), gibt es auch keine Kraft.

    Damit besagte zweite Ableitung gleich null ist, muss die Hubblekonstante mit ~1/t abfallen, da sie sich durch H = a’/a berechnet.
    Seit der Entdeckung der dunklen Energie spricht man von beschleunigter Expansion, weil a” positiv ist. H hingegen fällt seit dem Urknall und tut es immer noch.

  93. #93 Karl-Heinz
    16. September 2017

    @MartinB
    Ich wollte mit meinem Beispiel mit der Flugbahn nur andeuten, dass du dich ganz auf einen Raumpunkt konzentrierst, anstatt das Objekt (Galaxie) zu betrachten.

    Für mich logisch kling:
    Zwischen uns (Milchstraße) und Andromedagalaxie entsteht neuer Raum. Währe die Andromedagalaxie nicht mit der Milchstraße irgendwie kräftemäßig verbunden, so würde sie mit der Raumzeit mit fließen. Für uns sieht es aus als würde die Andromedagalaxie durch eine Kraft davongetragen werde. Halte ich aber die Andromedagalaxie auf konstanten Abstand, so muss ich dafür eine Kraft aufwenden.

  94. #94 Karl-Heinz
    16. September 2017

    @MartinB

    Dass bei konstanter Ausdehnungsrate keine Kräfte Objekte auseinanderreißen, steht auch in den verlinkten papers (und lässt sich wirklich nicht so schwer ausrechnen…)

    Natürlich werden keine Objekte auseinander gerissen, denn dafür ist die Kraft viel zu klein.
    Bei zwei Sauerstoffatomen würde meiner Meinung nach folgende Kraft wirken F= m*Abstand* a”(t)/a(t) = m*Abstand* H^2
    wobei a(t) der Skalenfaktor und H die Hubble Konstante ist.

  95. #95 MartinB
    17. September 2017

    @Karl-Heinz
    “Halte ich aber die Andromedagalaxie auf konstanten Abstand, so muss ich dafür eine Kraft aufwenden.”
    Nurt, wenn sich die Relativgeschwindigkeit der Galaxien ändert. Ansonsten musst du höchstens einmal eine Kraft aufwenden, um die beiden Galaxien relativ zueinander auf Geschwindigkeit Null zu bringen, danach nicht mehr. Siehe das Beispiel mit dem Auto und der Gummimatte.

    @Zimmermann
    Sory, mit diesen ganzen Begriffen wie Hubble-Parameter und hubble-Konstante und Skalenfaktor verheddere ich mich immer.
    Richtig ist, dass es keine Kraft gibt, wenn die 2. Ableitung des Skalenfaktor verschwindet, das steht ja auch in den papers so drin.

  96. #96 DerZimmermann
    17. September 2017

    @MartinB:
    Dann habe ich das schon richtig verstanden. Die ganzen Begrifflichkeiten sind bei dem Thema einfach leicht missverständlich.

    Wenn a” = 0 ist, gibt es keine Kraft. Das ist also genau dann der Fall (ergibt sich aus H = a’/a), wenn H mit 1/t abfällt. Anschaulich: Wenn Galaxie X sich über lange Zeit immer mit der gleichen Geschwindigkeit von uns entfernt (unabhängig von ihrer aktuellen Entfernung).

    Wenn H konstant ist, gilt a = e^(H*t), damit ist a” > 0 und es gibt eine Kraft. Anschaulich: Wenn über lange Zeit Objekte, die sich gerade in einer bestimmten Entfernung X befinden, immer die gleiche Zurückweichgeschwindigkeit haben.

  97. #97 Karl-Heinz
    17. September 2017

    @MartinB
    Das mit den Berifflichkeiten ist so ein Problem.
    Unter der zeitlichen Ausdehnungsrate verstehe ich den Hubbleparameter H(t) =a'(t)/a(t).
    Herr DerZimmermann versteht unter der Ausdehnungsrate nur a'(t).
    Wer von uns beiden hat nun recht?
    Danke KH

  98. #98 Karl-Heinz
    17. September 2017

    @MartinB

    Zur Veranschaulichung meiner oben gestellten Frage.

    R(t)=R0(1+k*t) Beispiel einer Ausdehnung
    k ist eine Konstante, t … Zeit

    R'(t)=R0*k

    a(t) = R(t)/R0 (ist so definiert)

    a(t) = 1+k*t

    a'(t) = k
    versus
    a'(t)/a(t)=H(t)=k/(1+k*t)

  99. #99 DerZimmermann
    17. September 2017

    @Karl-Heinz: Ich finde die üblichen Begriffsverwendungen auch nicht wirklich gut (womit ich wohl mit dir konform gehe). Ich versuche mich nur daran zu halten, um einheitlich zu bleiben. Wie man sich “seinen” Ausdehnungsrate-Begriff vorstellt bzw. definiert, ist ja jedem letztlich selber überlassen. Normalerweise ist nun mal a’ gemeint.

    Mit der Aussage “bei a” = 0 gibt es keine Kraft” sollte dazu eigentlich alles gesagt sein. Wie man diesen Zustand dann benennt, kann sich jeder selber aussuchen…

  100. #100 Karl-Heinz
    17. September 2017

    @DerZimmermann

    Wie man sich “seinen” Ausdehnungsrate-Begriff vorstellt bzw. definiert, ist ja jedem letztlich selber überlassen. Normalerweise ist nun mal a’ gemeint.

    Nein eben nicht. Fällt dir nicht auf, dass a'(t) in meinem in #98 angeführten Beispiel konstant ist, während a'(t)/a(t) zeitabhängig ist. Natürlich folgt für a”(t) =0 auch, dass a”(t)/a(t) =0.
    Ich bin deswegen so pingelig, weil meiner Meinung nach die Kraft proportional a”(t)/a(t) ist. Wie du meinem Beispiel entnehmen konntest ist nun a'(t) und a'(t)/a(t) nicht das selbe. Das gilt auch zwischen a”(t) und a”(t)/a(t).

  101. #101 DerZimmermann
    17. September 2017

    Na dann sind wir uns doch einig, oder?

    Mein Punkt war, wenn a” > 0 ist, gibt es eine “auseinandertreibende” Kraft, bei a” = 0 nicht. Und wenn H konstant ist, dann ist a” > 0, ergo gibt es eine Kraft; damit es keine gibt, muss H mit 1/t fallen.

    Ich wollte vorhin eben nur sagen: “Ausdehnungsrate” ist ein deutsches Wort, bei dem in diesem Kontext nicht sofort klar ist, was man damit meint. Daher sollte man sauber definieren, was damit gemeint ist (a’ oder H), sonst gibt es schnell Missverständnisse. Es hat sich halt nun mal so eingebürgert, dass damit a’ gemeint ist (das ist auch einfach ne Definition). Nirgends wird behauptet, unser Universum expandiere gebremst – obwohl H erwiesenermaßen fällt. Dagegen ist a’ tatsächlich bis vor einigen Mrd. Jahren gefallen und steigt seither wieder an.

  102. #102 Karl-Heinz
    17. September 2017

    @DerZimmermann
    Die große Frage, die im Raum steht ist, welche Definition für die Ausdehnungsrate sinvoller sein wird.
    a'(t)
    oder a'(t)/a(t) … mein Favorit
    Ich werde es dir Beweisen, wirst sehen. Smily

  103. #103 Karl-Heinz
    17. September 2017

    @DerZimmermann

    Beweis, dass Ausdehnungsrate gleich a'(t)/a(t) ist.
    Der Beweis ist ja gott sei Dank wirklich einfach.
    Als Analogie nehme ich die Wachstumsrate

    diskrete Wachstumsrate g=(A(t)-A(t0)) / A(t0)

    stetige Wachstumsrate w=A'(t)/A(t)

    fällt dir was auf???

    https://de.m.wikipedia.org/wiki/Wachstumsrate

  104. #104 Karl-Heinz
    17. September 2017

    @DerZimmermann

    Du hast damit recht gehabt, dass man diese Beziehung mit einfachen Mitteln herleiten kann.
    a”(t)/a(t) = H'(t) + H(t)^2
    ——————————————————-
    H'(t)=(a'(t)*a(t)^(-1))’ =
    a”(t)*a(t)^(-1) – a'(t) * a'(t) * a(t)^(-2)=>

    a”(t)/a(t) = H'(t) + H(t)^2
    ———————————————–
    Unter der Annahme, dass die Kraft bzw. Beschleunigung proportional a”(t)/a(t) ist folgt auch, dass die Kraft oder Beschleunigung
    proportional H'(t) + H(t)^2 ist.
    Wenn H(t) konstant ist –> H'(t)=0
    Wenn H(t) zeitlich grösser wird –> H'(t) > 0:
    Wenn H(t) zeitlich kleiner wird –> H'(t) kleiner 0

    Mir ist durchaus bewusst, dass all meine Folgerungen falsch sind, wenn meine Annahmen nicht richtig sind.
    Ich gehe aber davon aus, dass sie stimmen.
    a'(t)/a(t) und a”(t)/a(t) also Geschwindigkei und Beschleunigung können durchaus unterschiedliche Vorzeichen haben. Wenn das Universum beschleunigt kontrahiert , dann ist a'(t)/a(t) negativ aber a”(t)/a(t) ist positiv.
    In diesem Fall bremst die Kraft.
    MartinB behauptet, dass die Kraft 0 ist, wenn H(t) konstant ist und dass es nur dann eine Kraft gibt wenn H(t) sich zeitlich ändert.
    Ich persönlich sehe folgende Beziehung
    Beschleunigung bzw. Kraft ist proportional H'(t) + H(t)^2

  105. #105 Karl-Heinz
    17. September 2017

    @MartinB

    “Halte ich aber die Andromedagalaxie auf konstanten Abstand, so muss ich dafür eine Kraft aufwenden.”
    Nurt, wenn sich die Relativgeschwindigkeit der Galaxien ändert. Ansonsten musst du höchstens einmal eine Kraft aufwenden, um die beiden Galaxien relativ zueinander auf Geschwindigkeit Null zu bringen, danach nicht mehr. Siehe das Beispiel mit dem Auto und der Gummimatte.

    Ich werde es irgendwie schon noch schaffen mit einfachen Überlegungen zu beweisen, dass dies so nicht stimmt. Ich hoffe du hast nichts dagegen.

  106. #106 Karl-Heinz
    17. September 2017

    @MartinB

    Ziehst du die Gummimatte mit konstanter Geschwindigkeit auseinander?
    Dann wäre die Beschleunigung oder Kraft tatsächlich null, da H'(t)+H^2 gleich 0 ist.

  107. #107 DerZimmermann
    17. September 2017

    Ziehst du die Gummimatte mit konstanter Geschwindigkeit auseinander?

    Da hätten wir wieder das Thema mit den Begriffen. Was heißt “mit konstanter Geschwindigkeit auseinanderziehen”?

    Wenn am Ende des Gummibands eine Kurbel ist, an der jemand immer gleich schnell das Band aufwickelt und damit dehnt, dann ist H konstant und a exponentiell. Wenn hingegen jemand das Ende des Bandes in der Hand hat und mit gleichbleibender Geschwindigkeit losmarschiert, dann ist a’ konstant und H ~ 1/t.

    Ich hätte auch was zu #103 geschrieben, aber das kommt offenbar irgendwie nicht durch… mal schauen 🙂

  108. #108 DerZimmermann
    17. September 2017

    Ich persönlich sehe folgende Beziehung
    Beschleunigung bzw. Kraft ist proportional H'(t) + H(t)^2

    Das habe ich auch nie bestritten 😉
    (Genau genommen beschreibt dies die Kraft in Entfernung 1 ).
    Besonders anschaulich finde ich diese Darstellung allerdings nicht. Martin hat ja oben schon geschrieben, dass die Frage “ob Kraft oder nicht” einfach an a” hängt. Insofern sehe ich hier auch keinen Widerspruch 🙂

  109. #109 Karl-Heinz
    17. September 2017

    @DerZimmermann

    Wenn hingegen jemand das Ende des Bandes in der Hand hat und mit gleichbleibender Geschwindigkeit losmarschiert, dann ist a’ konstant und H ~ 1/t.

    Stimmt.
    Du wirst immer besser.

    a(t) = R(t)/R0=1+k*t
    a'(t) = k
    a”(t) = 0

    a'(t)/a(t)=H(t)=k/(1+k*t)
    a”(t)/a(t) =0

  110. #110 Karl-Heinz
    17. September 2017

    Besonders anschaulich finde ich diese Darstellung allerdings nicht. Martin hat ja oben schon geschrieben, dass die Frage “ob Kraft oder nicht” einfach an a” hängt. Insofern sehe ich hier auch keinen Widerspruch

    Überleg mal welche Aussagekraft die Beziehung Kraft ist proportional zu H'(t)+H(t)^2 besitzt. Zum Beispiel wenn H(t) = konstant und ungleich 0 ist. In diesem Fall existiert immer eine Kraft und sie ist immer positiv.

  111. #111 DerZimmermann
    17. September 2017

    Na das habe ich ja die ganze Zeit schon geschrieben…

    Bei konstantem H gibt es eine auseinanderschiebende Kraft; das ist ein Spezialfall von a” größer 0 (darum nennt man das beschleunigt). Bei a” gleich 0 und damit H ~ 1/t liegt genau die Wasserscheide zwischen zusammenziehender und auseinanderschiebender Kraft.
    Zur Vorstellung vom neu entstehenden Raum passt das eben gar nicht – danach müssten die Testkörper ja bei H größer 0 bzw. a’ größer 0 grundsätzlich auseinanderdriften.

  112. #112 DerZimmermann
    17. September 2017

    Autsch, halt, sorry – Kommando zurück: Natürlich müsste es heißen, dass der Testkörper bei konstantem H an seiner Stelle bleiben müsste (da konstante Geschwindigkeit relativ zum Hubble-Flow) und erst bei steigendem H weggetrieben werden sollte, wenn man die Vorstellung vom neu entstehenden Raum zugrundelegt.

    Wehe man tippt mal kurz unüberlegt was ein…

  113. #113 Karl-Heinz
    18. September 2017

    @DerZimmerman

    Bei a” gleich 0 und damit H ~ 1/t liegt genau die Wasserscheide zwischen zusammenziehender und auseinanderschiebender Kraft.

    Die Kraft ist für alle t genau dann 0, wenn gilt H'(t)+H(t)^2 =0.
    Die Lösung dieser Differentialgleichung ergibt
    H(t)= 1/(k+t) .

    a) H(t) =0 … triviale Lösung
    b) H(t) = 1/(k + t)

    Jetzt kann man sich überlegen, welche Vorzeichen H(t) und a”(t)/a(t) haben kann und was sie bedeuten.

    Kraft … Geschwindigkeit
    —————————————-
    0 … 0
    + … +
    + … –
    – … +
    – … –

  114. #114 Karl-Heinz
    18. September 2017

    @DerZimmermann

    Ok von mir aus …
    da a(t) immer >0 ist, ist eine Betrachtung a'(t) und a”(t) bezüglich > , gleich und kleiner 0 zulässig.

  115. #115 DerZimmermann
    18. September 2017

    Ich glaube immer noch, dass du dir das Leben da unnötig kompliziert machst… Die Kraft in Entfernung a ist proportional zu a”. Wenn ich das weiß, brauche ich keine Differentialgleichungen mehr lösen und sonst auch nix…

    Ich glaube, ich verstehe schon so ungefähr, wie du auf diesen Punkt kommst, weil ich früher auch immer gedacht habe, eigentlich müsste doch H als Ausdehnungsrate das Maß der Dinge sein – wenn man das Bild vom neu entstehenden Raum im Kopf hat. Aber ich verstehe inzwischen, warum man das so gewählt hat.
    Unbeeinflusste Expansion (leeres Universum): sinnvolle Bezeichnung: gleichbleibende Expansion. a’ ist dabei konstant.
    Universum (nur) mit Masse, d.h. zusammenziehender Einfluss: sinnvolle Bezeichnung: Gebremste Expansion. a’ sinkt in diesem Fall.
    Universum (nur) mit Dunkler Energie, d.h. auseinandertreibender Einfluss: sinnvolle Bezeichnung: Beschleunigte Expansion. a’ steigt in diesem Fall.
    Gleiches gilt für die Kräfte auf den Testkörper in Martins Experiment. a’ konstant: keine Kraft; a’ sinkend: zusammenziehende Kraft; a’ steigend: auseinandertreibende Kraft.

  116. #116 DerZimmermann
    18. September 2017

    Sorry, ich kann dir grad nicht antworten…. der nimmt meine Kommentare nicht 🙁

  117. #117 MartinB
    18. September 2017

    @Zimmermann,
    sorry,a us irgendeinem Grund mochte der Spamfilter deinen Kommentar nicht.

  118. #118 Karl-Heinz
    18. September 2017

    @DerZimmermann

    Ich glaube immer noch, dass du dir das Leben da unnötig kompliziert machst… Die Kraft in Entfernung a ist proportional zu a”. Wenn ich das weiß, brauche ich keine Differentialgleichungen mehr lösen und sonst auch nix…

    Wieso soll eine Differentialgleichungen kompliziert sein? Ich habe sie ja eh online gelöst.

    Ich glaube, ich verstehe schon so ungefähr, wie du auf diesen Punkt kommst, weil ich früher auch immer gedacht habe, eigentlich müsste doch H als Ausdehnungsrate das Maß der Dinge sein

    H(t) =a'(t)/a(t) ist noch immer das Maß der Dinge. Da a(t) ja immer > 0 ist kann man von mir aus auch auch folgern, wenn
    a’ > 0 dann gilt dies und jenes.
    Man darf aber nie vergessen, dass a’ eine absolute zeitliche Änderung zu einem bestimmten Zeitpunkt ist. Die relative Änderungsrate ergibt sich aber nur durch
    a'(t)/a(t).

  119. #119 Karl-Heinz
    18. September 2017

    Niels hat schon im Eintrag #3 auf eine Beziehung F proportional R(t) *[a‘‘(t)/a(t)]hingewiesen.
    Eigentlich schade, dass man anhand dieser Beziehung nicht schon nachgeprüft hat, ob das Beispiel Auto und Gummimatte in sich konsistent ist.

    https://scienceblogs.de/hier-wohnen-drachen/2014/03/21/wenn-der-raum-sich-ausdehnt/#comment-18504

    Wie schon bereits darauf hingewiesen, ist die Kraft tatsächlich 0, wenn die Gummimatte mit konstanter Geschwindigkeit an einem Ende auseinander gezogen wird.
    Sie ist aber nicht null, wenn sie mit konstanter Ausdehnungsrate sprich mit einer Rolle an einem Ende und konstanter Drehzahl auseinandergezogen wird.

  120. #120 Bernd
    18. September 2017

    Hallo.
    User Niels hat vor einiger Zeit mal Bilder für den Hubbleparameter und den Skalenfaktor und dessen Ableitungen erstellt.
    vielleicht hilft das weiter?

    https://scienceblogs.de/hier-wohnen-drachen/2010/09/19/wie-gross-ist-das-beobachtbare-universum/#comment-22557

    Weitere Bilder in #172 und #175.

  121. #121 Karl-Heinz
    18. September 2017

    @Bernd
    Danke Bernd
    Ich werde es mir gleich ansehen.

    WICHTIG!!!
    Nachtrag: wenn a'(t) zeitlich konstant ist, heißt das nicht unbedingt, dass H(t) =a'(t)/a(t) zeitlich konstant ist.

    Beispiel:
    a(t) =1+k*t
    a'(t) = k (zeitlich konstant !!! )
    a”(t) = 0

    a'(t)/a(t)=H(t)=k/(1+k*t) (zeitlich nicht konstant !!!)
    a”(t)/a(t) =0

  122. #122 Karl-Heinz
    18. September 2017

    @Bernd
    Nochmals danke für den Link.
    Ist ja eine wahre Fundgrube mit Substanz.
    Und das Plotten auf Wolfram|Alpha ist auch eine coole Sache.

  123. #123 DerZimmermann
    18. September 2017

    Ja, über den Beitrag damals mit dem beobachtbaren Universum bin ich auf die Sache gestoßen :-))

    WICHTIG!!!
    Nachtrag: wenn a'(t) zeitlich konstant ist, heißt das nicht unbedingt, dass H(t) =a'(t)/a(t) zeitlich konstant ist.

    ???
    Das schreibe ich doch schon die ganze Diskussion hier…

    H(t) =a'(t)/a(t) ist noch immer das Maß der Dinge. Da a(t) ja immer > 0 ist kann man von mir aus auch auch folgern, wenn
    a’ > 0 dann gilt dies und jenes.
    Man darf aber nie vergessen, dass a’ eine absolute zeitliche Änderung zu einem bestimmten Zeitpunkt ist. Die relative Änderungsrate ergibt sich aber nur durch
    a'(t)/a(t).

    Einmal probier ichs noch…. dann geb ich auf ^^
    Also: Deine grundlegende Annahme ist ja, dass es sich bei einer Raumexpansion in einem Universum grundsätzlich um eine Art exponentielles Wachstum handelt. Dann gilt natürlich a = e^(Wachstumsrate * t).

    Paradebeispiel für so ein Modell ist ein Bankkonto: Ich lege 100€ an und bekomme jedes Jahr 1% Zinsen, also erhöht sich mein Kontostand pro Jahr um den Faktor 1,01; die Wachstumsrate ist also 1,01. Es kann natürlich sein, dass meine Bank 2018 beschließt, die Zinsen auf 0,5% zu senken, dann bekomme ich nächstes Jahr nicht mehr, sondern weniger Zinsen als dieses Jahr. Wenn ich plötzlich 50€ abhebe, stimmt das ganze natürlich auch nicht mehr. Wenn aber nichts dergleichen passiert, dann steigt mein Kontostand exponentiell an mit einer festen Wachstumsrate.

    Was gibt es noch? Irgendwelche Bakterienkulturen, die sich alle X Tage verdoppeln, wachsen auch exponentiell, und zwar immer mit der gleichen Wachstumsrate, weil sich ein Bakterium eben alle X Tage teilt. Klar, wenn ich jetzt die Temperatur um 20 Grad senke, oder wenn die Nahrung knapp wird, oder etwas toxisches in meine Nährlösung kommt, dann verändert sich die Rate natürlich, und die Bakterien vermehren sich mit einer geringeren Rate. Aber wenn die Umgebungsbedingungen immer genau gleich bleiben, wächst meine Bakterienkultur mit einer festen Wachstumsrate.

    Es gibt aber auch Fälle, wo dieses Modell unsinnig ist. Beispiel: Ich gehe in die Arbeit und bekomme jeden Monat 2000€ aufs Konto. Angenommen, ich hebe nichts ab und bekomme auch keine Zinsen, dann erhöht sich mein Kontostand jedes Monat um 2000€.
    Jetzt kann ich natürlich ein wenig herumrechnen und finde heraus, wenn ich die Wachstumsrate einfach jedes Monat um den richtigen Wert senke, kann ich mir auch wieder etwas zusammenbasteln, das die Formel Kontostand = e^(Wachstumsrate*t) erfüllt – einfach weil ich die Wachstumsrate jedes Monat soweit nach unten korrigiere, dass immer der gleiche Absolutwert herauskommt.
    Es kann natürlich sein, dass mein Arbeitgeber so zufrieden mit mir ist, dass er mein Gehalt jeden Monat um so viel anhebt, dass ich die “Wachstumsrate” gar nicht mehr ändern muss. Vielleicht gewinne ich ja auch im Lotto. Oder meine Bank verlangt plötzlich Minuszinsen – dann sieht die Sache auch gleich wieder ganz anders aus.

    Handelt es sich also bei meinem Gehaltskontostand auch um ein exponentielles Wachstum, weil ich mir ja die Formel so hindrehen kann? Ich hoffe, jedem ist klar, dass das nicht der Fall ist. Es handelt sich hier im Grunde seines Wesens um ein lineares Wachstum – auch wenn ich mir – z.B. mit dem zufriedenen Chef von oben – etwas exponentielles daraus machen kann – das halte ich aber für sehr unwahrscheinlich (außer der Chef hat im Lotto gewonnen…?). Dieses Modell passt hier also nicht.

    Jetzt werfe ich mal einen Blick auf ein Universum.

    Ein leeres Universum, ohne Einflüsse auf seine Expansion, expandiert linear, d.h., der Skalenfaktor steigt linear an – genau wie mein Kontostand.

    Sobald das Universum nicht leer ist, wird die Expansion beeinflusst. Materie z.B. bremst die Expansion durch die Gravitation, sodass der Skalenfaktor langsamer steigt als linear.
    Dunkle Energie beschleunigt die Expansion, sodass das Universum schneller wächst als linear. Wenn ich annehme, dass die Dunkle Energie eine Zustandsgleichung von -1 hat – d.h. die absolute Menge an DE, die zwischen zwei mitbewegten Objekten liegt, wächst ständig an, da die Dichte gleich bleibt – , dann wächst auch die Beschleunigung ständig an, und zwar genau so, dass wir bei einer exponentiellen Expansion landen. Dies hat aber mit dem Expansionsmechanismus oder dem Raum selbst nichts zu tun, sondern ist allein auf den “schiebenden” Einfluss der Dunklen Energie zurückzuführen.
    Der Raum an und für sich, ohne Einflüsse, wächst *nicht* exponentiell.

  124. #124 MartinB
    18. September 2017

    @DerZimmermann
    Ein komplett leeres Universum ohne Materie (mal davon abgesehen, dass man die Expansion dann eh nicht beobachten kann) könnte aber auch statisch sein, oder?
    Es gilt doch. dass die zweite Ableitung des Volumens eines testvolumens proprotional zur eingeschlossenen Masse ist; das meinst du ja vermutlich auch mit linearer Expansion. Oder verstehe ich da was falsch?

  125. #125 Karl-Heinz
    18. September 2017

    @DerZimmermann

    Sorry, ich hatte nicht gewusst, dass dein Universum ohne alles, also auch ohne mich, dasjenige ist welches du favorisierst. Also dein Universum hat dann die Form
    a(t) = 1+k*t

    Wenn dein Universum auch noch statisch sein soll, dann ist k gleich 0 und hat die Form
    a(t) = 1
    a'(t) = 0
    a”(t) = 0
    a'(t)/a(t)=H(t)=0
    a”(t)/a(t) =0

  126. #126 DerZimmermann
    18. September 2017

    @MartinB: Ja, genau. Es hat ein festes a’ (was entsprechend auch 0 sein kann).
    Bei der Sache mit dem Testvolumen komme ich jetzt nicht wirklich mit.

    @Karl-Heinz: Ich favorisiere nichts. Ich versuche nur die Hintergründe des Expansionsmechanismus klar zu machen. Was passiert ohne Einflüsse, was bei bremsenden, was bei antreibenden Einflüssen.

  127. #127 Karl-Heinz
    18. September 2017

    @MartinB

    Mit lineare Expansion meint er sicher die Gummimatte, die er mit konstanter Geschwindigkeit auseinanderzieht. Irgendwann wird es ihm doch zu anstrengend und er muss innehalten und dann wird es so richtig statisch.

    @DerZimmermann
    Nicht persönlich nehmen. Ist nur ein Scherz. 😉

  128. #128 MartinB
    18. September 2017

    @Der Zimmermann
    Guckst du hier (Gl. 2)
    https://math.ucr.edu/home/baez/einstein/node3.html
    Generell eine der besten Internetseiten zur ART

  129. #129 Karl-Heinz
    18. September 2017

    @MartinB

    Die Gleichung sieht meiner Meinung nach fast so aus, wie die zweite Friedmann-Gleichung, wo Dichte und Druck vorkommt. Ich denke es ist dasselbe, nur in einer anderen Darstellung.

  130. #130 Karl-Heinz
    18. September 2017
  131. #131 Karl-Heinz
    18. September 2017

    upss …
    Eigentlich wollte ich das verlinken.
    Kosmologie der Friedmann-Gleichungen

  132. #132 MartinB
    19. September 2017

    @Karl-Heinz
    Die Gleichugn von Baez ist lokal, keine Gleichung für’s Universum, und gilt immer. In einem perfekt homogenen Universum gilt aber natürlich, dass das, was lokal passiert, auch global passiert.

  133. #133 DerZimmermann
    19. September 2017

    Die Gleichung sieht meiner Meinung nach fast so aus, wie die zweite Friedmann-Gleichung, wo Dichte und Druck vorkommt. Ich denke es ist dasselbe, nur in einer anderen Darstellung.

    Denke ich auch…. auch wenn ich grad nicht draufkomme^^
    Wenn man die Sache mit dem Druck mal außen vor lässt, entspricht die zweite Friedmanngleichung sowieso 1:1 dem Gravitationsgesetz nach Newton.

  134. #134 Karl-Heinz
    19. September 2017

    @MartinB

    Die Gleichung von Baez ist lokal, keine Gleichung für’s Universum, und gilt immer. In einem perfekt homogenen Universum gilt aber natürlich, dass das, was lokal passiert, auch global passiert.

    So ein Kugel mit den vielen Testpartikeln, wo die Gleichung voraussagt, dass die Testkugel zu schrumpfen beginnt, finde ich ausgesprochen cool. Aber was ist, wenn sie mich entdecken (gravitativ), läuft dann dieser Prozess genau so ab?
    Wenn ich schon ein rotes Tuch für die Gleichung bin, wie kannst du behaupten, sie gilt immer und überall?

  135. #135 DerZimmermann
    19. September 2017

    Nochmal kurz zurück zum ursprünglichen Experiment.

    Ich stelle mir den Raum wieder als Gummiband vor, das gedehnt wird, und meinen Testkörper als Spielzeugauto auf diesem Gummiband. Ich setze das Spielzeugauto auf das Gummiband z.B. in 1m Entfernung, halte es erst noch fest und lasse es dann los. Was passiert?

    Angenommen, H wäre konstant. Dann ist die Geschwindigkeit des Gummibandes in 1m Entfernung immer gleich, z.B. 1m/s. Ich würde das Auto beim Aufsetzen und Festhalten auf 1m/s relativ zum Gummiband beschleunigen. Da es reibungsfrei rollt, sollte es diese Geschwindigkeit für immer beibehalten und damit also immer genau auf 1m Entfernung zu mir bleiben. ==> Das tut es aber in Wirklichkeit nicht, da bei konstantem H auch a”>0 ist und damit das Auto eine Beschleunigung von mir weg erfährt.

    Angenommen, a’ wäre konstant, H damit fallend. Dann sinkt die Geschwindigkeit des Gummibandes in 1m Entfernung mit 1/t. Da da Auto beim Aufsetzen 1m/s in Richtung “ich” bekommen hat – relativ zum Gummiband – müsste es damit eigentlich langsam auf mich zu rollen. ==> Tut es aber nicht, sondern in diesem Fall bleibt der Abstand konstant.

    Irgendwo steckt in dieser Gummiband-Expansions-Denke ein Fehler…

  136. #136 Karl-Heinz
    19. September 2017

    @DerZimmermann

    Ich glaube mit dem Druck meint man den Strahlungsdruck, der auch eine Energieform dastellt und ebenfalls was zur Gravitation beiträgt.

  137. #137 Karl-Heinz
    19. September 2017

    @DerZimmermann
    Der Fehler steckt in den Folgerungen, wenn ursprünglich a'(t) mit a'(t)/a(t) verwechselt wird.
    Schade, dass ihr mir nicht glaubt.

  138. #138 DerZimmermann
    19. September 2017

    Der Druck kann auch von der Materie kommen (da macht man sich das Leben aber meistens leicht und nimmt an, dass sie keinen hat). Das entscheidende bei der Dunklen Energie ist ja gerade, dass sie negativen Druck hat.

    Wo habe ich denn irgendwas verwechselt? Ich habe beide Fälle explizit angesprochen.
    Dass die Aussagen hinter den Doppelpfeilen stimmen, sieht man einfach daran, dass sie aus der Tatsache sgn(F) = sgn(a”) folgen. Die haben wir ja schon lang und breit diskutiert.

  139. #139 Karl-Heinz
    19. September 2017

    @

    Dass die Aussagen hinter den Doppelpfeilen stimmen, sieht man einfach daran, dass sie aus der Tatsache sgn(F) = sgn(a”) folgen. Die haben wir ja schon lang und breit diskutiert.

    Herr Oberlehrer und was ist mit der Aussage, wenn Funktion zeitlich konstant, dann folgt?

  140. #140 Karl-Heinz
    19. September 2017

    @myself

    So ein Kugel mit den vielen Testpartikeln, wo die Gleichung voraussagt, dass die Testkugel zu schrumpfen beginnt, finde ich ausgesprochen cool. Aber was ist, wenn sie mich entdecken (gravitativ), läuft dann dieser Prozess genau so ab?
    Wenn ich schon ein rotes Tuch für die Gleichung bin, wie kannst du behaupten, sie gilt immer und überall?

    Sorry, ich glaube ich war da zu voreilig. Wenn man das Testvolumen bei der Gleichung von Baez sehr sehr klein macht, dann müsste sie trotz meiner Anwesenheit wie vorhergesagt ablaufen oder?

  141. #141 Karl-Heinz
    19. September 2017

    @DerZimmermann

    Ich mache dir einen Vorschlag.
    Warum nimmst du nicht die Gleichung von Baez und leitest das Verhalten eines leeren Universum (ohne Materie, Strahlung und Dunkle Energie) her. Also für mich ist so eine Herleitung ein klaks. Hauptschulenniveau ist, so denke ich, ausreichend.

  142. #142 DerZimmermann
    20. September 2017

    Herr Oberlehrer und was ist mit der Aussage, wenn Funktion zeitlich konstant, dann folgt?

    Sorry, bei dem Satz komme ich jetzt nicht mit.

    #141 Ich brauche da gar nix herleiten. Da brauch ich die zweite Friedmanngleichung nur anschauen, damit ich das sehe – und du kannst es gerne auch z.B. hier nachlesen: http://www.ita.uni-heidelberg.de/~dullemond/lectures/cosmology_2011/Chapter_4.pdf Seite 26/27.

    Ich verstehe immer noch nicht, wo ich in #135 deiner Meinung nach etwas falsches gesagt haben soll.

  143. #143 MartinB
    20. September 2017

    @Karl-Heinz
    “Aber was ist, wenn sie mich entdecken (gravitativ), läuft dann dieser Prozess genau so ab?”
    Wenn du innerhalb der Kugel bist, dann wird deine Masse und dein Druck ja berücksichtigt; wenn du außerhalb der Kugel bist, beeinflusst du nicht das Volumen (Einstein- oder Ricci-Tensor), sondern deine Anwesenheit führt zu einer Verzerrung der Kugel (Weyl-Tensor).

  144. #144 Bernd
    20. September 2017

    @MartinB
    @Alderamin

    Ist User Niels nicht mehr aktiv? Was ist mit ihm passiert?
    Der könnte das hier sehr schnell lösen, der hat das wohl beruflich gemacht?

    @Karl-Heinz
    @DerZimmermann
    Ich versteh ehrlich gesagt nicht, was das genau das probvlem ist und worüber ihr streitet.
    Wärt ihr so nett das vielleicht mal in einem Satz zusammenzufassen?

  145. #145 Karl-Heinz
    20. September 2017

    @Martin

    Wenn du innerhalb der Kugel bist, dann wird deine Masse und dein Druck ja berücksichtigt; wenn du außerhalb der Kugel bist, beeinflusst du nicht das Volumen (Einstein- oder Ricci-Tensor), sondern deine Anwesenheit führt zu einer Verzerrung der Kugel (Weyl-Tensor).

    Wenn ich die Kugel sehr klein mache, dann ist die Verzehrung linear. Bei einer grösseren Kugel könnte die Verzerrung auch nichtlinear sein.
    Sehe ich das so richtig?

  146. #146 Karl-Heinz
    20. September 2017

    @Bernd

    Ist User Niels nicht mehr aktiv? Was ist mit ihm passiert?

    Habe ich mich auch schon gefragt.
    Ich hoffe es geht ihm gut.

  147. #147 MartinB
    20. September 2017

    @Karl-Heinz
    Was genau meinst du mit einer linearen Verzerrung?
    Wie gesagt, eine Kugel aus Test-Teilchen, die keine Materie (oder Druck) einschließt, verzerrt sich (das regelt der Weyl-Tensor); dabei ändert sich anfänglich das Volumen nicht. (Sobald die Kugel verzerrt ist, ändert sich auch das Volumen, weil jetzt unterschiedliche Raumrichtungen unterschiedlich stark eingehen.)
    Das ist alles ausfürhlich auf den Seiten von Baez erklärt.

  148. #148 MartinB
    20. September 2017

    @alle
    Keine Ahnung, was mit Niels passiert ist.
    (Huhu, Niels, gibt es dich noch?)

  149. #149 DerZimmermann
    20. September 2017

    Ich versteh ehrlich gesagt nicht, was das genau das problem ist und worüber ihr streitet

    Ich auch nicht. Ich dachte eigentlich, wir hätten alles geklärt.

    Darum habe ich ja dann oben in #135 nochmal eine Frage angesprochen, die mich schon eine ganze Weile lang irritiert; in der Hoffnung dass vllt jemand eine Idee dazu hat.

  150. #150 Karl-Heinz
    20. September 2017

    @MartinB

    Was genau meinst du mit einer linearen Verzerrung?

    Das war nur so eine Überlegung von mir. Statt Materie und Druck stelle ich mir eine Kugel mit räumlich gleichverteilter Energiedichte vor. Diese Energiedichten haben untereinander gravitativen Einfluss und es läuft ein Prozess ab, der die Kugel schrumpfen lässt. Die räumliche Gleichverteilung bleibt während des Schrumpfungsprozesses aufrecht. Falls ich die Gleichung richtig interpretiere, so ist außerhalb der Kugel nix oder mögliche Energiedichten sind so verteilt, dass sie keinen Einfluss auf die Kugel haben, in etwa so wie das Schwerefeld im inneren der Erde, wo ich auch nur mehr die Masse betrachten muss, die unterhalb mir liegt.
    Jetzt zurück zu Kugel. Wenn draußen nix ist, dann schrumpft die Kugel. Wenn Außerhalb der Kugel außer dem Nichts ich mich zusätzlich befinde, dann habe ich gravitativen Einfluss auf die Kugel und der Prozess läuft dann nicht mehr exakt so ab, wie von der Gleichung beschrieben wird, die ja fordert das die Verteilung der räumliche Energiedichte innerhalb der Kugel gleichmäßig ist.
    Meine Überlegung ist, dass wenn die Kugel sehr klein ist, ich zuerst den Schrumpfungsprozess betrachte und auf das Ergebnis dann per Superposition meinen gravitativen Einfluss überlagere.
    Ist nur eine Gedankenspielerei von mir.

  151. #151 MartinB
    20. September 2017

    @Karl-Heinz
    “Falls ich die Gleichung richtig interpretiere, so ist außerhalb der Kugel nix oder mögliche Energiedichten sind so verteilt, dass sie keinen Einfluss auf die Kugel haben”
    Für die anfängliche Volumenänderung der Kugel ist es egal, was außerhalb der Kugel ist, das ist ja der Witz bei der Gleichung.
    Und ob du Energie oder Masse nimmst ist egal, E=mc²
    “Wenn Außerhalb der Kugel außer dem Nichts ich mich zusätzlich befinde, dann habe ich gravitativen Einfluss auf die Kugel und der Prozess läuft dann nicht mehr exakt so ab, wie von der Gleichung beschrieben wird, die ja fordert das die Verteilung der räumliche Energiedichte innerhalb der Kugel gleichmäßig ist.”
    Deine Anwesenheit hat *anfänglich* keinen Einfluss auf das Volumen der Kugel sondern nur auf die Verzerrung (Form) der Kugel. Auch später gilt dasselbe: Wenn ich aus der verzerrten Kugel ein kleineres kugelförmiges Teilvolumen betrachte, dann ist auch dessen Volumenänderung nur durch die Materie/Energie im Innern dieser Kugel bestimmt.

  152. #152 Karl-Heinz
    20. September 2017

    @DerZimmermann

    Das mit der Gummimatte kann man ja auch mit Zahlen ausrechnen. Ist manchmal besser als zu Argumentieren, wo oft was gefolgert wird, was dann nicht stimmt.
    Soll ich dir das mal vorrechnen?

  153. #153 Karl-Heinz
    20. September 2017

    @MartinB

    OK Martin
    Ich glaube dir.
    Mit der Tensorrechnung bin ich im Moment leider nicht so vertraut. Damit ich die Raumverzerrung so richtig verstehe, da werde ich am besten mal deine Artikeln lesen.

  154. #154 DerZimmermann
    20. September 2017

    @Karl-Heinz: Das ist ja hier gerade das Problem, dass beim ausrechnen (für ein Universum) was rauskommt, was zum Modell (gummimatte) nicht passt.

  155. #155 MartinB
    20. September 2017

    @Karl-Heinz
    Die Tensor-geschichten stehen da aber nicht wirklich drin, da musst du noch bis nächstes Jahr warten…

  156. #156 Karl-Heinz
    20. September 2017

    @MartinB

    Auch später gilt dasselbe: Wenn ich aus der verzerrten Kugel ein kleineres kugelförmiges Teilvolumen betrachte, dann ist auch dessen Volumenänderung nur durch die Materie/Energie im Innern dieser Kugel bestimmt.

    Dieser Satz gefällt mir sehr gut.

    Wegen der Energiedichte hätte ich aber noch eine Frage. Wenn die Kugel verzehrt wird, wird die Energiedichte, die in der unverzerrten Kugel ja über das Kugelvolumen (auch beim Schrumpfvorgang) immer gleichmässig verteilt ist, mit der Kugel mitverzehrt?

  157. #157 Karl-Heinz
    20. September 2017

    @MartinB
    Für irgendwas muss die Gleichung von Baez ja gut sein.
    Ich überlege mir gerade.
    Wenn ich den Raum als homogen und isotrop ansehe, dann kann ich doch jeden Raumpunkt ein Skalar lim V”(t)/V(t) zuordnen mit V(t) –> 0.
    Der Limes müsste doch existieren, oder?

  158. #158 MartinB
    20. September 2017

    @Karl-Heinz
    “Wenn die Kugel verzehrt wird, wird die Energiedichte, die in der unverzerrten Kugel ja über das Kugelvolumen (auch beim Schrumpfvorgang) immer gleichmässig verteilt ist, mit der Kugel mitverzehrt?”
    Wenn die Kugel nicht – wie bei Baez – aus quasi-masselosen (sehr sehr leichten) Testteilchen besteht, muss das sicher so sein.

    Und ja, die Formel gilt exakt nur für ein infinitesimales Volumen, sonst muss man annehmen, dass in der Kugel alles perfekt homogen ist.

    Das steht letztlich auch alles auf der Seite von Baez, wenn man sehr genau liest….

  159. #159 Karl-Heinz
    20. September 2017

    @MartinB
    Danke Martin

    Also wenn ich jetzt alles richtig verstanden habe, bestimmt das Umfeld mit seinem gravitativen Einfluss die Verzerrung meiner sehr kleinen Testkugel zu einem Ellipsoid. Die Veränderung meines Testvolumens V”(t)/V(t) mit der
    Zeit wird dann mit der Gleichung von Baez beschrieben, was als Fluss des Raumes zum Zeitpunkt t interpretiert werden kann.

    Ich hätte am Anfang nicht gedacht, dass so eine Testkugel zur Anschaulichkeit beiträgt.

  160. #160 Karl-Heinz
    20. September 2017

    @MartinB
    So nachdem das mit der kleinen Testkugel geklärt ist habe ich noch ein Attentat auf deine Gummimatte vor. Aber erst später, denn mein Handy Akku wird gerade leer.

  161. #161 MartinB
    20. September 2017

    @Karl-Heinz
    Besser nicht, ich habe im Moment wenig Zeit und bin ab morgen auch noch auf Dienstreise.

  162. #162 Karl-Heinz
    25. September 2017

    Dehnen eines Gummituches, wenn linker Teil fixiert und Rechts gedehnt wird.
    Fall a)
    Gummituch wird ab dem Zeitpunkt t=0 im Abstand R1 von einer Rolle mit konstanter Drehzahl n1 und damit mit konstanter Geschwindigkeit v1 gedehnt.

    t … Zeit
    R … Abstand eines beliebigen Punktes P von der Fixierung bei t=0 (Beginn der Dehnung des Gummituches)
    v1 … Geschwindigkeit des Punktes P (Beginn der Dehnung des Gummituches)
    s (R, t) … Bewegungsgleichung des Punktes P(R) in Abhängigkeit der Zeit

    Wir werden sehen, dass eine Länge in diesem Fall zeitlich exponentiell gedehnt wird.

    Ansatz:
    s (R, t) =R*e^(k*t)
    s‘(R, t) =k*R*e^(k*t)
    Anfangsbedingung: s‘(R=R1, t=0)= k*R1 = v1
    Damit folgt k = v1/ R1

    Damit ergibt der
    Weg: s (R, t) =R*e^(t*v1/R1)
    Geschwindigkeit: s‘ (R, t) = (v1/R1) * R * e^(t*v1/R1)
    Beschleunigung: s‘‘ (R, t) = (v1/R1)^2 * R * e^(t*v1/R1)
    momentane relative Änderungsrate der Geschwindigkeit = s‘ (R, t) / s (R, t) = v1/R1
    momentane relative Änderungsrate der Beschleunigung = s‘‘ (R, t) / s (R, t) = (v1/R1) ^2
    —————————————————————-
    Sonderfall t=0
    Weg: s (R, t=0) =R
    Geschwindigkeit: s‘ (R, t=0) = (v1/R1) * R
    Beschleunigung: s‘‘ (R, t=0) = (v1/R1)^2 * R
    momentane relative Änderungsrate der Geschwindigkeit s‘ (R, t=0) / s (R, t) = v1/R1
    momentane relative Änderungsrate der Beschleunigung s‘‘ (R, t=0) / s (R, t) = (v1/R1) ^2

    —————————————————————–
    Sonderfall für t wenn Punkt P( R) die Stelle R1 erreicht.
    Ansatz: s (R, t) = R1 = R*e^(t*v1/R1)
    R1/R = e^(t*v1/R1)
    ln(R1/R) = t*v1/R1
    t= (R1/v1) * ln(R1/R)

    Weg: s (R, t=(R1/v1) * ln(R1/R)) =R*e^((R1/v1) * ln(R1/R)*v1/R1) = R* R1/R = R1

    Geschwindigkeit: s‘ (R, t=(R1/v1) * ln(R1/R)) = (v1/R1) * R * e^((R1/v1) * ln(R1/R)*v1/R1) = (R1/R) * (v1/R1) * R = v1

    Beschleunigung: s‘‘ (R, t= (R1/v1) * ln(R1/R)) = (v1/R1)^2 * R * e^((R1/v1) * ln(R1/R)*v1/R1) =(v1/R1)^2 * R * R1/R = (v1/R1)^2 * R1

  163. #163 Karl-Heinz
    26. September 2017

    Bei der Bezeichnung der Änderungsrate ist mir ein Fehler passiert.
    Eine relative Änderungsrate ist ja wie folgt definiert. y'(t)/y(t)

    Die Bezeichnung würde ich daher wie folgt ändern.

    s‘ (R, t) / s (R, t) … relative Längenänderung
    oder Längenänderung bezogen auf die Länge

    s‘‘ (R, t) / s (R, t) … Geschwindigkeitsänderung bezogen auf die Länge oder
    Änderung der Längenänderung bezogen auf die Länge.

  164. #164 MartinB
    26. September 2017

    @Karl-Heinz
    Sorry, wie gesagt, keine Zeit, um da irgendwas nachzurechnen.

  165. #165 Karl-Heinz
    26. September 2017

    @MartinB

    Sorry, wie gesagt, keine Zeit, um da irgendwas nachzurechnen.

    Macht nix.
    In dem Beispiel sieht man eh nur, dass sich der markierte Punkt bei jedem Abstand, ausser dort wo das Gummituch eingespannt ist, beschleunigt entfernt, wenn gilt dass H(t) sprich in inserem Fall s‘ (R, t) / s (R, t) zeitlich konstant und nicht null ist.

    Ich denke mann kann sich das ganze ja angucken, wenn es zeitlich mal passt.
    Ich will ja nur zeigen, dass da was in deiner Argumentation nicht stimmt.

  166. #166 MartinB
    26. September 2017

    @Karl-heinz
    Willst du damit sagen, dass wenn zwei Punkte anfänglich in Ruhe sind und sich dann voneinander entfernen, dass die Realtivgeschwindigkeit im Laufe der Zeit zunimmt? Das ist natürlich richtig, wenn man eine konstante Dehngeschwindigkeit annimmt.

  167. #167 Karl-Heinz
    26. September 2017

    @MartinB

    Nein.
    Ich will damit sagen, dass in dem Punkt, wo sich das Teilchen augenblicklich befindet, die Geschwindigkeitsänderung für das Teilchen nicht 0 ist.
    In diesem Fall kann ein Teilchen durch eine einmalige Gegengeschwindigkeit nicht auf Position gehalten werden.

  168. #168 MartinB
    26. September 2017

    @Karl-Heinz
    Sorry, aber dann ist es falsch. Wenn ich eine konstante dehnrate habe, dann ändert sich der Abstand zwischen zwei Punkten in fester Entfernung mit konstanter Geschwindigkeit. Also kann ich das ebenfalls durch ne konstante Geschw. kompensieren.

  169. #169 Karl-Heinz
    26. September 2017

    @MartinB

    Grummel, Grummel …
    konstante dehnrate = y'(t)/y(t) = konstant
    Das heißt jetzt nicht unbedingt, dass y'(t) zeitlich konstant ist!
    Grummel Grummel …

  170. #170 MartinB
    26. September 2017

    @Karlheinz
    Konstante Dehnrate heißt:
    Der Abstand zwischen zwei Punkten wächst proportional zum Abstand der Punkte (für jeden Meter Abstand gibt es pro Sekunde x Mikrometer “neuen Raum), und diese Rate x ändert sich nicht mit der Zeit.
    Natürlich wird eine anfänglich vorgegebene Länge dadurch exponentiell gedehnt, das ist aber kein Widerspruch zu meiner Aussage.

  171. #171 Karl-Heinz
    26. September 2017

    @MartinB
    Ich gehe davon aus,dass ein Körper, der sich im Abstand R0 von mir befindet, sich mit dem expandierenden Raum mitbewegt, wenn nicht irgendwelche Kräfte auf diesen Körper wirken.Wenn sich der Raum mit einer konstante Dehnrate ausdehnt, kann ich sofort eine scheinbare Bewegungsgleichung angeben.
    R(t) =R0 * e^(H*t) angeben. Zu einem späteren Zeitpunkt t1 beschließe ich den Körper auf konstanten Abstand zu mir zu halten, indem ich einen Term v1* (t-t1) für >= t1 einfüge.
    Irgentwie klappt das aber nicht. Wo mache ich da einen Fehler?

  172. #172 MartinB
    26. September 2017

    @Karl-Heinz
    “Ich gehe davon aus,dass ein Körper, der sich im Abstand R0 von mir befindet, sich mit dem expandierenden Raum mitbewegt, wenn nicht irgendwelche Kräfte auf diesen Körper wirken.”
    Das gilt nur, wenn der Körper am Anfang mit dem Raum mitbewegt wird. Hat der Körper eine passende Anfangsgeschwindigkeit, dann kompensiert die die Ausdehnung.
    Deine Formeln verstehe ich nicht, warum v1*(t-t1)?
    Du misst zum betrachteten Zeitpunkt den Abstand, berechnest daraus die Geschwindigkeit, mit der sich ein punkt in diesem Abstand entfernen würde, und gibst die (negativ) dem Körper, dann bleibt er am Ort.

  173. #173 Karl-Heinz
    26. September 2017

    @MartinB

    R(t) =R0 * e^(H*t)
    R(t=0) = R0
    R(t=1) = R0 * e^H
    v1 =v(t=1) = R'(t)= H* R0 * e^H
    Für t>=1 mache ich es dann so wie du es gesagt hast.
    R(t) =R0 * e^(H*t) – v1 * ( t-1) =
    R0 * e^(H*t) – H* R0 * e^H *( t-1) = konstant für alle t >=1 ???

  174. #174 MartinB
    26. September 2017

    @Karl-Heinz
    Ich bin mir nicht sicher, ob ich deine Gleichugn richtig verstehe, aber so wie ich es sehe, steckt das Problem darin, dass du erst ne exponentielle Lösung berechnest, die nur für einen mitbewegten Beobachter gilt, und dann versuchst, in der die Geschwindigkeit mit nem konstanten term zu komensieren. Damit verwendest du unterschiedliche Bezugssysteme.
    In einer Entfernung von x ist die Gescheindigkeit zu jedem Zeitpuntk dieselbe, aber wenn du einen Punkt, der zur Zeit t am Ort x ist, immer weiter verfolgst, nimmt die Geschwindigkeit exponentiell zu. Das vermischt du in deinen Gleichungen unzulässig.

  175. #175 Karl-Heinz
    26. September 2017

    @MartinB

    Als Bezugsystem habe ich mich selbst genommen, also eigentlich nur ein Bezugsystem.

    Der exponentieller Term ist dadurch gegeben,
    weil wir ja im Moment, so hoffe ich jene Expansions betrachten, die die Bedingung s'(t)/s(t) = konstant = H erfüllt.

  176. #176 MartinB
    26. September 2017

    Ich geb auf…

  177. #177 Karl-Heinz
    26. September 2017

    @MartinB
    Eine Bitte hätte ich noch.
    Könntest du den Term, der aus deiner Sicht aus richtig ist, vervollständigen.
    geg.:
    Körper, welcher im Abstand D0 von mir aus gesehen, mit dem Raum mit fließt. Der Raum soll sich exponentiell ausdehnen.
    ges.: s(t)= ???
    Damit du siehst, dass ich nicht ganz so dumm bin, behaupte ich mal
    s(t=0) = D0 und
    ein Meter bleibt ein Meter für s(t)

  178. #178 MartinB
    26. September 2017

    @Karl-Heinz
    Nein, kann ich nicht, dazu müsste ich erstmal genau nachvollziehen, wie du s(t) definierst…

  179. #179 Karl-Heinz
    26. September 2017

    @MartinB
    Unter s(t) würde ich den Abstand zu mir verstehen. In etwa so, ich makiere einen Stab in einem Abstand von einem Meter zu mir und dieser Stab soll sich dann mit konstater Rate ausdehnen. Die Markierung wird sich von mir entfernen, d.h. der Abstand zu mir wird mit der Zeit grösser. s(t) ist eine Funktion der Zeit und genau diese Funktion suche ich.
    Wenn diese Überlegung mit der Expansion des Raumes aus welchen Gründen auch immer nichts zu tun hat, am besten sag einfach, dass diese Analogie Mist ist.

  180. #180 MartinB
    26. September 2017

    @Karl-Heinz

    Meinst du, du nimmst einen Stab. legst ihn neben dich, markierst einen Punkt in 1m entfernung und lässt den stab sich dann mit konstanter Rate ausdehnen, wobei Rate eine konstante Dehngeschwindigkeit (delta x/x) ist?
    Ja, dann entfernt sich dr Puntk auf dem Stab exponentiell von dir.
    Aber wen du zu jedem Zeitpunkt t immer einen Punkt betrtachtest, der jetzt zu diesem Zeitpunkt 1 Meter von dir entfernt ist, dann hat der Stab *an diesem Punkt* immer dieselbe Geschwindigkeit. Und deshalb kann ein Objekt, das sich an diesem Punkt befindet und eine bestimmte Geschwindigkeit relativ zjm Stabmaterial hat, auch kräftefrei am Ort verharren.

  181. #181 MartinB
    26. September 2017

    @Karl-Heinz
    Ich habe gerade unter der Dusche nachgedacht – ich glaube, dein Fehler ist, dass du im falschen Moment durch Null teilst.
    Hier nochmal die Sachlage, wie ich sie verstehe:
    Wir haben eine konstante Dehnrate epsdot, beispielsweise 1mm/s/m (Zwei Punkt, die einen Meter entfernt sind, entfernen sich in eiern Sekunde um einen Millimeter voneinander.
    Betrachten wir ein teilchen am Ort x und seine Entfernung zum Nullpunkt.
    Dann gilt:
    dx/dt = epsdot * x
    Dann machst du Separation der Variablen usw. und landest bei ner e-funktion für x(t).
    Tatsächlich hat das Teilchen am Ort x aber auch ne Geschwindigkeit v. Dann gilt
    dx/dt = epsdot * x + v
    Und für v=-epsdot x funktioniert die Separation der Variablen nicht mehr, weil rechts Null steht, stattdessen steht da direkt dx/dt=0, wie ich es ja auch die ganze Zeit sage.

  182. #182 Karl-Heinz
    26. September 2017

    @MartinB
    Danke für die rasche Antwort.
    Ich werde deine Argumente am späten Abend in Ruhe nochmals durchgehen.

    Ps: Ich wollte auf keinen Fall stressen.

  183. #183 DerZimmermann
    26. September 2017

    @Martin
    #180: Widersprichst du da nicht dir selbst? Die konstante Dehnrate entspricht einem konstanten Hubbleparameter. Dieser jedoch hat ein positives a” zur Folge wegen der exponentiellen Ausdehnung, wodurch (wie oben schon diskutiert) eine “wegtreibende” Kraft auf den Testkörper in 1m Entfernung wirkt => dieser kann also *nicht* durch eine einmalige Geschwindigkeit auf dieser Position gehalten werden. (Und genau in diesem Punkt sehe ich eben ein gravierendes Problem in der Gummiband- (bzw. Raum-Entstehungs-) Anschauung – weil danach müsste die konstante Geschwindigkeit ja reichen).

    #180: Woher soll denn das Teilchen diese zusätzliche Geschwindigkeit v haben? Es hat eine Zurückweichgeschwindigkeit durch die Ausdehnung, aber das ist ja gerade dein “epsdot * x”

  184. #184 Karl-Heinz
    26. September 2017

    @DerZimmermann

    Bevor du mich steinigst, dass folgende ist nicht mehr als ein Scherz.

    Auf die Reformation antworten wir mit einer Gegenreformation.
    Auf eine Expansion antworten wir mit einer Kontraktion.
    Mathematisch in etwa so ausgedrückt.
    s(t) = s0 * Expansion * Kontraktion
    s(t) = s0 * e^(H*t) * e^(-H*t) = s0
    s'(t) = 0 für jedes s0
    So, jetzt habe ich ein konstantes Universum ohne Ausdehnung, aber man beachte, dass die Geschwindigkeit der Expansion durch die Geschwindigkeit der Kontraktion aufgehoben wird. Was sagst zu meiner Idee? Ist sie nicht genial?

  185. #185 Karl-Heinz
    26. September 2017

    @DerZimmermann

    Der do is noch tiefer.

    Du mich hat es fast vom Hocker gehauen.
    Ich glaub Martin hat soeben in Gleichung #180
    eine kosmologische Geschwindigkeitskonstante eingeführt. 😉

  186. #186 Karl-Heinz
    26. September 2017

    Wenn gilt s'(t)/s(t) =H
    dann gilt
    s(t)=s0 * e^ (H * t )
    Wenn, das Teilchen in meinem Bezugsystem eine Geschwindigkeit v0 hat, dann könnte man s(t) so schreiben:
    s(t) = (s0 + v0 * t) * e^ (H * t )

  187. #187 Karl-Heinz
    27. September 2017

    @DerZimmermann

    Du hast Recht. Gummituch und Autos auf dem Gummituch sind keine gute Analogie um das Verhalten von Objekten, bei der Expansion des Raumes zu beschreiben.
    Objekte sofern keine äusseren Kräfte auf sie einwirken, fließen mit dem Raum mit. Dieses Verhalten zeigen, die Autos gegenüber dem Gummituch nicht. Sagen wir es einmal so, dass den Autos ziemlich egal ist, ob das Gummituch gestreckt wir oder nicht, solange Reibungslosigkeit herrscht. Für die Autos ist nicht ausschlaggebend was das Gummituch macht, sondern der echte Raum.

  188. #188 MartinB
    27. September 2017

    @DerZimmermann
    Ich verstehe deinen Kommentar nicht – aber ich habe jetzt auch wirklich genug Zeit in diese Diskussion gesteckt und im Moment nicht die Muße, mir nochmal alle Definitionen von Parametern anzugucken, da die Physik meiner Ansicht nach ziemlich klar ist.

  189. #189 DerZimmermann
    27. September 2017

    Ich dachte eigentlich auch schon ein paar mal dass eigentlich alles geklärt sein müsste…

    Der Punkt ist eben, dass in einem Universum mit konstantem Hubbleparameter tatsächlich eine auseindertreibende Kraft wirkt. Eben weil aus einem konstanten H automatisch ein positives a” (2. Ableitung des Skalenfaktors) folgt.

  190. #190 MartinB
    27. September 2017

    @Der Zimmermann
    Verstehe ich mal wieder nicht. Wenn der Hubble-Parameter 74km/sMpc beträgt, und ich mich in 1 Mpc Entfernung von dir befinde, dann kann ich mit einer relativgeschwindigkeit von 1km/s relativ zum mitbewegten Beobachter relativ zu dir ruhen, und daran ändert sich dann doch auch mit der Zeit nichts nichts.

  191. #191 Karl-Heinz
    27. September 2017

    @MartinB
    Auch wenn der mitbewegte Beobachter bei 1 Mpc Entfernung relativ zu dir ruht, wer sagt denn, dass er dort bleibt. Also wenn ich einen Stein senkrecht nach oben werfe, dann gibt es auch einen Anstand, wo der Stein relativ zu mir ruht.

  192. #192 DerZimmermann
    27. September 2017

    Verstehe ich mal wieder nicht. Wenn der Hubble-Parameter 74km/sMpc beträgt, und ich mich in 1 Mpc Entfernung von dir befinde, dann kann ich mit einer relativgeschwindigkeit von 1km/s relativ zum mitbewegten Beobachter relativ zu dir ruhen, und daran ändert sich dann doch auch mit der Zeit nichts nichts.

    Ich verstehs auch nicht.

    Aber du hast ja schon im Artikel oben gesagt, dass die Kraft auf den Testkörper (z.B. in 1Mpc Entfernung) von a” abhängt. Und bei konstantem H ist a” nun mal positiv. D.h. es wirkt eine “wegtreibende” Kraft auf den Körper.

    Die Frage ist nur, wie begründet/erklärt man dieses Rechenergebnis anschaulich?

  193. #193 Karl-Heinz
    27. September 2017

    Laut Literatur ist die Kraftwirkung proportional
    H'(t)+H(t)^2.
    Wenn H(t) zeitlich fällt kann es durchaus sein, dass H'(t)+H(t)^2 zu null wird und damit die Kraft auch null wird.

  194. #194 Karl-Heinz
    27. September 2017

    @DerZimmermann

    Ich verstehs auch nicht

    Wie wäre es mit Summe der Geschwindigkeiten ist 0 und einer Kraft? Körper bleibt natürlich nicht auf seinem Platz.

  195. #195 MartinB
    27. September 2017

    So, ich setze das mal auf die Liste der Dinge, die ich mir irgendwann nochmal gründlich angucken muss, wenn ich’s verstanden habe, sage ich Bescheid…

  196. #196 Karl-Heinz
    27. September 2017

    @MartinB
    Daumen hoch.
    So sollte Wissenschaft funktionieren.

    Ich hoffe, du hast unsere Kritik bzw. Bedenken im positiven Sinne entgegengenommen.

  197. #197 DerZimmermann
    27. September 2017

    Laut Literatur ist die Kraftwirkung proportional
    H'(t)+H(t)^2.
    Wenn H(t) zeitlich fällt kann es durchaus sein, dass H'(t)+H(t)^2 zu null wird und damit die Kraft auch null wird.

    Ja…. warum einfach wenn’s kompliziert auch geht^^

    a ist die Entfernung eines mitbewegten Testkörpers (das normiert man normalerweise passend).
    a’ damit seine Geschwindigkeit.
    a” damit seine Beschleunigung und damit proportional zur auf ihn wirkenden Kraft (Masse mal Beschleunigung). Wenn der Körper jetzt nicht mitbewegt werden soll, muss ich das noch durch a teilen, damit ich den Effekt durch die wachsende Entfernung rausrechne und immer die Beschleunigung in Entfernung des ursprünglichen a habe. Ergebnis: F ~ a”/a. Absolut logisch.

    Dass H’ + H^2 aufs gleiche rausläuft ist einfach die Definition von H mit ein bisschen ableiten und umformen. Ich sehe aber nicht wie man sich das direkt logisch herleiten soll (oder wie es das Verständnis fördern soll….).

    Ich persönlich glaube ja inzwischen, dass das Bild vom neu entstehenden Raum einfach Käse ist und man sich das ganze einfach eher wie eine Bewegung vorstellen muss.

  198. #198 Karl-Heinz
    27. September 2017

    @DerZimmermann

    a”(t)/a(t)=H’(t) + H(t)^2 ∝ Kraft

    Dass das H’ + H^2 das logische Verständnis fördern soll war ja nie meine Intention.
    H’ + H^2 soll ja nur beschreiben, wie sich die relative Änderungsrate der Ausdehnung und deren zeitliche Änderung auf die Kraft auswirkt.

  199. #199 Karl-Heinz
    27. September 2017

    @DerZimmermann

    Ich persönlich glaube ja inzwischen, dass das Bild vom neu entstehenden Raum einfach Käse ist und man sich das ganze einfach eher wie eine Bewegung vorstellen muss.

    Ich habe mich gerade wegen deiner Meinung verschluckt 😉

  200. #200 DerZimmermann
    29. September 2017

    Nehmen wir nochmal das Experiment von MartinB: Ich setze in einer bestimmten Entfernung von mir einen Testkörper aus. Damit alles ganz sauber ist, kann ich mir auch vorstellen, dass ich eines der Gaspartikel im Friedmannuniversum bin und der Testkörper ein anderer.

    Ich stelle mir den Raum jetzt wie ein Gummiband vor, das auseinandergezogen wird, und den Testkörper als kleines Fahrzeug, das reibungslos rollt. Aus meiner Beobachterperspektive sieht es so aus, als ob ich stillstehe und der Raum von mir weggezogen wird. Ich gehe von gleichbleibender Expansion aus (d.h., a‘ konstant, damit H ~ 1/t), also ein leeres Universum.
    Wenn ich den Testkörper zuerst fixiere, hat er zum Gummiband eine konstante Geschwindigkeit, die genau die Dehnung ausgleicht. Da aber bei konstanter Expansion (a‘ konstant) die Geschwindigkeit des Gummibandes an der „Testkörper-Stelle“ sinkt (der Testkörper bleibt ja in derselben Entfernung zu mir), müsste der Testkörper ja eigentlich beginnen, sich auf mich zuzubewegen.
    Man kann aber ausrechnen, dass er das nicht tun wird, sondern er bleibt für immer in dieser Entfernung. a‘‘ ist in diesem Szenario gleich 0, und damit auch die Kraft.

    Jetzt gehe ich von einem Dunkle-Energie-dominierten, beschleunigten Universum aus (d.h., H konstant, a ~ e^(H*t)). In diesem Fall bleibt die Geschwindigkeit des Gummis an der Testkörper-Entfernung immer gleich, und damit sollte der Testkörper immer an dieser Stelle bleiben, weil er ja die benötigte Relativgeschwindigkeit von mir bekommen hat.
    Man kann aber ausrechnen, dass er das nicht tun wird, sondern er wird beschleunigt von mir wegfliegen, und die Beschleunigung ist ~ a‘‘/a > 0.

    Bei einer *unbeeinflussten* Expansion bleibt H NICHT konstant, sondern fällt mit H ~ 1/t, eine gegebene Länge (z.B. 1 Mpc) expandiert pro Sekunde also morgen weniger als heute. Warum sollte das so sein, wenn es keine Einflüsse gibt und sich nichts geändert hat? Ist der neu entstandene Raum einfach irgendwie „anders“ als der ursprüngliche und dehnt sich nicht aus? Weshalb sollte er?

    Dagegen lässt sich ausrechnen, dass bei einer unbeeinflussten Expansion (keine Gravitation) a’ konstant bleibt – wie wenn ein kräftefreier Körper sich gleichförmig bewegt.

    Versteht ihr, was ich meine? Es passt alles einfach vorne und hinten nicht.
    Jetzt versuche ich es mal ganz anders.

    Ich stelle mir vor, dass alle Objekte beim Urknall einen initialen „Schubs“ bekommen haben, durch den sie auseinanderfliegen, und ansonsten wirkt einfach die Gravitation.

    Wenn ein Objekt sich mit konstanter Geschwindigkeit bewegt, dann tut es das für immer und ewig, wenn es keine Einflüsse gibt. In einem leeren Universum (ohne Masse, Strahlung, Dunkle Energie) fliegen die Objekte also nach dieser Vorstellung mit gleichbleibender Geschwindigkeit auseinander – und genau das tun sie nach den Friedmanngleichungen. PASST!

    Wenn ich jetzt meine zwei (masselosen) Testpartikel in dieses leere Universum hineinsetze, das mit konstantem a‘ expandiert, passiert natürlich nichts – sie bleiben einfach an der Stelle stehen, wo ich sie hinsetze. Die haben ja den „Schubs“ nicht mitbekommen. PASST!

    Jetzt kommt aber die Masse ins Spiel: Da die Objekte im Universum Masse haben, werden sie durch die Gravitation „zusammengezogen“, wenn also Masse im Universum ist, sollte die Geschwindigkeit sinken, und zwar entsprechend dem Gravitationsgesetz, genau wie beim Apfel, der vom Baum fällt. Und genau das tut sie – PASST! (kann man mit der 2. Friedmanngleichung nachrechnen).

    Jetzt stecke ich dunkle Energie in mein Universum. Die hat negativen Druck, und dieser bewirkt eine Art umgekehrte Gravitation – er treibt Objekte auseinander. Wenn ich also keine Masse in meinem Universum habe, dann müssten die Testobjekte ab dem Moment auseinandergetrieben werden (mit einer Kraft), in dem ich ein winziges bisschen Dunkle Energie hineinstecke. Und genau so ist es – PASST! Allein die Dunkle Energie kann eine nach “außen” gerichtete Kraft verursachen.

    Die Vorstellung vom neu entstehenden Raum brauche ich eigentlich nur, um klarzustellen, dass die Galaxien sich nicht gegenseitig Platz wegnehmen, sondern von allen Standpunkten aus einfach nach „außen“ wandern. Für die übrigen Phänomene kommt da nichts Brauchbares raus. Vielleicht kann man sich ja auch vorstellen, dass alle „Raumpunkte“ beim Urknall den „Schubs“ bekommen haben…?

    Es kann auch gut sein, dass ich hier auch Unsinn erzähle. Ich hab auch kein Problem damit, wenn mir das jemand sagt (im Gegenteil), aber dann hätte ich eben auch gerne eine Erklärung für die Widersprüche oben.

  201. #201 Alderamin
    29. September 2017

    @DerZimmermann

    Ohne neu entstandenen Raum bekommst Du aber ein Problem, wenn die Expansionsgeschwindigkeit gegen die Lichtgeschwindigkeit geht, denn man (also Masse) kann sich im Raum nicht so schnell wie das Licht bewegen. Aber es wird ja erwartet, dass hinter dem Hubble-Radius die Expansion von hier aus gesehen überlichtschnell erfolgt. Das geht nicht in einem nicht expandierenden Raum, in dem nur Materie auseinander fliegt.

    So wie ich das in meinem laienhaften Verständnis mitbekommen habe, geht man tatsächlich davon aus, dass die Materie vom Urknall her einen gewissen Schwung bekommen hat und deswegen expandiert. Dabei zieht sie aber den Raum mit sich mit, das dehnt ihn (im Prinzip rechnet man in der FLWR-Gleichung ja so, als ob der Raum homogen mit einer bestimmten Massedichte gefüllt sei, nix Sterne und Galaxien, eher wie ein Gas). Bevor man die dunkle Energie entdeckte, schien klar, dass die Expansion durch die wechselseitige Anziehung der Materie verlangsamt werden würde und sich bei mehr als der kritischen Dichte irgendwann umkehren müsste. Dann wäre der Raum auch wieder geschrumpft. Dank der dunklen Energie gibt das Vakuum aber auch selbst schon genug Gas, dass es dazu nicht kommen wird (wenn’s nicht noch etwas anderes Dunkles gibt, das die Verhältnisse mit der Zeit wieder umdreht).

    Wir haben also einerseits einen Mitzieheffekt durch die auseinander strebende Materie und andererseits eine abstoßende Graviation, die sich aus dem negativen Druck des Vakuums ergibt (irgendwo hatte ich mal eine Herleitung gefunden, warum ein negativer Druck im Energie-Impuls-Tensor [Diagonalelemente außer ganz oben links] eine abstoßende Gravitation verursacht). Beides bläht den Raum auf, das erste mit angezogener Handbremse, das zweite mit durchgetretenem Gaspedal. Das Gaspedal ist aber bekanntlich stärker als die Handbremse.

  202. #202 Karl-Heinz
    30. September 2017

    Ich würde sogar meinen, dass die Vorstellung vom neu entstanden Raum zwingend nötig ist, weil es ja von jedem Bezugssystem aus funktionieren soll.
    Machen wir ein Gedankenexperiment. Nehmen wir an der Raum expandiert im Augenblick nicht und ich bin das Zentrum und ordne rund um mich radial nach außen Testkörper an, die untereinander keiner gravitativen Wechselwirkung unterliegen. Sobald der Raum expandiert, sollen die Testkörper mit dem expandierenden Raum mitschwimmen. Auch die Eigenschaft, dass auf die Körper keine Kräfte wirken, wenn sie sich „lokal“ mit konstanter Geschwindigkeit bewegen, wollen wir beibehalten. Ebenso wollen wir die Begriffe Momentangeschwindigkeit und Momentanbeschleunigung etwas genauer unter die Lupe nehmen. Wenn ein Körper eine Momentangeschwindigkeit besitzt, dann befindet er sich nach der Zeit Δt um die Wegstrecke v*Δt mit derselben Geschwindigkeit versetzt wieder. Hat der Körper eine Momentangeschwindigkeit von 0, so bleibt er am selbigen Ort. Beide Aussagen gelten aber nur dann, wenn auf den Körper keine Beschleunigung (Kraft) wirkt. Die Momentanbeschleunigung verursacht eine Geschwindigkeitänderung innerhalb der Zeitspanne Δt. Wenn die Momentangeschwindigkeit 0 aber die Momentanbeschleunigung ungleich 0 ist, so bleibt der Körper auch nicht am selbigen Ort, sondern findet sich um ein kurzes Wegstück versetzt wieder. Jetzt betrachten wir wieder unsere Testkörper, die welche mit einem bestimmten Abstand zu mir angeordnet sind. Solange der Raum nicht expandiert halten alle Testkörper einen konstanten Abstand zu mir. Die Momentangeschwindigkeit und Momentanbeschleunigung sind sowohl aus meiner Sicht als auch aus der Sicht des Bezugssystems, wo sich die Körper befinden 0. So jetzt lassen wir den Raum sich linear ausdehnen. Unter einer linearen Expansion des Raumes verstehe ich, dass eine konstante Sphäre um mich mit konstanter Geschwindigkeit sich ausdehnt. Der innere und äußere Bereich der Sphäre wird entsprechen mitskaliert. Die Probekörper lasse ich mit dem Raum mitfließen und sie bewegen sich dadurch von mir aus gesehen radial mit konstanter Geschwindigkeit ohne Beschleunigung nach außen.
    Doch was sieht das Bezugsystem, wo sich der Körper gerade zuvor befunden hat. Da sich der Körper mit dem Raum mitbewegt stellen wir hier eine Geschwindigkeit und eine Beschleunigung von 0 fest.
    Geben wir anschließend jeden Körper eine entsprechende Gegengeschwindigkeit, die Radial zu mir zeigt, so kann man jeden Körper in Bezug auf mich, auf konstanten Abstand halten. Das Bezugsystem, wo sich der Körper ursprünglich befunden hat, entfernt sich mit konstanter Geschwindigkeit sowohl vom Körper als auch von mir.
    Soweit alles klar? Jetzt kommt aber der Hammer. Wir werden die Änderungsrate der Ausdehnung berechnen und sie ist wie wir sehen werden, zeitlich nicht konstant und nimmt mit der Zeit ab. Die Sphäre, die sich mit konstanter Geschwindigkeit ausdehnen soll, werde im mal als Formel anschreiben.
    R(t)=R0 * (1 + t * v0/R0) = R0+ v0 * t Ausdehnung der Sphäre
    R‘(t)= v0 … Geschwindigkeit mit der sich die Sphäre R0 ausdehnt.
    H(t)= R‘(t)/R(t) = v0/(R0 + v0 * t) relative Änderungsrate von der Ausdehnung der Sphäre
    Damit ergibt sich die Geschwindigkeit mit der sich der Raum im Abstand D zu mir ausdehnt
    v(D,t) = H(t)*D = D * v0 / (R0 + v0 * t)
    Probe: v(R0, t=0) = v0
    Fazit ist:
    Wenn ich also Objekte, welche bisher dem Fluss des Raumes (lineare Ausdehnung) gefolgt sind, eine einmalige Geschwindigkeit
    D * v0 / (R0 + v0 * t) radial nach innen zu mir verpasse, dann kann ich dieses Objekt auf konstanten Abstand in Bezug zu mir halten. Das funktioniert wie wir gesehenen haben, zumindest mal, wenn sich der Raum linear ausdehnt. Wenn t gleich 0 ist müsste ich den Objekten, die bisher dem Fluss des Raumes gefolgt sind, eine Geschwindigkeit radial nach innen mit v(D,t=0) = v0*D/R0 verpassen.
    Ob dass, auch funktioniert, wenn der Raum sich beschleunigt ausdehnt?
    MartinB sagt ja, ich sage nein.
    MartinB sagte ja auch, dass genau dann eine Kraft entsteht, wenn sich die relative Änderungsrate der Ausdehnung zeitlich ändert. Wie wir aber gesehen haben, ist unsere Änderungsrate zeitlich nicht konstant und dennoch entsteht keine Kraft.
    Wenn Interesse besteht kann ich das ganze ja mal mit einem Raum, der sich beschleunigt ausdehnt durchspielen.

    @ Alderamin
    Danke für deinen Kommentar

  203. #203 DerZimmermann
    30. September 2017

    Ohne neu entstandenen Raum bekommst Du aber ein Problem, wenn die Expansionsgeschwindigkeit gegen die Lichtgeschwindigkeit geht, denn man (also Masse) kann sich im Raum nicht so schnell wie das Licht bewegen.

    Das wird in der ART doch sowieso ein wenig aufgeweicht, oder? Solange es an verschiedenen Orten ist, sind überlichtschnelle Relativgeschwindigkeiten ja nicht prinzipiell ausgeschlossen. (Oder…?)

    Mit Licht bekomme ich sowieso ein Problem. Weil das wiederum scheint bei konstantem H durchaus auf konstantem Abstand “gehalten” zu werden – ohne dass jemand anschiebt (siehe Hubble-Radius / Ereignishorizont). Ich gehe aber mal davon aus, dass hier eher Licht die Sonderrolle hat und nicht die Materie. Evtl müsste man sich da noch mal Gedanken machen….

    (…) dass die Materie vom Urknall her einen gewissen Schwung bekommen hat und deswegen expandiert. Dabei zieht sie aber den Raum mit sich mit, das dehnt ihn.

    So ungefähr muss man sich das wahrscheinlich vorstellen. Gravitation ist ja hier eher eine Veränderung der Raumstruktur und keine Kraft. Aber es ändert nichts daran, dass der grundlegende Mechanismus der Expansion am ehesten einer Bewegung entspricht (was auch in dem PDF so beschrieben wurde, das ich hier mal verlinkt hatte: https://www.ita.uni-heidelberg.de/~dullemond/lectures/cosmology_2011/Chapter_2.pdf ).

  204. #204 Karl-Heinz
    30. September 2017

    Zum Abschluss noch ein Paper, auf welches
    Alderamin seit längerem schon hingewiesen hat.
    Ich klinke mich dann aus.

    In Gleichung (2) wird die Beschleunigung zwischen der Milchstraße und Andromeda beschrieben, und der zweite Term ist derjenige durch die Expansion des Universums (mit umgekehrtem Vorzeichen zum ersten, rein gravitativen Term).

    A dynamical model of the local cosmic expansion.
    https://arxiv.org/pdf/1405.0306.pdf

  205. #205 Alderamin
    30. September 2017

    @DerZimmermann

    Das wird in der ART doch sowieso ein wenig aufgeweicht, oder? Solange es an verschiedenen Orten ist, sind überlichtschnelle Relativgeschwindigkeiten ja nicht prinzipiell ausgeschlossen. (Oder…?)

    Doch? Es kommt weniger auf den Ort oder die Entfernung an, sondern vielmehr darauf, was die Raumzeit macht. In einer expandierenden Raumzeit ist kein Inertialsystem ziwschen weit entfernten Orten mehr gegeben. In einer gekrümmten Raumzeit wie in der Nähe eines schwarzen Lochs auch nicht (Licht läuft im Schwerefeld langsamer). Aber wenn man einen leeren, stationären Raum betrachten würde, in dem sich nur (beinahe) masselose Testpartikel im freien Fall befinden, dann hätte man ein Inertialsystem und dann dürfte sich nirgendwo ein Partikel in Bezug zu einem anderen Partikel mit Lichtgeschwindigkeit oder mehr bewegen.

  206. #206 DerZimmermann
    30. September 2017

    In Gleichung (2) wird die Beschleunigung zwischen der Milchstraße und Andromeda beschrieben, und der zweite Term ist derjenige durch die Expansion des Universums (mit umgekehrtem Vorzeichen zum ersten, rein gravitativen Term).

    Den zweiten Term kann man umformen zu 2 * M_Lambda * G / r^2. Damit lautet die Gleichung insgesamt also r‘‘ = – G*M_Materie / r^2 + G*2*M_Lambda/r^2.
    Zu Deutsch: Die Beschleunigung ergibt sich aus der zusammenziehenden Gravitation der Materie zusammen mit der abstoßenden Gravitation durch die dunkle Energie. (DE wirkt 2x so stark abstoßend wie Materie anziehend, das kommt durch den negativen Druck in drei Raumrichtungen). So simpel ist das. Und passt damit auch genau zu der Vorstellung oben.
    (Ich habe mir mal erlaubt, den Energiegehalt der DE als Masse hinzuschreiben).

    Aber wenn man einen leeren, stationären Raum betrachten würde, in dem sich nur (beinahe) masselose Testpartikel im freien Fall befinden, dann hätte man ein Inertialsystem und dann dürfte sich nirgendwo ein Partikel in Bezug zu einem anderen Partikel mit Lichtgeschwindigkeit oder mehr bewegen.

    Das habe ich ja auch nicht behauptet, oder? Mir ist natürlich schon bewusst, dass ich mich da jetzt nicht mehr wirklich auskenne und auf dünnem Eis bewege.

  207. #207 Karl-Heinz
    30. September 2017

    @DerZimmermann

    Deine Unformung der Gleichung kann ich nicht nachvollziehen. Was ist bei dir M_Lambda?

  208. #208 Karl-Heinz
    30. September 2017

    @DerZimmermann
    Sorry habe schon gesehen, du meinst mit M_Lambda die Masse der DE.
    Meine Frage daher: Wie berechnest du die Masse der DE für diesen speziellen Fall?

  209. #209 Karl-Heinz
    1. Oktober 2017

    @DerZimmermann

    DE wirkt 2x so stark abstoßend wie Materie anziehend

    Ich glaube, du verzapft gerade einen Blödsinn.
    Energie ist Energie auch wenn die DE in unserem Fall abstoßend wirkt. Dein Faktor von 2 kommt daher, dass du du manche Dinge falsch interpretierst.

  210. #210 DerZimmermann
    1. Oktober 2017

    Das Bild, das hinter der Gleichung steckt, ist auch wieder das einer Kugel, in der die Masse gleichmäßig verteilt ist; der eine betrachtete Punkt sitzt dabei im Mittelpunkt der Kugel, der andere Punkt am Rand. Dass dieses Modell hier benutzt wird, sieht man auch an dem ersten Schritt, der schon in dem Paper gemacht wird: von -4*pi*G*Dichte*r/3 zu -G*M/r^2. Dabei ist (4/3)*pi*r^3 das Volumen, und darüber erhält man dann aus der Dichte die Masse.

    Das Omega_Lambda ist der Dichteparameter der Dunklen Energie. Dichteparameter bedeutet Dichte/kritische Dichte, und letztere ist definiert als 3*H^2 / 8*pi*G. Das setze ich dann in den zweiten Term ein und bekomme H^2 * (Dichte_DE / (3*H^2 / 8*pi*G)) * r. Und daraus wird dann – mit ein bisschen Kürzen, Gleichung für Kugelvolumen einsetzen etc. – das was ich oben geschrieben habe.

    Ich glaube, du verzapft gerade einen Blödsinn.
    Energie ist Energie auch wenn die DE in unserem Fall abstoßend wirkt. Dein Faktor von 2 kommt daher, dass du du manche Dinge falsch interpretierst.

    Ich verzapfe hier keinen Blödsinn 😉
    Dunkle Energie ist selbstverständlich Energie. Darum wirkt sie auch anziehend. Allerdings hat sie negativen Druck (was eigentlich die Definition von Dunkler Energie ist), welcher abstoßend wirkt. Da das Universum homogen und isotrop sein soll, ist der negative Druck in allen 3 Raumrichtungen gleich und daher kommt ein Faktor 3 bei der Abstoßung (findet sich auch in der 2. Friedmanngleichung wieder). Das wird auch auf der Seite von baez so beschrieben. 1x anziehend plus 3x abstoßend ergibt 2x abstoßend.

  211. #211 Karl-Heinz
    1. Oktober 2017

    @DerZimmermann

    Sehe gerade, dass du mit dem Faktor 2 rechtgehabt hast.
    Komisch, hätte ich jetzt nicht erwartet.

    ρ =3H^2/(8πG)
    Kugelvolumen V=(4/3) * π * r^3
    Masse m = Kugelvolumen * ρ
    =(4/3) * π * r^3 * 3H^2/(8πG)
    =r^3 * H^2/(2G)=m
    => H^2 = m* 2G/r^3

    rechter Term von Gleichung2: H^2* r
    H^2* r = m* 2G/r^2

  212. #212 Karl-Heinz
    1. Oktober 2017

    @DerZimmermann
    Jetzt verstehe ich, du meinst eine Kombination der Eigenschaft der DE.
    Anziehend DE (gravitativ)+ abstoßend DE (Druck) => Term m* G/r^2 muss mit Faktor 2 multipliziert werden.

  213. #213 Karl-Heinz
    2. Oktober 2017

    Vielleicht sollte man auch dazusagen, dass wenn die gedachte Kugel expandiert, die Dichte der Materie annimmt aber die Dichte der Dunklen Energie gleich bleibt. Mit der Zeit gewinnt dann die Dunklen Energie die Oberhand und die Expansion ist nur mehr von der Dunklen Energie bestimmt.

  214. #214 DerZimmermann
    3. Oktober 2017

    Jetzt verstehe ich, du meinst eine Kombination der Eigenschaft der DE.
    Anziehend DE (gravitativ)+ abstoßend DE (Druck) => Term m* G/r^2 muss mit Faktor 2 multipliziert werden.

    So wie’s eben in der Formel steht, die die resultierende Kraft berechnet. Und dann brauche ich nicht mehr Raum auf mysteriöse Weise sich reproduzieren lassen und dadraus irgendwie eine Kraft ausrechnen – oder rumrätseln wie man jetzt Ausdehnung misst.
    Ich habe eine anziehende und eine abstoßende Komponente, und wenn sich beide genau die Waage halten, gibt es keine Kraft; ansonsten hat halt einer “Übergewicht”. Das hatte ich übrigens bei Florian drüben auch genau so schon geschrieben:

    Im Endeffekt kommt es da dann wohl darauf an, ob die Materie innerhalb der Kugel zwischen Milchstraße und Andromeda die dunkle Energie überwiegt – was genau dann der Fall ist, wenn es 1/2 mal soviel dunkle Energie dort gibt, wie Materie. Dann wird die Gravitation durch die Materie genau durch die “Abstoßung” der DE ausgeglichen (-> Einsteins Universum)

  215. #215 DerZimmermann
    4. Oktober 2017

    Das einzige, was da eben noch offen bleibt: Wie passt das Licht da hinein? Offenbar verhält es sich ja genau so wie man es von der Materie im Gummibandbeispiel erwarten würde: Bei konstantem H bleibt es quasi “stehen”. Hingegen wird ein Objekt mit einer festen Geschwindigkeit bei konstantem H zurückgetrieben.

  216. #216 Karl-Heinz
    7. Oktober 2017

    @DerZimmermann

    Das einzige, was da eben noch offen bleibt: Wie passt das Licht da hinein? Offenbar verhält es sich ja genau so wie man es von der Materie im Gummibandbeispiel erwarten würde: Bei konstantem H bleibt es quasi “stehen”. Hingegen wird ein Objekt mit einer festen Geschwindigkeit bei konstantem H zurückgetrieben.

    Wo soll da genau ein Problem sein. Im lokalen Raumgebiet gilt immer, dass die Lichtgeschwindigkeit gleich c ist. Die Wellenlänge wird durch die Ausdehnung des Raumes entsprechen mit dem Skalenfaktor mitskaliert.
    Natürlich kannst du das Ganze ab dem Zeitpunkt nicht mehr begreifen, wenn du folgenden Unsinn zur Maxime erhebst. „Und dann brauche ich nicht mehr Raum auf mysteriöse Weise sich reproduzieren lassen.“

  217. #217 DerZimmermann
    7. Oktober 2017

    Natürlich kannst du das Ganze ab dem Zeitpunkt nicht mehr begreifen, wenn du folgenden Unsinn zur Maxime erhebst. „Und dann brauche ich nicht mehr Raum auf mysteriöse Weise sich reproduzieren lassen.“

    Da steckt genauso viel (oder wenig) Unsinn drin wie in der Vorstellung vom aufgehenden Rosinenkuchen (oder dem Gummiband), die einem vorgaukelt, dass die Materie im Raum völlig passiv ist und mit der Ausdehnung nichts zu tun hat, während sie in Wahrheit die Ursache für das Verhalten der Expansion ist.

    Jeder Vergleich trägt nun mal nur bis zu einem gewissen Punkt. Darum sind oft mehrere verschiedene Vergleiche am besten geeignet, um einen Sachverhalt zu verstehen.

  218. #218 Karl-Heinz
    8. Oktober 2017

    @DerZimmermann
    Natürlich trägt die Materie im Modell zur Kontraktion des Raumes bei. Man sollte aber immer im Hinterkopf behalten, wann bewege ich mich in dem Raum versus mit dem Raum.

  219. #219 DerZimmermann
    8. Oktober 2017

    Tja… gibt es denn diese Unterscheidung? Kann ich (durch ein Experiment) feststellen ob sich ein Objekt mit dem Raum bewegt oder nicht? Anscheinend nicht.

  220. #220 Bernhard Kletzenbauer
    10. Oktober 2017

    @ DerZimmermann
    “Kann ich (durch ein Experiment) feststellen ob sich ein Objekt mit dem Raum bewegt oder nicht?”

    Wenn ich mich mit einem Objekt in einem ansonsten völllg leeren Weltraum befinde, kann ich nicht feststellen, ob ich mich bewege oder das Objekt.
    Aufgrund unserer Erfahrungswerte mit expandierendem Universum, Dopplereffekt und konstanter Lichtgeschwindigkeit können wir aber darauf schließen, daß die kosmologische Rotverschiebung des Lichts daher kommt, daß sich Lichtquelle, Photonen und Beobachter auch mit dem expandierenden Medium bewegen.
    In einem Labor ist von der kosmischen Expansion nichts wahrzunehmen. Wenn hier ein Lichtstrahl einen Dopplereffekt aufweist, dann liegt es daran, daß sich Lichtquelle oder Beobachter selbst im Raum bewegen.
    Durch direkte Beobachtung läßt sich also nicht feststelllen, ob sich etwas mit dem expandierenden Raum bewegt, oder selbst im Raum bewegt. Dazu braucht man einen absolut unbeweglichen Fixpunkt, den es aber nicht gibt.

  221. #221 Karl-Heinz
    11. Oktober 2017

    Kosmologische Kuriositäten Teil 1: Krümmung und Expansion
    https://www.ita.uni-heidelberg.de/research/bartelmann/files/2013SuW..0213…32S.pdf

    Kosmologische Kuriositäten Teil 2: Entfernungsbestimmung und Blick in die Vergangenheit
    https://www.ita.uni-heidelberg.de/research/bartelmann/files/2013SuW..0313…50S.pdf

  222. #222 Karl-Heinz
    11. Oktober 2017

    Sorry der oben genannter Link funktioniert nicht so ganz.
    Da der Dateiname nicht HTML konform ist, am besten diesen Link aufmachen und die Datei 2013SuW..0213…32S.pdf bzw. 2013SuW..0313…50S.pdf
    öffnen.

    https://www.ita.uni-heidelberg.de/research/bartelmann/files

  223. #223 MartinB
    11. Oktober 2017

    @Karl-Heinz
    Danke für die Links, muss ch mir bei Gelegenheit mal in Ruhe anschauen.

  224. #224 Karl-Heinz
    11. Oktober 2017

    @MartinB

    Bitte sehr.
    Dank zurück für deine Blogbeiträge.
    Ich lesen deine Beiträge sehr gerne 🙂

  225. #225 MartinB
    11. Oktober 2017

    @Karl-Heinz
    Ich habe die Texte zwar noch nict gelesen, aber eins der Bilder hat bei mir gerade einen Aha-Effekt ausgelöst, also nochmal danke.

  226. #226 MartinB
    4. November 2017

    @Karl-Heinz & DerZimmermann
    So, gestern habe ich mir mal die Zeit genommen, gründlich nachzudenken. Nach ungefähr 100 falschen Anläufen habe ich es dann eingesehen:
    Ihr habt recht, eine dauerhafte Kraft ziwschen zwei gebundenen Objekten gibt es nur, wenn der Skalenfaktor eine Beschleunigung hat. Solange der Skalenfaktor die Form
    a(t) = a_0 + a-dot t
    hat, gibt es keine Kraft. Am Ende sieht man das ganz leicht:
    Malt euch ein Raum-zeit-Diagramm so wie das in meinem alten Text hier:
    https://scienceblogs.de/hier-wohnen-drachen/2010/09/19/wie-gross-ist-das-beobachtbare-universum/
    (3. Bild im Text)
    Nehmt an, ihr wollt ein nach außen bewegtes Teilchen, das also auf einer der schrägen Linien läuft, einfangen von einem Teilchen aus, das im Ursprung sitzt. Dann müsst ihr es zunächst natürlich beschleunigen, damit seine Weltlinie im Diagramm gerade verläuft. Wie groß diese Beschleunigung ist, das hängt nicht nur vom Ort, sondern auch von der Zeit ab (und das hat mich genau auf die falsche Fährte geführt und das fehlerhafte Argument mit dem Hubble-Parameter verursacht). Aber wenn ihr es getan habt, dann läuft die Weltlinie dieses Teilchens senkrecht.
    Der Winkel zwischen dieser Weltlinie und all den anderen schrägen Linien ändert sich jetzt aber nicht mehr. Relativ zu allen anderen mitbewegten Teilchen gibt es also keine Beschleunigung, also braucht es auch keine Kraft. Einmaliges Anstupsen reicht, dann ist alles kräftefrei.

    (was mich rausgebracht hat war, dass die Kraft, mit der ich anstupsen muss, von der Zeit abhäng t- das ist aber nicht exakt die richtge Fragestellung.)

    Was die Frage angeht, ob ich durch ein Experiment feststellen kann, ob sich etwas relativ zum Raum bewegt: Nur für ein mitbewegtes Objekt ist der Raum homogen und isotrop, für jedes andere nicht. Wenn es also mitbewegte Objekte gibt, kann ich messen, in wie weit der Raum isotrop ist und damit feststellen, ob ein Teilchen in seinem Bezugssystem auch mitbewegt ist. Wenn ich es richtig im Kopf habe, vergeht für mitbewegte Objekte auch die maximale Eigenzeit, theoretisch kann ich also auch einfach die Zeitdilatation gegebüber einer Uhr messen, die mitbewegt ist. In zweifel könnte ich das Universum mit lauter Uhren in unterschiedlichen Bewegungszuständen anfüllen; die, die am langsamsten gehen, sind mitbewegt.

  227. #227 Niels
    5. November 2017

    Hallo, Leute.

    Ich leb noch, zwischenzeitlich war es aber tatsächlich mal recht knapp. Mittlerweile bin ich gesundheitlich allerdings langsam aber sicher wieder auf dem aufsteigenden Ast.

    Mit der ART oder Kosmologie habe und hatte ich noch nie beruflich zu tun, da bin ich auch nur Laie.
    Mittlerweile bin ich da außerdem mit Sicherheit auch deutlich weniger “drin” als noch 2014.
    Also bitte alle Aussagen mit Vorsicht genießen.

    @MartinB

    Ihr habt recht, eine dauerhafte Kraft ziwschen zwei gebundenen Objekten gibt es nur, wenn der Skalenfaktor eine Beschleunigung hat.

    ??
    Ja, aber genau darum dreht sich doch auch dein ganzer Artikel?

    Dort schriebst du doch:

    Wenn sich aber die Rate der Ausdehnung mit der Zeit ändert, dann sieht die Sache anders aus.
    [….]
    Eine Kraft entsteht durch die Expansion des Alls also nur dann, wenn sich die Expansionsrate ändert

    Wenn ich es richtig im Kopf habe, vergeht für mitbewegte Objekte auch die maximale Eigenzeit, theoretisch kann ich also auch einfach die Zeitdilatation gegebüber einer Uhr messen, die mitbewegt ist.

    Ja, das hast du völlig richtig im Kopf.

    In zweifel könnte ich das Universum mit lauter Uhren in unterschiedlichen Bewegungszuständen anfüllen; die, die am langsamsten gehen, sind mitbewegt.

    Das macht man im Prinzip ja auch immer stillschweigend ganz genau so.

    Wenn man davon spricht, dass der Urknall vor 13,7 Milliarden Jahren war, die Sonne 4,5 Milliarden Jahre alt ist, die Materie-Ära vor 4 Milliarden Jahren endete, … bezieht man sich damit immer auf die Zeit, die für einen mitbewegten Beobachter vergangen ist.

  228. #228 Niels
    5. November 2017

    @Alderamin @DerZimmermann @Karl-Heinz

    Ich verstehe euer Problem nicht. (Der Kommentarstrang ist allerdings auch ganz schön lang und verworren.)

    Vielleicht kann ich helfen, wenn ihr noch mal ganz konkret nachfragt?

    .
    Deswegen von mir erst mal nur etwas ganz Allgemeines, vielleicht hilft das ja trotzdem:

    In einem perfekten Friedmann-Universum gibt es keine Kräfte zwischen der Materie.

    Ganz egal, wie der Skalenfaktor aussieht.

    Wenn man ein perfekt isotropes und homogenes Universum betrachtet, sind zwangsläufig alle Teilchen mitbewegte Beobachter.

    Alle Teilchen bewegen sich entlang von Geodäten, fallen also frei. (Das ist die Definition von Geodäten).
    Damit wirken auf sie auch grundsätzlich keine Kräfte. (Das ist nämlich die Definition von frei fallend.)

    Dabei spielt es überhaupt keine Rolle, ob die Expansion beschleunigt ist oder nicht.
    (Klingt komisch, aber genau das kommt heraus, wenn man für ein perfekt homogenes und isotropes Universum die einsteinschen Feldgleichungen löst.)

    In einem echten Friedmann-Universum gibt es also keine Verklumpung, es gibt keine Sterne, keine Galaxien, kein gar nichts. Nur einzelne Teilchen, die sich durch die Expansion kräftefrei voneinander entfernen.

    Deswegen leben wir offensichtlich nicht in einem echten Friedmann-Universum, sonst würden wir nämlich gar nicht existieren.
    (Das liegt letztlich daran, dass es Quanteneffekte, insbesondere Vakuum-Fluktuationen, gibt. Die berücksichtigt die ART nicht.)

    Das Universum kann also nur näherungsweise ein Friedmann-Universum sein, die Robertson–Walker-Metrik gilt nur auf großen Skalen.

    In unserem lokalen, kleinen, durch die Gravitation gebundenen Klumpen liegt also keine Robertson-Walker-Raumzeit vor, sondern etwas unglaublich viel Komplizierteres.
    Dieser Klumpen inhomogener Materie beeinflusst schließlich die Robertson-Walker-“Hintergrundraumzeit”, und zwar nichtlinear, weil die Diff-Gleichungen der ART nichtlinear sind.

    Wählt man die Skala so groß, dass unser Klumpen gut durch ein Punktteilchen beschrieben wird, können wir wieder von einer reinen Friedmann-Raumzeit ausgehen. Deswegen können wir in der Regel eben doch die Friedmann-Gleichungen verwenden, weil bei der Kosmologie eigentlich immer mit gewaltigen Entfernungen arbeiten.

    Dieses Punktteilchen, das unseren ganzen Klumpen beschreibt, ist dann auf dieser Skala wieder einfach ein mitbewegter Beobachter, folgt also einer Geodäten und ist damit kräftefrei.

    Wenn wir jetzt wissen wollen, was in unserem Klumpen passiert, hilft uns eine Friedmann-Raumzeit also nur noch sehr begrenzt weiter.

    Am einfachsten macht man es sich, wenn man so tut, es liege auch hier weiter eine Friedmann-Raumzeit vor und die Materie würde über das Newtonsche Gravitationsgesetz wechselwirken.

    Das führt dann zu (hier zitiere ich mich selbst aus #3:

    Die Kraft ist proportional zu a‘‘ (t)/a(t), also zur zweiten Ableitung des Skalenfaktors geteilt durch den Skalenfaktor.

    Das kann man durch Herumstümpern mit Newton erstaunlich schnell erkennen.
    Man verwendet die lineare Hubble-Beziehung v = H * D.
    (v ist die “Rezessionsgeschwindigkeit”, H der Hubble-Parameter und D die Entfernung.)
    Das benennt man um zu
    R‘ (t) = H(t) * R(t)
    (R(t) ist der Abstand, R ‘(t) seine erste Ableitung.)
    Ableiten von R‘(t) ergibt die Beschleunigung
    R‘‘ (t) = H‘ (t) * R(t) + H(t) * R‘ (t)
    Wenn man jetzt die Definition des Hubble-Parameters H = a‘ (t)/a(t) einsetzt, bekommt man als Kraft, die durch die Expansion auf Körper einwirkt,
    F = m * R‘‘ (t) = m*R(t) *[a‘‘(t)/a(t)].
    (a(t) ist der Skalenfaktor)
    Wenn man das dann als äußere Störung ins newtonsche Zwei-Körper-Problem einsetzt und weiter vereinfacht, kommt man laut “Influence of global cosmological expansion on local dynamics and kinematics” auf folgendes Beispielergebnis:
    As an example, the deviation in the radius for an hypothetical spacecraft orbiting around the Sun at 100 AU would be just of the order of 1 mm.

    Natürlich ist das aber ein völlig illegitimer Ansatz.
    Es mag ja sein, dass das von der Größenordnung her hinkommen könnte.
    Ob allerdings die [a‘‘(t)/a(t)]-Gesetzmäßigkeit auch bei rein allgemein relativistischer Rechnung hinkommt ist eine ganz andere Frage.

    Eine rein relativistische Rechnung ist allerdings wahnsinnig schwierig, weil man nicht wirklich weiß, wie man auch nur halbwegs allgemein inhomogene Materieverteilungen im expandieren Universum im Rahmen der ART beschreibt.
    Leider ist es das aber eben die einzige sinnvolle Methode. Man muss die richtige Raumzeit finden können und dann knall hart die Gleichung der geodätische Abweichung lösen.

    Dafür gibt es viele verschiedene Abschätzungen, Ansätze und Vereinfachungen.
    Falls vorhanden, lässt man auch noch seinen Supercomputer ein paar Tage rennen.

    Man ist sich seit einem oder mittlerweile auch zwei Jahrzehnten mehr oder weniger einig, dass sich auch allgemein relativistisch wahrscheinlich eine [a‘‘(t)/a(t)]-Proportionalität müsste.
    (Ich hab mich seit 2014 aber überhaupt nicht mehr damit beschäftigt, da kann sich also schon etwas geändert haben.)

    100% gesichertes Wissen, dass sich bestimmt nicht mehr ändern wird, ist diese [a‘‘(t)/a(t)]-Proportionalität damit aber nicht.

  229. #229 Karl-Heinz
    5. November 2017

    @Niels

    Ich leb noch, zwischenzeitlich war es aber tatsächlich mal recht knapp. Mittlerweile bin ich gesundheitlich allerdings langsam aber sicher wieder auf dem aufsteigenden Ast.

    Es freut mich (uns), dass es dir wieder besser geht.

    Eine Kraft entsteht durch die Expansion des Alls also nur dann, wenn sich die Expansionsrate ändert

    Puh …
    Kann man, dass nicht ein bischen genauer definieren, was unter Expansionsrate gemeint ist. Ich verstehe darunter a'(t)/a(t) andere wiederum nur a'(t).

    Meinetwegen so:
    Expansionsrate a'(t)
    relative Expansionsrate a'(t)/a(t)

    Das man halt, dann bei der Diskussion auch die richtigen Begriffe verwendet. 😉

  230. #230 Karl-Heinz
    5. November 2017

    @Niels

    Es gilt auch folgende Beziehung
    a”(t)/a(t) = (a'(t)/a(t))^2 + (a'(t)/a(t))’

    ohne t, damit es leichter zum lesen ist.
    a”/a = (a’/a)^2 + (a’/a)’

    oder mit H(t) = a'(t)/a(t)
    a”/a = H^2 + H’

    https://de.m.wikipedia.org/wiki/Friedmann-Gleichung

    Ich hoffe, du kannst jetzt erkennen, warum ich darauf poche zwischen Expansionsrate und relative Expansionsrate zu unterscheiden.

  231. #231 Karl-Heinz
    5. November 2017

    @Niels

    Eine Kraft entsteht durch die Expansion des Alls also nur dann, wenn sich die Expansionsrate ändert

    Egal ob ich bei diesem Satz unter Expansionsrate die Expansionsrate a'(t) oder die relative Expansionsrate a'(t)/a(t) verstehe, der Satz ist, ich hoffe du gibst mir recht, falsch.

    Denn es gilt: a”/a = (a’/a)^2 + (a’/a)’

  232. #232 Karl-Heinz
    5. November 2017

    @
    Das hatte ich vergessen zu erwähnen.

    Die Beziehung
    a”/a = (a’/a)^2 + (a’/a)’
    kann man sich natürlich rein mathematisch herleiten. Ich persönlich finde die Formel gar nicht so schlecht, da man schon im Vorfeld überlegen kann, ob eine scheinbare Kraft auftritt oder nicht. Funktioniert aber nur wenn man mit der relative Expansionsrate handiert.

  233. #233 MartinB
    5. November 2017

    @Niels
    Das mit der Gesundheit klingt ja nicht so gut, ich hoffe, es geht wieder bergauf.

    Der Punkt in der Diskussion war, dass ich mich zwischendurch selbst verwirrt hatte und gedacht hatte, dass es eine Kraft nur dann nicht gibt, wenn der Hubble-Parameter konstant ist, weil dann die dehnrate zwischen zwei Punkten in fester Entfernung konstant ist. Bei dem Argument habe ich aber die relativgeschwindigkeit zwischen dem nicht mitbewegten Objekt und den mitbewegten Beobachtern nicht richtig berücksichtigt, das wurde mir jetzt klar (und man sieht es an dem einfache RZD sofort).

    @Karl-Heinz
    Nach der Logik meines Arguments gibt es keine Kraft genau dann, wenn a'(t)=konst ist, nur dann sind die Weltlinien mitbewegter Beobachter Geraden.

  234. #234 Karl-Heinz
    5. November 2017

    @MartinB

    Nach der Logik meines Arguments gibt es keine Kraft genau dann, wenn a'(t)=konst ist, nur dann sind die Weltlinien mitbewegter Beobachter Geraden.

    Ja natürlich.
    Wenn a'(t)=konst ist folgt auch, dass a”(t)=0
    und damit auch a”/a=0.

  235. #235 MartinB
    5. November 2017

    @Karl-Heinz
    Verwirrt-bin. Das war doch deine Frage am Ende von #229, die ich damit beantwortet habe?

  236. #236 Karl-Heinz
    5. November 2017

    @MartinB
    Ich wollte eigentlich nur sagen, dass es jetzt passt.

    Obwohl ich mich nicht verkneifen kann zu fragen: Wenn a'(t)=konst sei, welche Funktion hätte dann der Hubble-Parameter? 😉

  237. #237 MartinB
    5. November 2017

    @Karl-Heinz
    Was meinst du? Wenn a’=const, dann nimmt der Hubble-Parameter immer weiter ab.
    Deswegen mussten die leute beim steady-state-Modell auch einen exponentiellen Skalenfaktor haben, um einen konstanten Hubble-Parameter zu bekommen.

  238. #238 Karl-Heinz
    5. November 2017

    @MartinB
    In etwa so.

    a(t)= 1+k * t
    k = konstant
    a(t)’= k = konstant

    H(t)= a(t)’/a(t) = k/(1+k * t)
    Für sehr grosse t wird dann H gleich 0.

  239. #239 MartinB
    5. November 2017

    @Karl-Heinz
    Ja, so ist es. Sieht man auch im Diagramm: Je weiter man nach oben zu großen Zeiten geht, desto steiler verlaufne die Weltlinien an einem bestimmten Punkt, d.h. desto geringer ist dort die Geschwindigkeit.

  240. #240 Karl-Heinz
    5. November 2017

    @MartinB
    Coole Zeichnung
    Danke für deinen Beitrag.

    @Niels
    Gute Besserung, toi, toi, toi

  241. #241 Niels
    5. November 2017

    @MartinB

    Das mit der Gesundheit klingt ja nicht so gut, ich hoffe, es geht wieder bergauf.

    Danke.
    Es ist schon im Bergauf gehen und die Prognose sieht auch nicht schlecht aus.

    Der Punkt in der Diskussion war, dass ich mich zwischendurch selbst verwirrt hatte

    Mich hat eure ganze Diskussion von vorne bis hinten völlig verwirrt. 😉
    .

    @Karl-Heinz

    Gute Besserung, toi, toi, toi

    Danke.

    Es gilt auch folgende Beziehung
    a”(t)/a(t) = (a'(t)/a(t))^2 + (a'(t)/a(t))’

    Klar, das ist ja einfach die zweite Friedmanngleichung?
    Nennt man im Englischen auch Friedmann acceleration equation.

    Kann man, dass nicht ein bischen genauer definieren, was unter Expansionsrate gemeint ist.

    Ich glaube nicht, dass es hier eine allgemein anerkannte Definition gibt.

    Am ehesten würden Kosmologen so wohl den Hubble-Parameter nennen, also
    H(t) = a'(t)/a(t).
    Die Änderung der Expansionsrate ist dann
    H’ = a”/a – (a’/a)^2.

    Dann gibt es auch noch den deceleration parameter, der als
    q = – a”a/(a’^2)
    definiert ist.
    (Hat historische Gründe, heute würde man es acceleration parameter nennen und das Minuszeichen weglassen.)

    Ich persönlich kann damit aber echt wenig anfangen, ich kann weder mit H’ noch mit q denken.

    Letztlich geht es aber eigentlich nur darum, genau anzugeben, welche Koordinaten bzw. welches Entfernungsmaß man gerade verwenden.
    Bei Friedmann-Universen sind die beiden gebräuchlichsten Angaben für Strecken die mitbewegte Entfernung (comoving distance) and die proper distance.
    Dabei gilt einfach
    d_com = d_proper / a , .

    Ich verstehe a'(t) und a'(t)/a(t) damit einfach als genau den selben Parameter, nämlich den Hubble-Parameter / die Expansionsrate.
    Nur eben in zwei verschiedenen Entfernungsmaßstäben ausgedrückt.

    Kann man aber natürlich auch gerne Expansionsrate und relative Expansionsrate nennen.
    Das ist ja letztlich nur eine Kurzbezeichung für genau den Unterschied, den man mit der comoving und proper distance machen will.

    Ich hoffe, du kannst jetzt erkennen, warum ich darauf poche zwischen Expansionsrate und relative Expansionsrate zu unterscheiden.

    Ehrlich gesagt nur noch vom Prinzipellen her.
    Wenn man in der Materie ein bisschen drin ist, ist es eigentlich immer völlig offensichtlich, ob gerade comoving distance oder proper distance verwendet wird, auch ohne dass das ausdrücklich angegeben werden müsste.
    In kosmologischen Papers wird das deswegen auch notorisch schlampig gehandhabt.

    Schlechter wissenschaftlicher Stil ist das aber natürlich trotzdem und sollte auf jeden Fall vermieden werden, da hast du absolut recht.
    Von anderen Bereichen, bei denen ich mich nur flüchtig auskenne, weiß ich auch, wie absolut ärgerlich so eine Praxis sein kann. Und wenn jemand seine Begriffe nicht sauber definiert, ist das immer das erste, was ich ihm um die Ohren haue.
    Tut mir leid, dass ich das hier auch nicht besser gehandhabt habe.

    Ich persönlich finde die Formel gar nicht so schlecht, da man schon im Vorfeld überlegen kann, ob eine scheinbare Kraft auftritt oder nicht. Funktioniert aber nur wenn man mit der relative Expansionsrate handiert.

    Ich verstehe nicht ganz, was du hier meinst.

    Wie gesagt, im perfekten Friedmann-Universum gibt es keine Kräfte zwischen der Materie.
    Ganz egal, wie irgendeine Formel genau aussieht.

    Darüber hinaus ergibt der Begriff der Scheinkraft außerhalb der newtonschen Physik keinen Sinn mehr.
    Oder man muss alternativ man auch die Gravitation als Scheinkraft bezeichnen usw.
    Eigentlich ist der Begriff der Scheinkraft in der ART aber eher weniger nützlich und wird daher nicht mehr verwendet.

    Warum es für die Frage nach der Existenz real wirkende Kräfte eine Rolle spielen sollte, in welchem Entfernungsmaßstab man sich das Ganze anschaut (Funktioniert aber nur wenn man mit der relative Expansionsrate handiert.), ist mir grundsätzlich nicht klar, sorry.

    Eine Kraft entsteht durch die Expansion des Alls also nur dann, wenn sich die Expansionsrate ändert

    Egal ob ich bei diesem Satz unter Expansionsrate die Expansionsrate a'(t) oder die relative Expansionsrate a'(t)/a(t) verstehe, der Satz ist, ich hoffe du gibst mir recht, falsch.

    Denn es gilt: a”/a = (a’/a)^2 + (a’/a)’

    Verstehe ich nicht, sorry.

    Änderung der Rate = zweite Ableitung, oder?

    Und wenn man auf der linken Seite a” = 0 einsetzt, muss sich
    a”/a = (a’/a)^2 + (a’/a)’ = 0
    ergeben, oder?
    Oder stehe ich gerade auf dem Schlauch?

    Wenn a'(t)=konst sei, welche Funktion hätte dann der Hubble-Parameter?

    Na ja, H (in proper distance) geht dann offensichtlich gegen Null.

    In unserem Universum mit beschleunigter Expansion macht sich diese übrigens dadurch bemerkbar, dass der Hubble-Parameter gegen einen Grenzwert abnimmt, nämlich ungefähr H(end) = 60 km/(s*Mpc).
    Das liegt daran, dass die Expansion rein exponentiell erfolgt, sobald die dunkle Energie völlig überwiegt.
    Die Ableitung einer Exponentialfunktion ist natürlich wieder eine Exponentialfunktion.

    Das Ganze als Schaubild:
    Zeitliche Entwicklung des Hubble-Parameters H(t)
    (Auf der x-Achse ist die Zeit in Milliarden Jahren aufgetragen.)

    Ganz allgemein gilt:
    Der Hubble-Paramter ist definiert als H (t) = a’(t)/a(t).
    Solange der Nenner schneller zunimmt als der Zähler nimmt der Hubble-Parameter ab.
    Wenn Zähler und Nenner prozentual gleichschnell zunehmen, bleibt der Hubble-Parameter konstant.
    a(t) ist eine Art Maß für die Größe des Universums. a’(t) ist eine Art Maß, wie schnell sich die Größe des Universums verändert.

    Mathematisch ist doch mit H (t) = a’(t)/a(t) folgendes möglich:

    a) a(t) wächst langsamer an als a’(t)
    Zum Beispiel expandiert das Universum vom Zeitpunkt A bis zum Zeitpunkt B auf den dreifachen Radius.
    Die Rate, mit der sich der Abstand zwischen zwei Galaxien durch die Expansion ändert, ist am Zeitpunkt B viermal so groß wie zum Zeitpunkt A.
    Die “Geschwindigkeit” a’(t) ist also jetzt viermal so groß.
    H (A) ist der Hubble-Parameter zum Zeitpunkt A, H(B) der Hubble-Parameter zum späteren Zeitpunkt B.
    Offensichtlich ergibt sich H(B) = a’(B)/a(B) = [4*a'(B)]/[3*a(B)] = 4/3 * H(A)
    Also hat der Hubble-Parameter mit der Zeit zugenommen.

    b) a(t) wächst schneller an als a’(t)
    Zum Beispiel expandiert das Universum vom Zeitpunkt A bis zum Zeitpunkt B auf den vierfachen Radius.
    Die Rate, mit der sich der Abstand zwischen zwei Galaxien durch die Expansion ändert, ist am Zeitpunkt B dreimal so groß wie zum Zeitpunkt A.
    Die “Geschwindigkeit” ist also jetzt dreimal so groß.
    H (A) ist der Hubble-Parameter zum Zeitpunkt A, H(B) der Hubble-Parameter zum Zeitpunkt B.
    Offensichtlich ergibt sich H(B) = a’(B)/a(B) = [3*a'(B)]/[4*a(B)] = 3/4 * H(A)
    Also hat der Hubble-Parameter mit der Zeit abgenommen.
    Obwohl die Größe des Universums zugenommen hat und obwohl es jetzt schneller expandiert, als vorher.

    c)
    a(t) und a’(t) wachsen gleichschnell.
    Also beide beispielsweise um das dreifache.
    H(B) = H(A)

    In einem beliebigen beschleunigt expandierenden Universum sind alle drei Beispiele möglich. Da muss sich nix zusammenziehen.
    Wenn wir uns unser spezielles Universum anschauen, ist es aber (zufällig ?) so, dass früher und heute b) zutrifft, irgendwann in Zukunft gilt aber für alle Ewigkeit c).
    Zumindest nach den momentanen kosmologischen Modellen.

  242. #242 MartinB
    5. November 2017

    @Niels
    Danke für die Ausführungen (und weiter alles Gute).

    Das hier hat mich überrascht/verwirrt:
    “Eigentlich ist der Begriff der Scheinkraft in der ART aber eher weniger nützlich und wird daher nicht mehr verwendet.”
    Ist nicht ein Witz der ART genau der, dass die Schwerkraft eine Scheinkraft ist, die nur dadurch zu Stande kommt, dass ich z.B. auf der Erdoberfläche stehend ein in Wahrheit beschleunigtes Bezugssystem verwende? Genau so funktioniert doch das Äq-Prinzip: Im beschleunigten Fahrstuhl gibt es eine Scheinkraft, die die Sachen nach unten fallen lässt, und die Situation ist genau äquivalent zu der auf der Erdoberfläche, wo ein scheinbar ruhender Beobachter in Wahrheit permanent von der Geodäten wegbeschleunigt wird.

  243. #243 MartinB
    5. November 2017

    PS
    Was Karl-Heinz meint steckt in deinen Formeln drin:
    Am ehesten würden Kosmologen so wohl den Hubble-Parameter nennen, also
    H(t) = a'(t)/a(t).
    Die Änderung der Expansionsrate ist dann
    H’ = a”/a – (a’/a)^2.
    Danach ist H’ auch dann ungleich Null, wenn a”=0 ist. Das war ja letztlich genau meine Verwirrung. Kräftefrei für gebundene Materie ist eine Expansion nach meinem verständnis nur bei a”=0, aber dann ist H’ nicht Null. Für H’=0 muss ja a(t) ne Exponentialfunktion sein.

  244. #244 Niels
    5. November 2017

    @MartinB @Karl-Heinz

    Ist nicht ein Witz der ART genau der, dass die Schwerkraft eine Scheinkraft ist, die nur dadurch zu Stande kommt[…]

    Doch, da hast du völlig recht. Ich habe mich blöd ausgedrückt und zu kurz gefasst.

    Ich wollte darauf hinaus, dass es in der ART nicht hilfreich ist, zwischen Gravitations”kraft” als “echter Kraft” und zwischen Trägheitskräften wie etwa der Zentrifugalkraft zu unterscheiden.
    (Ich hoffe, das passt so. Ich hab diese Begriffe nicht mehr richtig drauf. Wie nennt man eine Kraft, die keine Scheinkraft ist, richtig?)

    Wie von dir schon zusammengefasst ist das in der ART keine sinnvolle Unterscheidung mehr ist.

    Und da man in der ART mit differentialgeometrischen Begriffen wie Geodäten und geodätischer Abweichung argumentiert, wird der Begriff Scheinkraft auch nie verwendet.

    Zumindest nicht in den Papern und der weiterführenden Literatur, mit denen ich mit zuletzt fast ausschließlich befasst habe.
    Außer eben bei so total unwichtigen Dingen wie dem Äq-Prinzip und Einsteins gesamter Grundmotivation für die ART, was ich dämlicherweise komplett übersehen hatte.
    [Aber wer braucht schon Wald, wenn er Bäume hat und wer das Äq-Prinzip, wenn man doch auch mit Dingen wie Tangentialbündeln argumentieren kann. ;-)]

    Danach ist H’ auch dann ungleich Null, wenn a”=0 ist. Das war ja letztlich genau meine Verwirrung. Kräftefrei für gebundene Materie ist eine Expansion nach meinem verständnis nur bei a”=0, aber dann ist H’ nicht Null.

    Oh, danke schön.
    Das ist mir tatsächlich noch nie bewusst aufgefallen, obwohl es völlig offensichtlich sein sollte.

    Sorry, MartinB und vor allem Karl-Heinz. Damit wird der Unterschied zwischen deiner “Expansionsrate” und deiner “relativen Expansionsrate” dann natürlich doch absolut entscheidend.
    Tut mir leid, dass ich das überhaupt nicht kapiert habe und damit ist eine Menge des von mir Geschriebenen leider völlig Thema verfehlt…

    Hm, muss mal in Ruhe nachdenken, ob mir dazu etwas Clevereres einfällt als das, was MartinB schon dazu bemerkt hat.
    Im Moment rührt sich leider nichts.

  245. #245 MartinB
    5. November 2017

    @Niels
    Dann sind wir uns ja einig.

    Vielleicht kann man das auch so ausdrücken:
    Bei Newton gilt erstmal immer Newton II: Wo Beschleunigung, da Kraft. Und dann merkt man, dass das in bösen nicht-Inertialsystemen nicht klappt und führt das Konzept “Scheinkraft” ein, damit es dann doch wieder gilt.

    Bei Einstein unterscheidet man nicht mehr zwischen gutem und bösen Koordinatensystemen und sagt immer
    Beschleunigung =Kraft + Koordinatenbeschleunigung
    (wobei die Koordinatenbeschleunigung gleich -Gamma^i_jk v^jv^k ist)
    und es gibt halt manchmal geschickt gewählte Koordinatensysteme, in denen die Koordinatenbeschleunigung Null ist, aber das ist nichts, worüber man besonders nachdenken sollte.

    Und deine Ausführungen zu Hubble etc. waren zwar vielleicht knapp an der Frage vorbei, aber trotzdem auf jeden Fall hilfreich für mich.

  246. #246 Niels
    5. November 2017

    @MartinB

    Vielleicht kann man das auch so ausdrücken […]

    Prima. Gefällt mir sehr gut.

    Und deine Ausführungen zu Hubble etc. waren zwar vielleicht knapp an der Frage vorbei, aber trotzdem auf jeden Fall hilfreich für mich.

    Gern geschehen.
    Wobei du eine Kurvendiskussion von a’/a aber bestimmt wahnsinnig nötig hattest.

  247. #247 MartinB
    6. November 2017

    @Niels
    “Prima. Gefällt mir sehr gut.”
    Dann werde ich das mal nachher sauber aufschreiben für…
    “Wobei du eine Kurvendiskussion von a’/a aber bestimmt wahnsinnig nötig hattest.”
    Auch wenn ich das wahrscheinlich selbst ausrechnen könnte – ist immer gut zu sehen, wie du so etwas systematisch machst und die Bestätigung zu haben, dass das auch so geht; manchmal übersieht man bei solchen Rechnungen ja irgendwas wie ich bei meinen Argument mit dem Hubble-Parameter hier in der Diskussion.

  248. #248 Alderamin
    6. November 2017

    @Niels

    Gute Genesung auch von mir, ist ja schlimm zu hören.

    Und danke für die tolle Zusammenfassung. Eine dumme Frage habe ich aber noch:

    Wie isses denn in unserem Universum, ist a” da = 0? Doch eher nicht, oder? Die Expansion beschleunigt sich doch. Und wenn die dunkle Energie nicht wäre, nähme sie ab (tat sie ja in den ersten 7 Milliarden Jahren). Quintessenz also, es gibt doch eine kleine Kraft durch die Expansion. Oder?

  249. #249 Karl-Heinz
    6. November 2017

    @Alderamin

    Niels wird mich hauen, wenn ich den Begriff scheinbar verwende.
    Aber was solls.

    Ich sage mal aus dem Stegreif:
    H … Hubble-Parameter
    D … Abstand
    v … scheinbare Geschwindigkeit
    a’/a = scheinbare Beschleunigung

    v = H*D
    a’/a = H^2 * D (H’ kann man vernachlässigen)

    Bin schon gespannt, was Niels antwortet. 😉

  250. #250 Karl-Heinz
    6. November 2017

    myself

    sorry ich meinte
    a’’/a = scheinbare Beschleunigung
    a’’/a = H^2 * D (H’ kann man vernachlässigen)

  251. #251 Karl-Heinz
    6. November 2017

    myself
    Sorry so ein Mist

    sorry ich meinte
    a’’/a * D = scheinbare Beschleunigung
    a’’/a * D = H^2 * D (H’ kann man vernachlässigen)

    Ist mir jetzt aber peinlich.

  252. #252 MartinB
    6. November 2017

    @Alderamin
    Nee, bei uns ist a” nicht Null, im beschleunigt expandierenden Universum ist es größer als Null.
    “Quintessenz also, es gibt doch eine kleine Kraft durch die Expansion. Oder?”
    Ja, es gibt eine kleine Kraft z.B. auf Materie, die wird in verschiedenen papern und Büchern auch vorgerechnet. Solange die Expansion aber nicht massiv beschleunigt (Stichwort “big rip”) ist das vernachlässigbar.

  253. #253 Karl-Heinz
    6. November 2017

    @Alderamin

    Der unten stehender Text stammt von
    Welt der Wissenschaft Kosmologische Kuriositäten
    Krümmung und Expansion

    Kollaps einer Kugel
    Dass sich das Universum ausdehnt, bedeutet aber keineswegs, dass sich Körper wie die Erde oder Galaxien mit ihm ausdehnen. Der Grund dafür liegt in der Entstehung von Strukturen, die durch ihre Schwerkraft zusammengehalten werden und für die unsere Galaxie ein Beispiel ist. Im sehr jungen Universum gab es jedoch noch keine Strukturen, die durch ihre eigene Schwerkraft zusammengehalten wurden. Stattdessen erfüllte den Raum eine nahezu homogen verteilte Mischung aus dunkler und aus uns bekannter gewöhnlicher Materie. Diese Mischung hatte überall fast dieselbe mittlere Dichte. Jedoch waren schon Gebiete angelegt, in denen die Dichte minimal erhöht war, und andere, in denen sie geringfügig niedriger war. Stellen wir uns ein solches leicht überdichtes Gebiet im frühen Universum als eine gleichmäßig mit Gas und dunkler Materie gefüllte Kugel vor. Solche gedachten Kugeln werden Halos genannt. Während die Dichteparameter des Universums die Anfangsbedingungen für seine Ausdehnung setzen, sind die Anfangsbedingungen für die Entwicklung des Halos durch seine eigenen Dichteparameter gegeben. Die Dichte unseres Universums ist nach heutigem Kenntnisstand zu klein, als dass es wieder kollabieren könnte. Würde sie aber eine bestimmte, kritische Dichte überschreiten, könnte in ferner Zukunft ein Kollaps eintreten. Der betrachtete Halo ist jedoch bereits etwas dichter als seine Umgebung. Nach den Friedmann-Gleichungen, die im Halo ebenso wie außerhalb gelten, dehnt er sich ein wenig langsamer aus. Sein Volumen wächst deswegen langsamer als das eines vergleichbaren Ausschnitts aus dem umgebenden Universum. Dadurch verstärkt sich der Unterschied zwischen den Dichteparametern im Halo und außerhalb. Seine Ausdehnung verlangsamt sich daher zusehends. Falls die anfängliche Überdichte im Halo groß genug war, kommt seine Ausdehnung irgendwann zum Stillstand und kehrt sich in einen Kollaps um. Man sagt, dass sich der Halo zu diesem Zeitpunkt von der kosmischen Ausdehnung abkoppelt. Betrachten wir dieses Argument noch einmal von seinem Ende her: Der sich erst ausdehnende, später kollabierende Halo mag am Ende eine Galaxie oder eine andere kosmische Struktur enthalten. Anfänglich expandierte der Halo zwar mit einer ähnlichen Rate wie das umgebende Universum auch. Damit im weiteren Verlauf aber etwa eine Galaxie aus ihm werden kann, muss er zu einem stabilen Gebilde kollabieren. Daher wissen wir, dass er sich von der Ausdehnung des Universums unabhängig gemacht haben muss. Der Halo kann als ein Ausschnitt des Univer­sums betrachtet werden, in dem dieselbe Friedmann-Gleichung auf Grund leicht erhöhter Dichteparameter zu einem anderen Ausdehnungsverhalten geführt hat. Heute nimmt sein Inneres nicht mehr an der Expansion Teil.

  254. #254 MartinB
    6. November 2017

    @Karl-Heinz
    Das ist natürlich wieder ein etwas anderer Fall – ich habe hier ja explizit Dinge angeguckt, die nicht durch die Raumzeitkrümmung zusammengehalten werden.
    Für das Sonnensystem gelten ähnliche Überlegungen, das steht schon im oben verlinkten Text von Baez:
    https://math.ucr.edu/home/baez/physics/Relativity/GR/expanding_universe.html

    PS: Generel bitte möglichst keine langen Texte einfach von irgendwo rüberkopieren wegen Copyright; ich lasse es hier mal als Zitat stehen, in der Annahme, dass die kolleginnen von SdW da kein Problem haben…

  255. #255 Niels
    7. November 2017

    @Alderamin

    Gute Genesung auch von mir

    Vielen Dank.

    Wie isses denn in unserem Universum, ist a” da = 0?

    Natürlich nicht.
    Die Definition von beschleunigt ist a” > 0.

    Oder komplizierter:
    Wenn in der Physik etwas positiv oder negativ beschleunigt stattfindet, bedeutet das, dass man dessen zeitliche Entwicklung mit Hilfe einer Exponentialfunktion beschreiben kann.
    Die Ableitung einer Exponentialfunktion ist natürlich wieder eine Exponentialfunktion. Die Ableitung dieser Ableitung ebenfalls, usw.

    Und wenn die dunkle Energie nicht wäre, nähme sie ab (tat sie ja in den ersten 7 Milliarden Jahren).

    Die Expansion? Die Änderungsrate der Expansion?
    .

    Das ist dann die Sache mit kritischen Dichte.

    Materiedichte wirkt abbremsend (negativ beschleunigend),
    Dunkle Energie wirkt (positiv) beschleunigend.

    Mit der Expansion verteilt sich die Materie auf ein immer größeres Volumen, die Materiedichte wird also immer kleiner. Die Dichte der dunklen Energie bleibt aber gleich, soweit wir momentan wissen.
    Deswegen überwiegt im späten Universum die beschleunigende Wirkung der dunklen Materie, das Universum expandiert beschleunigt.

    Universen ohne dunkle Energie aber mit Materie sind zu Anfang immer in der Expansion stark von ihrer Materie gebremst, a” ist also negativ.

    Anschließend kommt es auf das Verhältnis von Materiedichte zur berühmten kritischen Materiedichte an.

    [[Nicht verwechseln mit der sogenannten kritischen Dichte.
    Wenn man die Summe aus dunkler Energiedichte + Materiedichte mit dieser kritischen Dichte vergleicht, lassen sich Aussagen über die globale Geometrie des Universums ableiten.
    Ist es offen, geschlossen, begrenzt, gekrümmt…?

    Bevor die beschleunigte Expansion des Universums entdeckt wurde, hieß die kritische Materiedichte einfach nur kritische Dichte.
    Leider wird das heutzutage oft immer noch so gehandhabt. Welche der beiden unterschiedlichen kritischen Dichten jeweils gemeint ist, darf man sich dann aus dem Kontext erarbeiten.]]

    .

    Universen ohne dunkle Energie kollabieren, falls ihre Massendichte oberhalb der kritischen Dichte liegt (a” also kleiner 0), die Expansion läuft mit der Zeit gegen Null, wenn es exakt die kritische Dichte hat und sie geht gegen einen Grenzwert größer Null, falls die Materiedichte kleiner als die kritische Dichte ist. (a’ läuft also gegen einen konstanten Wert und damit dann a” sehr schnell gegen Null.)
    .
    .

    Zur Veranschaulichung dieses Bildchen der Zeitentwicklung des Skalenfaktors verschiedener Universen:

    [[Zum Diagramm:
    Der Urknall war unterschiedliche lange Zeit vor “Now”, d.h. die jeweiligen Universen sind zum Zeitpunkt “Now” unterschiedlich alt. Zeit vor dem Urknall gibt es nicht. Beispielsweise beschriebt die grüne Kurve ein Universum mit einem Alter von 10 Mrd. Jahren zum Zeitpunkt “Now”.
    Diese Darstellung kommt daher, dass der Ersteller der Grafik wollte, dass die “Relative size of the universe” (also der Wert des Skalenfaktors a beim Zeitpunkt “Now”) gerade für alle Universen exakt 1 ist. ]]

    .
    .

    Nochmal in Kurzform:

    Es gibt drei Möglichkeiten für Universen ohne dunkle Energie (In der Grafik Ωv=0)

    1) Big Crunch, orange (Kollaps des Universums).
    Materiedichte > “kritische Materiedichte”
    => a nimmt zu dann wieder ab
    => a” kleiner 0, negative Beschleunigung (Abbremsung).

    2) Big Freeze, blau (ewige konstante Expansion).
    Materiedichte kleiner “kritische Materiedichte”
    => a nimmt zu
    => a’ nimmt ab gegen = konst (> 0)
    => a” erst kleiner 0, dann = 0.

    3) Big Freeze, grün (Expansion läuft gegen Null).
    Materiedichte = “kritische Materiedichte”
    => a nimmt zu aber läuft sehr langsam gegen a = konst
    => a’ nimmt ab gegen = 0
    => a” erst kleiner 0, dann = 0.

  256. #256 Niels
    7. November 2017

    Und wenn die dunkle Energie nicht wäre, nähme sie ab (tat sie ja in den ersten 7 Milliarden Jahren).

    a’ nimmt ab.
    a” kleiner Null.

    Hier mal Schaubilder für unser Universum (also mit dunkler Materie), zu beachten ist beim Universumsalter von 7 Milliarden Jahren der Umkehrpunkt von a’ bzw. der Vorzeichenwechsel von a”.

    Zeitliche Entwicklung der Änderung des Skalenfaktors (“Geschwindigkeit”), also von also von a’(t), bis 20 Milliarden Jahre nach dem Urknall.

    Zeitliche Entwicklung der zweiten Ableitung des Skalenfaktors (Beschleunigung, also von a’’(t), bis 15 Milliarden Jahre nach dem Urknall.

    Quintessenz also, es gibt doch eine kleine Kraft durch die Expansion. Oder?

    Ziemlich wahrscheinlich schon. Aber klitzeklitzeklein.
    bzw.
    This requires further study.
    bzw.
    However, the general problem of gaining a qualitative and quantitative understanding of how the cosmological dynamics influences local systems remains challenging, with only partial clues being so far provided by exact solutions to the field equations of General Relativity.
    .

    @MartinB

    Ja, es gibt eine kleine Kraft […]
    Solange die Expansion aber nicht massiv beschleunigt (Stichwort “big rip”) ist das vernachlässigbar.

    Ich weiß nicht, ob man das so einfach direkt vergleichen kann.

    Der Big Rip beruht ja darauf, dass das beobachtbare Universum irgendwann durch diese ominöse Phantomenergie so stark schrumpfen würde, dass ein Elektron und sein Atomkern sich nicht mehr in ihrem jeweiligen beobachtbaren Universum befinden.
    Und irgendwann eben sogar ein Quark nicht mehr im selben beobachtbaren Universum wie seine bisherigen Partner im Hadron.

    D.h. selbst wenn die Kernkräfte beliebig groß wären, würden die Quarks nicht mehr zusammenhalten.
    Sie sind kausal ja gar nicht mehr verbunden und damit kann auch grundsätzlich keine physikalische Wechselwirkung mehr stattfinden.
    Jedenfalls nicht, wenn sich diese nicht schneller als mit Lichtgeschwindigkeit ausbreiten kann.

    Vereinfacht gesagt zerreißen Strukturen im Big Rip genau dann, wenn der Hubble-Parameter so groß wird, dass die Konstituenten der Struktur durch die Expansion mit einer Relativgeschwindigkeit größer der Lichtgeschwindigkeit auseinander getrieben werden.

    Das kann man für verschiedene Wechselwirkungen und Elementarteilchen vermutlich auch jeweils irgendwie einem Zahlenwert für eine wirkende Kraft zuordnen.

    Finde ich aber wenig hilfreich.
    Selbst wenn die starke Kernkraft sofort im Anschluss unendlich stark würde, könnte das eine Quark ja trotzdem das anderes Quark mehr einfangen, dass sich jetzt für immer außerhalb seines beobachtbaren Universums befindet.

    Da versagt dann meiner Meinung nach der Kraftbegriff aus der klassischen Mechanik völlig.
    Beim kosmologischen Ereignishorizont redet man ja auch nicht von irgend welchen Kräften, die da beteiligt sind.

  257. #257 MartinB
    7. November 2017

    @Niels
    “Vereinfacht gesagt zerreißen Strukturen im Big Rip genau dann, wenn der Hubble-Parameter so groß wird, dass die Konstituenten der Struktur durch die Expansion mit einer Relativgeschwindigkeit größer der Lichtgeschwindigkeit auseinander getrieben werden.”
    Ja, aber ohne dass die Beschleunigung massiv zunimmt, würde ja die Anfangsgeschwindigkeit eines gebundenen Teilchens gegenüber der Expansion ausreichen, um das immer zusammenhalten.
    MaW: Wenn zwei Objekte jetzt zusammengehalten werden, dann können sie in der Zukunft durch die Expansion nur auseinandergerissen werden, wenn die Expansionsgeschwindigkeit zunimmt. Und solange die Geschwindigkeit nicht extrem zunimmt, würden die zusammenhaltenden Kräfte ja für eine entsprechende Beschleunigung sorgen.

  258. #258 Niels
    8. November 2017

    @Karl-Heinz
    Sorry, hatte vergessen, dir zu antworten.

    a’’/a * D = scheinbare Beschleunigung
    a’’/a * D = H^2 * D (H’ kann man vernachlässigen)

    Warum sollte man H vernachlässigen können?

    Ich plotte einfach mal beides:

    Zeitliche Entwicklung von a’’/a

    Zeitliche Entwicklung von H^2

    H^2 = a’’/a ist also offensichtlich völlig falsch, man muss zwingend mit
    a’’/a = H^2 + H
    rechnen.

    .

    Was ist D für eine Entfernung? Proper distance, oder?

    In der Kosmologie gibt es schon den Begriff der Rezessionsgeschwindigkeit v.

    Den Hubble-Parameter kann man auch formal als das Verhältnis zwischen dieser Rezessionsgeschwindigkeit v (eines Objektes in proper distance D) und der Entfernung D zu diesem Objekt auffassen.
    H = v/D bzw. v = H * D

    Die comoving distance X ist definiert als
    X = D/a
    wobei a der Skalenfaktor ist.
    Die comoving distance ist dadurch genau so definiert, dass sie zwischen mitbewegten Beobachtern konstant bleibt.

    Es gilt also
    v = D(t)/dt = d[ X(t)*a(t) ] / dt = X’(t) * a(t) + a’(t) * X(t)

    Für zwei mitbewegte Objekte ist X’(t) = 0, also wie gewünscht
    v = a’(t) * X(t) = a’(t) * D(t)/a(t) = [a’(t)/a(t)] *D(t)
    = H(t) * D(t)

    Wenn man gerne noch eine Beschleunigung haben will, würde ich einfach dieses Rezessionsgeschwindigkeit v noch einmal ableiten, also v berechnen.

    Für mitbewegte Beobachter also einfach

    v = a’’ * X = a’’ * D/a

    Also

    v’ = D * a’’/a
    Also Funktion ist das aber natürlich identisch zu a’’, weil für mit bewegte Beobachter eben wie gesagt per Definition X(t) = const gilt.

    Langer Rede kurzer Sinn:
    A) scheinbare Beschleunigung = a’’ (falls D proper distance)
    B) scheinbare Beschleunigung = a’’ / a^2 (falls D comoving distance)

    A bringt keine neuen Erkenntnisse.
    Bei B habe ich keine Ahnung, was mir diese Funktion sagen sollte.
    Sieht als Graph übrigens so aus: Zeitliche Entwicklung von a’’ / a^2
    .

    [Wenn es um zwei Beobachter geht, die nicht beide mitbewegt sind, müssen wir
    v = X’(t) * a(t) + a’(t) * X(t) ableiten und es wird komplizierter.]
    .

    Wegen mir darfst du dieses v’ gerne scheinbare Beschleunigung nennen, ist aber eben als Funktion schlicht identisch zur zweiten Ableitung des Skalenfaktors a’’.

    Wobei natürlich dieses v, die Rezessionsgeschwindigkeit v, und die proper distance D alle keine Messgrößen sind, sondern unter Anwendung der ART und Berücksichtigung der Expansion aus der beobachtbaren Rotverschiebung berechnet werden müssen.

  259. #259 Alderamin
    8. November 2017

    @Niels

    Danke für die ausführliche Erklärung. Wollte nur noch mal sicher gehen, ob denn nun eine (winzige) Kraft durch die Expansion existiert oder nicht.

    Und wenn die dunkle Energie nicht wäre, nähme sie ab (tat sie ja in den ersten 7 Milliarden Jahren).

    Die Expansion? Die Änderungsrate der Expansion?
    […]
    a’ nimmt ab.

    Das meinte ich mit der abnehmenden Expansion in einem Universum ohne dunkle Energie, die Expansion würde da gebremst. Logisch, a” muss dann kleiner Null sein. Heute ist a” größer Null. Zwischendurch war a” auch mal kurz genau Null.

    Daraus würde also folgern dass es heute (mindestens theoretisch) eine sehr kleine, auseinandertreibende Kraft gibt, bei einem Weltalter von ca. 7 Milliarden Jahren einen Zeitpunkt, wo es diese Kraft nicht gab, und davor eine Zeit, wo die Kraft in Gegenrichtung wirkte, also “nach innen”.

    Das ist nun auch wieder interessant. Das war dann aber nach meinem Verständnis keine mit der dunklen Energie vergleichbare Kraft, die Atome und Planetensysteme (sehr schwach) zusammengedrückt hätte, sondern nur die wechselseitige Gravitation der Materie im Großen, man rechnet ja bei Friedmann so, als ob der Raum gleichmäßig mit verdünnter Materie ausgefüllt sei, was er im Kleinen nicht ist. Tatsächlich wirkte die dunkle Energie im Kleinen da auch schon minimal abstoßend, wie heute, aber im Großen nicht abstoßend genug, um a” in Summe > 0 zu machen. Das war erst nach 7 Milliarden Jahren der Fall, als sich die Materie weit genug verdünnt hatte.

    Dann wäre es auch am korrektesten, wenn man nicht sagt, Objekte die durch Kräfte gebunden sind, spüren nichts von der Expansion, sondern die Bindungskräfte sind auf kurze Distanzen so viel größer als die auseinandertreibende Kraft der dunklen Energie (und nur dieser!), dass letztere keine messbare Auswirkung hat. Erst über große Distanzen wird sie so stark (bzw. die Gravitationskräfte zwischen den Galaxien so klein), dass sie überwiegt und die Galaxien auseinander treibt.

    Würde ich mir dann so merken, wenn nochmal jemand danach fragt.

    Deine Plots sollten übrigens mal in einem eigenen Artikel gesichert werden, die findet man in den Kommentaren später mit Google so schlecht wieder. (Eigentlich solltest Du mal ein Buch schreiben 😉 😉 😉 )

  260. #260 Alderamin
    8. November 2017

    @myself

    ob denn nun eine (winzige) Kraft durch die Expansion existiert oder nicht.

    Natürlich durch die Änderungsrate der Expansion (“Expansion” war hier mehr als Name des dynamischen Prozesses schlechthin gemeint).

  261. #261 Karl-Heinz
    8. November 2017

    @Niels

    Warum sollte man H’ vernachlässigen können?

    Du hast natürlich vollkommen Recht. Mein Fehler 😉

    Ich hatte deine zeitliche Entwicklung des Hubble-Parameters H(t) genommen und H^2 und H‘(t) gebildet und beide Kurven miteinander verglichen. Worauf ich vergessen habe ist H(t) auf die Einheit [1/s] umzuwandeln. Ich muss zugeben, dass ich mich doch über das Ergebnis einigermaßen gewundert habe, dann aber leider zur Kenntnis genommen habe.

    Die Einheiten [km^2/ (s^2 * Mpc^2)] von H^2 sind halt doch ein bisschen anders als die Einheiten [km/(s^2 * Mpc)] von H‘(t) und deswegen dürfen die Beträge natürlich nicht miteinander verglichen werden.

    Danke für den Hinweis.

  262. #262 Niels
    8. November 2017

    @Alderamin

    Daraus würde also folgern dass es heute (mindestens theoretisch) eine sehr kleine, auseinandertreibende Kraft gibt, bei einem Weltalter von ca. 7 Milliarden Jahren einen Zeitpunkt, wo es diese Kraft nicht gab, und davor eine Zeit, wo die Kraft in Gegenrichtung wirkte, also “nach innen”.
    […]
    Tatsächlich wirkte die dunkle Energie im Kleinen da auch schon minimal abstoßend, wie heute, aber im Großen nicht abstoßend genug, um a” in Summe > 0 zu machen. Das war erst nach 7 Milliarden Jahren der Fall, als sich die Materie weit genug verdünnt hatte.

    Alles richtig.
    Wie jetzt schon öfter erwähnt ist diese Kraft proportional zu a‘‘(t)/a(t), der zeitliche Verlauf dieser Kraft ist also in #258 schon geplottet.

    Der Übersichtlichkeit halber nochmal:

    zeitlicher Verlauf der Kraft, die aufgrund der Änderung der Expansionsrate wirkt, für Zeiten von 7 Milliarden Jahre bis 20 Milliarden Jahre nach dem Urknall.

    zeitlicher Verlauf der Kraft, die aufgrund der Änderung der Expansionsrate wirkt, für Zeiten bis 40 Milliarden Jahre nach dem Urknall.

    Gegen einen Grenzwert läuft das Ganze, weil die Expansion exponentiell erfolgt, sobald die dunkle Energie überwiegt. Die Ableitung einer Exponentialfunktion ist natürlich wieder eine Exponentialfunktion, die zweite Ableitung ebenfalls.

    Die auseinandertreibende Kraft ist also beschränkt.
    Für etwa 20 Milliarden Jahre nach dem Urknall bis in alle Ewigkeit wirkt eine maximale, sich nicht mehr verändernde auseinandertreibende Kraft.
    Wobei heute bei einem Universumsalter von 13,7 Milliarden Jahren schon gerundete 80% dieser auseinandertreibenden Maximalkraft wirken.

    Das war dann aber nach meinem Verständnis keine mit der dunklen Energie vergleichbare Kraft, die Atome und Planetensysteme (sehr schwach) zusammengedrückt hätte, sondern nur die wechselseitige Gravitation der Materie im Großen

    Kann man so sehen.
    Oder so, dass dunkle Energie letztlich auch nur über wechselseitige Gravitation wirkt.

    Genauer:
    Die Frage, ob es eine “vergleichbare” Kraft ist oder nicht, ist ein bisschen schwierig und eher philosophisch.

    Die ART beruht ja auf den Einsteinsche Feldgleichungen.
    Diese sind, wenn man man alle Konstanten auf 1 setzt und ohne Kosmologische Konstante (Λ = 0) einfach

    G = T

    G ist der Einstein-Tensor
    T ist der Energie-Impuls-Tensor

    Die linke Seite beschreibt die Geometrie der Raumzeit, die rechte Seite den Materie- bzw. Energie-Gehalt der Raumzeit.

    Bei Einführung einer zusätzlichen (kosmologischen) Konstanten hat man die Wahl, diese Konstante entweder auf der rechten oder auf der linken Seite dazuzuaddieren.
    Also entweder

    a) G + Λ*g = T
    b) G = T + T_Λ = T_ges

    g ist der metrische Tensor
    T_Λ ist der Energie-Impuls-Tensor des Vakuums

    Man könnte also nach b) sagen, dass das im wesentlichen das Gleiche ist.
    Der Unterschied ist nur, dass der eine Effekt verschwindet, wenn man den Vakuum-Energiedichte-Eintrag des Gesamt-Energie-Impuls-Tensor gleich Null setzt.
    Der andere, wenn man den Materie-Energiedichte-Eintrag des Gesamt-Energie-Impuls-Tensor gleich Null setzt.

    Oder man argumentiert mit Hilfe von a), dass das ein grundlegend unterschiedlicher Effekt ist, weil Λ eben zur “Geometrie-Seite” der Gleichung gehört. Und eben nicht zur “Materie-Seite”.

    Dann wäre es auch am korrektesten, wenn man nicht sagt, Objekte die durch Kräfte gebunden sind, spüren nichts von der Expansion, sondern die Bindungskräfte sind auf kurze Distanzen so viel größer als die auseinandertreibende Kraft der dunklen Energie (und nur dieser!), dass letztere keine messbare Auswirkung hat.

    Richtig.

    Erst über große Distanzen wird sie so stark (bzw. die Gravitationskräfte zwischen den Galaxien so klein), dass sie überwiegt und die Galaxien auseinander treibt.

    Wenn du mit sie die Expansion meinst, ist das richtig.
    Falls sie die auseinandertreibende Kraft abkürzt, ist es falsch.

    Über große Distanzen sind die Galaxien in guter Näherung mitbewegte Beobachter.

    Wie gesagt, mitbewegte Beobachter bewegen sich entlang von Geodäten, fallen also frei.
    (Das ist die Definition von Geodäten).
    Damit wirken auf sie auch grundsätzlich keine Kräfte. (Das ist nämlich die Definition von frei fallend.)
    .

    Karl-Heinz
    Prima.
    Sonst dann soweit alles klar?

  263. #263 Karl-Heinz
    8. November 2017

    @Niels

    Prima.
    Sonst dann soweit alles klar?

    Fragen kommen noch, so bald ich mehr Luft habe.
    Während der Arbeitszeit ist man doch ein bisschen eingeschränkt. 😉

    Grüße KH

  264. #264 Niels
    8. November 2017

    @MartinB

    MaW: Wenn zwei Objekte jetzt zusammengehalten werden, dann können sie in der Zukunft durch die Expansion nur auseinandergerissen werden, wenn die Expansionsgeschwindigkeit zunimmt. Und solange die Geschwindigkeit nicht extrem zunimmt, würden die zusammenhaltenden Kräfte ja für eine entsprechende Beschleunigung sorgen.

    Kein Einwand.
    .

    Nochmal anders formuliert, worum es mir geht:
    .

    Beim Big-Rip schrumpft der kosmologische Ereignishorizont gegen Null.
    Zur Zeit T habe der dieser Ereignishorizont E_k den Wert
    E_k = d = 1.
    Zur Zeit T berechnen wir die diskutierte zu a’’/a proportionale “kosmologische Kraft” F_k zu
    F_k (T) = 2.

    Eine hypothetische Kraft F_h verbindet zwei Elementarteilchen ξ1 und ξ2, ihr Abstand R in dieser Bindung habe ebenfalls den Wert d,
    R = d = 1.
    Die Stärke der Bindung für diesen Abstand d sei
    F_h (d) = 10000 Einheiten.

    .

    Es gilt
    F_h (d) > F_k (T).
    Die Bindung sollte halten, oder?

    Da der kosmologische Ereignishorizont aber für Zeiten t größer als T auf einen Wert kleiner d schrumpft, werden die beiden Elementarteilchen ξ1 und ξ2 dennoch auseinandergerissen.

    Sie sind kausal eben nicht mehr verbunden. Da spielt es schlicht keine Rolle, welche konkreten Werte F_h und F_k annehmen.

    Deswegen hat meiner Ansicht der Big Rip höchstens indirekt etwas mit der in diesem Artikel besprochenen Kraft F_k aufgrund der Änderung der Expansionsrate des Universums zu tun.

    Oder mache ich da einen Denkfehler?

  265. #265 DerZimmermann
    9. November 2017

    Hallo zusammen 🙂

    Ich leb noch, zwischenzeitlich war es aber tatsächlich mal recht knapp. Mittlerweile bin ich gesundheitlich allerdings langsam aber sicher wieder auf dem aufsteigenden Ast.

    Dann auch von mir gute Besserung… klingt ja ziemlich heftig 😐

    Ich verstehe euer Problem nicht. (Der Kommentarstrang ist allerdings auch ganz schön lang und verworren.)

    Vielleicht kann ich helfen, wenn ihr noch mal ganz konkret nachfragt?

    Ja, es gab einen Haufen Missverständnisse über Begrifflichkeiten etc….

    Was ich mich eigentlich nur noch gefragt hatte: Wie müsste man sich das ganze anschaulich mit einem Vergleich oder Modell vorstellen?
    Bei Materie verhält sich die Expansion ja eher wie eine Art Auseinanderbewegung. Das Gummibandbeispiel funktioniert dabei nicht. Für Photonen, also Licht, scheint das Gummibandmodell aber doch zu stimmen. Wie könnte man sich diese Diskrepanz veranschaulichen?

  266. #266 MartinB
    9. November 2017

    @Niels
    “Oder mache ich da einen Denkfehler?”
    Vermutlich nicht.
    Erst wollte ich so argumentieren: Beim Big Rip divergiert doch der Skalenfaktor; divergiert dann nicht a”/a und damit auch die Kraft?
    Aber du kannst natürlich immer argmentieren, dass du die Konstanten immer so anpassen kannst, dass eine Kraft über eine endliche Distanz ja vor der Singularität immer größer sein kann als die Kraft durch die Expansion? Das klingt für mich logisch.

  267. #267 Alderamin
    9. November 2017

    @Niels

    Alles richtig. […] Kann man so sehen. […] Richtig.

    Danke. 😀

    Wenn du mit sie die Expansion meinst, ist das richtig.
    Falls sie die auseinandertreibende Kraft abkürzt, ist es falsch.

    Über große Distanzen sind die Galaxien in guter Näherung mitbewegte Beobachter.

    Wie gesagt, mitbewegte Beobachter bewegen sich entlang von Geodäten, fallen also frei.
    (Das ist die Definition von Geodäten).
    Damit wirken auf sie auch grundsätzlich keine Kräfte. (Das ist nämlich die Definition von frei fallend.)

    Einspruch. Ich sehe ein, dass a” nicht entfernungsabhängig ist, also auch die Kraft nicht zunimmt (aber auch nicht ab). Ich sehe auch ein, dass nicht gebundene Objekte frei fallen. Aber eine ISS fällt auch frei und die Schwerkraft wirkt auf sie und beschleunigt sie auf eine Kreisbahn. Ich weiß, das ist die Newtonsche Deutung, in der ART folgt sie kräftefrei einer Geodäte.

    Aber das tun Objekte im All, die nicht mechanisch daran gehindert werden, doch auch, es gibt doch keinen Unterschied zwischen einem Planeten, der seinen Stern umkreist, auf den nur eine winzige Kraft der dunklen Energie wirkt, die unmessbar ist, und einer weit entfernten, einsamen Galaxie, die fast keine Schwerkräfte von benachbarten Galaxien spürt, und die dennoch der kleinen Kraft der dunklen Energie ausgesetzt ist. Sie bewegt sich dann halt auf Geodäten, die durch alle Einflüsse (Schwerkraft, dunkle Energie) in Summe entstehen. Oder nicht? Klang jetzt nämlich so, als wenn sich mit der Entfernung grundsätzliches ändert. Ändert sich aber nur der Einfluss anderer Galaxien, denke ich.

  268. #268 Niels
    9. November 2017

    @MartinB

    Aber du kannst natürlich immer argmentieren, dass du die Konstanten immer so anpassen kannst, dass eine Kraft über eine endliche Distanz ja vor der Singularität immer größer sein kann als die Kraft durch die Expansion?

    Stimmt. Genau das ist die Idee, die ich in #256 zu vermitteln versucht habe.
    .

    @DerZimmermann

    Ich kenne neben dem Gummiband nur noch die Luftballon- sowie die Napfkuchen-Veranschaulichung.
    Diese drei Analogien haben unterschiedliche Stärken und Schwächen, am besten finde ich persönlich das Napfkuchen-Modell.

    Glücklicherweise muss ich dazu nichts schreiben, der erste Google-Treffer ist gleich ein prima Artikel genau dazu.

    Markus Pössel auf den SciLogs:
    Kosmische Expansion: Gummiband oder Rosinenkuchen?

  269. #269 Niels
    9. November 2017

    @MartinB
    Oh, mein Kommentar ist in der Moderation.
    Absolut keine Ahnung worans liegen könnte. Der Spamfilter ist offenbar immer noch ziemlich schlecht? Oder mag scienceblogs mittlerweile keine Links zur Konkurrenz mehr?

  270. #270 Niels
    9. November 2017

    @MartinB
    Der Spamfilter mag mich gerade nicht?

  271. #271 Niels1
    9. November 2017

    @MartinB

    Es gibt gerade Probleme mit allen meinen Kommentarversuchen und der Moderation?

  272. #272 MartinB
    10. November 2017

    @Niels
    Leider habe ich erst sehr spät gesehen, dass da was in der Moderation hängt (ich habe die mail-Benachrichtigung abgeschaltet, weil das zeitweilig Überhand nahm, und war die letzten Tage wenig am Rechner). Ich hoffe, jetzt ist alles da.

  273. #273 MartinB
    10. November 2017

    PS: Warum unser Filter plötzlich auch die simpelsten Kommentare nicht mehr mag, weiß ich ach nicht, aber es gab eine Flut von Spamzeugs…

  274. #274 DerZimmermann
    10. November 2017

    @DerZimmermann

    Ich kenne neben dem Gummiband nur noch die Luftballon- sowie die Napfkuchen-Veranschaulichung.
    Diese drei Analogien haben unterschiedliche Stärken und Schwächen, am besten finde ich persönlich das Napfkuchen-Modell.

    Glücklicherweise muss ich dazu nichts schreiben, der erste Google-Treffer ist gleich ein prima Artikel genau dazu.

    Ja… so weit war ich auch schon 😉

    Ich erlaube mir mal mich selbst zu zitieren 🙂

    Ich setze in einer bestimmten Entfernung von mir einen Testkörper aus. Damit alles ganz sauber ist, kann ich mir auch vorstellen, dass ich eines der Gaspartikel im Friedmannuniversum bin und der Testkörper ein anderer.
    (…)
    Jetzt gehe ich von einem Dunkle-Energie-dominierten, beschleunigten Universum aus (d.h., H konstant, a ~ e^(H*t)). In diesem Fall bleibt die Geschwindigkeit des Gummis in der Entfernung des Testkörpers immer gleich, und damit sollte der Testkörper immer an dieser Stelle bleiben, weil er ja die benötigte Relativgeschwindigkeit einmalig von mir bekommen hat.
    Man kann aber ausrechnen, dass er das nicht tun wird, sondern er wird beschleunigt von mir wegfliegen, und die Beschleunigung ist ~ a‘‘/a > 0.

    Die Modelle kenne ich. Die sind ganz praktisch um einen Eindruck zu bekommen… aber wenn es darum geht, ein Gefühl für den Expansionsmechanismus zu bekommen, funktioniert es offenbar nicht mehr.

    Das Gummituch (und soweit ich sehe auch die anderen Modelle) kommt anscheinend nur für Photonen einigermaßen hin (??), die bei konstantem H tatsächlich in einer entsprechenden Entfernung festsitzen.

    Und das war auch mit der Anstoß für die Diskussion hier.

  275. #275 Niels
    11. November 2017

    @Alderamin

    es gibt doch keinen Unterschied zwischen einem Planeten, der seinen Stern umkreist, […] und einer weit entfernten, einsamen Galaxie

    Doch, genau das ist der Unterschied.

    Im Fall der Galaxie betrachten wir eine einsame Galaxie.
    Also eine Struktur, die für uns einfach ein einzelnes Teilchen darstellt.
    Wir nehmen an, dass auf dieses Teilchen keine (resultierende) newtonsche Gravitationskraft wirkt.

    Im Falle des Planetensystem betrachten wir einen Planeten, der seinen Stern umkreist.
    Also eine Struktur, die sich aus zwei Teilchen zusammensetzt.
    Diese Teilchen sind durch newtonsche Gravitationskräfte gebunden.

    Wenn sich Planet und Stern allein im intergalaktischen Raum befinden und wir das ganze Planetensystem wie im Fall der Galaxie einfach zu einem einzelnen Teilchen der addierten Masse vereinfachen, auf das keine resultierende Gravitationskraft wirkt, dann gibt es keinen Unterschied.

    Sie bewegt sich dann halt auf Geodäten, die durch alle Einflüsse (Schwerkraft, dunkle Energie) in Summe entstehen.

    Richtig.

    Wir betrachten momentan gar keine anderen Wechselwirkungen außer der durch die ART beschriebenen Gravitation, es bewegt sich also grundsätzlich alles ausschließlich entlang von Geodäten.

    Eigentlich hätte ich sauber in ART-Sprechweise immer über geodätischer Abweichung und der Relativbeschleunigung der geodätischen Abweichung sprechen müssen.
    (Macht zu meiner Verteidigung aber auch sonst niemand.)

    Klang jetzt nämlich so, als wenn sich mit der Entfernung grundsätzliches ändert. Ändert sich aber nur der Einfluss anderer Galaxien, denke ich.

    Na ja, der Inhalt des Universum besteht aber aus Materie (und dunkler Energie), aus sonst nichts.
    Wenn sich deren Einfluss auf unseren jeweiligen gedachten Beobachter ändert ist das also kein “nur”.

    Es hängt auch nicht unbedingt mit der Entfernung, sondern vielmehr mit der Skala zusammen, auf der wir die Dinge betrachten.
    .

    “Ungebundene Systeme”

    Auf großen Skalen ist das Universum mit einzelnen, identischen Teilchen (Supergalaxienhaufen) gefüllt, die alle den gleichen Abstand voneinander haben.
    Nach dem Newtonschen Gravitationsgesetz gleichen sich die anziehenden Gravitationskräfte daher vollständig aus.
    Die resultierende Kraft auf jedes Teilchen ist Null, es liegt ein ideales Gas vor.

    “Gebundene Systeme”

    Auf kleinen Skalen gibt es gravitativ gebundene Strukturen.
    Nach dem Newtonschen Gravitationsgesetz üben die betrachteten Teilchen Netto-Kräfte aufeinander aus.
    .

    Oder komplizierter und in ART-Sprech:
    Grundsätzlich ändert sich der Grad der Isotropie und der Homogenität des betrachteten Ausschnittes des Universums und damit die Form der Raumzeit, die auf dieser Ebene resultiert.
    .

    “Ungebundene Systeme”

    Auf Ebene der Supergalaxienhaufen ist das Universum nach unserem Modell homogen und istotrop.
    Hier liegt eine praktisch perfekte Robertson-Walker-Raumzeit vor.

    Es gibt daher zwischen Supergalaxienhaufen keine geodätischer Abweichung allein aufgrund der Änderung der Expansionsrate des Universums.
    Und damit natürlich auch keine Relativbeschleunigung allein aufgrund der Änderung dieser Expansionsrate.

    “Gebundene Systeme”

    Innerhalb eines Planetensystems oder einer Galaxie ist das Universum nicht homogen und nicht isotrop.
    Die Raumzeit unterscheidet sich daher deutlich von einer perfekten Robertson-Walker-Raumzeit.

    In solchen Bereichen gibt es allein aufgrund der Änderung der Expansionsrate des Universums eine geodätischer Abweichung.
    Damit dann auch allein aufgrund der Änderung der Expansionsrate des Universums eine Relativbeschleunigung zwischen den Masseteilchen.

  276. #276 Karl-Heinz
    11. November 2017

    @Niels

    Neben dem Big Rip gibt, wo es in endlicher Zeit zu einer Singularität kommt, gibt es ja auch noch den Little Rip, wo es in endlicher Zeit zu keiner Singularität kommt, aber trotzdem alles zerrissen wird.

    Was hältst du vom Little Rip?

    https://artikel.vsda.de/astronomie-ohne-teleskop-grosse-risse-kleine-risse

  277. #277 Karl-Heinz
    12. November 2017

    @DerZimmermann

    Verwirrt von der Expansion des Universums? Damit sind Sie nicht allein – selbst Astronomen verstehen den Urknall nicht immer richtig. 😉

    https://homepage.univie.ac.at/Michael.Berger/lit/urknall.pdf

  278. #278 Alderamin
    13. November 2017

    @Niels

    “Ungebundene Systeme”

    Auf großen Skalen ist das Universum mit einzelnen, identischen Teilchen (Supergalaxienhaufen) gefüllt, die alle den gleichen Abstand voneinander haben.
    Nach dem Newtonschen Gravitationsgesetz gleichen sich die anziehenden Gravitationskräfte daher vollständig aus.
    Die resultierende Kraft auf jedes Teilchen ist Null, es liegt ein ideales Gas vor.

    Was mich zu einer anderen Frage führt. Ungefähr dieses Argument brachte neulich mal jemand bei Astrodicticum Simplex und stellte damit in Zweifel, warum die Expansion (ohne dunkle Energie) sich verlangsamen sollte. Man liest halt immer, die Masse im All zieht sich gegenseitig an und bei mehr als der kritischen Dichte würde die Expansion sich irgendwannzu einer Schrumpfung mit Big Crunch umkehren, aber wenn der Raum (auf großen Skalen annähernd) homogen mit Materie ausgefüllt ist, dann gibt es ja keine Nettokraft mehr, welche die Galaxien zueinander ziehen könnte, die Kräfte aus allen Richtungen addieren sich überall zu 0.

    Ich hatte es bisher immer so interpretiert, dass die Masse, die aus dem Urknall mit einem gewissen Impuls auseinandertreibend hervorgegangen ist (das mit dem Impuls ist dabei nicht von mir, das habe ich auch so als Aussage von Kosmologen gefunden), den Raum mit sich zieht und er deswegen expandiert (auch ohne dunkle Energie), wobei die Gravitation zwischen den Galaxien diese Expansion bremst.

    Kann man also die Verlangsamung der Expansion mit Newtonschen Argumenten gar nicht begründen? Und wenn das so ist, kann man irgendwie anschaulich begründen, woher die Rückstellkraft in der ART resultiert? Bei flachem Universum?

  279. #279 Alderamin
    13. November 2017

    @Karl-Heinz #277

    Klasse Artikel, den habe ich mal gebookmarkt. Steht fast alles drin (auch die auseinandertreibende Kraft, die gebundene Objekte zwar unmerklich vergrößer, aber nicht trennen kann).

    Außer der Antwort auf meine Frage in #278…

  280. #280 MartinB
    13. November 2017

    @Alderamin
    Wenn das UNiversum homogen mit Materie angefüllt ist, dann ist an jedem Punkt im Universum die Energiedichte größer als Null. Damit ist überall (linke Seite der Einstein-Gleichung) der Ricci-Tensor nicht Null, und Geodäten von massiven Teilchen müssen sich zwangsläufig stärker annähern, als sie es ohne diesen Effekt tun würden.
    Deswegen sind Argumente mit Kräften ja auch immer ein bisschen gefährlich, wie Nils schon sagte:
    “Eigentlich hätte ich sauber in ART-Sprechweise immer über geodätischer Abweichung und der Relativbeschleunigung der geodätischen Abweichung sprechen müssen.”
    Im Moment empfehle ich diese Seiten hier sehr (ab nächstem Jahr empfehle ich dann vermutlich was anderes…):
    https://math.ucr.edu/home/baez/einstein/
    für deine Frage ist das hier relvant:
    https://math.ucr.edu/home/baez/einstein/node3.html

  281. #281 Alderamin
    13. November 2017

    @MartinB

    Die Einstein-Gleichung mit Ricci-Tensor, metrischem Tensor und Krümmungsskalar ist mir eine Nummer zu groß, habe gerade mal den Energie-Impuls-Tensor verstanden, aber Deine Links bringen mich weiter, denn unter

    https://math.ucr.edu/home/baez/einstein/node6.html

    steht genau das gesuchte (wobei die Drücke P in erster Näherung im Universum keine Rolle spielen, in zweiter Näherung sind sie negativ und genau die dunkle Energie, soweit ich das von anderswo richtig verstanden habe).

    Bin ja sehr gespannt, was Du uns nächstes Jahr bescherst. Hier im Blog oder in gedruckter Form im Buchladen??

  282. #282 MartinB
    13. November 2017

    Für ne Blogaktion müsste ich mich nicht nen Jahr einigeln und das Bloggen quasi an den Nagel hängen (so wenig wie im Moment habe ich ja noch nie gebloggt).

  283. #283 Alderamin
    13. November 2017

    🙂

  284. #284 Albrecht Storz
    Mannheim
    20. Januar 2018

    Ich versuche es auch noch hier, da dieser Thread neuer ist, und kopiere hier meine Frage aus
    https://scienceblogs.de/astrodicticum-simplex/2014/07/14/wie-gross-ist-das-universum/#comment-1438685
    ein:
    Ich würde gerne ein Thema aufgreifen, dass schon angesprochen wurde, aber mE nicht abschließend beantwortet wurde:
    Ich verstehe es so, dass sich der Raum ausdehnt, aber Materie (die ja im Raum ist) sich nicht ausdehnt.
    Das macht für mich keinen Sinn.
    Andererseits ist natürlich ganz klar: wenn sich ALLES ausdehnt, so dehnen sich auch unsere Maßstäbe (ja, sogar wir selbst uns) aus, und somit wäre für uns keine Veränderung erkennbar.
    Wenn sich aber der Raum ausdehnt, die sich darin befindliche Materie aber nicht, so befindet sich die Materie nicht nur im Raum, sondern auch noch irgendwo anders, wozu sich die Materie in Relation dazu konstant verhalten würde.
    Ich möchte es mit dem immer wieder bemühten Gummiband veranschaulichen:
    Auf dem Gummiband sind Punkte markiert. Wenn man nun das Gummiband zieht, entfernen sich die Punkte voneinander. Nun ist Materie aber eben nicht punktförmig. Wenn sich nun auf dem Gummiband Punkthaufen befinden, wobei jeweils ein Punkthaufen einem materiellen Objekt entsprecht, so werden eben auch diese Punkthaufen auseinander gezogen: sie vergrößern sich in ihrer Ausdehnung.
    Wenn nun ein solcher Punkthaufen ein Meterstab ist, so vergrößert sich dieser Meterstab genau proportional so, wie das gesamte Band vergrößert wird. Messe ich damit, sind aber alle gemessen Abstände so wie vor der Dehnung. ZB (einfachster Fall) mein Meterstab ist vor UND nach der Dehnung genau 1 Meterstab lang.
    Wie erklärt die Kosmologie diesen Widerspruch? Nämlich dass wir eine Raumausdehnung messen können mit Maßstäben die selber im Raum ausgedehnt sind. Ist die Ausdehnung des Raumes lokal von der darin enthaltenen Masse abhängig? Dehnt sich relativ leerer Raum mehr aus als Raum der viel Massen enthält?
    Tut mir Leid, aber dieses Konzept ist für mich nicht schlüssig.
    [Tschuldigung, falls die Frage hier schon aufgeworfen und beantwortet wurde, habe denn Thread noch nicht durch]

  285. #285 Albrecht Storz
    Mannheim
    20. Januar 2018

    Wenn ich das so lese bekomme ich den Eindruck, dass man als Lösung des Problems annimmt, dass Materie auf eine gewisse Art vom Raum unabhängig wäre. Sprich: Materie ist nicht vollständig der unterlegten Metrik (die sich ja durch die Raumdehnung ändert) unterworfen.
    Ist das nicht ein arg extremistisches Konzept? Insbesondere soll sich die Schwerkraft unabhängig vom Raum verhalten. Sprich: wenn sich der Raum ausdehnt, würde die Schwerkraft nicht dieser Ausdehnung unterworfen und damit die Masseteilchen gegen die Ausdehnung zusammen halten.
    Das läuft auf ein völlig anderes, unklares Raumkonzept wie wir es bisher kennen, hinaus. Ist das nicht etwas arg mit Kanonen auf Spatzen geschossen, nur um die Rotverschiebung der fliehenden Galaxien zu erklären? Klar, das Phänomen soll erklärt werden. Aber warum wird nur dieses eine Konzept diskutiert? Gibt es denn keine anderen Modelle?

  286. #286 Alderamin
    20. Januar 2018

    @Albrecht

    Hast Du den Artikel gelesen? Und die Kommentare? (wenigstens überflogen)

    Die Raumexpansion verursacht eine sehr, sehr, sehr kleine auseinandertreibende Kraft, weil die Expansionsrate nicht konstant ist, aber zwischen Objekten wirken immens viel größere Bindungskräfte, inklusive der gewöhnlichen Schwerkraft, die diese Kraft mehr als überwinden. Maßstäbe expandieren nicht, und Jupiter kreist vielleicht weniger als einen Atomdurchmesser weiter von der Sonne weg, weil der Raum expandiert, aber entfernt sich nicht beständig von ihr.

    Und Schwerkraft ist nicht unabhängig vom Raum bzw. der Raumzeit, sondern laut allgemeiner Relativitätstheorie eine Scheinkraft, die gerade von der Raumkrümmung verursacht wird.

    Dafür, dass der Raum expandiert und die Rotverschiebung ferner Galaxien durch die Raumexpansion verursacht wird, gibt es zig unabhängige Belege; abgesehen davon, dass nach der ART der Raum nur expandieren oder kollabieren kann, gibt es die Hintergrundstrahlung, das begrenzte Alter der Sterne, die mit der Entfernung abnehmende Menge von Metallen im Sternenlicht, das Fehlen naher Quasare etc etc., die die Richtigkeit der Urknalltheorie belegen. Und nein, es gibt keine alternative Theorie mehr, die all das erklären könnte und nicht so etwas exotisches wie permanente Entstehung von Materie aus dem Nichts bräuchte.

    Wem der Urknall absurd erscheint, dem muss die Steady State Theorie völlig irre erscheinen.

  287. #287 MartinB
    20. Januar 2018

    @Albrecht Storz
    Materie ist in der Tat nicht an Raumpunkte angeheftet – sonst gäbe es ja keine Bewegung. Wir können uns relativ zueinander und zum Raum bewegen, und damit zwei Objekte sich relativ zueinander mit konstanter Geschwindigkeit bewegen, braucht es keinerlei Kraft. Das gilt schon in der Newtonschen Physik; Newton selbst sprach zwar vom absoluten Raum, aber die Gleichungen der Newtonschen Physik sagen tatsächlcih etwas anderes, nämlich dass der Raum auch dort relativ ist: Es gibt keine Möglichkeit, zu zwei verschiedenen Zeiten “denselben” Punkt im Raum zu identifizieren. (Man kann tatsächlich sogar die Newton-Theorie der Schwerkraft über gekrümmte Raumzeiten formulieren, das nennt sich Newton-Cartan-Theorie.)

    Wenn der Raum sich mit konstanter Rate (die Feinheiten der Ratendefinition haben wir oben definiert, spare ich mir hier) ausdehnt, und ich möchte zwei Objekte in konstantem Abstand halten, dann muss ich eines der beiden Objekte einmal anschubsen, damit es sich relativ zum expandierenden Raum mit der passenden Geschwindigkeit bewegt. Sobald es das tut, brauche ich nichts weiter mehr zu machen.

    Dass die Schwerkraft unabhängig vom Raum ist, ist nicht richtig, die Schwerkraft ist ja nur unsere Art, die Raumzeitkrümmung wahrzunehmen. Objekte wie z.B. eine Sonne mit ihren Planeten werden aber im Rahmen der ART durch beispielsweise die Schwarzschildlösung beschrieben, die enthält keinen Ausdehnungsterm, das heißt, diese Systeme dehnen sich erstmal nicht aus.

    Komplizierter wird es erst dann, wenn sich die Expansion des Universums ändert, dann können Objekte auch tatsächlich zerrissen werden.

    Ich gebe aber zu, dass das alles nicht ganz leicht zu verstehen ist – es ist aber die beste Beschreibung des Universums, die wir haben. Aber Konzepte wie die Raumzeitkrümmung und die Bewegung von materie darin zu verstehen, ist nicht einfach. Vielleicht hilft es auch, die anderen Artikel zur ART zu lesen (rechts bei den Artikelserien verlinkt), insbesondere bei den Gravitationswellen haben wir das in ähnlicher Weise auch diskutiert.

  288. #288 MartinB
    20. Januar 2018
  289. #289 Albrecht Storz
    Mannheim
    22. Januar 2018

    Danke für die Antworten @Alderamin und @MartinB.

    Logischer wird es für mich leider dadurch noch nicht. Der einem Objekt unterlegte Raum, bzw. der Raum, der vom Objekt eingenommen wird, soll sich also krümmen, dehnen, verwinden, verringern, vergrößern können, ohne dass das Objekt davon tangiert wird?

    Natürlich ist Materie nicht an Raumpunkten angeheftet. Das ist für mich aber auch ein völlig falsches Bild.

    Ein anderer Versuch: ich nehme den Raum weg, an dem ein Objekt sich aufhält (Gedankenexperiment). Dann soll das Objekt immer noch da sein? Aber Ausdehnung ist eben durch “Raum” und durch sonst nichts, gegeben. Ein Objekt ohne den Raum, den es einnimmt, wäre auf nicht einmal mehr einen Punkt reduziert, es wäre weg, da einfach keine “Seinsebene” mehr für das Objekt vorhanden wäre.

    Ich sehe das so, dass Raum genau das ist, was Ausdehnung ermöglicht. Ohne Raum keine Ausdehnung, keine Abstände, keine Metrik, ja, keine physische Existenz.

    Wenn nun dieser Raum unabhängig von dem Raum ist, den zB ein Objekt ausmacht (abgesehen von anderen Eigenschaften wie Farbe, …), worin soll sich denn das Objekt ausdehnen, also befinden, also verortet sein? In einem “anderen Raum”?

    Übrigens muss ich auf eine Wortverwirrung hinweisen: ich rede hier von der Ausdehnung eines Objektes nicht in dem Sinne einer Veränderung seiner Maße sondern in dem Sinne von
    “die Maße der räumlichen Dimensionen eines Objekts, die Abmessung”
    https://de.wikipedia.org/wiki/Ausdehnung

    Dies noch:
    “Der Raum (von mhd. rûm „das nicht Ausgefüllte“, „freier Platz“) ist eine grundlegende Komponente der Wirklichkeit. Raum definiert sich als Ausdehnung in Höhe, Länge und Breite. Philosophisch strittig ist, ob der Raum „an sich“, unabhängig von Wahrnehmung und Vorstellung, existiert oder lediglich eine Anschauungsform des wahrnehmenden Subjekts ist, ob also mathematischer Raum, physikalischer Raum und der Raum der Erfahrung zusammenfallen.”
    https://de.wikipedia.org/wiki/Raum_(Philosophie)

    Wird denn gar nicht erkannt, dass eine angenommene Variabilität und Unabhängigkeit des Raumes von den Ausdehnungen eines Objektes zu schwierigsten philosophischen (und damit auch kommunikativen) Problemen führen muss?

    Noch ein Gedankenexperiment: Der gesamte Kosmos würde sich mit einem Schlag vergrößern oder verkleinern: Wieso sollten wir das Wahrnehmen wenn alles gleich und gleichzeitig davon betroffen ist? Unsere Maßstäbe würden ja entsprechend mit vergrößert / verkleinert. Das ist so ähnlich, wie wenn sich der Zeitverlauf für den gesamten Kosmos verändern würde: auch das könnten wir nicht wahrnehmen, da auch unsere Uhren entsprechend anders gehen würden.

    Die beschriebene Veränderung des Raumes ohne eine entsprechende Veränderung der Maße würde unterschiedliche Metriken erfordern. Wie soll so etwas möglich, ja denkbar sein?

    Hmm, ich glaube mein Punkt ist schwer zu erklären.

    Ich möchte nur noch darauf hinweisen, dass man ja auch aus der allg. Relativitätstheorie “Veränderungen” des Raumes kennt. Aber dort ist es so, wie es mir auch plausibel erscheint: Ein Maßstab ist unter Gravitationseinwirkung eben kürzer als ohne, Uhren sind verlangsamt.
    Indem der Raum sich verändert, werden aber hierbei die Ausdehnungen von Objekten mit verändert (und der Gang von Uhren).

    Ich halte das Konzept eines Raumes, der sich unabhängig von dem sich darin befindlichen Sein (Materie, Energie, Ereignisse, Bewegung, …) verändern kann für die völlige Negierung unseres gesamten Weltbildes. Diese Art von Raum hat nichts mit unserem klassischen Verständnis von Raum (Dimension, “Ausdehnung”, Abstand, Metrik) zu tun.

  290. #290 Alderamin
    22. Januar 2018

    @Albrecht Storz

    Der einem Objekt unterlegte Raum, bzw. der Raum, der vom Objekt eingenommen wird, soll sich also krümmen, dehnen, verwinden, verringern, vergrößern können, ohne dass das Objekt davon tangiert wird?

    Genau so ist das. Anderes Beispiel sind die Gravitationswellen, die von den LIGO- und VIRGO-Detektoren gemessen wurden. Da verformte sich die Raumzeit, aber nicht die Erde bzw. die Tunnelschächte, in denen ein Laserstrahl hin- und herlief, um den Abstand der Spiegel zu messen (hierbei wurde der Lichtweg in zwei rechtwinklig zueinander stehenden Röhren gemessen). Wenn sich die Erde mit der Gravitationswelle (=fortschreitende Raumzeit-Deformation) verformt hätte, hätte man nichts gemessen. Martin hat das ausführlich erklärt.

    Wird denn gar nicht erkannt, dass eine angenommene Variabilität und Unabhängigkeit des Raumes von den Ausdehnungen eines Objektes zu schwierigsten philosophischen (und damit auch kommunikativen) Problemen führen muss?

    Die Natur kümmert sich nicht um menschliche Philospophie. Oder wie Feynman sagte: wenn Deine Theorie nicht zu den Daten passt, ist sie falsch.

    Noch ein Gedankenexperiment: Der gesamte Kosmos würde sich mit einem Schlag vergrößern oder verkleinern: Wieso sollten wir das Wahrnehmen wenn alles gleich und gleichzeitig davon betroffen ist?

    Das wurde oben in den Kommentaren indirekt schon angesprochen. Wenn sich die Expansionsrate ändert, gibt es eine Kraft. Wenn sie sich extrem ändert, ist die Kraft auch extrem, und dann kann es einen “Big Rip” geben, der alles zerreisst (den gibt es auch, wenn die Expansionsrate allmählich immer weiter bis ins Unermessliche ansteigt). Haben wir aber im Universum derzeit nicht, die Rate ändert sich nur sehr langsam, die Kraft ist winzig.

    Die beschriebene Veränderung des Raumes ohne eine entsprechende Veränderung der Maße würde unterschiedliche Metriken erfordern. Wie soll so etwas möglich, ja denkbar sein?

    Dies Kosmologen sprechen von “Mitbewegter Entfernung” (comoving distance), wenn sie eine Metrik verwenden, die mit der Raumexpansion mitwachsen soll, und von “Eigendistanz” (proper distance), wenn die Metrik nicht expandieren soll. So geht das. Im Kleinen braucht man so was nicht. Es gibt noch weitere kosmologische Entfernungsmaße: Lichtlaufzeitentfernung (klar), Leuchtkraftentfernung (abgeleitet aus der Helligkeit einer Standardkerze, z.B. einer bestimmten Art von Supernovae), Winkelausdehnungsentfernung (abgeleitet aus der Größe eines Standardlineals, z.B. korrelierte Bereiche in der kosmischen Hintergrundstrahlung), die im Kleinen alle zusammenfallen und im Großen alle verschieden sind.

    Ich möchte nur noch darauf hinweisen, dass man ja auch aus der allg. Relativitätstheorie “Veränderungen” des Raumes kennt. Aber dort ist es so, wie es mir auch plausibel erscheint: Ein Maßstab ist unter Gravitationseinwirkung eben kürzer als ohne, Uhren sind verlangsamt.

    Wüsste nicht, dass die ART etwas an der räumlichen Ausdehnung von starren Objekten ändert; die SRT ändert die Länge von bewegten Objekten in Bewegungsrichtung, aus Sicht eines ruhenden Beobachters; da sich aber jeder als ruhend definieren kann, sieht jeder den anderen verkürzt. Dabei wird aber nicht wirklich etwas komprimiert. Und auch kein Raum gekrümmt.

    Indem der Raum sich verändert, werden aber hierbei die Ausdehnungen von Objekten mit verändert (und der Gang von Uhren).

    Der Raum krümmt sich um Massen, aber abgesehen von Gezeitenkräften und ihrem Eigengewicht (wenn sie auf der Oberfläche der Masse liegen) werden starre Objekte davon nicht verformt (im freien Fall merken sie nichts). Nur die Zeit läuft langsamer.

  291. #291 Albrecht Storz
    Mannheim
    22. Januar 2018

    @Alderamin

    Zu Deiner früheren Post eine Detailfrage:
    Du: “und Jupiter kreist vielleicht weniger als einen Atomdurchmesser weiter von der Sonne weg, ”

    Die Anhebung einer Umlaufbahn erfordert eine gewisse Kraft, auch wenn sie noch so gering ist. Und natürlich wird nicht nur die Jupiterumlaufbahn angehoben, sondern Zigmilliarden andere Umlaufbahnen auch. Das ist dann schon eine Menge Energie, die da erforderlich ist.
    Du: “Die Raumexpansion verursacht eine sehr, sehr, sehr kleine auseinandertreibende Kraft,”

    Woher stammt diese Kraft? Oder muss man das so verstehen, dass die Expansion diese Kraft erzeugt? Wie sieht es dann mit der Thermodynamik aus? Gilt die nur noch lokal, nicht global?

    Und zu Deiner letzten Post:
    Du schreibst:
    “Wüsste nicht, dass die ART etwas an der räumlichen Ausdehnung von starren Objekten ändert; ”

    Ich finde dazu folgendes:

    “Maßstäbe, die von ∞ in die Umgebung schwerer Massen gebracht werden, schrumpfen im
    Vergleich zu den bei ∞ befindlichen Maßstäben um den Faktor (1 + φ/c2
    )”

    siehe
    https://www.dieter-heidorn.de/Physik/SS/K10_Relativitaet/SRT_Einfuehrung/K07_ART/ART_Einfuehrung.pdf

    Ist das falsch?

    Übrigens finde ich den Ausdruck “starre Objekte” zwar sehr eindrücklich, aber eher verwirrend. Es ist schon klar, dass wir nicht von elastischen Objekten reden. Was ist ein “starres Objekt”, was ein “nichtstarres” in diesem Zusammenhang?

  292. #292 Alderamin
    23. Januar 2018

    @Albrecht Storz

    Die Anhebung einer Umlaufbahn erfordert eine gewisse Kraft, auch wenn sie noch so gering ist. Und natürlich wird nicht nur die Jupiterumlaufbahn angehoben, sondern Zigmilliarden andere Umlaufbahnen auch. Das ist dann schon eine Menge Energie, die da erforderlich ist.
    Du: “Die Raumexpansion verursacht eine sehr, sehr, sehr kleine auseinandertreibende Kraft,”

    Es ging darum, dass die Kraft, die lediglich aus der Änderung der Expansionsrate herrührt, keine messbare Auswirkung hat. Und das hat sie nicht.

    Woher stammt diese Kraft? Oder muss man das so verstehen, dass die Expansion diese Kraft erzeugt?

    Siehe oben, #228 ff.
    Kurz gesagt: a(t) ist der Skalenfaktor des Universums, also die Größe des Universums relativ zu heute (bezogen auf den Abstand zweier mit der Expansion bewegter Punkte: wenn die heute die Entfernung x haben, hatten sie beim Skalenfaktor 1/2 vor y Milliarden Jahren den Abstand x/2).

    a'(t) ist die Ableitung des Skalenfaktors nach der Zeit, also die Geschwindigkeit der Expansion des Universum. Wenn a(t), die relative Größe des Universums zur Zeit t, nicht konstant ist, dann ist a'(t) ungleich 0. a'(t)/a(t) ist die normierte Expansionsgeschwindigkeit, das ist die berühmte Hubble-Konstante.

    a”(t) ist die zweite Ableitung des Skalenfaktors nach der Zeit, d.h. die Änderung dieser Geschwindigkeit, also die Beschleunigung oder Verzögerung der Expansion.

    Wenn a”(t) nicht Null ist, ergibt sich eine Kraft, die proportional zu a”(t)/a(t) ist.
    Die Änderung der Expansionsgeschwindigkeit erzeugt die Kraft.

    Vereinfacht gesagt stammt die Expansion aus dem “Schwung”, den die Materie nach dem Urknall erhielt (plus der dunklen Energie, die das Vakuum zusätzlich auseinander treibt, aber grundsätzlich würde die Expansion auch ohne dunkle Energie funktionieren). Da sich alles gegenseitig anzieht, ist die Expansionsrate nicht konstant. Wenn die Expansionsrate (genauer gesagt, a'(t)/a(t)) konstant wäre, könnte man mit konstanter Geschwindigkeit im gleichen Abstand von einem anderen Objekt verbleiben. Wenn aber die Expansionsrate nicht konstant ist (ewa a”(t) > 0), dann kann man das nicht, d.h. es treibt einen von dem anderen Objekt weg, wenn man nicht Gas gibt. Das Wegtreiben sieht aus wie eine Kraft, die einen wegzieht, die man nur mit fortwährendem Gegenschub kompensieren kann.

    Wie sieht es dann mit der Thermodynamik aus? Gilt die nur noch lokal, nicht global?

    Du meinst die Energieerhaltung? Schwieriges Thema. Die einen sagen, in der ART gilt sie nur lokal, weil expandierende Raumzeiten kein Intertialsystem sind. Ein Photon, das einen langen Weg zurücklegt, erleidet eine Wellenlängenvergrößerung, verliert also Energie. Die anderen sagen, die potenzielle Energie im Schwerefeld zählt als negative Energie, d.h. das Photon klettert gewissermaßen gegen die Expansion bergauf und baut immer mehr potenzielle Energie auf (zwei aus großer Entfernung aufeinander zu fallen gelassenene Objekte gewinnen ja auch mehr Geschwindigkeit, je größer die Anfangsentfernung = Fallhöhe ist; je mehr sich der Raum aufbläht, desto mehr Fallgeschwindigkeit könnten sie sich zurückholen).

    “Wüsste nicht, dass die ART etwas an der räumlichen Ausdehnung von starren Objekten ändert; ”

    Ich finde dazu folgendes:

    “Maßstäbe, die von ∞ in die Umgebung schwerer Massen gebracht werden, schrumpfen im
    Vergleich zu den bei ∞ befindlichen Maßstäben um den Faktor (1 + φ/c2)”

    Ok, scheint zu stimmen (habe auch noch eine andere Quelle gefunden). Den Effekt kannte ich in der Tat nicht. So tief stecke ich in der ART nicht drin. Der Blogchef kennt sich da besser aus.

    Was ist ein “starres Objekt”, was ein “nichtstarres” in diesem Zusammenhang?

    Vielleicht besser: “durch Kräfte gebundenes Objekt”. Ein Stück Stahl ist durch die Kristallkräfte im Metallgitter gebunden. Ein Tropfen Wasser ist durch die Wasserstoffbrückenbindungen zwischen den Molekülen gebunden. Die Erde ist durch ihre Gravitation gebunden. Das Sonnensystem gleichfalls an die Sonne. Auch die Milchstraße und ihre Nachbargalaxien (Lokale Gruppe) sind durch Schwerkraft aneinander gebunden. Erst in Entfernungen, wo die Hubble-Expansion in der Größenordnung der Fluchtgeschwindigkeit von Galaxienhaufen erreicht, ist die Bindung nicht mehr gegeben. Ab diesem Maßstab, ca. 20 Megaparsec, drifttet alles auseinander.

  293. #293 Albrecht Storz
    Mannheim
    23. Januar 2018

    Nochmal zum Raumbegriff.

    Ich habe das Gefühl, dass es bei der hier beschriebenen Kosmologie zwei verschiedene Arten von “Raum” geben muss. Es gibt den leeren Raum, und es gibt den Raum, den ein Objekt einnimmt. Diese beiden Arten von Raum verhalten sich anscheinend unterschiedlich.
    Versuchen wir uns mal von dem etwas schwierigen Begriff des Raumes zu lösen und betrachten wir nur eine Dimension davon. Dann sind wir auch näher bei dem gerne bemühten Modell des gedehnten Gummibandes für die Expansion.

    Wir können also alles auf einer Achse, zB der x-Achse des kartesischen Koordinatensystem betrachten.

    Wir haben nun einen willkürlichen Nullpunkt, zB “hier”, wir haben eine Position 1 (Strecke [0,1] ist also die Einheit) und wir haben zB den Punkt 13 Mrd. zur Zeit t0 (jetzt).

    Nun nehmen wir zur Vereinfachung jede Fluchtbewegung weg und betrachten diese Metrik zu einem Zeitpunkt t1. Wir nehmen nun an, dass sich eine Galaxie zum Zeitpunkt t0 am Punkt 13 Mrd. befindet,und fragen nun nach der Position dieser Galaxie zum Zeitpunkt t1 wobei _nur kosmische Expansion_ zwischen t0 und t1 stattgefunden hat.

    Wenn sich die Galaxie zum Zeitpunkt t1 nicht mehr an der Position 13 Mrd. sondern an der Position 13 Mrd. + x befindet, sie sich aber nicht bewegt hat, sondern der Abstand zwischen 0 und der Galaxie “gewachsen” ist (Raumexpansion), so hätte die Raumexpansion keinen Einfluss auf das Anfangsstück [0,1] gehabt.
    Hat aber die Expansion einen proportionalen Einfluss auf die Einheit [0,1], so wird die Galaxie immer noch bei 13 Mrd. sein, da die Einheit [0,1] mit gewachsen ist.
    13 Mrd. x 1 gibt eben immer 13 Mrd.

    Übrigens: in welchem der Entfernungsmaße die Du, Alderamin, aufzählst (“Es gibt noch weitere kosmologische Entfernungsmaße: Lichtlaufzeitentfernung (klar), Leuchtkraftentfernung (abgeleitet aus der Helligkeit einer Standardkerze, z.B. einer bestimmten Art von Supernovae), Winkelausdehnungsentfernung”) geht eigentlich nicht die SI-Basiseinheit “Länge” ein? Sprich: die Länge des Urmeter, gerne mittels Lichtausbreitung im Vakuum gemessen).
    Wenn die Expansion auf alles wirkt, wirkt sie auch auf unsere Einheit [0,1]. Und mit dieser Einheit messen wir. Wenn sie aber nicht auf die Einheit [0,1] wirkt, haben wir ein sehr strange universe.

  294. #294 DerZimmermann
    23. Januar 2018

    @Albrecht Storz

    Woher stammt diese Kraft? Oder muss man das so verstehen, dass die Expansion diese Kraft erzeugt?

    Man kann sich das (finde ich) auch erheblich einfacher vorstellen.

    Die Kraft kommt von der Dunklen Energie (DE). Wenn es in unserem Universum keine gäbe, dann gäbe es auch keine (auseinandertreibende) Kraft. Die Kraft, die zwischen zwei Punktteilchen wirkt, kannst du einfach aus der Dichte der Dunklen Energie und dem Abstand zwischen den beiden Teilchen berechnen.

    Was deinen letzten Beitrag angeht, da müsstest du noch spezifizieren, wie du du die Strecke [0,1] markierst, sodass du sie als Maßstab verwenden kannst.
    Ein Universum mit gleichmäßiger Ausdehnung (ich nehme mal an, dass du ein solches meinst, also eines mit a’ = 0) hat seinen “Schwung” in Form eines festen Ausdehnungswertes beim Urknall mitbekommen.
    D. h. konkret: Wenn auf Position 1 beim Urknall schon ein Objekt war (bzw. entstanden ist), dann bewegt es sich ebenfalls entsprechend dem Ausdehnungswert mit gleichbleibender Geschwindigkeit von dir weg.
    Wenn du nachträglich (z. B. mit deiner Hand) dort hinlangst und ein Objekt hinsetzt, wird sich das überhaupt nicht bewegen (bzw. eher sogar noch auf dich zu, da du es ja durch deine Gravitation anziehst).

  295. #295 MartinB
    23. Januar 2018

    @Albrecht Storz
    Ich habe das – mangels Zeit – nicht alles gelesen, aber zwei Hinweise:
    1. Man kann die ART auch rein als Theorie in einer ungekrümmten Raumzeit auffassen, in der Kärfte wirken. In dieser Formulierung kann man dann sagen, dass Maßstäbe verzerrt werden. Oder man fasst die ART als Theorie der gekrümmten Raumzeit auf, dann sind Maßstäbe Maßstäbe.
    Das oben gebrachte Zitat halte ich trotzdem in der Form für falsch (aber es sol ja auch eine simple Einführung sein, das ist ja wahrlich keine wissenschaftliche Veröffentlichung): Auf der Erde zum Beispiel ändert sich der Maßstab (in Schwarzschild-Metrik) nur in radialer, aber nicht in Umfangsrichtung; bei Aussagen wie “Maßstäbe ändern sich” oder “Uhren gehen langsamer” muss man auch immer aufpassen, worauf man sich bezieht. Von einem entfernten Beobachter aus gesehen geht eine Uhr auf der Erde langsamer; aber so etwas ist immer gefählrich und ich empfehle, möglichst immer nur lokale Experimente anzugucken.
    2, Mir scheint, dass du an verschiedenen Stellen Koordinatenabstände und physikalische Abstände durcheinanderwirfst. Wenn z.B eine GRavitationswelle auf eine Ansammlung von freien Atomen trifft, ändern sich deren Koordinaten in der üblicherweise verwendeten Metrik gar nicht, nur die Abstände ändern sich. Ähnliches gilt beim expandieren Universum.

  296. #296 Alderamin
    23. Januar 2018

    @Der Zimmermann

    Die Kraft kommt von der Dunklen Energie (DE). Wenn es in unserem Universum keine gäbe, dann gäbe es auch keine (auseinandertreibende) Kraft.

    Stimmt, sonst könnte a” nur negativ sein. Eine Expansion a’ >0 gäbe es aber trotzdem, nur würde sie sich verlangsamen.

    @Albrecht Storz

    Übrigens: in welchem der Entfernungsmaße die Du, Alderamin, aufzählst (“Es gibt noch weitere kosmologische Entfernungsmaße: Lichtlaufzeitentfernung (klar), Leuchtkraftentfernung (abgeleitet aus der Helligkeit einer Standardkerze, z.B. einer bestimmten Art von Supernovae), Winkelausdehnungsentfernung”) geht eigentlich nicht die SI-Basiseinheit “Länge” ein?

    Im Prinzip in alle, es hängt ja davon ab, wie man die Länge misst. Ein Urmeter ist winzig, da hat man beide Ende in 3 Nanosekunden Lichtlaufzeit Abstand, man kann den Meter neben ein Vergleichsobjekt legen und dann praktisch gleichzeitig an beiden Ende die Länge ablesen.

    Angenommen, Du hättest ein extrem langes Maßband und willst damit 1 Mrd. Lichtjahre abmessen. Wenn Du das Maßband erst abwickeln muss, dauert es schon mal mindestens 1 Mrd. Jahre, um es abzuwickeln. Bis dahin ist die Galaxie, deren Entfernung Du messen willst, schon wieder ein Stück weiter weg. Um das andere Ende abzulesen, brauchst Du wieder 1 Mrd. Jahre und Du bekommst Schwierigkeiten mit dem Moment, zu welchem die Messung durchgeführt werden soll, weil jemand, der am anderen Ende ablesen würde, einen relativ zu Dir verlangsamten Zeitablauf hätte (durch die fortwährende Expansion werden alle Perioden, die über die Strecke beobachtet werden, zeitlich gedehnt, das ist nichts anderes als die kosmologische Rotverschiebung, die betrifft nicht nur Licht, sondern auch z.B. die Dauer von Supernovae-Lichtkurven oder Gamma Ray Bursts).

    Die nachher abgelesene Länge des Maßbands wäre die Eigendistanz (allerdings vor 1 Mrd. Jahren). Misst du die Länge mit Lichtpulsen, wie mit einem Laser-Entfernungsmesser, dann bekommst Du die Lichtlaufzeitdistanz heraus. Triangulierst Du die Entfernunung mit einem Theodoliten, ergibt sich die Winkeldurchmesserdistanz. Und misst Du die Helligkeit einer Lichtquelle bekannter Helligkeit, dann die Leuchtkraftdistanz.

    Diese Maße lassen sich ineinander umrechnen, und sie sind wichtig, weil es eben verschiedene Möglichkeiten zur Entfernungsmessung gibt (insbesondere Leuchtkraft- und Winkeldurchmesserentfernung beziehen sich auf konkrete Messmethoden). Will man die Rotverschiebung z in Abhängigkeit von der Entfernung bestimmen, dann muss man eine einheitliche Entfernung ansetzen, und da nimmt man die Eigendistanz, die man direkt nicht messen kann. Für die Umrechnung benötigt man eine Formel, die die Entwicklung des Skalenfaktors beschreibt, d.h. ein konkretes Modell für die Expansion des Universums.

  297. #297 Albrecht Storz
    Mannheim
    27. Januar 2018

    At Alderamin und alle anderen

    Es tut mir Leid, aber wir werden nicht zusammen kommen können.
    Wenn der Raum sich verändern (expandieren oder was auch immer) kann, ohne dass dies Auswirkungen auf die darin eingebetteten Objekte, insbesondere Maßstäbe hat, warum dann nicht zB auch Veränderungen des Zeitablaufs für möglich halten, ohne dass diese Einfluss auf den Lauf der Uhren hätte? Beides erscheint mir genau so unsinnig.

    Aber – vor allem auch – wird dadurch schlicht jedes Beobachten und Messen zum schlechten Scherz: Messen ist immer ein Vergleichen anhand von Maßstäben.
    Wenn aber keine eindeutige Relation zwischen den benutzen Maßstäben und den damit gemessenen Objekten oder Zuständen besteht, kann man das Messen genauso gut lassen.
    Nehmen wir ein ganz plattes Beispiel mit lokalem Bezug 😉 : Ein Zimmermann misst die Höhe eines Dachstuhles aus. Allerdings wird der Dachstuhl danach höher (oder der Maßstab des Zimmermanns schrumpft): was ist seine Messung dann noch wert?

    Nun kommt vielleicht das Argument: der Zimmermann wüsste ja, wie sich der Dachstuhl vergrößert (oder sein Maßstab verkleinert) und könnte dies dann bei seinen Berechnungen und Ausführungen berücksichtigen.

    Aber weiß er denn diese Funktionen (also die Veränderung des Maßstabs gegenüber dem Raum, abhängig von Parametern wie Zeit oder Entfernung, etc.) genau? Wie will er sie wissen – ohne zu messen? Und wenn er misst (“misst er Mist”, hätte ich fast gesagt) um die Parameter zu ermitteln treten ja eben diese Fehler wieder auf.

  298. #298 DerZimmermann
    27. Januar 2018

    Wenn der Raum sich verändern (expandieren oder was auch immer) kann, ohne dass dies Auswirkungen auf die darin eingebetteten Objekte, insbesondere Maßstäbe hat, warum dann nicht zB auch Veränderungen des Zeitablaufs für möglich halten, ohne dass diese Einfluss auf den Lauf der Uhren hätte? Beides erscheint mir genau so unsinnig.

    Die Expansion “sagt”, dass die Entfernung zwischen weit entfernten Objekten (solchen, die vom Urknall “an ihren Platz gestellt” wurden) im Raum zunimmt. Nicht mehr und nicht weniger – und das kann man auch messen.
    Wenn dein Maßstab aus zwei solchen (man sagt auch “mitbewegten”) Objekten besteht, dann wächst er gleichmäßig mit (mit einem solchen Maßstab gemessene Distanzen würde man als “mitbewegte Entfernung” bezeichnen). Wenn dein Maßstab aus zwei nachträglich in einem bestimmten Abstand platzierten Objekten besteht, verändern sie ihren Abstand nicht (gleiches gilt natürlich auch z. B. für einen starren Stock); mit ihnen könnte man dann die “Eigendistanz” zweier mitbewegter Objekte messen – der Wert, der mit der Ausdehnung kontinuierlich ansteigt.
    (Ich bin jetzt der Einfachheit halber mal von einem gleichförmig expandierenden Universum mit a” = 0 ausgegangen).

    Ein Zimmermann misst die Höhe eines Dachstuhles aus. Allerdings wird der Dachstuhl danach höher (oder der Maßstab des Zimmermanns schrumpft): was ist seine Messung dann noch wert?

    Naja – er weiß dann eben, wie hoch der Dachstuhl zum Zeitpunkt der Messung war. Dann kann er ja ne Stunde warten und nochmal messen und damit (im Falle einer linearen Ausdehnung) sogar die Ausdehnungsrate bestimmen.

  299. #299 MartinB
    27. Januar 2018

    @Albrecht Storz
    Es gibt leider keine Möglichkeit, es anders zu sagen: Du hast schlicht nicht verstanden, was eine Ausdehnung des Raums (oder eine Änderung des Zeitablaufs) bedeutet, da hakt es bei dir irgendwo (leider kann ich deinen Kommentaren nicht entnehmen, wo genau).

    Ein Problem besteht vielleicht darin, dass du glaubst, der Abstand zweier Raumpunkte sei immer eindeutig. Das ist er nicht mal in der Newtonschen Physik:
    Nehmen wir an, ein Raumschiff fliegt an der Erde vorbei und ist einen Tag später beim Mond. Der räumliche Abstand dieser beiden Ereignisse hängt selbst in der Newtonschen Physik vom Bezugssystem ab: Von der Erde aus gesehen sind es so etwa 380000km, vom Raumschiff aus gesehen ist er Null.

    Und nur wenn sich die Maßstäbe nicht bei jeder Änderung des Raums mitändern, können wir sinnvoll von einer Ausdehnung des Raums reden, sonst würden wir das ja nie bemerken können.

    Ich verweise nochmal auf die beiden texte hier, die das Problem sehr detailliert auseinandernehmen:
    https://scienceblogs.de/hier-wohnen-drachen/2016/02/14/was-bedeutet-es-wenn-sich-der-raum-verzerrt/
    https://scienceblogs.de/hier-wohnen-drachen/2016/02/21/noch-einmal-die-raumverzerrung/

  300. #300 Albrecht Storz
    Mannheim
    28. Januar 2018

    Nun, vielleicht kann man solche Themen einfach nicht auf diese Art abhandeln. Vielleicht entsteht dadurch einfach zu viel Verwirrung – und auch, fürchte ich – ein hohes Maß an “Scheinverstehen”. Da sind dann Leute, die etwas vehement verteidigen obwohl sie es gar nicht kapiert haben. Ich kenne das leider zu gut.

    Ich möchte um das zu untermauern auf etwas hinweisen, was mir zumindest unvollständig wenn nicht sogar falsch dargestellt vorkommt:
    in den ganzen Ausführungen finde ich viel die Rede von Entfernung. Aber nirgends einen (deutlichen) Hinweis, dass kosmologische Entfernungen sich auf die Entfernung zwischen uns heute und den beobachteten Objekten ZU EINEM FRÜHEREN ZEITPUNKT beziehen.
    Nehmen wir eine Galaxie für die HEUTE aus der Rotverschiebung eine Entfernung von 13 Mrd. Lichtjahren ermittelt wird. Wir erhalten somit eine “Nachricht” (Information) über eine Gegebenheit von VOR 13 Mrd. Jahren.
    Nun wird es spannend: denn die Galaxie war, mit Expansion, nicht einmal vor 13 Mrd. Jahren dort wo wir sie heute sehen. Die 13 Mrd. Jahre Expansion hat sie ja nur scheinbar dorthin gerückt. Wohlgemerkt: an einen Ort vor 13 Mrd. Jahren!
    Aufgrund der Expansion muss die Galaxie vor 13 Mrd. Jahren viel näher gewesen sein. Und heute, nach 13 Mrd. Jahren, muss sie mit Expansion viel weiter weg sein (wobei das reden über den heutigen Zustand wahrscheinlich keinen Sinn macht. Wir können einfach gar nichts über den “heutigen” Zustand in 13 Mrd. Lichtjahren Entfernung wissen!).

    Alles verwirrend genug. Wie gesagt: was mich stört, was mich irritiert, ist, bei der Rede über kosmische Entfernungen (Raum) völlig den Zeitaspekt weg zu lassen. Ich habe gelernt, dass es die verschiedenen, UNTERSCHIEDLICHEN Entfernungen gibt, die sich alle AUF EIN OBJEKT beziehen.
    Sind das nun Aussagen über ein Objekt zu UNTERSCHIEDLICHEN Zeiten? Oder Aussagen über ein Objekt zu EINER Zeit, der Entfernungsbegriff evtl. einfach keinen Sinn macht oder zumindest völlig anders verstanden werden muss wie wir im Alltag ihn verstehen?
    Wie gesagt, vielleicht lässt sich das alles einfach nicht in normaler, dem Alltag verhafteten Sprache ausdrücken. Auf jeden Fall Danke für die Mühe, die ihr Euch gemacht habt.

  301. #301 DerZimmermann
    28. Januar 2018

    Nun, vielleicht kann man solche Themen einfach nicht auf diese Art abhandeln. Vielleicht entsteht dadurch einfach zu viel Verwirrung – und auch, fürchte ich – ein hohes Maß an “Scheinverstehen”. Da sind dann Leute, die etwas vehement verteidigen obwohl sie es gar nicht kapiert haben. Ich kenne das leider zu gut.

    Wenn du damit meinst, dass gerade bei diesem Thema viel Unsinn und unsinnige Erklärungen kursieren – da bin ich ganz bei dir.

    Sind das nun Aussagen über ein Objekt zu UNTERSCHIEDLICHEN Zeiten? Oder Aussagen über ein Objekt zu EINER Zeit, der Entfernungsbegriff evtl. einfach keinen Sinn macht oder zumindest völlig anders verstanden werden muss wie wir im Alltag ihn verstehen?

    Das ist eigentlich ganz einfach.
    Die tatsächliche, aktuelle Entfernung nennt man “Proper Distance” (“Eigendistanz”).
    Die Entfernung, die das Objekt zu dem Zeitpunkt hatte, als es das Licht ausgesendet hat, das wir heute sehen, nennt man “Angular diameter distance” (“Winkeldurchmesserentfernung”).

    Dann gibt es noch ein paar Dinge, die zwar “Entfernung” heißen, aber mit einer Entfernung eigentlich nichts zu tun haben. Z. B.
    – Laufzeitentfernung: Die Strecke, die das Licht des Objektes, das uns jetzt gerade erreicht, zurückgelegt hat. Liegt irgendwo zwischen Eigendistanz und Winkeldurchmesserentfernung.
    – Leuchtkraftentfernung: Gibt die Entfernung an, die man herausbekommen würde, wenn man aus der Helligkeit des Objektes auf seine Entfernung schließen wollte, ohne die Expansion mit einzuberechnen.

  302. #302 Bernhard Kletzenbauer
    Reichelsheim
    28. Januar 2018

    “…Nun wird es spannend: denn die Galaxie war, mit Expansion, nicht einmal vor 13 Mrd. Jahren dort wo wir sie heute sehen. Die 13 Mrd. Jahre Expansion hat sie ja nur scheinbar dorthin gerückt…”

    Das ist ein sehr wichtiger Beitrag zum Verständnis.
    Es gibt noch mehr Verwirrung wenn wir zurückrechnen wo ein sehr altes Objekt früher war, in Bezug zu unserem jetzigen, irdischen Beobachterstandpunkt – und wo jüngere Objekte waren, vor nicht so langer Zeit.

    Ich nehme mal nicht das Extrem mit 13 Milliarden. Aus Zeitmangel werde ich auch nicht genau nachrechnen, sondern nur schätzen.
    Wir haben beispielsweise eine alte Galaxie, deren Licht 10 Mrd. Jahre bis zu uns brauchte. Als das Licht vor 10 Mrd. Jahren losgeschickt wurde, befand sich diese Galaxie nur etwa 1 Mrd. Lichtjahre vom späteren Standpunkt des Planeten Erde entfernt. Aber damals war sie hier noch nicht sichtbar.
    Dann haben wir eine junge Galaxie, deren Licht nur 2 Mrd. Jahre bis zu uns braucht. Die war, als ihr Licht (8 Mrd. Jahre später) losgeschickt wurde, vielleicht auch 1 Mrd. Lichtjahre vom späteren Standpunkt des Planeten Erde entfernt.
    Wir haben also am Nachthimmel 2 Galaxien in zurückgerechnet 1 Mrd. Lichtjahren Entfernung.
    Natürlich erkennt man an der Rotverschiebung, welche die ältere- und welche die jüngere Galaxie ist.
    Aber durch den verspäteten Start des Lichts von der jüngeren Galaxie erreicht es uns zeitgleich mit dem Licht der älteren Galaxie. Es liegen aber 8 Milliarden Jahre Zeit zwischen beiden Licht-Startzeiten.
    Die ältere Galaxie ist längst hinter dem irdischen Sichthorizont verschwunden, während die jüngere Galaxie relativ “nahe” zur Erde steht und auch sehr lange brauchen wird, bis sie hinter dem Sichthorizont verschwinden wird.
    All das wird noch komplizierter durch die beschleunigte Expansion des Universums.
    Es wäre vielleicht ein schönes Thema für eine Doktorarbeit. Eine Animaton erstellen, die 2 Objekt-Standorte zeigt wenn:
    – sie ihr Licht losschicken,
    – das Licht bei uns ankommt.

  303. #303 MartinB
    28. Januar 2018

    “Aber nirgends einen (deutlichen) Hinweis, dass kosmologische Entfernungen sich auf die Entfernung zwischen uns heute und den beobachteten Objekten ZU EINEM FRÜHEREN ZEITPUNKT beziehen”
    Es ist immer eine Frage, was du betrachtest. In der Astronomie verwendet man ja ein Bezugssystem auf der Basis von “mitbwegten Beobachtern”, da kann man natürlich auch fragen, was der räumliche Abstand zwischen zwei Galaxien ist, die beide die gleice Zeitkoordinate haben. Das ist natürlich nicht dasselbe wie anzugucken, woher das Licht kommt, das ich jetzt beobachte, dazu gab’s hier ja auch mal nen Artikel “wie groß ist das beobachtbare Universum”.
    Nicht jeder Blogartikel kann auf alles eingehen.

  304. #304 Alderamin
    29. Januar 2018

    @Albrecht Storz

    in den ganzen Ausführungen finde ich viel die Rede von Entfernung. Aber nirgends einen (deutlichen) Hinweis, dass kosmologische Entfernungen sich auf die Entfernung zwischen uns heute und den beobachteten Objekten ZU EINEM FRÜHEREN ZEITPUNKT beziehen.
    Nehmen wir eine Galaxie für die HEUTE aus der Rotverschiebung eine Entfernung von 13 Mrd. Lichtjahren ermittelt wird. Wir erhalten somit eine “Nachricht” (Information) über eine Gegebenheit von VOR 13 Mrd. Jahren.
    Nun wird es spannend: denn die Galaxie war, mit Expansion, nicht einmal vor 13 Mrd. Jahren dort wo wir sie heute sehen. Die 13 Mrd. Jahre Expansion hat sie ja nur scheinbar dorthin gerückt. Wohlgemerkt: an einen Ort vor 13 Mrd. Jahren!
    Aufgrund der Expansion muss die Galaxie vor 13 Mrd. Jahren viel näher gewesen sein. Und heute, nach 13 Mrd. Jahren, muss sie mit Expansion viel weiter weg sein (wobei das reden über den heutigen Zustand wahrscheinlich keinen Sinn macht. Wir können einfach gar nichts über den “heutigen” Zustand in 13 Mrd. Lichtjahren Entfernung wissen!).

    Das war genau das, was ich oben angedeutet hatte:

    Die Kosmologen sprechen von “Mitbewegter Entfernung” (comoving distance), wenn sie eine Metrik verwenden, die mit der Raumexpansion mitwachsen soll, und von “Eigendistanz” (proper distance), wenn die Metrik nicht expandieren soll. So geht das. Im Kleinen braucht man so was nicht. Es gibt noch weitere kosmologische Entfernungsmaße: Lichtlaufzeitentfernung (klar), Leuchtkraftentfernung (abgeleitet aus der Helligkeit einer Standardkerze, z.B. einer bestimmten Art von Supernovae), Winkelausdehnungsentfernung (abgeleitet aus der Größe eines Standardlineals, z.B. korrelierte Bereiche in der kosmischen Hintergrundstrahlung), die im Kleinen alle zusammenfallen und im Großen alle verschieden sind.

    Nochmal Klartext: Die Entfernung aufgrund der Lichtlaufzeit (13 Milliarden Jahre in Deinem Beispiel) wird als “Lichtlaufzeitentfernung” bezeichnet. Das ist die Entfernung, die das Licht zurückgelegt hat, während der Raum vor und hinter ihm expandierte. Wie bei einer Ameise, die über einen Ballon von einem aufgemalten Punkt zu einem anderen läuft, während der Ballon aufgeblasen wird. Zu jedem Zeitpunkt auf dem Weg wächst die Gesamtentfernung zwischen den Punkten, aber für den Restweg der Ameise ist nur der Teil relevant, der noch vor ihr liegt; der hinter ihr liegende wächst, ohne einen Einfluss auf die Zeit bis zum Ziel zu haben. Deswegen liegt die zurückgelegte Strecke nachher zwischen der Anfangsentfernung und derjenigen am Ende der Strecke. Lichtlaufzeitentfernung.

    Was interessanter ist, ist die Entfernung zu einem bestimmten Zeitpunkt, die ein gedachtes Maßband ergeben würde. Das nennt sich Eigendistanz und kann nur berechnet, nicht gemessen werden, denn jede Messung von der Erde aus hat unvermeidlicherweise eine gewisse Lichtlaufzeitverzögerung. Man kann die Helligkeit von Standardkerzen oder von Standardlinealen verwenden (bei supermassiven Schwarzen Löchern gab’s da irgendeine Methode unabhängig von Supernovae, müsste ich zuerst raussuchen, wie das funktionierte). Auch die “Leuchtkraft” von Gravitationswellenereignissen hat man schon verwendet (auf die man wohl aus der Frequenz des Ereignisses durch Modellierung schließen konnte). Jedenfalls ergibt sich aus der Formel für die Entwicklung der Expansion des Universums (die Entwicklung des Skalenfaktors) aus solchen Messungen die aktuelle Eigendistanz.

    Will man die Eigendistanz zu einem anderen Zeitpunkt wissen (etwa zu der Zeit, als das Licht ausgesendet wurde) braucht man den Skalenfaktor zu diesem Zeitpunkt. Der ist ziemlich einfach aus der Rotverschiebung z zu bestimmen: z=0 heißt keine Verschiebung, z=1 eine Verschiebung auf die doppelte Wellenlänge, z=3 auf die dreifache etc., also auf das z+1-fache. Und um den Faktor z+1 ist dann auch das Universum während der Lichtlaufzeit gewachsen, denn die Lichtwellenlänge wuchs genau um diesen Faktor mit. Vergleicht man die Wellenlänge mit der ursprünglichen (was man leicht kann, man verwendet ja z.B. die Linien des Wasserstoffs, die eine bestimmte Anordnung haben, welche man wiedererkennt), dann hat man sofort den Skalenfaktor a=1/(z+1). Bei einer Galaxie mit z=10 (ungefähr aktueller Rekord) war das Weltall also 11-mal kleiner als heute. Die kosmische Hintergrundstrahlung hat ca. z=1080, also war das Weltall damals 1080-mal kleiner (und auch 1080-mal heißer als die Hintergrundstrahlung heute ist, 2,8 K -> es waren damals ca. 3000 K).

    Schließlich noch die mitbewegte Entfernung: wenn man ein bestimmtes (sehr großes) Volumen im Universum betrachten will und wissen will, wie sich die Objekte darin relativ zueinander bewegen, wenn man die Expansion abzieht, dann betrachtet man die mitbewegte Entfernung. Dazu friert man die aktuelle Eigendistanz zum heutigen Zeitpunkt einfach ein (oder dividiert die Eigendistanz zur jeweiligen Zeit durch den Skalenfaktor zu dieser Zeit). Dann ist ein bestimmtes Volumen immer gleich groß und enthält dieselben Galaxien, über die gesamte Lebensdauer des Universums.

    Die verschiedenen Entfernungen kann man ineinander umrechnen, z.B. mit dem Cosmic Distance Calculator. Man gibt oben links z ein und drückt dann auf “Flat” für ein flaches Universum. Die Dichte der Materie (OmegaM) relativ zur kritischen Dichte für ein flaches Universum (1) und die aktuelle Hubble-Konstante in km/s/Megaparsec sind schon eingetragen; wenn man auf “Flat” tippt, wird die Dichte der Vakuumenergie (OmegaVac, Dunkle Energie) auf 1-OmegaM gesetzt, so ist der derzeitige Stand des Lambda-CDM-Modells (das Weltall ist flach, OmegaM+OmegaVac=1). Die Knöpfe für “Open” und “General” erlauben es, die Distanzen für ein Weltall ohne Vakuumenergie (Open) oder für beliebige Vakuumenergie (General) auszurechnen (für den letzten Fall kann man deren Dichte im Feld über “General” verändern).

    Rechts erscheinen dann
    * das Alter des Universums laut den eingegebenen Omega- und Hubble-Werten,
    * das Alter des Universums zur Zeit, als das Licht mit Rotverschiebung z auf den Weg ging,
    * die Lichtlaufzeit,
    * die mitbewegte Entfernung = der aktuellen Eigendistanz (die mal 1/(z+1) wäre dann die Eigendistanz zur Zeit, als das Licht auf den Weg ging)
    * die Winkeldurchmesserentfernung, wenn man ein Standardlineal vermessen würde
    * wieviel Strecke ein Winkel von einer Bogensekunde (1″) in diesem Abstand ausmachen würde (wenn man beispielsweise den Durchmesser einer fernen Galaxie wissen möchte, deren Sehwinkel in ” man gemessen hat)
    * und die Leuchtkraftentfernung, die man benötigt, wenn man die Helligkeit z.B. einer Supernova ausrechnen möchte (die mit dem Quadrat dieser Entfernung in der Helligkeit abnimmt).

    Nochmal: auf kurze Entfernung wären alle diese Maße gleich, aber da sich das Weltall ständig vergrößert, werden die verschiedenen Maße auf unterschiedliche Weise verändert und sie driften auseinander. Wenn man will, ist die Eigendistanz die “echteste” Entfernung, die aktuelle Raumentfernung (wenn ich das bei Martin richtig gelernt habe, ist das eine “raumartige Geodäte” von hier bis zu diesem Punkt). Die kann man aber leider nur indirekt aus anderen messbaren Entfernungen berechnen, wenn man weiß, wie das Weltall in der Vergangenheit expandiert ist. Und das verrät einem das Lambda-CDM-Modell mit den Parametern, die man oben links eintippt. Die werden seit Jahren immer genauer bestimmt.

  305. #305 Albrecht Storz
    Mannheim
    29. Januar 2018

    “Nochmal Klartext: Die Entfernung aufgrund der Lichtlaufzeit (13 Milliarden Jahre in Deinem Beispiel) wird als “Lichtlaufzeitentfernung” bezeichnet.”

    Ich übersetze: “Wir sehen ein Galaxie in einer Entfernung von 13 Mrd. Licht-Jahren. So sah die Galaxie also vor 13 Mrd. Jahren aus.”
    Gleicheitig wird aber gesagt, dass wir das Bild einer viel jüngeren, näheren Galaxie sehen würden. Denn die Winkeldurchmesserentfernung, eine unmittelbar (ohne große Rechenleistung und ohne zuhilfenahme von Zusatzannahmen) erfahrbare Information über das Objekt, ist viel kleiner.

    Nun misstraut man der unmittelbaren Erfahrung und macht statt dessen komplexe Annahmen über eine Expansion des Raumes (die sich auf duch Kräfte gebundene Objekte wie Maßstäbe nicht auswirken würde, etc.), anstatt komplexe Annahmen über mögliche Gründe für die Verschiebung der Spektrallinien nach Rot zu machen. Oder sogar anstatt ganz einfache Annahmen über die Verschiebungen der Spektrallinien zu machen: dass nämlich entferntere Galaxien einfach eine höhere Fluchtgeschwindigkeit besitzen könnten.

    Und warum das Ganze? Weil es zB nicht sein darf, aus irgend welchen Gründen nicht sein kann, dass wir in (der Nähe) der Mitte des Weltalls sitzen könnten?

    Verstehe ich das ganz falsch?

  306. #306 MartinB
    30. Januar 2018

    @AlbrechtStorz
    “Verstehe ich das ganz falsch?”
    Ja. Ich werdde jetzt aber keinen Artikel über die vielen vielen Gründe schreiben, weshalb wir wissen, dass sich das Universum ausdehnt, davon gibt es genügend.

  307. #307 MartinB
    30. Januar 2018

    PS
    Ich finde es immer wieder erstaunlich, für wie dämlich manche Menschen Wissenschaftler halten. Glaubst du ernsthaft, wir reden seit 90 Jahren über Expansion nur weil wir alle ganz fest daran glauben, dass die Erde nicht im Mittelpunkt des Universums sitzen kann, und du bist der erste, der sich fragt, ob man die Beobachtungen nicht auch anders erklären könnte als durch Expansion?

  308. #308 Alderamin
    30. Januar 2018

    @Albrecht Storz

    Ich übersetze: “Wir sehen ein Galaxie in einer Entfernung von 13 Mrd. Licht-Jahren. So sah die Galaxie also vor 13 Mrd. Jahren aus.”

    So weit richtig.

    Gleicheitig wird aber gesagt, dass wir das Bild einer viel jüngeren, näheren Galaxie sehen würden. Denn die Winkeldurchmesserentfernung, eine unmittelbar (ohne große Rechenleistung und ohne zuhilfenahme von Zusatzannahmen) erfahrbare Information über das Objekt, ist viel kleiner.

    Jünger ist klar, weil das Licht 13 Milliarden Jahre alt ist. Größer erscheint die Galaxie, weil in der Ferne ein früher kleineres Universum auf den Außenrand unseres heute größeren Universums projiziert erscheint. Das geht ja gar nicht anders. Näher erscheint die Galaxie auch schon deswegen, weil sie sich seit der Zeit, zu der ihr heute hier eintreffendes Licht auf den Weg ging, schon viel weiter entfernt hat. Auch das ist unvermeidlich. Das stimmt ja sogar schon im Sonnensystem, wo die Lichtlaufzeit ein paar Stunden beträgt. Was unsere Raumsonden da draußen anstellen, erfahren wir mit Zeitversatz. Bis dahin haben sie sich schon weiter bewegt. Man kann sich auf den Standpunkt stellen, “jetzt” ist nur, was man jetzt auch sieht. Man kann aber auch ausrechnen, was instantan zur selben Zeit am entfernten Ort wäre. Daraus resultieren bei bewegten Objekte zwei verschiedene Orte bzw. Entfernungen.

    Nun misstraut man der unmittelbaren Erfahrung

    Welche unmittelbare Erfahrung hast Du mit Milliarden Lichtjahre großen Strukturen?

    und macht statt dessen komplexe Annahmen über eine Expansion des Raumes (die sich auf duch Kräfte gebundene Objekte wie Maßstäbe nicht auswirken würde, etc.),

    Das Objekte an der Expansion nicht teilnehmen müssen, folgt doch trivial aus dem Artikel oben.

    anstatt komplexe Annahmen über mögliche Gründe für die Verschiebung der Spektrallinien nach Rot zu machen. Oder sogar anstatt ganz einfache Annahmen über die Verschiebungen der Spektrallinien zu machen: dass nämlich entferntere Galaxien einfach eine höhere Fluchtgeschwindigkeit besitzen könnten.

    Und wir sitzen exakt im Zentrum? Warum ist dann in jeder Richtung ganz außen der Feuerball zu sehen, ganz symmetrisch zu uns? Warum sind Standardlineale wie Strukturen in der Hintergrundstrahlung dann entsprechend vergrößert? Das kann man ja messen. 100000 Lichtjahre erscheinen in der Hintergrundstrahlung unter einem Sehwinkel von 1° (wenn ich mich da richtig erinnere). Der 5500-mal nähere Andromedanebel mit auch etwas um die 100000 LJ Durchmesser erscheint 4° groß. Wie erklärst Du den Lupeneffekt in der Hintergrundstrahlung?

    Wie Martin sagt, die Kosmologen wissen schon, warum sie gewisse Annahmen treffen oder verwerfen können. An diesem Gebäude haben sehr viele Leute sehr lange gebaut, und viele auch immer wieder versucht, es einzureißen. Aus jedem dieser Versucht ging es gestärkt hervor.

    Und warum das Ganze? Weil es zB nicht sein darf, aus irgend welchen Gründen nicht sein kann, dass wir in (der Nähe) der Mitte des Weltalls sitzen könnten?

    Weil es konsistent mit den Beobachtungen und der Relativitätstheorie ist. Die erlaubt nämlich kein still stehendes Universum, es muss expandieren oder kollabieren. Expandieren es tut.

    Verstehe ich das ganz falsch?

    Offenbar.

  309. #309 Albrecht Storz
    Mannheim
    30. Januar 2018

    “Ich finde es immer wieder erstaunlich, für wie dämlich manche Menschen Wissenschaftler halten. Glaubst du ernsthaft, wir reden seit 90 Jahren über Expansion …”

    MartinB,
    Wissenschaftler sind tatsächlich genauso dämlich wie Nichtwissenschaftler. Das sage ich, und dazu stehe ich. Und dass das Argument mit den Wissenschaftlern und den 90 Jahren irgend etwas anderes bedeuten könnte, als dass die heilige Inquisition der römisch-katholischen Kirche eben doch Recht gehabt hätte, sollte Dir wohl schon klar sein. Immerhin hielt sich das Konzept nicht nur 90 Jahre sondern über Jahrhunderte und wurde von zig Millionen Menschen, darunter Päpste, Theologen, Philosophen, Naturforschern, Priestern, Mönchen und vielen, vielen anderen Gläubigen verteidigt.

    Ich finde es erstaunlich, dass Du Dir scheinbar nicht darüber bewusst bist, dass das ganze Gedankengebäude der Raumexpansion auf mindestens zwei Dogmen aufbaut:
    – Das Weltall ist homogen
    – Das Weltall ist isotrop
    (ich würde noch als Drittes hinzufügen: – wir können nicht im oder nahe am Mittelpunkt des Weltalls sitzen)
    Das nennt man das Kosmologische Prinzip.
    https://de.wikipedia.org/wiki/Kosmologisches_Prinzip

    Ich lese dort, dass das olberssche Paradoxon für die Urknall-Theorie (wohl mit Raumexpansion, also Lambda-CDM-Modell) sprechen würde. Aber genauso wird es mE aufgelöst durch zB ein auseinanderstrebendes Weltall, dessen Raum “nur” mit den am weitesten entfernten Objekten “am Rand” wächst. Das ist aber nur so eine Idee. Es gibt beliebig viele andere Modelle, die dieses (spezielle) Problem lösen können.

  310. #310 MartinB
    30. Januar 2018

    “Es gibt beliebig viele andere Modelle, die dieses (spezielle) Problem lösen können.”
    Name ten, sagt man in solchen Fällen.
    Und nicht vage, hingehustete Ideen (ich stell mir mal vor, das Weltall wächst nur am Rand), sondern sauber durchgerechnete Modelle, die zu den Beobachtungen und allen Messdaten passen.

    Der Dogma-Vorwurf wäre möglicherweise ein Argument, wenn es nicht nachgewiesenermaßen in der Vergangenheit immer auch andere Modelle gegeben hätte – auch solche, bei denen das All nicht homogen und isotrop ist. Dumm nur, dass diese Modelle einfach nicht funktionieren.

    Und so Dinge wie homogen und isotrop kann man messen – ich sage mal WMAP als Beispiel, Alderamin wird dich bestimmt gern mit weiteren Daten zuschütten.

  311. #311 Albrecht Storz
    Mannheim
    30. Januar 2018

    Alderamin,

    Du hast von meiner Post eventuell einiges missverstanden.

    Die unmittelbare Erfahrung von der ich spreche ist die unmittelbare Erfahrung des Lichts das durch ein Teleskop HIER und HEUTE auf ein Auge trifft. Gut, heutzutage ist zwischen Auge und Teleskop auch schon eine Menge Verarbeitung geschaltet.
    Aber das ist alles immer noch viel weniger artifiziell, also immer noch viel unmittelbarer, als etwa Ergebnisse aus Simulationsmodellen in die hunderte von Parametern und Annahmen aufgrund von (Ziel-)Vorstellungen einfließen.
    Von dem her halte ich die Ansicht, dass der Winkeldurchmesser eine der klarsten und unverfälschbarsten und am wenigsten falsch interpretierbaren Informationen über unseren Kosmos ist, für durchaus vertretbar.

    “Jünger ist klar, weil das Licht 13 Milliarden Jahre alt ist. Größer erscheint die Galaxie, weil in der Ferne ein früher kleineres Universum auf den Außenrand unseres heute größeren Universums projiziert erscheint. Das geht ja gar nicht anders. Näher erscheint die Galaxie auch schon deswegen, weil sie sich seit der Zeit, zu der ihr heute hier eintreffendes Licht auf den Weg ging, schon viel weiter entfernt hat.”

    Das ist für mich einigermaßen kryptisch.
    Zuerst einmal “jünger”. Ich habe damit gemeint, dass wir, wie FF glaube ich auch geschrieben hat, ein jüngeres Bild (also weniger alt als 13 Mrd. Jahre) sehen würden. Wir würden nämlich ein Bild der Galaxie sehen, wie sie entsprechend der Winkeldurchmesserentfernung ENTFERNT und ALT war.
    Ich versuche es mal metaphorisch: uns würde ein Photo der Galaxie, im Alter entsprechend der Winkeldurchmesserentfernung, überbracht werden, die Auslieferung des Photos erfolgt allerdings erst sehr verspätet, nämlich erst nach verstreichen der Zeit entsprechend der Lichtlaufzeitentfernung.

    Was Du mit “auf den Außenrand unserer Galaxie projiziert” meinst ist mir allerdings wirklich schleierhaft.
    Dass die Galaxie näher erscheint (im Vergleich zu wann oder was?) verstehe ich mangels zugeordnetem Argument nicht. Aus dem nachfolgenden Beispiel schließe ich, dass Du die triviale Ortsveränderung während der Lichtlaufzeit meinst. Genau hier schlägt aber wieder die immanente Verwirrung zu.
    Die Galaxie soll doch HIER und JETZT Aufgrund des Bildes das wir empfangen haben näher erscheinen als es der Lichtlaufstrecke entspricht.

    Hat eigentlich noch niemand Sprachmodi entwickelt, aufgrund derer diese ständigen Missverständnisse vermieden werden könnten? Man muss sich winden und drehen, damit klar ist, ob man über die HEUTE HIER vorliegende Information oder über den DAMALS DORT oder den HEUTE DORT vorliegenden Zustand spricht, etc., und dann gibt es auch noch mehrere DORT und entsprechend mehrerer DAMALS entsprechend den verschiedenen Entfernungs-/Zeit-Angaben.
    Das wäre doch mal wirklich eine schöne Aufgabe: die Entwicklung von ein-eindeutigen Sprachkonventionen zur Diskussion von Relationen zwischen Beobachtungen und Beobachtungsgegenständen in relativistischen, kosmologischen Modellen mit multiplen Entfernungsmaßen.

  312. #312 Albrecht Storz
    30. Januar 2018

    ““Es gibt beliebig viele andere Modelle, die dieses (spezielle) Problem lösen können.”
    Name ten, sagt man in solchen Fällen.”

    MartinB,
    mag sein, dass man das sagt. Allerdings ist meine Aussage mehr als trivial. Das ist Wissenschaftstheorie. Genau so, wie man durch eine endliche Anzahl von Punkten beliebig viele verschiedene Kurven legen kann, wenn man ausreichend viele freie Kurvenparameter hat (zB Polynomzug), so kann man auch eine endliche Anzahl von Beobachtungen mit beliebig vielen verschiedenen Modellen modellieren, wenn man beliebig komplexe Modelle zulässt.
    Urknall mit Inflation, Lambda-cdm, Raumexpansion, dunkle Materie und Energie, etc., das ist gewiss kein einfaches, elegantes Modell wie es zB ein Einstein gesucht hätte.

  313. #313 MartinB
    31. Januar 2018

    @Albrecht STorz
    “Hat eigentlich noch niemand Sprachmodi entwickelt, aufgrund derer diese ständigen Missverständnisse vermieden werden könnten? ”
    Wozu hat dir denn Alderamin die Terminologie erklärt? Dein Unverständnis ist nicht die Schuld derjenigen, die die Theorien aufstellen.

    “Das ist Wissenschaftstheorie.”
    Nein, deine Aussage ist nicht trivial, sie ist schlicht falsch. Es gibt (zumndest meines Wissens) keine alternative Erklärung der Expansion, die alle Fakten (Galaxienverteilung, Elemente im Universum, WMAP, Rotverschiebung, Übereinstimmung mit der Allg. RT etc. etc.) richtig vorhersagt, falls es doch eine gibt, sag, wo die veröffentlicht ist.

    Das mit der Wissenschaftstheorie ist allerdings trivial – aber man solltes an physikalische Theorien auch nicht größere Forderungen stellen als an andere Theorien. Wenn du gegen den Tisch stößt und ein Glas darauf umfällt, dann redest du dich auch nicht damit raus, dass es ja theoretisch auch eine Erklärung geben könnte, nach der dein Stoßen gegen den Tisch gar keine Konsequenzen hatte und das ganze von unsichtbaren Aliens getan wurde. Wenn wir so argumentieren, müssen wir alle wissenschaftlichen Theorien ablehnen, nicht nur die, die dir aus irgendeinem Grund nicht gefällt, dann können wir aber auch morgens nicht mehr aufstehen, weil wir ja nie wissen können, ob die Welt nicht in Wahrheit ganz anders funktioniert.

    Im übrigen hast du ja was von “Dogma” gesagt, das ist noch wieder ne andere Argumentation – du verschiebst die Torpfosten von Post zu Post. Das wird mir deshalb jetzt auch zu nervig, ich steige hier aus.

  314. #314 Alderamin
    31. Januar 2018

    @Albrecht Storz

    Die unmittelbare Erfahrung von der ich spreche ist die unmittelbare Erfahrung des Lichts das durch ein Teleskop HIER und HEUTE auf ein Auge trifft.

    Die wird auch vom Weltall verzerrt wie durch eine Linse, s.u.

    Aber das ist alles immer noch viel weniger artifiziell, also immer noch viel unmittelbarer, als etwa Ergebnisse aus Simulationsmodellen in die hunderte von Parametern und Annahmen aufgrund von (Ziel-)Vorstellungen einfließen.

    Auf diese Weise kann man die Entstehung von Galaxien reproduzieren, aber das Lambda-CDM-Modell, das die Expansion im Großen beschreibt, braucht nur 6 Parameter, die man auf verschiedene Weisen messen kann, und die Friedmann-Gleichung, die aus der allgemeinen Relativitätstheorie hergeleitet wird (bzw. ihre Lösung für den speziellen Fall eines homogen mit Materie gefüllten Universums ist). Setzt man die Parameter in die Gleichung ein, folgt die Expansionsgeschichte des Universums und der Zusammenhang zwischen den Entfernungsmaßen.

    Von dem her halte ich die Ansicht, dass der Winkeldurchmesser eine der klarsten und unverfälschbarsten und am wenigsten falsch interpretierbaren Informationen über unseren Kosmos ist, für durchaus vertretbar.

    Sie ist nicht weniger unverfälschbar als die Leuchtkraftentfernung. Aber durchaus verschieden von ihr, weil sie von der Expansion des Universums anders beeinflusst wird.

    Zuerst einmal “jünger”. Ich habe damit gemeint, dass wir, wie FF glaube ich auch geschrieben hat, ein jüngeres Bild (also weniger alt als 13 Mrd. Jahre) sehen würden. Wir würden nämlich ein Bild der Galaxie sehen, wie sie entsprechend der Winkeldurchmesserentfernung ENTFERNT und ALT war.

    Nein! Die Winkeldurchmesserentfernung unterliegt ledglich einer optischen Verzerrrung der Größe von Objekten, wie beim Blick durch eine Glaskugel oder ein Glas Wasser. Das Alter einer Galaxie kann man beispielsweise an der Sternentstehungsrate und dem Gehalt an schweren Elementen abschätzen. Dadurch, dass das Bild der Galaxie verzerrt wird, ändern sich die beobachtete Sternentstehungsrat oder die Spektrallinien schwerer Elemente in keinster Weise (nicht einmal die Rotverschiebung). NUR der Winkeldurchmesser. Und der wird mit der Entfernung absurd groß.
    Die Leuchtkraft unterliegt einer entsprechenden, aber anderen Verzerrung und ergibt demgemäß einen anderen Wert, wenn man so recht, als gäbe es die Verzerrung nicht.

    Ich versuche es mal metaphorisch: uns würde ein Photo der Galaxie, im Alter entsprechend der Winkeldurchmesserentfernung, überbracht werden, die Auslieferung des Photos erfolgt allerdings erst sehr verspätet, nämlich erst nach verstreichen der Zeit entsprechend der Lichtlaufzeitentfernung.

    Nein, die Lichtlaufzeit ist gänzlich unbeeinflusst von der Verzerrung der Galaxie. Diese ganzen Entfernungsmaße sind die Werte, die man herausbekommt, wenn man mit den verschiedenen Entfernungsmessmethoden so misst und rechnet, wie man es von einer nicht-expandierenden Raumzeit gewohnt ist. Die Entfernungen sind aber in Wahrheit gar nicht verschieden, sondern verzerrt, und müssen korrigiert werden, um die Effekte der expandierenden Raumzeit herauszurechnen. So wie man den Winkeldurchmesser eines Objekts im Teleskop durch die Vergrößerung teilen muss, um den wahren Winkeldurchmesser zu bestimmen.

    Was Du mit “auf den Außenrand unserer Galaxie projiziert” meinst ist mir allerdings wirklich schleierhaft.

    Ich meinte auf den Außenrand des Universums projiziert. In einem gewöhnlichen, nichtexpandieren Raum nimmt der absolute Durchmesser eines Sehwinkels linear zu (Strahlensatz), d.h. wenn ich den Mond mit der Kuppe des kleinen Fingers bei gestrecktem Arm abdecken kann, haben sie den gleichen Sehwinkel, aber der Mond ist 200 Millionen mal weiter weg als die Armlänge und somit auch 200 Millionen mal größer als die Fingerkuppe (mal über den kleinen Finger gepeilt 😉 ). Und somit sieht eine Struktur in der kosmischen Hintergrundstrahlung in einer Entfernung (Eigendistanz) von 42 Milliarden Lichtjahren auch riesig groß aus. Verfolgt man die Sichtlinie aber in die Vergangenheit zurück, dann war das, was man sieht, damals viel kleiner und die Strahlen, die den Sehwinkel begrenzen, laufen nicht geradlinig sondern gekrümmt. Ich weiß nicht, wie ich’s mit Worten besser beschreiben soll, aber am Firmament erscheint eine ursprünglich kleine Kugel, die uns umgibt, heute viel größer, wenn man sie mit dem heutigen Radius bemisst aber mit der damaligen Ausdehnung sieht.

    Hat eigentlich noch niemand Sprachmodi entwickelt, aufgrund derer diese ständigen Missverständnisse vermieden werden könnten?

    Ich hatte den Wikipedia-Artikel zu den Begriffen verlinkt. Da die Effekte im normalen Leben nicht beobachtbar sind, haben sie im Alltagsleben auch keinen Alltagsnamen. Wir haben auch nicht 90 Namen für Schnee, wie die Eingeboren der Arktis (angeblich).

    Man muss sich winden und drehen, damit klar ist, ob man über die HEUTE HIER vorliegende Information oder über den DAMALS DORT oder den HEUTE DORT vorliegenden Zustand spricht, etc., und dann gibt es auch noch mehrere DORT und entsprechend mehrerer DAMALS entsprechend den verschiedenen Entfernungs-/Zeit-Angaben.

    Nein, man muss einfach wissen, wie die Entfernungen gemessen wurde und was man damit ausdrücken will. Dazu muss man natürlich intus haben, was sie meinen und wie sie von der Raumexpansion beeinflusst werden.

    Das wäre doch mal wirklich eine schöne Aufgabe: die Entwicklung von ein-eindeutigen Sprachkonventionen zur Diskussion von Relationen zwischen Beobachtungen und Beobachtungsgegenständen in relativistischen, kosmologischen Modellen mit multiplen Entfernungsmaßen.

    Unter den Leuten, die mit ihnen zu tun haben, sind die Begriffe klar und sauber definiert. Anderen steht frei, die Begriffe zu lernen. Man könnte Chirurgen auch vorwerfen, warum sie zum Skalpell nicht “Messer” sagen. Grund wird sein, dass es viele Arten von Messern gibt, und das Skalpell ein sehr spezifisches Messer ist, das der Chirurg genau so benötigt, deshalb hat es einen spezifischen Namen, den man als Nicht-Chrirug nicht wissen muss (außer, man kommuniziert mit Chirurgen oder über deren Arbeit).

  315. #315 Alderamin
    31. Januar 2018

    @myself

    wenn man so recht, als gäbe es die Verzerrung nicht.

    rechnet war natürlich gemeint.

  316. #316 Albrecht Storz
    31. Januar 2018

    Alderamin,

    das mit “auf den Außenrand unserer Galaxie projiziert” war natürlich ein Lapsus. Entschuldige bitte.
    Verstehen tu ich diese Redeweise (mit “Universum” statt “Galaxie”) trotzdem nicht.

    Aber eines ist mir inzwischen auf alle Fälle klar geworden, nämlich dass der eindeutige Zusammenhang zwischen Raum und Zeit der “klassisch-relativistischen Physik” durch die Lichtgeschwindigkeit, in einem expandierenden Universum aufgehoben wird.
    In der klassisch-relativistischen Physik ist ja eindeutig klar, dass wir ein Ereignis, das wir HIER und HEUTE in zB 10 Mrd. Lichtjahren Entfernung beobachten, sich auch dort und vor genau 10 Mrd. Lichtjahren ereignet hat (von Gravitationslinseneffekte und andere möglichen, relativ geringfügigen Verzerrung abgesehen).
    In der Lambda-CDM-Kosmologie wird diese Beziehung (Entfernung/c = vergangene Zeit seit dem Ereignis) zugunsten einer äußerst komplizierten Beziehung aufgegeben. Ist die Theorie tatsächlich so gut und gesichert, dass man zu solchen Opfern bereit ist?

  317. #317 Alderamin
    31. Januar 2018

    @Albrecht Storz

    In der Lambda-CDM-Kosmologie wird diese Beziehung (Entfernung/c = vergangene Zeit seit dem Ereignis) zugunsten einer äußerst komplizierten Beziehung aufgegeben. Ist die Theorie tatsächlich so gut und gesichert, dass man zu solchen Opfern bereit ist?

    Die Empirie gibt nichts anderes als eine komplexe Entwicklung des Skalenfaktors her, und man kann diese ja auch begründen (wechselseitige Gravitation mit der ART, konstante Abstoßung durch die dunkle Energie; bei höherer Materiedichte überwog zunächst die Gravitation und verlangsamte die Expansion, bis die Materiedichte so weit abgenommen hatte, dass die Abstoßung durch die dunkle Energie die Oberhand gewann).

    Es gibt mehrere durchaus sehr unterschiedliche Belegketten, die alle das gleiche Weltalter (das aus der Historie des Skalenfaktors folgt) bestätigen, wie etwa das Alter der ältesten Sterne oder die Ergebnisse von Messungen baryonischer Oszillationen in der Hintergrundstrahlung und Galaxienverteilung. Man beachte, dass dies quantitative Ergebnisse sind, die hervorragend zu den Ergebnissen der üblichen Standardkerzenmessungen für die Beziehung Entfernung-Rotverschiebung passen. Da kann man Zufall wohl ausschließen.

    Man muss da aber nichts opfern, in einem expandierenden Universum ändern sich nun einmal die Entfernungen laufend. Entfernung/c = Laufzeit gilt eben nur bei zeitlich einigermaßen konstanter Entfernung.

  318. #318 Albrecht Storz
    31. Januar 2018

    MartinB hat sich ausgeklinkt. Trotzdem möchte ich noch darauf hinweisen, dass auch arrivierte Wissenschaftler an als gesichert geltenden Theorien Zweifel vortragen können (ohne deshalb gleich eine vollständig ausgearbeitete Alternative vorweisen zu müssen). Hier geht es halt um die Inflationstheorie die aber doch eng mit der Theorie der Expansion verwoben ist:

    https://backreaction.blogspot.de/2017/10/is-inflationary-universe-scientific.html

    “There’s no warning sign you when you cross the border between science and blabla-land. But inflationary model building left behind reasonable scientific speculation long ago. I, for one, am glad that at least some people are speaking out about it. And that’s why I approve of the Steinhardt et al criticism. ”

    Dr. S. Hossenfelder

    Die Verweigerung eines Disputs ist auf jeden Fall keine Lösung. Aber natürlich respektiere ich es, wenn jemand besseres (oder einfach anderes) zu tun hat. Don’t worry.

  319. #319 Albrecht Storz
    31. Januar 2018

    Alderamin,
    eines versteh ich dann aber nicht: warum gibt man weiterhin bezüglich eines bestimmten kosmologischen Ereignisses immer nur einen einzigen Wert für dessen Verortung in der Raumzeit an?
    Nehmen wir zB die Supernova ASASSN-15lh
    https://de.wikipedia.org/wiki/ASASSN-15lh

    Es wird (lt. Wiki) eine Entfernung (leider ohne weitere Spezifizierung) von 3,8 Milliarden Lichtjahre angegeben. Warum wird nicht der Vollständigkeit halber auch die Zeitkoordinate, die ja nicht 3,8 Mrd. Jahre betragen kann, angegeben?

    Hier
    https://www.rochesterastronomy.org/sn2015/index.html#2015L
    wird ein (z=0.2326) angegeben. Die Rotverschiebung?
    Wenn ich das richtig verstanden habe, kann man damit aber nur die verschiedenen Entfernungsmaße umrechnen, aber nicht das Alter berechnen.

  320. #320 DerZimmermann
    1. Februar 2018

    Hier (…) wird ein (z=0.2326) angegeben. Die Rotverschiebung?
    Wenn ich das richtig verstanden habe, kann man damit aber nur die verschiedenen Entfernungsmaße umrechnen, aber nicht das Alter berechnen.

    Wenn ich die Laufzeitentfernung habe, habe ich doch automatisch auch das Alter. Aus der Rotverschiebung (dem Maß, das man auch wirklich konkret messen kann) kann ich alles andere berechnen, darum wird auch oft nur die Rotverschiebung z angegeben.

    Wiki gibt als Quelle für die Entfernungsabschätzung den “Spiegel” an…. dass es solche Nachrichtenmagazine mit diesen Unterscheidungen nicht so genau nehmen, sondern lieber möglichst beeindruckende Zahlen präsentieren wollen, ist leider fast immer so.

  321. #321 MartinB
    1. Februar 2018

    @Albrecht STorz
    Inhaltlich bin ich raus, aber das kann ich jetzt doch nicht stehen lassen, weil du andere für dich zu vereinnahmen versuchst.
    Sabine Hossenfelder ist eine brillante Physikerin – die aber wahrlich nicht taugt, um deine vollkommen schwammigen “Ich habe zwar den Urknall und all das nicht verstanden, aber ich bin dagegen”-Ansichten zu unterstützen.

    Wie vereinbarst du übrigens den Post mit deiner Behauptung, dass man ja in der Physik völlig im Dogma gefangen sei? Was geht mich mein Geschwätz von gestern an, oder wie? Bevor man versucht, etablierte und (wie Alderamin ja immer wieder mit unendlicher Geduld zeigt, gut belegte) Theorien zu hinterfragen, sollte man vielleicht erstmal überlegen, ob das eigene Denken wenigstens halbwegs konsistent ist.

  322. #322 Albrecht Storz
    1. Februar 2018

    MartinB,
    ich vereinnahme niemanden und versuche es auch nicht. Dr. Hossenfelder ist skeptisch bzgl. der Inflation und verweist in dem Zusammenhang auch auf andere, die diese Skepsis teilen (etwa Steinhardt). Es erschließt sich mir nun überhaupt gar nicht, warum ich nicht auf andere verweisen sollte, die ebenso wie ich an einer verbreiteten Überzeugung zweifeln.

    Ich frage mich so langsam, wozu Du diesen Blog hier betreibst. Verstehst das als eine reine Werbeveranstaltung nach dem Motto: wer an dem “Produkt” (LCDM) Zweifel hegt hat hier nichts verloren?

  323. #323 Albrecht Storz
    1. Februar 2018

    DerZimmermann,

    die Laufzeitentfernung verstehe ich als eine Zeitangabe, die sich auf den Lauf des Lichtes bezieht (also auf das DAZWISCHEN …), nicht auf den Zeitpunkt des Ereignisses (… also nicht auf das DORT).

    Den Unterschied könnte man vielleicht metaphorisch mit dem Unterschied zwischen der Entfernung zwischen zwei Punkten vergleichen, einmal wenn man der Straße folgt (Laufzeitentfernung) und zum anderen die Luftlinie zwischen den zwei Punkten.

    Ich muss inzwischen den Eindruck haben, dass MartinB meine Aktivität hier nicht mag.
    Insofern danke ich für alle Inputs und die aufgewandte Zeit. Vielleicht hat dem einen oder anderen der Diskurs ja sogar etwas gebracht, und wenn es nur das Schärfen der eigenen Anschauung und der eigenen Argumente sei.

  324. #324 Albrecht Storz
    1. Februar 2018

    PS zur letzten Post als Versuch einer Verdeutlichung:
    wenn man eine Galaxie beobachtet und zB deren Metallizität bestimmt, so muss doch dieser beobachteten Eigenschaft auch einer Zeit zugeordnet werden um sie mit anderen Beobachtungen an anderen Orten und in anderen Zeiten in Zusammenhang zu bringen.
    Dazu nützt aber die Laufzeitentfernung nicht unmittelbar, da der beobachtete Zustand der Galaxie ja zu einem späteren Zeitpunkt so war, als es die Laufzeitentfernung suggeriert.

  325. #325 MartinB
    1. Februar 2018

    @Albrecht Storz
    Was ich nicht mag, sind
    1. Kommentatoren, die zwar von einem Thema keine Ahnung haben, aber trotzdem meinen, schlauer zu sein als all die Leute, die besagtes Thema seit Jahren studieren (siehe auch den letzten Kommentar, glaubst du ernsthaft, es wäre den Profis neu, dass man immer überlegen muss, wie Entfernung und Zeit zusammenhängen), also das klassisce Dunning-Kruger-Syndrom und
    2. Kommentatoren, die ständig neue Ideen einwerfen, auf Gegenargumente aber nicht eingehen und
    3. Kommentatoren, die sich selbst innerhalb weniger Posts widersprechen (einerseits ist die Physik dogmatisch, aber komischerweise gibt es kritische Blogs…)
    Du fällst leider in alle drei Kategorien.

  326. #326 Alderamin
    1. Februar 2018

    @Albrecht Storz

    eines versteh ich dann aber nicht: warum gibt man weiterhin bezüglich eines bestimmten kosmologischen Ereignisses immer nur einen einzigen Wert für dessen Verortung in der Raumzeit an?

    Die Astronomen geben im allgemeinen z an (also die Rotverschiebung), weil die sich am leichtesten messen lässt und unabhängig von den Parametern des Lambda-CDM-Modells ist. Die Medien verwechseln andauernd Lichtlaufzeitentfernung und Eigendistanz, weil sie den Unterschied nicht kennen. Eigendistanz ist das, was man eigentlich wissen will (wie weit ist die Galaxie gerade jetzt entfernt?)

    Nehmen wir zB die Supernova ASASSN-15lh

    Es wird (lt. Wiki) eine Entfernung (leider ohne weitere Spezifizierung) von 3,8 Milliarden Lichtjahre angegeben. Warum wird nicht der Vollständigkeit halber auch die Zeitkoordinate, die ja nicht 3,8 Mrd. Jahre betragen kann, angegeben?

    In der englischen Wikipedia steht auch die Rotverschiebung z=0,2326. Der Cosmic Distance Calculator schmeißt dafür eine Leuchtkraftentfernung von 3,818 Milliarden Lichtjahren heraus, das entspricht sehr genau dem Eintrag in der Wikipedia. Eigendistanz und Lichtlaufzeitentfernung sind deutlich kleiner (3,59 und 2,89 Milliarden Lichtjahre). Sieht also so aus, als wenn die 3,8 Milliarden Lichtjahre die Leuchtkraftentfernung sind.

    Hier
    […]
    wird ein (z=0.2326) angegeben. Die Rotverschiebung?
    Wenn ich das richtig verstanden habe, kann man damit aber nur die verschiedenen Entfernungsmaße umrechnen, aber nicht das Alter berechnen.

    Genau, die Rotverschiebung, siehe englische Wikipedia. Das Alter des Lichts (2,791 Milliarden Jahre) und des Weltalls zu dieser Zeit (10,923 Milliarden Jahre) spuckt der Cosmic Distance Calculator aus. Um zu wissen, wie alt die Galaxie ist, muss man wissen, wann sie entstanden ist (man könnte beispielsweise das Alter ihrer Kugelsternhaufen ganz gut abschätzen, aber sie ist dafür viel zu weit weg; ansonsten könnte man die Häufigkeit von Metallen messen; ich weiß aber nicht, ob eine solche Messung vorliegt). Für das Alter einer Galaxie interessiert man sich eher selten, höchstens wenn sie auffällig jung ist und sehr viele Sterne bildet. Die Muttergalaxie der zitierten Hypernova hat eine sehr niedrige Sternentstehungsrate und dürfte daher recht alt sein. Die meisten Galaxien sind innerhalb der ersten Milliarde Jahre nach dem Urknall entstanden, also wird sie wohl um die 10 Milliarden Jahre alt sein, so wie man sie jetzt sieht.

  327. #327 george hystrix
    terra
    3. Juni 2018

    Je einfacher etwas ist, desto komplizierter muss man es machen. Sonst machts ja zu wenig her.
    Dass sowas wie ein Uhrknall, vielleicht sogar ein Urknall stattgefunden hat, ist stark anzunehmen.
    Die Expansion des Raumes hingegen ist eine Spielerei. Ungefähr, wie wenn die Schrödingerkatze über die Außenwelt philosophieren wollte. Das Viech ist tot, und fertig. Und wenn nicht tot, dann bald.

    Die Materie ist ein räumlich erstrecktes “Etwas”. Somit sind Atome, Quarks, Planeten, Sterne, Katzen, Aliens, Physiker, deren Unterwäsche … alle sind sie räumlich erstreckte Entitäten.
    Und, siehe da, auch die Maßstäbe, mit denen man Entfernungen misst, sind räumlich erstreckt.
    Dämmert euch was, ihr Hubbler? Alle erdenklichen Längenmessvorrichtungen sind notwendigerweise im Gleichschritt mit dem guten alten Zollstock, und dieser dehnt sich im gleichen Maße aus wie das “Universum”.
    Jede Raumexpansionsbehauptung ist Unsinn, denn sie ist eo ipso unmessbar.

  328. #328 Karl-Heinz
    3. Juni 2018

    @george hystrix
    Du argumentierst saudumm. 😉

  329. #329 Alderamin
    3. Juni 2018

    @george hystrix

    Alle erdenklichen Längenmessvorrichtungen sind notwendigerweise im Gleichschritt mit dem guten alten Zollstock, und dieser dehnt sich im gleichen Maße aus wie das “Universum”.

    Ach, daher die Rotverschiebung…! Oder, Moment… nee…

  330. #330 MartinB
    4. Juni 2018

    @george
    Nein, so ist es nicht. Dass der raum expandiert bedeutet, dass teilchen, die sich kräftefrei bewegen, sich voneinander entfernen. Atome expandieren aber nicht, eeil sie durch kräfte zusammengehalten werden.
    Dazu gibt es hier eine ausführliche erklärung in einem der arzikel zu gravitationswellen.

  331. #331 Albrecht Storz
    5. Juni 2018

    Man beachte, dass in der Hypothese der Rotverschiebung durch Raumexpansion ZWEI Annahmen stecken die BEIDE unbeweisbar sind:

    a) Es gäbe eine Raumausdehnung, ABER nur dort, wo wir sie nie direkt nachweisen werden können.

    b) Licht würde beim Durcheilen eines solchen angenommenen, expandierenden Raum rotverschoben werden.

    Beide Hypothesen sind weder falsifizierbar noch verifizierbar.
    a) weil sich diese Raumausdehnung erst weit außerhalb unserer Milchstraße bemerkbar machen würde. Und dort werden wir nie hinkommen.

    b) weil wir in keinem Labor heute oder in der fernsten Zukunft Licht durch “expandierenden Raum” werden schicken können.

    So etwas ist weitab von jeder Wissenschaft. Das ist purer Mystizismus.

    Ich finde das sehr, sehr seltsam. Wir kennen genug Effekte durch die Licht rotverschoben wird. Und weitere (vernünftige und auf dem Boden unserer Physik stehende) sind denkbar. Warum braucht man diese Mythen?

  332. #332 Karl-Heinz
    5. Juni 2018

    @Albrecht Storz

    Nun dann erkläre mal die Kosmische Rotverschiebung, die ja durchaus messbar ist. 😉

    Komme mir aber bitte nicht mit der Lichtermüdung!

  333. #333 MartinB
    5. Juni 2018

    @Albrecht
    Wenn du eine bessere Theorie hast, die alle Beobachtungsdaten erklären kann, dann veröffentliche sie (aber bitte sauber durchgerechnet). In der Physik ilt, dass die gewinnt, deren Modell die Beobachtungen am schlüssigsten erklären kann. Bisher sind irgendwelche Theorien, die Lichtermüdung oder ähnliches annehmen, nicht schlüssig und lassen sich nicht mit allen Daten in Einklang bringen.

    Im übrigen müsstest du, wenn du annimmst, dass sich das Universum nicht ausdehnt, erklären, wie sich das mit der ART in Einklang bringen lassen soll, in der ein statisches Universum nicht ohne weiteres geht. (UNd falls du uns gleich erzählen willst, dass die ART ohnehin falsch ist, lass es einfach.)

  334. #334 george hystrix
    6. Juni 2018

    Viel Feind viel Ehr.
    Ausgenommen Karlheinz – dieses Etwas argumentiert weder sau noch dumm, sondern überhaupt nicht. Daher auch nicht ehreinträglich. Also, Dummi, marsch zurück in deinen Koben.
    Was die Rotverschiebung anbelangt, so ist dieser Dopplereffekt der nach der Anfangsexplosion sich voneinander entfernenden Explosionsprodukte bestens damit vereinbar, dass sich das Ganze in einem übergeordneten Raum abspielt.
    Die Kräfte zwischen den Teilchen (zB des Metermaßstabs) sind wie alles andere ein Aspekt des Gefüges der Raum-Zeit-Materiewelt.

  335. #335 MartinB
    6. Juni 2018

    @george
    Die Rotverschiebung ist kein Dopplereffekt, ansonsten wäre sie nur durch die Geschwindigkeit bei Aussendung des Lichts bestimmt, das passt aber nicht zu den Messdaten.

  336. #336 Albrecht Storz
    6. Juni 2018

    Schon mal jemandem aufgefallen, dass die (bekannte und erwiesene) Frequenzverschiebung durch den so genannten Dopplereffekt nichts ist, was dem Licht irgendwie aufgeprägt wäre (also nichts ist, was dem Licht unterwegs geschieht), sondern rein eine Folge des Empfangs ist. Entscheidend dabei ist die Relativgeschwindigkeit zwischen Sender und Empfänger (welche eigentlich? Die zur Zeit der Aussendung, oder die zum Zeitpunkt des Empfangs. Da beides nicht gleichzeitig ist, steht die Frage im Raum: die Relativgeschwindigkeit von wann? Das würde mich auch mal interessieren.)

    Die Rotverschiebung durch Raumexpansion, die eine Ad-Hoc-Hypothese ohne jede physikalische Grundlage ist, wäre aber dem Licht aufgeprägt, praktisch eine Veränderung des Lichtes an sich während dieses durch den expandierenden Raum fliegt? (Würde man neben dem Lichtstrahl herfliegen müsste man dessen zunehmendes Röterwerden ja praktisch wahrnehmen können, oder?).
    Auch für so etwas gibt es keinerlei sinnvolle Analogien in der uns bekannten Physik. (Das Wechselwirken von Photonen mit anderen Teilchen hat freilich auch so einen konkreten Effekt, scheint mir aber doch auf einem anderen Blatt zu stehen.)

    Und keine Mensch hat irgend eine Ahnung, wie sich eine, falls überhaupt möglich, reale Raumexpansion auf irgend etwas auswirken würde. Das ist schlimmer als Kaffesatz-Lesen was uns da als Kosmologie verkauft wird.

    Wie viel Hokuspokus und Zauberei will man uns eigentlich noch zumuten?

  337. #337 MartinB
    6. Juni 2018

    @Albrecht Storz
    Es ist immer wieder faszinierend zu sehen, für wie doof manche Leute den Rest der Menschheit halten. Ja, alle diese Dinge sind Überlegungen, über die man nachgedacht hat (und über die jede Physikerin mal nachdenkt, um dann aber nicht beim “Oh-mein-Gott-das-verstehe-ich-nicht-auf-Anhieb-also-muss-es-falsch-sein” stehen zu bleiben.)

    Für neue Phänomene muss es übrigens keine Entsprechungen geben, deswegen sind sie nämlich auch neu.

    “Und keine Mensch hat irgend eine Ahnung, wie sich eine, falls überhaupt möglich, reale Raumexpansion auf irgend etwas auswirken würde”
    Doch. Dazu gibt es eine Theorie, die sich “Allgemeine Relativitästheorie” nennt und mit der genau solche Dinge im Detail berechnet werden können.

    Weitere Kommentare von Dir bitte nur noch mit irgendeinem sinnvollen Inhalt – bloßes “Das kann alles nicht sein, weil ich das doof finde” ist hier unerwünscht.

  338. #338 Alderamin
    6. Juni 2018

    Wie kann man den Doppler-Effekt vom kosmologischen Doppler unterscheiden? Durch die unterschiedliche Entfernung, die sich für ein bestimmtes z ergibt. Besonders krass sieht man das bei der Winkeldurchmesserentfernung für große z (Rotverschiebungen).

    Die kosmische Hintergrundstrahlung hat beispielsweise eine Rotverschiebung von ca. 1080 (heute 2,8 Kelvin, waren ursprünglich mal ein Plasma von 3000 K = ca. 1080*2,8K). Sie zeigt Korrelationen, die einen Energieaustausch durch Strahlung belegen, auf ca. 1° Winkelabstand (WMAP und PLANCK-Messungen).

    Nach dem aktuellen ΛCDM-Modell (Dunkle Energie, kalte dunkle Materie) mit Hubble-Konstante H0=69,6 km/s/Mpc spuckt der Kosmologie-Kalkulator (siehe Link unter “Winkeldurchmesserentfernung”, ganz am Ende) eine Winkeldurchmesserentfernung von 12,9 Mpc aus und eine Bogensekunde hat einen Durchmesser von 0,063 kpc. 1° hat 3600″, entspricht also 3600*0,063 kpc=226,8 kpc = 740 kLJ (1 pc = 3,26 LJ). Das Weltalter der Hintergrundstrahlung ist 380.000 Jahre nach dem ΛCDM-Modell. Strahlung konnte also eine Kugel mit 380 kLJ Radius oder 760 kLJ Durchmesser in dieser Zeit ausleuchten. Passt (das ca. 1° heißt ja nicht 1,0000°).

    In folgendem schönen Papier hat sich jemand die Mühe gemacht, nicht weniger als 60 verschiedene Vorschläge für die Ursache der kosmologischen Rotverschiebung durchzurechnen. Auch der Dopplereffekt kommt dran (3.1, S. 5). Die Winkeldurchmesserentfernung ist dafür sehr einfach zu rechnen, dA = DH*z und DH ist (weiter oben, 2.1) c/H0. Ergo dA = c/H0*z. Kann man zu Fuß rechnen: dA = 3e+5 km/s / (69,6 km/s/Mpc) * 1080 = 4.655.172 Mpc (ja, Mega-Parsec!). Also 4,6 Tpc (4,6e+12 pc).

    Welche Strecke s macht 1° in dieser Entfernung aus? Das wäre der Winkel in Radians * Entfernung. Also s = π/180 *1° *4,6e12 pc = 8e+10 pc = 2,6e+11 LJ oder 260 Milliarden Lichtjahre. Um durch Strahlungsaustausch Korrelationen über 1° Winkelabstand hin zu bekommen, hätte das Weltall also schon zur Zeit der Entstehung der Mikrowellen-Hintergrundstrahlung 130 Milliarden Jahre alt sein müssen. Das ist offenbar Quatsch.

    Also ist die Rotverschiebung der Hintergrundstrahlung kein Doppler. Ähnliches kann man für Leuchtkraftentfernung und Supernovae-Entfernungen rechnen, mit denen sich die Entfernung von Galaxien messen lassen. Fig. 2 im schönen Papier zeigen, wie der Doppler und ΛCDM hier für große z auseinanderlaufen. In meinem Artikel (“Winkeldurchmesserentfernung”) gibt es ein Bild unter dem Unterpunkt “Leuchtkraftentfernung” das zeigt, wie die Supernovae-Helligkeiten genau auf der grünen ΛCDM-Kurve liegen und folglich nicht auf der Doppler-Kurve in Fig. 2. Damit fällt auch hier der Doppler als Erklärung aus.

    Man kann also sehr wohl unterscheiden, mit welcher Rotverschiebung man es in der Kosmologie zu tun hat, und das wissen die Profis schon lange.

  339. #339 george hystrix
    6. Juni 2018

    @Martin B, # 330 #
    Hab mir die Sache nach meinem locker eingeklopften 334/vielfeindvielehr nochmal durch meine Kalkganglien wandern lassen.
    Die zweitleerreichste Passage in deinem “soistesnicht” bezieht sich auf die kräftefreien Teilchen. Aha, sowas gibts? Wie amputierst du den Teilchen die unvermeidliche Gravitation, aber auch die zwei anderen WW? Der arme hubble-Raum wäre also ein Zitterpudding, der da expandiert und dort, einige Angström oder Lichtnanosekunden oder sonstwas weiter nicht? Ein Anblick grässlich und gemein, drum zog ihn der Senat auch ein.

    Am leerreichsten und ultimativ sogar ist indes die Feststellung … “raum expandiert bedeutet, dass teilchen, die sich kräftefrei bewegen, sich voneinander entfernen.” —–>> dieses sich-voneinander-entfernen kann nur in einem ÜBERGEORDNETEN BEZUGSRAUM vor sich gehen … “sich (voneinander) entfernen” ist die Perspektive des Hyper-räumlichen Messtechnikers!
    ~~
    Soweit ich es im Moment zu erkennen glaube, besteht eine andere Widerlegung des “in Atomen wird der Raum am Expandieren gehindert”-Argumentes im Gedankenexperiment ‘zwei Messlatten’ — die eine ist aus Neutronen, die zweite aus Osmium, die dritte aus schlichtem Eisen und die vierte ist ein Hologramm oder aus Zuckerwatte oder sonst etwas undichtem. Naja, ich sprach ja von zwei, also nehmen wir die Eisen- und die Zuckerschneelatte. Beide seien geeicht.
    Die eiserne hält den Raum eisern im Zaum und lässt ihn nicht expandieren, und die Wattelatte gibt ihm freie Fahrt? Je nachdem, welche ich zur Hubblefalsi/verifizierung verwende, bekomme ich konträre Ergebnisse?
    Hic haeret aqua … da hapert es.

  340. #340 Karl-Heinz
    6. Juni 2018

    @george hystrix

    Unser dreidimensionaler Raum muss nicht unbedingt in einem höherdimensionalen Raum eingebettet sein.

  341. #341 Alderamin
    6. Juni 2018

    @george hystix

    dieses sich-voneinander-entfernen kann nur in einem ÜBERGEORDNETEN BEZUGSRAUM vor sich gehen … “sich (voneinander) entfernen” ist die Perspektive des Hyper-räumlichen Messtechnikers!

    Nö, sich voneinander entfernen heißt, Licht braucht zunehmend länger für die Strecke zwischen beiden Objekten, ferne Objekte erscheinen mit kleinerem Sehwinkel, Helligkeiten von Lichtquellen nehmen ab.

    Soweit ich es im Moment zu erkennen glaube, besteht eine andere Widerlegung des “in Atomen wird der Raum am Expandieren gehindert”-Argumentes im Gedankenexperiment ‘zwei Messlatten’

    Nimm’ zwei andere Messlatten: ein Stück festen Boden und eine Vakuumröhre. In der Röhre hängen zwei Spiegel, zwischen denen ein Laserstrahl hin- und herläuft. Kommt eine Raumzeitwelle durch das System gelaufen, dann verkürzt und verlängert sich periodisch der Abstand im Vakuum, aber nicht der im Erdboden. Deshalb misst man zwischen den Spiegeln, die am Erdboden festgemacht sind, schwankende Lichtlaufzeiten. Das Ding nennt sich LIGO – und funktioniert! Eine der damit detektierten Quellen wurde mit Teleskopen am Himmel gefunden.

    Die selbe Theorie, die die Gravitationswellen vorhergesagt hat, beschreibt die Raumexpansion. Beide folgen aus der Annahme, dass Beschleunigung und Gravitation dasselbe sind. Und dass die Lichtgeschwindigkeit in allen Inertialsystemen konstant ist. Das reicht schon aus, um die ganze Relativitätstheorie daraus zwingend herzuleiten. Samt Raumexpansion.

  342. #342 MartinB
    7. Juni 2018

    @Alderamin
    Danke für den paper-Verweis, schön, das mal sauber gerechnet zu bekommen.

    @georg
    “bezieht sich auf die kräftefreien Teilchen. Aha, sowas gibts? ”
    Ja, zumindest in sehr guter Näherung. Die Reichweite der Kernkräfte ist winzig, die elektromagnetische Kraft ist so stark, dass die meisten Objekte im kosmischen Maßstab elektrisch neutral sind, und die Gravitation ist nach der ART ja gerade keine Kraft – Objekte folgen kräftefrei ihren Bahnen in der gekrümmten Raumzeit.

    “dieses sich-voneinander-entfernen kann nur in einem ÜBERGEORDNETEN BEZUGSRAUM vor sich gehen ”
    Warum? Wenn ich einen Metalklotz aufheize, entfernen sich auch alle Atome voneinander wegen der thermischen Ausdehnung, da brauche ich auch keinen übergeordneten Bezugsraum. Das ist ein durchaus brachbare Analogie (allerdings auch nicht mehr).
    Man kann die Geometrie gekrümmter Räume komplett ohne übergeordnete Räume beschreiben, das wusste schon Gauß (“theorema egregium”).

    “Die eiserne hält den Raum eisern im Zaum und lässt ihn nicht expandieren, und die Wattelatte gibt ihm freie Fahrt?”
    Nein. Die Atome in der eisernen Stange haben gegenüber kräftefreien Teilchen eine Geschwindigkeit – Teilchen sind nämlich nicht an Raumpunkten festgenagelt, sondern können sich im und gegen den Raum bewegen – du bist ja auch nicht immer am selben Ort.
    Das steht oben im Text bereits, wie das mit Materie genau geht, habe ich sehr ausführlich hier erklärt:
    https://scienceblogs.de/hier-wohnen-drachen/2016/02/14/was-bedeutet-es-wenn-sich-der-raum-verzerrt/
    https://scienceblogs.de/hier-wohnen-drachen/2016/02/21/noch-einmal-die-raumverzerrung/
    Stell dir vor, da ist eine Metallstange, und das Universum fängt plötzlich an, zu expandieren. Dann werden die Atome der Metallstange zunächst durch die Expansion beschleunigt, aber die Kräfte in der Stange hindern die Atome daran, dieser Beschleunigung zu folgen, sie behalten ihren Abstand deshalb bei.

  343. #343 MartinB
    7. Juni 2018

    PS
    Im übrigen gilt das, was ich schon in #337 gesagt habe: Glaubt ihr eigentlich ernsthaft, dass sich in den letzten 100 Jahren, seit es die ART gibt, niemand diese Fragen gestellt hat?
    Die richtige Reaktion in der Wissenschaft auf “das kann doch gar nicht sein” ist nicht “alle anderen sind doof und die Wissenschaft ist falsch” sondern erst mal “da muss ich wohl was falsch verstanden haben”.
    Revolutionen kann es nur geben, wenn man erst mal verstanden hat, wogegen man eigentlich ist.

  344. #344 Albrecht Storz
    7. Juni 2018

    Interessant dass man hier nicht einmal Antwort zu einer einfachen, ernst gemeinten Frage bekommt.

    (Zur Erinnerung, die Frage war: wie wird eine Relativgeschwindigkeit für Rotverschiebung definiert wenn (was ja die Regel ist) Aussendung und Empfang zu verschiedenen Zeiten erfolgt, also sich die Geschwindigkeiten während der Übertragung ändern?)

    MartinB: Du generierst Dich wie ein vormittelalterlicher Priester: “GLAUBT GEFÄLLIGST! Wenn ihr kein Mathe könnt, selber Schuld!” (früher war es eben statt dessen Nicht-Lesen-Können oder Unkenntnis des Lateins was den “Unwissenden” den Weg zur wahren Weisheit versperrt hat). Auf Hinweis von Unklarheiten oder Unlogik, auf Fragen die sich eben aus den INTERPRETATIONEN der Mathematik (wie sie in populärwissenschaftlichen Medien wie diesem hier verbreitet werden) ergeben wird mit so einer Art “Du Dummie, Du nervst.” reagiert.
    Sie praktizieren hier öffentlich antiskeptizistische, vorwissenschaftliche, voraufklärerische Dogmenapologisierung. Ich dachte die Menschheit wäre ein kleines Stück weiter gekommen … ;-(

  345. #345 DerZimmermann
    7. Juni 2018

    @MartinB / @alderamin: Ich bin sehr beeindruckt, mit welcher Energie ihr bereit seid, hier solche “Fragen” zu beantworten und zu diskutieren.

    Niemand hat das Recht darauf, auf Fragen eine Antwort zu bekommen (zumindest wüsste ich nicht dass jemand hier z. B. für Antworten bezahlt würde). Wenn hier jemand Fragen beantwortet, tut er das in seiner Freizeit und aus gutem Willen.

  346. #346 MartinB
    7. Juni 2018

    @ALbrecht
    Alderamin hat dir sehr ausführlich geantwortet.

    Und nein, die Antwort von mir war kein “glaub gefälligst”, sondern ein, “denk doch erst mal länger als 2 Minuten nach, bevor du Tausende Physikerinnen zu Deppen erklärst.”

  347. #347 Albrecht Storz
    7. Juni 2018

    MartinB: ich sehe keine Antwort von Alderamin auf meine Frage.

    Alderamin: nur weil die “Raumexpansion” mit der Relativitätstheorie vereinbar ist, bedeutet das in keinster Weise, dass damit “Raumexpansion” “beschrieben” werden würde. Dein Formulierung ist wenigstens fragwürdig bis irreführend.

    Der Urknall ist auch mit der Theorie von der Existenz des Spaghetti-Monster vereinbar …

  348. #348 Karl-Heinz
    7. Juni 2018

    @Albrecht Storz

    Hallo Albrecht
    Mich würde interessieren, wie dein Verhältnis zur Mathematik ist.
    Stehst du mit ihr auf Kriegsfuß?
    Ist aber kein Vorwurf von mir. 😉

  349. #349 MartinB
    7. Juni 2018

    @DerZimmermann
    Ich antworte vor allem deshalb auf so etwas, weil ich annehme, dass hier auch andere Leute mitnehmen – bei den AlbrechtStorzens dieser Welt ist argumentieren meist nutzlos, aber andere können dann zumindest sehen, dass diese scheinbaren Probleme längst diskutiert und ausgeräumt sind.

    @AlbrechtStorz
    Hast du in das von Alderamin verlinkte paper reingeschaut?

    Und die Expansion ist nicht nur mit der ART vereinbar, sondern ist eine Vorhersage der ART (um die zu vermeiden hatte Einstein ja extra noch die kosmologische Konstante eingebaut, weil er sich ein statisches Universum dachte – ja, auch Einsteins machen Fehler).

  350. #350 Alderamin
    7. Juni 2018

    @Albrecht Storz

    Die ART lässt nur drei Möglichkeiten für einen Raum, der mit frei fallender Materie angefüllt ist, zu:
    1) der Raum expandiert
    2) der Raum kollabiert
    3) der Raum wird durch eine kosmologische Konstante (aka Dunkle Energie) und eine perfekt austarierte, überall gleiche Dichte in einer feinen Balance gehalten. So wie ein Bleistift, der auf der Spitze steht. Diese letzte Idee hatte Einstein ernsthaft in Erwägung gezogen und später als seine größte Eselei bezeichnet, weil er ansonsten die Raumexpansion vorhergesagt hätte.

    Einen Kollaps würden wir an der Blauverschiebung der Galaxien sehen. Wir sehen aber eine Rotverschiebung. Also muss es sich um Fall 1 handeln.

    Die Materie zerrt den Raum mit sich (Merksatz: Materie sagt der Raumzeit, wie sie sich zu krümmen hat, die Raumzeit der Materie, wie sie sich zu bewegen hat) und zieht ihn auseinander. Um rotierende Massen zerren diese die Raumzeit im Kreis herum mit sich (Lense-Thirring-Effekt). Sich umkreisende Massen werfen Wellen in der Raumzeit. Ist alles gemessen und nachgewiesen worden.

    Dass wir es nicht mit simplem Doppler zu tun haben, erkennt man aber eben auch schon daran, dass die beobachteten Winkeldurchmesser von Standardlinealen wie den Strukturen in der Hintergrundstrahlung (Galaxiendurchmesser tun’s im Prinzip auch, sind aber schwieriger absolut zu bestimmen) und die Helligkeiten von Standardkerzen wie Typ-Ia-Supernova-Helligkeiten anders von der Rotverschiebung abhängen, als das bei simplem Doppler der Fall wäre. Das zeigt das in #338 verlinkte Papier.

    Es reicht in der Wissenschaft nicht, qualitative Aussagen zu machen. Man muss die Aussagen einer Hypothese quantifizieren und dann durch Messungen überprüfen. Qualitativ mag die kosmologische Rotverschiebung wie Doppler aussehen, quantitativ passt diese Hypothese aber nicht zu den Messungen. Zu den Messungen passt aber ΛCDM mit einem ganz bestimmten Mix von Materie und Dunkler Energie.

    Und ob man, wenn man neben einem Lichtstrahl herflöge, diesen röter werden würde sehen können – man kann neben keinem Lichtstrahl herfliegen, nach dem Relativitätsprinzip haut der Lichtstrahl immer mit Lichtgeschwindigkeit vor einem ab, egal wie schnell man sich bewegt. Wenn man aber den Lichtstrahl unterwegs von ein paar Beobachtern messen lassen würde und die dann mit weniger als Lichtgeschwindigkeit der Reihe nach besuchen und befragen würde, dann würden die einem bestätigen, dass der Lichtstrahl zunehmend röter geworden sei. Das ist technisch nicht ganz einfach umsetzbar, deswegen muss man sich damit begnügen, zunehmend weiter entfernte (und automatisch damit auch in der Vergangenheit liegende) Quellen anzuschauen. Das tut man z.B. bei den Supernovae, die sind alle gleich hell und weit genug zu sehen (bis z=10). Nach Fig. 2 ergibt die Leuchtkraftentfernung für z=10 im ΛCDM 30 in irgendeiner nicht spezifizierten Einheit (sind laut Kosmologierechner ungefähr 100.000 Mpc), für Doppler hingegen ungefähr 70. Das würde man bemerken!

    Die Leuchtkraftentfernung ist diejenige, die man erhält, wenn man die Standardformel anwendet, dass die Helligkeit quadratisch mit der Entfernung abnimmt. Sie nimmt aber kosmologisch darüber hinaus noch quadratisch mit 1+z ab, da die Frequenz und die Zahl der Photonen pro Zeiteinheit um jeweils 1+z kleiner werden. Deswegen kommen viel größere Entfernungen heraus, als wenn man etwa von der Lichtlaufzeit ausgeht.

  351. #351 Albrecht Storz
    7. Juni 2018

    Karl-Heinz: mein Verhältnis zur Mathematik ist sehr gut.

    Alderamin: Du vertrittst hier eine Über-Interpretation von ART.

    Aber nehmen wir einmal an, dass es tatsächlich keinen materiefreien Raum geben könnte (was aber keiner weiß, bei Deiner Darstellung aber eine implizite Voraussetzung ist), so gäbe es zu der “Lösung” dass Raum sich an jeder Stelle lokal ausdehnt (was in einer globalen Expansion, von mir aus noch von lokalen Gegebenheiten abhängig, resultieren würde und was dem heutigen Lambda-CDM-Modell entspricht) noch die Lösung, dass sich Raum nur “an seinen Rändern” ausdehnt. Sprich: indem Materie auseinander flöge würde der Raum immer größer werden – ohne dass dabei etwa eine Grenze, ein tatsächlicher Rand, vorhanden sein müsste (jeder Versuch, diesen “Rand” irgendwie zu fassen, würde diesen ja weiter hinausschieben, man denke an den Esel und die Mohrrübe).

    Aber die wesentlich trivialere Lösung ist die, dass ein unendlicher Raum vorliegt in dem Materie entsprechend Deinem 1), 2) und 3) angeordnet und bewegt ist.

    Oder eben, noch viel naheliegender: ein ewiger, unendlicher Kosmos, in dem Materie relativ gleichverteilt ist und sich daher im Mittel nichts daran ändert.

    Du packst neben ART noch weitere Annahmen zusammen (Urknall, Raumexpansion, “Raum ohne Materie unmöglich”) und kommst damit zu Deinen Behauptungen, die so von keinem seriösen Wissenschaftler unterschrieben werden könnten. Die Aussagen aus ART beziehen sich einzig und alleine auf das Verhalten der Materie (und deren Verhalten unter Gravitationswirkung) und die Verkopplung mit dem Verhalten von Raum in diesem Zusammenhang ist eine pure Hypothese (ich weiß genau, dass dieser Punkt hier wieder massiv missverstanden wird, aber es ist mir auch so langsam Leid alles zu erklären und trotzdem nirgendwo auf den Versuch von verstehen wollen zu stoßen).

    Ich habe Dir bis jetzt vier alternative Modelle vorgelegt. Wahrscheinlich wirst Du keinen sinnvollen Gedanken an auch nur eines dieser Modelle verschwenden weil für Dich ja schon fest steht, welches einzig und allein richtig sein muss. Das ist sehr traurig.

    Und von Gedankenexperimenten hast Du auch noch nie was gehört (mein “neben Lichtstrahl herfliegen”) … Also mir wird es hier einfach zu doof. Selbst auf eine triviale Frage bekommt man keine Antwort, es wird aber faktenwidrig das Gegenteil behauptet. Sehr, sehr traurig das Ganze.

  352. #352 MartinB
    7. Juni 2018

    @Albrecht
    “Aber die wesentlich trivialere Lösung ist die, dass ein unendlicher Raum vorliegt in dem Materie entsprechend Deinem 1), 2) und 3) angeordnet und bewegt ist.”
    Diese sogenannte Milne-Lösung passt aber nicht zu den Messdaten, da müsste in mitbewegter Zeitkoordinate der Raum hyperbolisch gekrümmt sein, was definitiv nicht der Fall ist.

    Dinge wie Expansion des Alls sind keine Annahmen, sondern die besten Erklärungen für Beobachtungen, wie Alderamin ja ausführlich erklärt hat.

    “Ich habe Dir bis jetzt vier alternative Modelle vorgelegt. ”
    Nein. Du hast Worte hingeschrieben. Du hast nicht vorgerechnet, wie du die Messdaten, die Alderamin präsentiert hat, mit diesen Modellen reproduzieren kannst. Das geht nämlich schlicht nicht (beispiel Milne-Universum oder die Erklärung der Rotverschiebung, siehe Alderamins Post.) Physik kann man zwar mit Worten vermitteln, aber um Theorien zu prüfen muss man rechnen und quantitativ auswerten.

    Keines von deinen alternativen “Modellen” ist irgendwie neu – all diese Ideen hatten andere auch und haben sie schließlich verworfen, weil ihre Konsequenzen nicht zu den Messungen passen. Dass du das nicht weißt, macht die Sache nicht besser…

  353. #353 Alderamin
    7. Juni 2018

    @Albrecht Storz

    Alderamin: Du vertrittst hier eine Über-Interpretation von ART.

    Es war Einstein höchstpersönlich, der diese Interpretation vertrat, genau wie die anderen auch (Lichtablenkung im Schwerefeld, verlangsamte Zeit im Schwerefeld, Gravitationswellen,…).

    Aber nehmen wir einmal an, dass es tatsächlich keinen materiefreien Raum geben könnte

    Es spielt keine Rolle, ob es den geben könnte – unser Raum ist nicht materiefrei. Es gibt zwar Lücken zwischen den Massepunkten, aber die überbrückt die Schwerkraft problemlos. In den Einsteinschen Feldgleichungen kann man deshalb näherungsweise so rechnen, als ob die Materie den Raum gleichmäßig erfüllt (was sie ganz zu Beginn des Universums tatsächlich getan hat).

    so gäbe es zu der “Lösung” dass Raum sich an jeder Stelle lokal ausdehnt (was in einer globalen Expansion, von mir aus noch von lokalen Gegebenheiten abhängig, resultieren würde und was dem heutigen Lambda-CDM-Modell entspricht) noch die Lösung, dass sich Raum nur “an seinen Rändern” ausdehnt.

    Nein, denn dann müsste der innere Bereich ja kollabieren (man kann sich den äußeren einfach wegdenken, dann ist der “Rand” des inneren außen (wobei das reale Universum keinen Rand hat).

    Nach der ART muss ein Raumvolumen, das eine positive Materiedichte hat, (positiver Druck tut’s übrigens auch) eine anziehende Kraft entwickeln. Die Materie kann sich dem entweder durch hinreichenden Schwung nach außen entziehen (wie ein nach oben geworfener Stein), oder sie fällt frei entlang der zusammenlaufenden Geodäten zu einem gemeinsamen Punkt hin. Das ist ziemlich komplexes Zeug, das man ohne Physik-Studium und Spezialvorlesungen nicht wird nachvollziehen können.

    Aber die wesentlich trivialere Lösung ist die, dass ein unendlicher Raum vorliegt in dem Materie entsprechend Deinem 1), 2) und 3) angeordnet und bewegt ist.

    Wie schon mehrfach gesagt, passt das nicht zu den Beobachtungen Man kann aus dem Zusammenhang zwischen Supernova-Helligkeiten und der Rotverschiebung bzw. der Größe von Strukturen in der kosmischen Hintergrundstrahlung sicher ausschließen, dass die Materie einfach nur auseinander fliegt. Und die ART verbietet es sogar.

    Du packst neben ART noch weitere Annahmen zusammen (Urknall, Raumexpansion, “Raum ohne Materie unmöglich”)

    Urknall: Ohne Urknall hast Du ja nicht mal Deinen Doppler, ich dachte, wenigstens darüber bestünde Einigkeit. Was sonst noch den Urknall belegt ist die Hintergrundstrahlung (Feuerball), die Elementhäufigkeit von Helium, Deuterium und Lithium (Nukleogenese), die Reionisation, die Galaxienentwicklung (Sternentstehungsrate, Quasare, Wachstum durch Kollisionen), das Alter der ältesten Sterne, die zunehmende Metallizität bei Sternen, die Strukturbildung, und, fast hätte ich’s vergessen, nebenbei, auch die Rotverschiebung.

    Die Raumexpansion setze ich nicht voraus, sondern begründe, warum die Rotverschiebung nur auf sie zurückgeführt werden kann.

    Und dass ein “Raum ohne Materie unmöglich” sei, habe ich wo genau gesagt? Davon weiß ich nichts. Es ist nur halt so, dass unser Raum Materie enthält. Und ein mit Materie gefüllter Raum kann nur expandieren, kollabieren, oder perfekt balanciert sein, wobei letzteres ziemlich unwahrscheinlich ist.

    Die Aussagen aus ART beziehen sich einzig und alleine auf das Verhalten der Materie (und deren Verhalten unter Gravitationswirkung) und die Verkopplung mit dem Verhalten von Raum in diesem Zusammenhang ist eine pure Hypothese

    Dieselben Formeln, die erklären, warum gemäß der ART ein Stein zu Boden fällt, erklären das Verhalten des Universums. Das eine kann nicht falsch sein, wenn das andere richtig ist. Das darzulegen ist allerdings ausgeschlossen in einem Post. Aber der Blogchef hier hat ein Buch angekündigt, das dies leisten kann, denke ich.

    aber es ist mir auch so langsam Leid alles zu erklären und trotzdem nirgendwo auf den Versuch von verstehen wollen zu stoßen

    Wie soll man denn etwas verstehen, von dem man weiß, dass es vollkommen falsch ist?

    Ich habe Dir bis jetzt vier alternative Modelle vorgelegt. Wahrscheinlich wirst Du keinen sinnvollen Gedanken an auch nur eines dieser Modelle verschwenden weil für Dich ja schon fest steht, welches einzig und allein richtig sein muss. Das ist sehr traurig.

    Erklären Deine Modelle die Beobachtungen? Kannst Du was vorrechnen? Kannst Du daraus eine Arbeit machen, die nach Peer Review veröffentlicht wird? Nein? Warum soll ich irgendwelche Behauptungen glauben, die nicht einmal ein messbares Ergebnis produziert haben? Da halte ich mich lieber an die mit wissenschaftlichen Methoden gewonnenen Erkenntnisse (selbst, wenn ich sie nicht bis zum letzten i-Tüpfelchen nachvollziehen kann – andere haben das getan und sie für mich geprüft). Nicht böse sein…

    Und von Gedankenexperimenten hast Du auch noch nie was gehört (mein “neben Lichtstrahl herfliegen”)

    Ich hab’s ja extra so modifiziert, dass es funktionieren würde. Nur ist das kein Gedankenexperiment gewesen, weil man ja nicht erschließen kann, was rauskommt (wie z.B. bei Einsteins Lichtuhr in einem bewegten Raumschiff – da kann man direkt die Zeitdilatation draus berechnen).

    Selbst auf eine triviale Frage bekommt man keine Antwort

    Was genau war denn die triviale Frage? Ob Licht unterwegs rotverschobener wird? Die ebenso triviale Antwort lautet: Ja!

    Sehr, sehr traurig das Ganze.

    Traurig finde ich immer die, die von Physik kaum eine Ahnung haben, aber glauben, an den Theorien der größten Koryphäen rütteln zu können. Das ist so unglaublich naiv.

  354. #354 MartinB
    7. Juni 2018

    “Traurig finde ich immer die, die von Physik kaum eine Ahnung haben, aber glauben, an den Theorien der größten Koryphäen rütteln zu können. Das ist so unglaublich naiv.”
    Und hinzu kommt die Annahme, dass noch nie eine vorher auf die Idee gekommen ist, das vielleicht einfach “alles auseinanderfliegt” oder “Licht müde wird” oder was auch immer. Als wären diese Ideen nicht total naheliegend und als würde nicht so ziemlich jede, die anfängt, sich mit der Expansion des Alls zu beschäftigen, genau diese Fragen stellen.

    Und wenn man das schon so annimmt, dan könnte man sich wenigstens die Mühe machen, mal die Suchmaschine der Wahl anzutreten und zu schauen, was es an Erklärungen zum Thema gibt und ob diese Fragen nicht vielleicht doch längst geklärt sind.

    Stattdessen schlägt man mit Sprüchen wie
    “Das ist schlimmer als Kaffesatz-Lesen was uns da als Kosmologie verkauft wird. Wie viel Hokuspokus und Zauberei will man uns eigentlich noch zumuten?”
    in einem Blog auf und spielt dann, wenn’s argumentativ eng wird, plötzlich den armen Märtyrer, dem ja nicht mal einfache Fragen beantwortet werden (dabei die gegebenen Antworten natürlich geflissentlich ignorierend) und dessen “Gedankenexperimente” nicht ernst genommen werden – auch wenn die dummerweise auf einer Annahme beruhen, die der gesamten Physik zuwiderläuft…

    Irgendwie schon faszinierend.

  355. #355 george hystrix
    7. Juni 2018

    Jörgls gestörtes Verhältnis zur Mathematik mögen die Gestörten beurteilen, die meine beschiss-bescheidenen Mathematik-publikationen blickgewürdigt haben … richtig verstehen tuts ja immer nur einer – der Autor selbst … und DER nicht einmal. Aber, gernstens zugegeben … meine zwischen Zahlentheorie und Vektorpurzelbäumen herumturnenden Spielereien waren turmhoch tief unter den hier zitierten Winkelspiegelungen der Photonen. Doch eines bleibt mir unerschütterlich im Magen liegen, ganz ohne Mathesis (“die Kunst”) im engeren Sinn: ein sich-voneinander-entfernen kann nur von außen, also von einem anderen Koordinatensystem aus beurteilt werden als dem gepeinigten Koordinatensystem, dessen x1/x2/x3-Achsen mitsamt der t-Achse gedehnt werden wie im Prokrustesbett. An diesem Salto seitwärts ändert keine Rotz-verschiebung oder sonstwas was. Es muss sich eben ein gescheiterer finden als ich alter Störungsintegrierer des x-Körperproblems, der das alles auf einen Nenner bringt. Und sagt nicht, Einsteindergroße. Der wars nicht.

    Oder ich versuchs mal so: es klebt das Spiel der Irrungen am Mythos des Raumes. Es gibt das Wort “Raum”, und daher ist es heilige Pflicht, Theorien so prokrustenisch wie das Koordinatensystem in diesen widersprüchlichen Pseudobegriff hineinzuzwängen, koste es die Rotschiebung, was es wolle.

  356. #356 george hystrix
    7. Juni 2018

    Nehmen wir mal an, es gäbe ein Universum, das aus einem einzigen “Etwas” bestünde. Man könnte sich ein Teilchen, ein Energiequantum oder wie Faust “die” Kraft oder “das” Wort vorstellen. Oder, verdammt, das Brahman.
    Plötzlich (natürlich nur ein Hilfs-Zeit-Wort) erscheint ein Gott und sagt zum Teilchen: “Tu irgendwas”. Aber das Teilchen will nicht explodieren, es setzt sich lediglich in Bewegung. Das heißt, Raum und Zeit entstehen spontan und ermöglichen “Bewegung”. Tja, Rotverschiebung gibts keine, denn das Teilchen weiß ja nicht, dass es nach irgendwessen Gesetzen was abstrahlen müsste. Es gibt die Urbewegung und sonst nix. Sie findet im Gottesraum statt und nicht in dem “Raum” des Teilchens. Expandiert mit dieser Bewegung des Eremitis sein Raum? Eremiti wird antworten: keine Spur. Mein Raum bin ich, und ob er expandiert, soll meinetwegen Gott beurteilen.
    Und versuche JA keiner dem eigensinnigen Eremiti aufzuoktroyieren, es solle Rotverschiebung mimen.

  357. #357 MartinB
    8. Juni 2018

    @georghystrix
    ” ein sich-voneinander-entfernen kann nur von außen, also von einem anderen Koordinatensystem aus beurteilt werden”
    Wie das. Man kann nicht messen, dass sich zwei Objekte voneinander entfernen? Die können keine Signale austauschen, keine Lichtlaufzeiten messen, keine Maßbänder ausrollen?

    Natürlich kann man “Raum” selbst nicht direkt messen. Deswegen misst man kräftefreie Teilchen, die sich auf Geodäten im Raum bewegen. Genauso wie ich auf der Erde die Krümmung der Erdoberfläche zum Beispiel daran sehen kann, dass zwei Flugzeuge, die am Äquator nach Norden starten, sich einander annähern, oder daran, dass ich Dreiecke mit Winkelsumme größer als 180 Grad zeichnen kann.
    Der Rest deines Textes enthält nicht mal ansatzweise so etwas wie Argumente, insofern lohnt es sich nicht, darauf einzugehen.

  358. #358 Spritkopf
    8. Juni 2018

    @george hystrix

    Doch eines bleibt mir unerschütterlich im Magen liegen, ganz ohne Mathesis (“die Kunst”) im engeren Sinn: ein sich-voneinander-entfernen kann nur von außen, also von einem anderen Koordinatensystem aus beurteilt werden als dem gepeinigten Koordinatensystem, dessen x1/x2/x3-Achsen mitsamt der t-Achse gedehnt werden wie im Prokrustesbett. An diesem Salto seitwärts ändert keine Rotz-verschiebung oder sonstwas was.

    Wenn ich Ihre vom Bemühen um größtmögliche Unverständlichkeit getragene Aneinanderreihung von Worten mal umformulieren darf, sagen Sie Folgendes:

    “Eine Rotverschiebung, sprich eine Ausdehnung des Raums, lässt sich innerhalb des Raumes gar nicht messen, da sich alle physischen Maßstäbe ebenfalls ausdehnen würden.”

    Korrigieren Sie mich, wenn ich Ihre Annahme falsch formuliert habe.

    Warum das mit den physischen Maßstäben Quatsch ist, hat Martin u. a. hier erklärt.

    Und leider ist Ihre Behauptung, dass man gar keine Rotverschiebung messen könne, genauso Quatsch. Die Physiker messen sie nun einmal. Was sagen Sie nun?

    Oder kann es sein, dass Sie noch nicht mal wissen, was bei dieser Rotverschiebung eigentlich gemessen wird und warum man daraus den Schluss ziehen kann, dass es sich um Licht handelt, welches rotverschoben wurde?

  359. #359 Albrecht Storz
    8. Juni 2018

    MartinB, #352: Ich finde nichts auf Anhieb zur “Milne-Lösung” ( https://de.wikipedia.org/wiki/Edward_Arthur_Milne ) darum lasse ich das jetzt so mal stehe.

  360. #360 Albrecht Storz
    8. Juni 2018

    Alderamin: “Es spielt keine Rolle, ob es den geben könnte – unser Raum ist nicht materiefrei.”

    Wenn Du ein besseres Verständnis von dieser Thematik hättest, hättest Du verstanden, worum es mir in diesem Zusammenhang gegangen ist: es ist nämlich ein wesentlicher Aspekt der Betrachtung ob man bei ART davon ausgeht, ob es völlig materiefreien Raum geben kann – oder nicht. Und bei einigen Interpretationen von ART bezüglich unseres Kosmos spielt genau der Punkt eine wichtige Rolle.
    So ist es sicherlich ein Unterschied, ob es um den von uns mit Materie gefüllt gesehenen Kosmos herum noch unendlich viel Raum gibt der tatsächlich völlig leer ist (und so weit entfernt, dass auch noch keine Auswirkungen der “hiesigen Materie” dort angekommen ist) – oder eben nicht.
    Bei Lambda-CDM geht man idR davon aus, dass dem nicht so wäre. Und auch hier gilt: etwas genaues weiß man nicht!

  361. #361 MartinB
    8. Juni 2018

    @Albrecht Storz
    Die Milne-Lösung wird zum Beispiel bei Rebhan sehr ausführlich erklärt, Wikipedia hat auch was dazu
    https://en.wikipedia.org/wiki/Milne_model
    Ist letztlich ein leeres kosmologisches Modell, das nach den Regeln der SRT funktioniert.

    “So ist es sicherlich ein Unterschied, ob es um den von uns mit Materie gefüllt gesehenen Kosmos herum noch unendlich viel Raum gibt der tatsächlich völlig leer ist (und so weit entfernt, dass auch noch keine Auswirkungen der “hiesigen Materie” dort angekommen ist) – oder eben nicht.”
    Für welche Messungen ist das ein Unterschied?

  362. #362 MartinB
    8. Juni 2018

    PS:
    Das Milne-Modell zeigt übrigens auch, dass man ohne Probleme einen “leeren” (materiefreien) Raum in der ART annehmen kann – die ART wird dann allerdings zur SRT, weil die Raumzeit dann nicht mehr gekrümmt ist.
    Es ist allerdings (durch die Messungen) klar, dass unser Universum nicht zu dieser Annahme passt.

  363. #363 Albrecht Storz
    8. Juni 2018

    Alderamin, ” In den Einsteinschen Feldgleichungen kann man deshalb näherungsweise so rechnen, als ob die Materie den Raum gleichmäßig erfüllt (was sie ganz zu Beginn des Universums tatsächlich getan hat).”

    Und das Letztere weiß man woher jetzt so genau?

    “Urknall: Ohne Urknall hast Du ja nicht mal Deinen Doppler, …”

    Was heißt hier “mein Doppler”? Der Dopplereffekt tritt ganz genau dann auf, wenn Sender und Empfänger sich gegeneinander bewegen. Dazu braucht es keinen Urknall. Es gibt auch andere Möglichkeiten warum etwas auseinander strebt.

    Und genau da sind wir wieder bei meiner jetzt damit zum dritten mal gestellten Frage:
    Es gibt in der Relativitätstheorie keine Gleichzeitigkeit. Wie will man unter diesen Bedingungen eine Relativgeschwindigkeit bestimmen? Nimmst Du dann die Geschwindigkeit des Senders (gegenüber was) zum Zeitpunkt der Aussendung und die Geschwindigkeit des Empfängers (gegenüber was?) zum Zeitpunkt des Empfangs?
    Relativgeschwindigkeit ist im Zusammenhang mit nicht vorhandener Gleichzeitigkeit offensichtlich ein Problem.

  364. #364 Albrecht Storz
    8. Juni 2018

    MartinB, “Für welche Messungen ist das ein Unterschied?”

    Für keine. Aber für unsere Modellbildung, für die Hypothesenerstellung, für die Ideenwelt die wir entwickeln können.

    Ein kurzer Blick auf dieses Milne-Modell zeigt mir, dass das wohl nichts mit dem zu tun hat, auf was ich abziele. Ich versuche klar zu machen, dass das Urknall-Modell zumindest teilweise den Gedanken impliziert, dass es keinen leeren Raum ohne Materie (oder deren Auswirkung, also Gravitation) geben könne. Danach würde sich der Raum zusammen mit der Materie ausbreiten.
    Und genau das weiß niemand und hat noch kein Mensch irgendwo beobachtet.
    Neben dem kosmologischen Prinzip ist aber dies eine oft genug nur stillschweigend vorausgesetzte Annahmen.

  365. #365 MartinB
    8. Juni 2018

    @AlbrechtStorz
    Wenn es keine messbaren Konsequenzen hat, ist es egal, was man annimmt. Kann vielleicht die Intuition ankurbeln oder so, aber wenn zwei Modelle für alle Messungen dasselbe vorhersagen, dann kann man zwischen ihnen auch nicht unterscheiden.

    “Ich versuche klar zu machen, dass das Urknall-Modell zumindest teilweise den Gedanken impliziert, dass es keinen leeren Raum ohne Materie (oder deren Auswirkung, also Gravitation) geben könne”
    Und ich versuche klarzumachen, dass das falsch ist und dass das Milne-Modell genau einen solchen Fall beschreibt und damit eindeutig zeigt, dass diese Annahme falsch ist. Natürlich könnte man einen vollkommen leeren Raum niemals messen, messen können wir nur das Verhalten von Objekten.

    Was die Relativgeschwindigkeit angeht: Du kannst doch in der FRW-Metrik (oder jeder anderen) die Geodäten zweier Objekte ausrechnen, die mit einem Zeitunterschied von z.B. einer Sekunde ausgesendet werden und die sich mit Lichtgeschwindigkeit bewegen (beispielsweise zwei Gamma-Pulse oder was auch immer). Dann kannst du am anderen Ende gucken, mit welchem zeitlichen Abstand die beiden Signale ankommen, das gibt dir die Rotverschiebung, denn eine Welle mit einer Frequenz von anfänglich 1Hz hätte ja genau denselben Abstand der Wellenberge oder -täler. Insofern verstehe ich nicht, welches Problem du da siehst.

  366. #366 MartinB
    8. Juni 2018

    PS
    Und wenn dir die ganze Raumexpansionsgeschichte suspekt ist, kannst du dich natürlich auf den Standpunkt zurückziehen, dass die ART eigentlich eine ganz normale Feldtheorie beshreibt, bei der die Kräfte dank Äquivalenzprinzip uns nur vorgaukeln, die Raumzeit sei gekrümmt und würde sich ausdehnen. Das ist mathematisch vollkommen in Ordnung, siehe meine “ist die Raumzeit gekrümmt”-Serie oder das Buch zur ART von Weinberg (oder die Feynman Lectures on Gravitation). Da spielt sich alles auf einer Minkowski-Raumzeit ab, die wir aber direkt niemals messen können.

  367. #367 Alderamin
    8. Juni 2018

    @Albrecht Stolz

    Und das Letztere weiß man woher jetzt so genau?

    Man braucht sich nur die Hintergrundstrahlung anzusehen. Das ist die Materie, wie sie zu Beginn des Universums verteilt war, lediglich mit krass rotverschobener Wellenlänge. Man sieht einen glühenden Feuerball von 3000 K mit minimalsten Strukturen im Millionstel Kelvin-Bereich.

    Das ist eigentlich ganz simpel: je weiter man in die Fern schaut, desto früher schaut man in die Vergangenheit. Und Irgendwo ganz weit weg sieht man den Feuerball des Urknalls.

    Dazu braucht es keinen Urknall. Es gibt auch andere Möglichkeiten warum etwas auseinander strebt.

    Ein Beispiel bitte. Alles strebt in alle Richtungen von uns weg, vollkommen symmetrisch, umso schneller, je weiter weg. Bitte erkläre, wie das anfing und woher die Geschwindigkeit stammt.

    Es gibt in der Relativitätstheorie keine Gleichzeitigkeit. Wie will man unter diesen Bedingungen eine Relativgeschwindigkeit bestimmen?

    Man braucht keine Gleichzeitigkeit, um die Relativitätsgeschwindigkeit zu bestimmen, der Zeitablauf der anderen Seite interessiert dazu doch überhaupt nicht. Es reicht vollkommen aus, die Rotverschiebung einer bekannten Wellenlänge zu messen und in die relativistische Dopplerformel zu stecken (dann bekommt man natürlich auch ein Doppler-Ergebnis; die Raumexpansion sagt aber, dass die Galaxien sich kaum bewegen, nur ein wenig umeinander durch die Gegend treiben, da bewegt sich eigentlich nichts, nur der Raum wächst; wie man das unterscheiden kann, siehe oben, ich erklär’s nicht nochmal).

    Nimmst Du dann die Geschwindigkeit des Senders (gegenüber was) zum Zeitpunkt der Aussendung und die Geschwindigkeit des Empfängers (gegenüber was?) zum Zeitpunkt des Empfangs?

    Relativgeschwindigkeit heißt, man kann seine eigene Geschwindigkeit zu 0 setzen und misst die Geschwindigkeit der anderen Seite relativ zu sich selbst. Wenn man in einer Milchstraße auf einer Erde sitzt, die um die Sonne kreist, kann man allerdings messen, wie man sich relativ zur Hintergrundstrahlung bewegt und sich selbst diese Geschwindigkeit zuschreiben. Und die dann mit der gemessenen Relativgeschwindigkeit verrechnen. Simple Vektoraddition.

    Relativgeschwindigkeit ist im Zusammenhang mit nicht vorhandener Gleichzeitigkeit offensichtlich ein Problem.

    Nö, wieso? jeder misst für sich. A sieht B mit v vorbeifliegen. Dann sieht B A mit -v vorbeifliegen. Jeder für sich.

    Das ist doch hochtrivial. Ich dachte, wir kritisieren die Allgemeine Relativitätstheorie, und jetzt reden wir über Galileo-Transformationen…?

    Die Gleichzeitigkeit ist nur ein Problem, wenn man sich mit dem bewegten Beobachter über externe Vorgänge unterhält, über deren Reihenfolge dieser und man selbst verschiedener Auffassung sein könnte.

  368. #368 Albrecht Storz
    9. Juni 2018

    Alderamin, “Ein Beispiel bitte. Alles strebt in alle Richtungen von uns weg, vollkommen symmetrisch, umso schneller, je weiter weg. Bitte erkläre, wie das anfing und woher die Geschwindigkeit stammt.”

    warum fragst Du mich das? Beim Inflationsmodell werden doch von Dir anerkannt, soweit ich das weiß, genau solche Dinge erklärt:

    “Zur Erklärung der Dynamik der Inflation wird ein skalares Quantenfeld benötigt, das räumlich homogen ist und eine endliche Energiedichte aufweist. Wenn sich das Feld zeitlich genügend langsam ändert (nämlich in Richtung einer Verringerung der Energiedichte), so hat es negativen Druck und verhält sich effektiv wie eine kosmologische Konstante, führt also zu einer beschleunigten Expansion des Universums.”
    https://de.wikipedia.org/wiki/Inflation_(Kosmologie)

    Das ist doch nicht auf meinem Mist gewachsen!

    Und nun wirst Du erzählen, dass so ein “negativer Druck”, aus welchen Gründen auch immer, nur nach einem Urknall auftreten könne, oder wie?
    Fakt ist: Es werden Kräfte diskutiert, die so ein auseinander Streben erklären könnten, unabhängig von einem Urknall.
    Es scheint Methode zu sein, dass man zur Stützung des Standard-Modells jede beliebige Annahme heranziehen darf, diese aber bei Alternativmodellen von vorne herein ausgeschlossen werden, wie es gerade passt.

  369. #369 Albrecht Storz
    9. Juni 2018

    Alderamin, “Traurig finde ich immer die, die von Physik kaum eine Ahnung haben, aber glauben, an den Theorien der größten Koryphäen rütteln zu können. Das ist so unglaublich naiv.”
    ich rüttle nicht an Theorien großer Koryphäen sondern an Erklärungen von Leuten die nicht durchblicken, sich aber als Priester irgend einer halb verstandenen Lehre generieren.
    Warum ist es eigentlich so schwer zu begreifen, dass kein vernünftiger Mensch etwas gegen die Aussage haben kann, dass das Standardmodell, dass Lambda-CDM, dass Relativitätstheorie sicher aktuell die beste Erklärung für unsere Welt und den Kosmos liefert (auf jeden Fall die bestuntersuchte Erklärung wenn man nach Manpower und Finanzierung geht) aber diese Theorien und Hypothesen eben keine durchgängig gesicherten Wahrheiten darstellen.
    Jemand der so auftritt (wie ich es kritisiere) ist nicht weit von der Dogmenbildung des Mittelalters entfernt, wo auch “Auserwählte” der gläubigen Gemeinde wortreich vermittelten, wie gefälligst die Wahrheit auszusehen hat – und Skeptiker wurden genauso wie hier gedisst, ausgegrenzt, in Extremfällen verjagt oder sogar getötet.

  370. #370 Albrecht Storz
    9. Juni 2018

    MartinB, ““Ich versuche klar zu machen, dass das Urknall-Modell zumindest teilweise den Gedanken impliziert, dass es keinen leeren Raum ohne Materie (oder deren Auswirkung, also Gravitation) geben könne”
    Und ich versuche klarzumachen, dass das falsch ist”

    also ist auch die Bahauptung von Alderamin #353 falsch, dass “die Materie den Raum gleichmäßig erfüllt (was sie ganz zu Beginn des Universums tatsächlich getan hat).”
    oder wie wird dieser Widerspruch jetzt wieder weg erklärt?

  371. #371 MartinB
    9. Juni 2018

    @AlbrechtStorz
    Verstehe dein Argument nicht: Auch in der Inflation ist es doch der Raum, der sich – entsprechend der Friedmann-Gleichung – ausdehnt. Ich dachte, du wolltest eine Erklärung für das Auseinanderfliegen präsentieren, die einen anderen Mechanismus verwendet.

    Kannst du vielleicht nochmal auf den Punkt bringen, was eigentlich genau dein Problem ist, so langsam blicke ich nciht mehr durch.

    Und nein, hier wird niemand getötet – aber wer hier arrogant aufschlägt, auf Gegenargumente nicht eingeht und deutlich macht, vom Thema wenig Ahnung zu haben, muss sich schon ein wenig Kritik gefallen lassen – und wenn du mit Angriffen wie “Hokus-Pokus” und “Kaffeesatzleserei” einsteigst, dann hast du keinen Grund, dich wegen unserer Kritik jetzt plötzlich zum Märtyrer zu erklären – wie man in den Wald reinruft…

  372. #372 Albrecht Storz
    9. Juni 2018

    Alderamin, #367, “Nö, wieso? jeder misst für sich. A sieht B mit v vorbeifliegen. Dann sieht B A mit -v vorbeifliegen. Jeder für sich.”

    Was hat das damit zu tun? Wir haben einen Sender, zB eine Galaxie in 13 Mrd. Lichtjahren Entfernung und einen Empfänger, wir, heute. Wie wird nun die Relativgeschwindigkeit zwischen der Galaxie und uns definiert? da es keine Gleichzeitigkeit gibt, kann man nicht sagen “jetzt”, zum Zeitpunkt x, gilt Relativ-Geschwindigkeit y.

    Und Dein Ausflucht in irgend ein Internet-Programm und “relativistische Dopplerformel” hilft da auch nichts. Es geht nämlich um das Prinzipielle. Denn darauf basiert so eine Formel.
    Also nochmal: welche Zeitpunkte werden benutzt um die Relativgeschwindigkeit zwischen zwei Objekten zu bestimmen? Wir sind uns doch hoffentlich einig, dass “Relativgeschwindigkeit zwischen A und B” einen Zusammenhang zwischen Zuständen von A und B herstellt? Und da spielt Zeit eben eine Rolle. Im Alltag gehen wir von einer Gleichzeitigkeit aus wenn zB Auto A mit Auto B zusammen stößt. Aber Licht von A kommt nicht gleichzeitig zur Aussendung schon bei B an. In kosmischen Dimensionen sogar dramatisch unterschiedlich.

    Und bitte: Wenn Du nicht verstehst was ich meine, dann antworte auch nicht auf meine Frage. Das bringt niemandem etwas.

  373. #373 Karl-Heinz
    9. Juni 2018

    @Albrecht Storz

    Einfache Frage für dich: wie wird die Geschwindigkeit (relative Geschwindigkeit) von einem Auto mit der Laserpistole gemessen?

  374. #374 Albrecht Storz
    9. Juni 2018

    MartinB, “Verstehe dein Argument nicht: Auch in der Inflation ist es doch der Raum, der sich – entsprechend der Friedmann-Gleichung – ausdehnt. Ich dachte, du wolltest eine Erklärung für das Auseinanderfliegen präsentieren, die einen anderen Mechanismus verwendet.”

    wo ist da der negative Druck? Warum kann negativer Druck nur zugleich mit Raumexpansion auftreten?

    “Kannst du vielleicht nochmal auf den Punkt bringen, was eigentlich genau dein Problem ist, so langsam blicke ich nciht mehr durch.”

    Dass das Lamda-CDM-Modell EIN Erklärungsversuch ist. Und das das stimmen kann, oder auch nicht. Und dass wenn man in der Kosmologie nur noch danach trachtet, das gerade am besten gefallende Modell so lange zurecht zu parametrisieren, bis es zu allen (aktuell bekannten) Beobachtungsdaten passt, damit nichts gewonnen ist, denn aus so eine Modell lässt sich nichts ableiten, genauso wenig wie aus einem supergefitteten Spline, der nämlich 100% zu den gemessenen Punkten passt, aber auch nichts weiter.

  375. #375 Albrecht Storz
    9. Juni 2018

    Karl-Heinz, die Antwort: Dopplerverschiebung im Lichtspektrum beißt sich hier in den Schwanz. Denn genau um diese Frage geht es doch im Kern: kann die Rotverschiebung des Lichtes ferner Quellen nur und nicht anders erklärt werden als durch Relativgeschwindigkeit zwischen Sender und Empfänger.
    Ich hoffte in diesen Kreisen wäre bekannt, wie viele physikalische Effekte wir kennen, die zu einer Rotverschiebung in Spektren führen. Neben gravitativen Wirkungen sind es Wechselwirkungen mit anderen Teilchen oder Feldern.

    Aber insbesondere kann man nicht ein Phänomen zur Begründung seiner Ursachen heran ziehen.

    Wenn die Rotverschiebung die Relativgeschwindigkeit zwischen A und B anzeigt, so muss trotzdem die Frage geklärt werden können: Relativgeschwindigkeit wann?
    Und anscheinend kann ja nicht EIN Zeitpunkt dazu heran gezogen werden, da in ART keine Gleichzeitigkeit existiert.

    Ist die Frage denn so schwer zu verstehen?

  376. #376 MartinB
    9. Juni 2018

    @AlbrechtStorz
    “Warum kann negativer Druck nur zugleich mit Raumexpansion auftreten? ”
    Negativer Druck erzeugt nun mal Raumexpansion, das folgt aus der ART. Prinzipiell kannst du aber negativen Druck und so viel Materiedichte haben, dass der Raum nicht schrumpft, das war ja genau Einsteins Ansatz der kosmologischen Konstante, um das Universum stabil zu halten.

    “Dass das Lamda-CDM-Modell EIN Erklärungsversuch ist.”
    Ja, sicher. Es ist im Moment der beste, den wir haben. Wenn du morgen ein besseres Modell aufstellst, applaudiere ich als erster und schreibe nen entsprechenden Blogeintrag, sobald ich die Veröffentlichung gelesen habe. Es muss “nur” all die Dinge erklären, die die herkömmlichen Modelle auch erklären können.

    “Wenn die Rotverschiebung die Relativgeschwindigkeit zwischen A und B anzeigt”
    Aber das ist doch genau der Punkt – was die Rorverschiebung anzeigt, ist die Veränderung eines Lichtsignals, das sich durch ein expandierendes Universum ausbreitet (siehe meine Erklärung oben). Wer behauptet denn, dass man daraus eindeutig eine Relativgeschwindigkeit definieren kann und wofür wäre so eine Relativgeschwindigkeit sinnvoll? Ist mir gerade nicht klar.

    “Ich hoffte in diesen Kreisen wäre bekannt, wie viele physikalische Effekte wir kennen, die zu einer Rotverschiebung in Spektren führen. Neben gravitativen Wirkungen sind es Wechselwirkungen mit anderen Teilchen oder Feldern.”
    Ja, und bisher hat niemand es geschafft, ein Modell aufzustellen, dass die Rotverschiebung mit diesen Mitteln erklärt und u allen Beobachtungen passt. Wenn dir das gelingt, siehe oben… Aber immer nur zu behaupten “Es gibt doch ganz viele andere mögliche Erklärungen” aber nie mal konkret zu sagen, wie die aussehen sollen, ist schon etwas wenig.

  377. #377 Karl-Heinz
    9. Juni 2018

    @Albrecht Storz
    Bei einer Laserpistole wird bei der Messung ein Folge von kurzen Impulsen ausgesandt.
    Bleibt der zeitliche Abstand zwischen den Impulsen gleich groß wie beim Aussenden, dann ruht das zu messende Objekt. Jedenfalls kann bei einer zeitlichen Differenz des Impulsabstandes die Relativegeschwindigkeit bestimmt werde. Bei der Laserpistole wird natürlich noch über die Laufzeit die Entfernung ermittelt. Bei einer zu grossen Entfernung würden unter Umständen auch andere Autos mit gemessen werden.
    Auf was will ich eigentlich hinaus:
    Ich hoffe ich konnte mit diesem wirklich sehr einfachen Beispiel zeigen, wie man relaive Geschwindigkeit misst.
    Beim Doppler Effekt ist es halt keine Folge von Impulsen sondern Licht.

  378. #378 erik||e oder wie auch immer . . . ..
    10. Juni 2018

    Warum kann negativer Druck nur zugleich mit Raumexpansion auftreten?

    Wenn ich exp( i * π ) + 1 = 0 voraussetze, negativen Druck und Raumexpansion in einem gemeinsamen Bezugssystem betrachte, dann sollte die Mathematik die Physik (und die Ph die Ma) bestätigen.
    Es fehlt noch eine topologische Geometrie der Quantengravitation, welche die Bewegung im Raum und einem singulären negativen Druck beschreibt . . . ..
    . . . .. meine Herren und Physikerinnen – das ist mein Aufruf an die Spezialisten . . . .. schreiten Sie ans Werk 🙂

  379. #379 Karl-Heinz
    10. Juni 2018

    @Albrecht Storz

    Wenn die Rotverschiebung die Relativgeschwindigkeit zwischen A und B anzeigt, so muss trotzdem die Frage geklärt werden können: Relativgeschwindigkeit wann?
    Und anscheinend kann ja nicht EIN Zeitpunkt dazu heran gezogen werden, da in ART keine Gleichzeitigkeit existiert. Ist die Frage denn so schwer zu verstehen?

    Warum nimmst du statt der Zeit nicht die Raumzeit (Ort und Zeit = Ereignis).
    Damit wäre die Beschreibung des Doppler-Effektes für alle Bezugsystem eindeutig.
    Ich nehmen an, du meinst die zwei Ereignisse Aussendung und Empfangen des Lichtes.
    Also wo ist jetzt genau der Widerspruch?
    Ich denke du hast ohne nachzudenken mal so auf die Schnelle einen Widerspruch konstruiert.

  380. #380 Karl-Heinz
    10. Juni 2018

    @ erik||e oder wie auch immer . . . ..

    Wenn ich exp( i * π ) + 1 = 0 voraussetze, negativen Druck und Raumexpansion in einem gemeinsamen Bezugssystem betrachte, …

    Ok ich versuch es mal.
    Zeiger R mit den Komponenten (1, i*0) um 180 Grad in der komplexen Ebene drehen und 1 addieren. (1, i*0) –> (-1, i*0) –> (0, 0) =0.
    So das hätte ich geschafft. Was aber hat diese mathematische Operation mit Druck und Raumexpansion zu tun hat?
    Kann es sein, dass du einfach sinnlos Wörter aneinander reihst und uns dieses, als etwas Tiefsinniges verkaufst? 😉

  381. #381 erik||e oder wie auch immer . . . ..
    10. Juni 2018

    @Karl-Heinz

    Was aber hat diese mathematische Operation mit Druck und Raumexpansion zu tun hat?

    Genau diese Antwort würde eine Beschreibung der Quantengravitation geben . . . ..

    Kann es sein, dass du einfach sinnlos Wörter aneinander reihst und uns dieses, als etwas Tiefsinniges verkaufst?

    . . . .. nein 🙂

  382. #382 Albrecht Storz
    10. Juni 2018

    Wenn hier niemand die Problematik erkennen kann die sich aus einer Relativgeschwindigkeit ohne Gleichzeitigkeit ergibt, lassen wir das halt.

    Ein kleiner Hinweis (trotzdem) zum Verständnis könnte sein, dass das Relativprinzip gilt, also beide Objekte keine absolute Bewegung kennen können. Also das Licht auch nicht irgendwelche Bewegungs-Informationen über den Zustand des Senders zum Empfänger transportieren kann.

    Woher “weiß” das Licht beim Empfänger, welche Rotverschiebung es beim Empfang anzeigen muss?

    und Karl-Heinz, dass Deine Laserpistole hier völlig fehl am Platz ist wirst Du hoffentlich selber Dir erschließen können.

    Ich sehe aber ein, dass ich hier völlig fehl am Platz bin …

    MartinB, “Wenn du morgen ein besseres Modell aufstellst, applaudiere ich als erster und schreibe nen entsprechenden Blogeintrag, sobald ich die Veröffentlichung gelesen habe. Es muss “nur” all die Dinge erklären, die die herkömmlichen Modelle auch erklären können.”, Haha, Superwitz. 🙁
    Wie kommst Du darauf dass man nur etwas kritisieren dürfte wenn man etwas “besseres” anzubieten hat?

    “Ja, und bisher hat niemand es geschafft, ein Modell aufzustellen, dass die Rotverschiebung mit diesen Mitteln erklärt und u allen Beobachtungen passt.”

    Weil alles Geld und Manpower in das Passendmachen des aktuell bestgehypten Modells geht?
    Tatsache ist und bleibt, dass wir Rotverschiebung auf verschiedene Wege experimentell erzeugen und beobachten können (womit diese physikalischen Vorgänge als ziemlich gesichert gelten sollten) – nur nicht mittels Raumexpansion.
    Also lieber eine windige Hypothese verfolgen als auf gesichertes Wissen setzen?

    Na, von mir aus könnt ihr ja machen was ihr wollt …

  383. #383 MartinB
    10. Juni 2018

    @AlbrechtStorz
    “Woher “weiß” das Licht beim Empfänger, welche Rotverschiebung es beim Empfang anzeigen muss?”
    Das muss es nicht wissen, weil die kosmologische Rotverschiebung kein Dopplereffekt ist. Zwei Wellenberge werden mit einem bestimmten räumlichen Abstand ausgesandt und bewegen sich beide entlang von lichtartigen Geodäten. Durch die Expansion ändert sich der Abstand der Wellenberg und die beiden kommen mit größerem Abstand bei uns an. Das ist die Rotverschiebung, dazu braucht man keine Relativgeschwindigkeit zu irgendeiner Zeit.

    “Wie kommst Du darauf dass man nur etwas kritisieren dürfte wenn man etwas “besseres” anzubieten hat?”
    Sachlich fundiert kritisieren ist o.k. – aber das Problem, das du siehst, existiert ja wie mehrfach gesagt nicht.

    Da du aber ja behauptest hast, es gäbe “verschiednee Wege”, die Rotverschiebung zu erklären, solltest dus chon in der Lage sein, die aufzuzählen. (Und ein Blick in die Literatur würde dir zeigen, dass alle die Möglichkeiten, die nach der aktuellen Physik naheliegende Alternativen sein könnten, längst untersucht und ausgeschlossen sind.)

  384. #384 Karl-Heinz
    11. Juni 2018

    @Albrecht Storz

    und Karl-Heinz, dass Deine Laserpistole hier völlig fehl am Platz ist wirst Du hoffentlich selber Dir erschließen können.

    hmm…
    Ich dachte an eine folge von Impulsen mit einer bestimmten Periodendauer mit dem das Auto bestrahlt wird. Wenn sich das Auto bewegt müsste sich die Periodendauer beim Empfang ändern. Am besten werde ich mal nachrechnen. Bei dir habe ich das Gefühl, dass du ziemlich borniert bist.

  385. #385 Niels
    11. Juni 2018

    Und anscheinend kann ja nicht EIN Zeitpunkt dazu heran gezogen werden, da in ART keine Gleichzeitigkeit existiert. Ist die Frage denn so schwer zu verstehen?

    Keine Ahnung, warum darauf noch niemand konkret eingegangen ist.

    Das ist nämlich überhaupt kein Problem.
    Für alle Beobachter, die mit der Expansion des Universums mitbewegt werden, vergeht die selbe Zeit.
    Das heißt ihre Uhren gehen gleich. Sie sind sich einig, wie alt das Universum ist, wann Keplers Supernova explodiert ist, …

    Es gibt in der ART also tatsächlich keine universelle Gleichzeitigkeit von Ereignissen, über die sich alle denkbaren Beobachter einig sein können.
    Gerade für die Beobachter, auf die es in unserem Fall ankommt, gibt es sie aber eben in sehr guter Näherung dann doch.

  386. #386 Alderamin
    11. Juni 2018

    @Albrecht Storz

    “Ein Beispiel bitte. Alles strebt in alle Richtungen von uns weg, vollkommen symmetrisch, umso schneller, je weiter weg. Bitte erkläre, wie das anfing und woher die Geschwindigkeit stammt.”

    warum fragst Du mich das?

    Weil Du behauptest, ein Auseinanderstreben der Materie ginge ohne Urknall. Da Du anscheinend mehr Fantasie hast als ich, frage ich Dich, wie das funktionieren soll. Alles fliegt auseinander, aber einen Zeitpunkt, wo alles superdicht gepackt war und explosionsartig auf den Weg geschickt wurde, gab es nicht. Das wäre nämlich der Urknall.

    Beim Inflationsmodell werden doch von Dir anerkannt, soweit ich das weiß, genau solche Dinge erklärt:

    Das ist ja gerade der “Knall” beim Urknall. Weißt Du eigentlich, worüber Du redest?

    Es scheint Methode zu sein, dass man zur Stützung des Standard-Modells jede beliebige Annahme heranziehen darf, diese aber bei Alternativmodellen von vorne herein ausgeschlossen werden, wie es gerade passt.

    Das Standardmodell (der Kosmologie) braucht die Inflation nicht unbedingt. Aus den Beobachtungen (Entwicklung des Skalenfaktors) folgt, dass der Raum expandiert und alles in einem superdichten Zustand begonnen hat. Wie dann die ersten Bruchteile der ersten Sekunde aussahen, darüber gibt es verschiedene Hypothesen (die Inflation ist eine davon). Egal welche davon stimmt oder nicht, damit ändert sich nichts an der Raumexpansion und dem superdichten Anfangszustand, aka “Urknall”.

    Warum ist es eigentlich so schwer zu begreifen, dass kein vernünftiger Mensch etwas gegen die Aussage haben kann, dass das Standardmodell, dass Lambda-CDM, dass Relativitätstheorie sicher aktuell die beste Erklärung für unsere Welt und den Kosmos liefert (auf jeden Fall die bestuntersuchte Erklärung wenn man nach Manpower und Finanzierung geht) aber diese Theorien und Hypothesen eben keine durchgängig gesicherten Wahrheiten darstellen.

    Eigentlich hat niemand was anderes behauptet… gesicherte Wahrheiten gibt es in der Naturwissenschaft nicht. Das ist so selbstverständlich, dass man es nicht nach jedem Satz nochmal erwähnen muss. Allerdings ist auch kein besseres Modell als ΛCDM in Sicht, im Gegenteil, das Modell wird immer wieder bestätigt. Es gibt derzeit genau drei Dinge, die nicht ins Bild passen:

    1) H0 aus der Hintergrundstrahlung ist signifikant verschieden von H0 aus näher gelegenen Galaxienmessungen.
    2) Die DM-Teilchen sind noch nicht gefunden
    3) Es gibt keine Theorie, die die Dunkle Energie auf Basis der Quantenfeldtheorie beschreiben könnte.

    Zu 1) gibt es noch keine gute Idee. Zu 2): neue Teilchen wurden im letzten Jahrhundert andauernd gefunden und aus dem Asysmmetrie von Materie und Antimaterie weiß man, dass das Standardmodell der Quantenphysik nicht der Weisheit letzter Schluss sein kann. Was dann vermutlich auch die Lösung für 3) und 1) ist.

    Dass wir noch nicht alles wissen, heißt nicht, dass alles andere falsch ist. Solange ΛCDM die Beobachtungen ansonsten hervorragend beschreibt.

    Wir haben einen Sender, zB eine Galaxie in 13 Mrd. Lichtjahren Entfernung und einen Empfänger, wir, heute. Wie wird nun die Relativgeschwindigkeit zwischen der Galaxie und uns definiert? da es keine Gleichzeitigkeit gibt, kann man nicht sagen “jetzt”, zum Zeitpunkt x, gilt Relativ-Geschwindigkeit y.

    Die Geschwindigkeit ferner Galaxien ist bis auf lokale Eigenbewegungen 0. Was man beobachtet ist die akkumulierte Rotverschiebung über die Lichtlaufzeit, und die sagt direkt aus, um wieviel das Universum seit der Aussendung des Lichts gewachsen ist (Skalenfaktor = 1/(1+Rotverschiebung) ). Die Rotverschiebung hängt davon ab, wie lange das Licht im Raum unterwegs war und um wieviel das Universum in dieser Zeit gewachsen ist, mit Gleichzeitigkeit hat das absolut nichts zu tun, eher im Gegenteil. Wenn man den Skalenfaktor für verschiedene z gemessen und die Abhängigkeit in eine Formel gepackt hat, folgt daraus auch die Lichtlaufzeit und damit, wie schnell das Universum gewachsen ist (Hubble-Paramter = a'(t)/a(t)).

    Und bitte: Wenn Du nicht verstehst was ich meine, dann antworte auch nicht auf meine Frage.

    Wenn Du nicht verstehst, was man Dir erklärt, dann frag’ halt weiter…

    wo ist da der negative Druck? Warum kann negativer Druck nur zugleich mit Raumexpansion auftreten?

    Ist hier anschaulich erklärt und hier hergeleitet. Ja, man muss die ART verstehen, um das nachvollziehen zu können, aber dann ist es zwingend. Positiver Druck wirkt übrigens aus dem gleichen Grund anziehend und sorgt dafür, dass Sterne zu Neutronensternen kollabieren, wenn ihr Energievorrat aufgebraucht ist.

    Der negative Druck, der sich bei einer gewissen Vakuumenergie ergibt, ist ein Erklärungsversuch für die Dunkle Energie. Sollte er sich als falsch herausstellen, ist die Dunkle Energie (der Effekt der beschleunigten Expansion) aber immer noch da, man kann ihn ja direkt beobachten (Entwicklung des Skalenfaktors).

    Dass das Lamda-CDM-Modell EIN Erklärungsversuch ist. Und das das stimmen kann, oder auch nicht. Und dass wenn man in der Kosmologie nur noch danach trachtet, das gerade am besten gefallende Modell so lange zurecht zu parametrisieren, bis es zu allen (aktuell bekannten) Beobachtungsdaten passt

    Und die Alternative soll sein, eine “Theorie” zu postulieren, die überhaupt nicht zu den Beobachtungsdaten passt? Dass die Materie im Universum sich in einen leeren Raum ausbreitet, der nicht expandiert? *Kopfkratz*

    Nörgeln ist immer supereinfach, dafür muss man nichts gelernt haben. Wohl aber, wenn man das notwendige tun will, nämlich was besseres vorzuschlagen.

  387. #387 Albrecht Storz
    13. Juni 2018

    “Nörgeln ist immer supereinfach, dafür muss man nichts gelernt haben. Wohl aber, wenn man das notwendige tun will, nämlich was besseres vorzuschlagen.”

    Das ist alles hier einfach nur lächerlich. Gehabt Euch wohl …

  388. #388 Albrecht Storz
    13. Juni 2018

    Perlen wohin geworfen:

    “The scientific approach to the situation would be to choose a model, determine the parameters that best fit observations, and then revise the model as necessary – ie, as new data comes in. But that’s not what cosmologists presently do. Instead, they have produced so many variants of models that they can now “predict” pretty much anything that might be measured in the foreseeable future.

    It is this abundance of useless models that gives rise to the criticism that inflation is not a scientific theory. And on that account, the criticism is justified. It’s not good scientific practice. It is a practice that, to say it bluntly, has become commonplace because it results in papers, not because it advances science.”

    https://backreaction.blogspot.com/2017/10/is-inflationary-universe-scientific.html

    Ihr praktiziert hier die selbe Antiwissenschaft, aber eben auf einem niedrigeren Niveau: Kritik wird zuerst einmal gar nicht nachvollzogen. Als angebliche “Widerlegung” werden dann unverstanden und unverdaute Wissensfetzen wortreich aneinander gereiht um dann in dem Urteil zu münden, damit wäre alle Kritik widerlegt. Und überhaupt hätten Nichtwissenschaftler überhaupt nichts zu melden und außerdem würde alles in der Mathematik stehen, die, wenn man sie nicht beherrscht, einen sowieso als ernst zu nehmenden Person ausschließt.

    Schmort ruhig weiter in Eurem Saft. Und praktiziert weiter Eure Religion und Eure Mainstream-Wissenschaftler-Anbetung ohne dass ich stören würde. Ich bin es Leid mit soviel Arroganz und Unverständnis konfrontiert zu sein.

  389. #389 Alderamin
    13. Juni 2018

    @Albrecht Storz

    Frau Hossenfelder hat absolut recht!

    The scientific approach to the situation would be to choose a model

    Ok, haste gemacht.

    determine the parameters that best fit observations

    Haste nicht gemacht! Um die observations, die Dir genannt wurden hast Du Dich einen Dreck geschert, da bist Du mit keinem Wort drauf eingegangen. Finde den Fehler!

    and then revise the model as necessary

    Ist aber nicht necessary, ΛCDM beschreibt die Beobachtungen ja korrekt, Doppler nicht.

    Der Rest bezieht sich auf die Inflation, die war aber nicht das Thema. Torpfosten schön stehen lassen, der Ball ging vorbei. Meterweit.

    Kritik wird zuerst einmal gar nicht nachvollzogen.

    Schon, aber die Argumente dagegen wurden nicht beachtet. Nicht jede Kritik ist berechtigt.

    Als angebliche “Widerlegung” werden dann unverstanden und unverdaute Wissensfetzen wortreich aneinander gereiht

    Ach’ da hängt der Hase, Du hast die Antworten nicht verstanden… mach’ Dich halt schlau und frag’ nach. Bevor man sagt “was die Wissenschaft da sagt, ist alles Käse” könnte man ja mal den Ansatz versuchen “Ich habe das soundso verstanden. Könnte es nicht sein, dass…?”. Dann bekommt man hier auch freundlich Antwort.

    Wie gesagt, nörgeln ist einfach, verstehen und mitdenken ist was ganz anderes, bei den Großen mitreden nochmal eine andere Nummer (was keiner der hier Anwesenden für sich beanspruchen wird, da bin ich absolut sicher).

    Und überhaupt hätten Nichtwissenschaftler überhaupt nichts zu melden

    Wer keine Ahnung hat, lieber mal.. usw. halten.

    und außerdem würde alles in der Mathematik stehen, die, wenn man sie nicht beherrscht, einen sowieso als ernst zu nehmenden Person ausschließt.

    Tja, wer ein großer Literaturkritiker werden will, sollte lesen können. Und wer ein NaWi-Kritiker werden will, sollte rechnen können. In beiden Fällen kann er sonst nicht verstehen, was er da eigentlich kritisiert.

    Schmort ruhig weiter in Eurem Saft. Und praktiziert weiter Eure Religion und Eure Mainstream-Wissenschaftler-Anbetung ohne dass ich stören würde. Ich bin es Leid mit soviel Arroganz und Unverständnis konfrontiert zu sein.

    Wer keine Argumente mehr hat, der hat zumindest noch ein paar Beleidigungen, die für ihn selbst wie Argumente aussehen mögen. Für andere eher nicht.

  390. #390 MartinB
    13. Juni 2018

    ” Und überhaupt hätten Nichtwissenschaftler überhaupt nichts zu melden und außerdem würde alles in der Mathematik stehen, die, wenn man sie nicht beherrscht, einen sowieso als ernst zu nehmenden Person ausschließt.”
    Ja, das ist in weiten teilen richtig. Ich sollte mich zum Beispiel nicht zur Frage äußern, wie man ein fußballspiel gewinnt, weil ich davon keine Ahnung habe, und sollte meiner Autoschrauberfirma auch nicht sagen, wo sie in meiner Karre was wie reparieren sollen, weil ich davon nichts verstehe.
    Es ist in der Welt tatsächlich so, dass man Wissen braucht, um Dinge zu verstehen und möglicherweise zu verbessern.

    PS: Das war jetzt der wievielte Kommentar, in dem du angekündigt hast, hier nicht mehr zu kommentieren?

  391. #391 Martin Anneken
    8. Juni 2019

    Meine Überlegungen, hauptsächlich zur Expansion, scheinen ein Jahr verspätet zu sein, sind aber wohl immer noch aktuell.
    Ich habe nun alle Kommis durch und fange mal mit dem letzten an:

    @MartinB # 390
    “Ja, das ist in weiten teilen richtig. Ich sollte mich zum Beispiel nicht zur Frage äußern, wie man ein fußballspiel gewinnt, weil ich davon keine Ahnung habe, und sollte meiner Autoschrauberfirma auch nicht sagen, wo sie in meiner Karre was wie reparieren sollen, weil ich davon nichts verstehe.
    Es ist in der Welt tatsächlich so, dass man Wissen braucht, um Dinge zu verstehen und möglicherweise zu verbessern. ”

    Der letzte Satz spiegelt meine Motivation wieder, mich in der Physik umzuschauen. Ich will >die WeltDerZimmermann #123<

    "Wenn t gleich 0 ist müsste ich den Objekten, die bisher dem Fluss des Raumes gefolgt sind, eine Geschwindigkeit radial nach innen mit v(D,t=0) = v0*D/R0 verpassen.
    Ob dass, auch funktioniert, wenn der Raum sich beschleunigt ausdehnt?
    MartinB sagt ja, ich sage nein.
    MartinB sagte ja auch, dass genau dann eine Kraft entsteht, wenn sich die relative Änderungsrate der Ausdehnung zeitlich ändert. Wie wir aber gesehen haben, ist unsere Änderungsrate zeitlich nicht konstant und dennoch entsteht keine Kraft."

    Mir ist es verborgen, wie du bei einer Zeit von 0 eine Geschwindigkeit oder Beschleunigung generierst. Allerdings leuchtet mir ein, dass dabei dann auch keine Kraft entsteht.
    Vielleich hilft es ja, den Kommi #170 (auch #168) von MartinB aufmerksam zu lesen. Er hat sich bisher sehr viel Mühe gemacht, seine Gedanken mit verständlichen Worten darzulegen:

    "Konstante Dehnrate heißt:
    Der Abstand zwischen zwei Punkten wächst proportional zum Abstand der Punkte (für jeden Meter Abstand gibt es pro Sekunde x Mikrometer neuen Raum), und diese Rate x ändert sich nicht mit der Zeit.
    Natürlich wird eine anfänglich vorgegebene Länge dadurch exponentiell gedehnt, das ist aber kein Widerspruch zu meiner Aussage."

    @Aldemarin #386
    "Die Geschwindigkeit ferner Galaxien ist bis auf lokale Eigenbewegungen 0. Was man beobachtet ist die akkumulierte Rotverschiebung über die Lichtlaufzeit, und die sagt direkt aus, um wieviel das Universum seit der Aussendung des Lichts gewachsen ist (Skalenfaktor = 1/(1+Rotverschiebung) )."

    Eine Geschwindigkeit von 0 für ferne Galaxien ist für mich neu. Ich habe immer gedacht, sie sei in die Rorverschiebung integriert. Kannst du mir das näher begründen (ein Link auf deutsch tut's auch)?

  392. #392 Martin Anneken
    Rheine
    8. Juni 2019

    Dieses Schreibprogramm mag keine langen Texte, oder ich mache irgend welche Formatierungs-Fehler. Ich versuche mal die fehlende Zwischen-Passage nachzuholen.
    ………….
    Der letzte Satz spiegelt meine Motivation wieder, mich in der Physik umzuschauen. Ich will die Welt verstehen, soweit es meine Bildung (Volksschule), Ausbildung (Schlosser) und Weiterbildung (Elektriker und Funk-Mechaniker) erlauben.
    Falls ich mit meinen Meinungen nicht richtig liege, soll gerne korrigiert werden – man lernt ja nie aus.

    @MartinB #41
    “Meine Frage ist, ob es einen Unterschied macht, wenn ich *alles* im Universum mit einer zusätzlichen konstanten Geschwindigkeit versehe. Wenn ich sämtliche Galaxien, Teilchen etc. mit einem zusätzlichen Geschwindigkeitsvektor versehen würde, wäre daraus irgendein Effekt beobachtbar?”

    Eine universal beaufschlagte Geschwindigkeit wäre durch keinen Effekt beobachtbar. Das liegt nicht an der Lorentz-Invarianz, sondern am Machschen Prinzip. Das heißt, dass die Gesamtheit aller Energie und Materie den Nullpunkt IM Raum darstellt, ohne jedoch an ihn gebunden zu sein.
    Ungefähr die selbe Schiene nutzt du in deinem Kommi #226, in dem es im letzten Absatz heißt:
    “Nur für ein mitbewegtes Objekt ist der Raum homogen und isotrop, für jedes andere nicht. Wenn es also mitbewegte Objekte gibt, kann ich messen, in wie weit der Raum isotrop ist und damit feststellen, ob ein Teilchen in seinem Bezugssystem auch mitbewegt ist.”

    Ich denke mal, der Lorentz-Faktor hat mit der kosmischen Expansion nichts zu tun, da diese eine Raum-Expansion ist und keine Raum-Zeit-Expansion.

    @DerZimmermann #123
    “1) Ich lege 100€ an und bekomme jedes Jahr 1% Zinsen, also erhöht sich mein Kontostand pro Jahr um den Faktor 1,01; die Wachstumsrate ist also 1,01 … dann steigt mein Kontostand exponentiell an mit einer festen Wachstumsrate.
    2) Ich … bekomme jeden Monat 2000€ aufs Konto, … dann erhöht sich mein Kontostand jedes Monat um 2000€. … Es handelt sich hier im Grunde seines Wesens um ein lineares Wachstum
    3) Jetzt werfe ich mal einen Blick auf ein Universum.
    Ein leeres Universum, ohne Einflüsse auf seine Expansion, expandiert linear, d.h., der Skalenfaktor steigt linear an – genau wie mein Kontostand.”

    Ich denke nicht. Die Expansion ist exponentiell wie 1), nur die Geschwindigkeit steigt linear mit der Entfernung, so wie auch die Zinsen linear mit dem Kapital steigen. Das gleicht einer gleichmäßigen (linearen) Beschleunigung.

    @MartinB # 180
    “Aber wen du zu jedem Zeitpunkt t immer einen Punkt betrtachtest, der jetzt zu diesem Zeitpunkt 1 Meter von dir entfernt ist, dann hat der Stab *an diesem Punkt* immer dieselbe Geschwindigkeit. Und deshalb kann ein Objekt, das sich an diesem Punkt befindet und eine bestimmte Geschwindigkeit relativ zjm Stabmaterial hat, auch kräftefrei am Ort verharren.”

    Dem stimme ich zu bis auf den Vermerk, dass diese Geschwindigkeit wegen dem Orts- und Zeit-PUNKT nur infinitesimal sein kann. Diesem Sachverhalt kommst du im nachfolgendem Kommi #181 ja schon näher mit dx/dt. Vielleicht hilft dieser Sachverhalt ja weiter.

    @MartinB #190
    “Wenn der Hubble-Parameter 74km/sMpc beträgt, und ich mich in 1 Mpc Entfernung von dir befinde, dann kann ich mit einer relativgeschwindigkeit von 1km/s relativ zum mitbewegten Beobachter relativ zu dir ruhen,”

    Ist da ein Flüchtigkeitsfehler? Nicht 1 km/s sondern 74 km/s (relativ zum expansiv mitbewegten Beobachter).

    @DerZimmermann #200
    “Da aber bei konstanter Expansion (a‘ konstant) die Geschwindigkeit des Gummibandes an der Testkörper-Stelle sinkt (der Testkörper bleibt ja in derselben Entfernung zu mir), müsste der Testkörper ja eigentlich beginnen, sich auf mich zuzubewegen.”

    Da widersprichst du dir selbst. Da der TK-Abstand konstant bleibt, muss auch die Geschwindigkeit des Gummibandes an der TK-Stelle gleichbleiben, denn es wird ja auch ein konstanter HP (Hubble-Parameter) vorausgesetzt.
    Damit stimmt auch der Folge-Absatz nicht, in dem die DE in die Betrachtung dazu kommt:

    “In diesem Fall bleibt die Geschwindigkeit des Gummis an der Testkörper-Entfernung immer gleich, und damit sollte der Testkörper immer an dieser Stelle bleiben, weil er ja die benötigte Relativgeschwindigkeit von mir bekommen hat.”

    Das trifft genau auf die unbeeinflusste (ohne DE) konstante Expansion zu (siehe auch den Kommi #170 von MartinB).
    Konstant heißt, die HP-Zahl bleibt gleich. Damit ist die Längen-Zunahme einer Einheits-Strecke pro Zeit-Einheit gemeint, aber nicht eine gleichbleibende Geschwindigkeit für diese Einheits-Strecke, auch wenn das Ergebnis identisch ist.
    Anschaulich HP = (km/Mpc) /s
    jedoch nicht HP = (km/s) / Mpc.
    Deine angeführten Friedmann-Gleichungen kannst du kaum für dein relativ leeres Universum anwenden, wenn du wenigstens nicht auch den HP mit einbeziehst. Es PASST also nicht, da du die Expansion allgemein verneinst, sie nicht mitrechnest und nur die DE als expansionstreibend erachtest.

    @Karl-Heinz #202
    “So jetzt lassen wir den Raum sich linear ausdehnen. Unter einer linearen Expansion des Raumes verstehe ich, dass eine konstante Sphäre um mich mit konstanter Geschwindigkeit sich ausdehnt. Der innere und äußere Bereich der Sphäre wird entsprechen mitskaliert. Die Probekörper lasse ich mit dem Raum mitfließen und sie bewegen sich dadurch von mir aus gesehen radial mit konstanter Geschwindigkeit ohne Beschleunigung nach außen.”

    Diese Interpretation ist aber falsch, denn sie stellt eine anfängliche entfernungs-abhängige Geschwindigkeits-Verteilung IM Raum dar, die zeitinvariant ist. Das entspricht aber nicht der kosmologischen Expansion. Das wird in deinem nächsten Absatz deutlich, in dem du meinst:

    “Das Bezugsystem, wo sich der Körper ursprünglich befunden hat, entfernt sich mit konstanter Geschwindigkeit sowohl vom Körper als auch von mir.”

    Die kosmologische Expansion ist keine zeitlich konstante Geschwindigkeit eines wie auch immer festgelegten Raumpunktes oder Testkörpers. Sie ist nur infinitesimal konstant beim Durchqueren einer darauf bezogenen Entfernung und deshalb auch zeitlich infinitesimal.
    Bei einer passenden Gegenbewegung ist diese Geschwindigkeit deshalb konstant, weil sie bei konstantem Abstand erfolgt, womit sie auch zeitlich konstant ist.
    Dieser Irrtum rührt vielleicht daher, dass einfach angenommen wird, die Expansion sei linear – ist sie aber nicht, wie ich schon oben weiter geäußert habe im Kommi an “DerZimmermann #123”

    “Wenn t gleich 0 ist müsste ich den Objekten, die bisher dem Fluss des Raumes gefolgt sind, eine Geschwindigkeit radial nach innen mit v(D,t=0) = v0*D/R0 verpassen.
    ………. weiter siehe oben

    Ich denke mal, dass man da noch durchblickt, und danke euch für euer Verständnis. M.A.

  393. #393 MartinB
    9. Juni 2019

    @MartinAnneken
    “Das liegt nicht an der Lorentz-Invarianz, sondern am Machschen Prinzip. Das heißt, dass die Gesamtheit aller Energie und Materie den Nullpunkt IM Raum darstellt, ohne jedoch an ihn gebunden zu sein.”
    Ich rede ja hier von der Situation, dass ich Materie quasi als Staub (der keine Rückwirkung auf die Raumzeit hat) verteile – da gilt das Machsche Prinzip auf keinen Fall. (Ob es sonst generell gilt, ist meines Wissens unklar.) Und inzwischen habe ich dazugelernt und weiß sicher, dass sich etwas anderes ergeben würde, wenn man den Staub mit konstanter Geschwindigkeit bewegt.

    “da diese eine Raum-Expansion ist”
    Sollte man so nicht sehen – Raum und Zeit sind immer verbunden, und ob etwas Raum-Expansion oder Raumzeit-Expansion ist, hängt davon ab, welches Koordinatensystem du nimmst.

    “Ist da ein Flüchtigkeitsfehler? Nicht 1 km/s sondern 74 km/s (relativ zum expansiv mitbewegten Beobachter”#Ja, da hab ich mich wohl vertan.

    Den rest des Kommentares verstehe ich nicht so richtig (habe im Moment aber auch nicht sehr viel Zeit.)

    Ansonsten verweise ich mal auf mein Buch, da hab ich ein ganzes Kapitel über die Expansion, und das habe ich geschrieben, als ich deutlich mehr wusste als seinerzeit beim Schreiben dieses Blogartikels und der Kommentare hier…
    https://www.springer.com/de/book/9783662572924

  394. #394 Karl-Heinz
    10. Juni 2019

    @Martin Anneken

    Ich habe nur auf das Modelluniversum (Milne Modell) Bezug genommen. Mal gucken, ob ich es noch schaffe den Skalenfaktor und andere Größen aus den Ärmel zu schütteln.

    a(t) = c · t / (c · t0) = t/t0
    t0 … Jetztzeit
    a'(t)= 1/t0
    H(t) = a'(t) / a(t) = (1/t0)/(t/t0)=1/t
    a”(t)/a(t) =0